You are on page 1of 273

SỬ DỤNG ÁNH XẠ ĐỂ CHỨNG MINH ĐẲNG THỨC,

BẤT ĐẲNG THỨC TỔ HỢP

Định nghĩa 1
Cho hai tập hợp 𝑋 và 𝑌 . Một ánh xạ 𝑓 từ tập 𝑋 đến tập 𝑌 là một quy tắc đặt tương ứng mỗi
phần tử 𝑥 của 𝑋 với một (và chỉ một) phần tử 𝑦 của 𝑌 . Phần tử này được gọi là ảnh của 𝑥 qua
ánh xạ 𝑓 và được ký hiệu là 𝑦 = 𝑓 (𝑥).
Tập hợp 𝑋 được gọi là tập xác định của 𝑓 . Tập hợp 𝑌 được gọi là tập giá trị của 𝑓.
Định nghĩa 2
i) Ánh xạ 𝑓 : 𝑋 −→ 𝑌 được gọi là đơn ánh nếu với 𝑎, 𝑏 ∈ 𝑋 mà 𝑎 ̸= 𝑏 thì 𝑓 (𝑎) ̸= 𝑓 (𝑏). Nói một
cách khác ánh xạ 𝑓 là một đơn ánh nếu và chỉ nếu với 𝑎, 𝑏 ∈ 𝑋 mà 𝑓 (𝑎) = 𝑓 (𝑏) thì suy ra 𝑎 = 𝑏.
ii) Ánh xạ 𝑓 : 𝑋 −→ 𝑌 được gọi là toàn ánh nếu với mỗi phần tử 𝑦 ∈ 𝑌 đều tồn tại một phần
tử 𝑥 ∈ 𝑋 sao cho 𝑓 (𝑥) = 𝑦. Như vậy 𝑓 là toàn ánh nếu và chỉ nếu 𝑓 (𝑋) = 𝑌 .
iii) Ánh xạ 𝑓 : 𝑋 −→ 𝑌 được gọi là song ánh giữa 𝑋 và 𝑌 nếu và chỉ nếu nó vừa là đơn ánh và
vừa là toàn ánh. Như vậy 𝑓 là song ánh nếu với mỗi 𝑦 ∈ 𝑌 tồn tại duy nhất một phần tử 𝑥 ∈ 𝑋
sao cho 𝑦 = 𝑓 (𝑥).
Định lí 1. Cho 𝐴 và 𝐵 là hai tập hợp hữu hạn.
∙ Nếu có một đơn ánh 𝑓 : 𝑋 −→ 𝑌 thì |𝑋| ≤ |𝑌 |.
∙ Nếu có một toàn ánh 𝑓 : 𝑋 −→ 𝑌 thì |𝑋| ≥ |𝑌 |.
∙ Nếu có một song ánh 𝑓 : 𝑋 −→ 𝑌 thì |𝑋| = |𝑌 |.
1. Sử dụng song ánh để đếm tổ hợp
Ví dụ 1. Cho 𝑋 = {1, 2, ..., 𝑛}. Có bao nhiêu tập con 𝑘 phần tử của X sao cho trong mỗi
tập con không chứa 2 số nguyên liên tiếp.
Lời giải
Gọi A là tập tất cả các tập con 𝑘 phần tử của X mà trong mỗi tập không chứa 2 số nguyên
liên tiếp và B là tập tất cả các tập con của tập 𝑌 = {1, 2, ..., 𝑛 − (𝑘 − 1)}. Ta xây dựng song
ánh từ A đến B như sau: Lấy 𝑆 = {𝑠1 , 𝑠2 , ..., 𝑠𝑘 } ∈ 𝐴 (không mất tổng quát có thể giả sử
𝑠1 < 𝑠2 < ... < 𝑠𝑘 ) đặt tương ứng với 𝑓 (𝑆) = {𝑠1 , 𝑠2 − 1, 𝑠3 − 2, ..., 𝑠𝑘 − (𝑘 − 1)}. Dễ chứng minh
𝑘
đây là một song ánh. Từ đó có 𝐶𝑛−𝑘+1 tập thoả yêu cầu đề bài.
Ví dụ 2. Hãy tính trung bình cộng của tất cả các số N gồm 2014 chữ số thỏa mãn N chia
hết cho 9 và các chữ số của N được lập từ 𝑋 = {1, 2, ..., 8}
Lời giải
Gọi M là tập các số thỏa yêu cầu đề bài.
Ta xây dựng một ánh xạ đi từ M đến M như sau: Với mỗi 𝑁 = 𝑎1 𝑎2 ...𝑎2014 ∈ 𝑀 dặt 𝑓 (𝑁 ) =
𝑏1 , 𝑏2 , ..., 𝑏2014 với 𝑏𝑖 = 9 − 𝑎𝑖 với mọi 𝑖 = 1, 2, ..., 2014. Vì 𝑁 + 𝑓 (𝑁 ) = 99...9 (2014 số 9) chia
hết cho 9 và N chia hết cho 9 nên suy ra 𝑓 (𝑁 ) cũng chia hết cho 9. Do đó 𝑓 là một ánh xạ đi

Nguyễn Ngọc Duy - Trường Phổ Thông Năng Khiếu ĐHQG TPHCM 1
từ M vào M. Hơn nữa dễ thấy 𝑓 là một song ánh. Từ đó suy ra
∑︁ ∑︁
2 𝑁= (𝑁 + 𝑓 (𝑁 )) = |𝑀 |.99...9.
𝑁 ∈𝑀 𝑁 ∈𝑀

Vậy trung bình cộng của các số trong M là 99...9 : 2.


Ví dụ 3. Cho các số tự nhiên 𝑘, 𝑛, 𝑚 thỏa điều kiện 1 < 𝑘 ≤ 𝑛, 𝑚 > 1. Hỏi có bao nhiêu
bộ sắp thứ tự (𝑎1 , 𝑎2 , ..., 𝑎𝑘 ) của 𝑛 số tự nhiên đầu tiên mà mỗi bộ đều thỏa mãn ít nhất một
trong hai điều kiện sau:
i) Tồn tại 𝑖, 𝑗 ∈ {1, 2, ..., 𝑘} sao cho 𝑖 < 𝑗 và 𝑎𝑖 > 𝑎𝑗 .
ii) Tồn tại 𝑖 ∈ {1, 2, ..., 𝑘} sao cho 𝑎𝑖 − 𝑖 không chia hết cho 𝑚.
Lời giải
Gọi A là tập tất cả những chỉnh hợp chập 𝑘 của 𝑛 số tự nhiên đầu tiên, A’ là tập các chỉnh hợp
thỏa yêu cầu đề bài.
.
Đặt 𝐵 = {(𝑎1 , 𝑎2 , ..., 𝑎𝑘 ) ∈ 𝐴 : 𝑎1 < 𝑎2 < ... < 𝑎𝑘 và 𝑎𝑖 − 𝑖 .. 𝑚, 𝑖 = 1, 2, ..., 𝑘},
.
𝐵 ′ = {(𝑏 , 𝑏 , ..., 𝑏 ) : 𝑏 < 𝑏 < ... < 𝑏 , 𝑏 ∈ {1, 2, ..., 𝑛 − 𝑘 + 𝑘𝑚}, 𝑏 ..𝑚, ∀𝑖 = 1, 2, ...𝑘}.
1 2 𝑘 1 2 𝑘 𝑖 𝑖

Xét ánh xạ 𝑓 đi từ B đến B’ như sau: với mỗi (𝑎1 , 𝑎2 , ..., 𝑎𝑘 ) ∈ 𝐵 ta cho tương ứng với 𝑓 (𝐵) =
(𝑎1 − 1 + 𝑚, 𝑎2 − 2 + 2𝑚, ..., 𝑎𝑘 − 𝑘 + 𝑘𝑚). Khi đó 𝑓 là song ánh từ B đến B’ nên |𝐵| = |𝐵 ′ | =
𝐶⌊𝑘𝑛−𝑘 ⌋+𝑘 .
𝑚
Mà 𝐴′ = 𝐴 ∖ 𝐵 nên |𝐴′ | = |𝐴| − |𝐵| = 𝐴𝑘𝑛 − 𝐶⌊𝑘𝑛−𝑘 ⌋+𝑘
𝑚
2. Sử dụng song ánh để chứng minh đẳng thức tổ hợp
Ví dụ 4. Gọi 𝑎𝑛 là số các xâu nhị phân độ dài 𝑛 không chứa ba bit 0, 1, 0 liên tiếp. Gọi
𝑏𝑛 là số các xâu nhị phân độ dài 𝑛 không chứa bốn bit 0, 0, 1, 1 hoặc 1, 1, 0, 0 liên tiếp. Chứng
minh rằng 𝑏𝑛+1 = 2𝑎𝑛 với mọi số nguyên dương 𝑛.
Lời giải
Gọi 𝐴𝑛 , 𝐵𝑛 lần lượt là tập các xâu nhị phân độ dài 𝑛 thỏa điều kiện thứ nhất và thứ hai. Với
mỗi xâu nhị phân (𝑥1 , 𝑥2 , ..., 𝑥𝑛 ) ta cho tương ứng với một xâu nhị phân (𝑦0 , 𝑦1 , ..., 𝑦𝑛 ) xác định
bởi 𝑦0 = 0 và
𝑦𝑖 = 𝑥1 + 𝑥2 + ... + 𝑥𝑖 𝑚𝑜𝑑 2, 𝑖 = 1, 2, ..., 𝑛. (*)

Khi đó
𝑥𝑖 = 𝑦𝑖 − 𝑦𝑖−1 𝑚𝑜𝑑 2, 𝑖 = 1, 2, ..., 𝑛.

Dễ thấy (*) là một song ánh giữa tập tất cả các xâu nhị phân độ dài 𝑛 và tập tất cả các xâu
nhị phân độ dài 𝑛 + 1 trong đó có bit đầu tiên là 0. Hơn nữa xâu nhị phân (𝑥1 , 𝑥2 , ..., 𝑥𝑛 ) có
3 bit 0,1,0 liên tiếp theo thứ tự này khi và chỉ khi xâu nhị phân tương ứng (𝑦0 , 𝑦1 , ..., 𝑦𝑛 ) có 4
bit liên tiếp theo thứ tự là 0,0,1,1 hoặc 1,1,0,0. Nói cách khác một xâu nhị phân thuộc 𝐴𝑛 sẽ
tương ứng với một xâu nhị phân thuộc 𝐵𝑛+1 và bắt đầu bằng bit 0. Vì số xâu nhị phân thuộc
vào 𝐵𝑛+1 bắt đầu bằng bit 0 đúng bằng một nửa số xâu nhị phân thuộc vào 𝐵𝑛+1 do đó ta có
𝑏𝑛+1 = 2𝑎𝑛 (điều phải chứng minh).

Nguyễn Ngọc Duy - Trường Phổ Thông Năng Khiếu ĐHQG TPHCM 2
Ví dụ 5. Gọi M là số số nguyên dương trong hệ thập phân có 2𝑛 chữ số trong đó có 𝑛
chữ số 1 và 𝑛 chữ số 2. Gọi N là số số nguyên dương có 𝑛 chữ số trong hệ thập phân trong đó
chỉ có các chữ số 1, 2, 3, 4 và số chữ số 1 bằng số chữ số 2. Chứng minh |𝑀 | = |𝑁 |.
Lời giải
Ta sẽ chứng minh rằng có một song ánh đi từ N vào M. Lấy một số bất kỳ thuộc vào N, ta sẽ
cho tương ứng với một số thuộc vào M theo cách: đầu tiên viết hai số này liền nhau ta sẽ được
một số có 2𝑛 chữ số , sau đó các chữ số 3 ở 𝑛 chữ số đầu tiên được đổi thành chữ số 1, các chữ
số 3 ở 𝑛 chữ số tiếp theo được đổi thành chữ số 2. Tương tự, các chữ số 4 ở 𝑛 chữ số đầu tiên
được đổi thành chữ số 2, các chữ số 4 ở 𝑛 chữ số tiếp theo được đổi thành các chữ số 1. Với cách
làm này ta thu được một số thuộc vào M và hiển nhiên đây là một đơn ánh. Để chứng minh
đây là một song ánh ta xây dựng ánh xạ ngược như sau:
Với một số có 𝑛 chữ số 1 và 𝑛 chữ số 2 ta cắt đôi 𝑛 chữ số đầu và 𝑛 chữ số cuối sau đó đặt
chúng song song nhau và thực hiện phép cộng như sau:

1 + 1 = 1, 2 + 2 = 2, 1 + 2 = 3, 2 + 1 = 4.

Khi đó ta sẽ thu được một số thuộc vào N. Như vậy có một song ánh giữa hai tập M và N nên
|𝑀 | = |𝑁 |.
Ví dụ 5. Cho 𝑋 = {1, 2, ..., 𝑛}. Một tập con A của X được gọi là tập béo nếu mỗi phần
tử của A đều không nhỏ hơn số phần tử của nó. Tập rỗng cũng là một tập béo. Đặt 𝑎𝑛 là số các
tập béo của X mà trong mỗi tập không chứa hai số liên tiếp, 𝑏𝑛 là số các tập con của X mà hai
phần tử bất kỳ hơn kém nhau ít nhất 3 đơn vị. Chứng minh 𝑎𝑛 = 𝑏𝑛 .
Lời giải
Gọi A là họ các tập béo thỏa yêu cầu đề bài, B là họ các các tập con của X có tính chất
hai phần tử bất kỳ hơn kém nhau 3 đơn vị. Ta thiết lập một ánh xạ 𝑓 đi từ A đến B như
sau: giả sử 𝑥 = {𝑎1 , 𝑎2 , ..., 𝑎𝑘 } ∈ 𝐴, ta có thể giả sử 𝑘 ≤ 𝑎1 < 𝑎2 < 𝑎3 < ... < 𝑎𝑘 ≤ 𝑛. Đặt
𝑏1 = 𝑎1 − 𝑘 + 1, 𝑏2 = 𝑎2 − 𝑘 + 2, ..., 𝑏𝑘 = 𝑎𝑘 . Khi đó

𝑎𝑖+1 ≥ 𝑎𝑖 + 2, 𝑖 = 1, 2, ..., 𝑘 − 1.

Suy ra 𝑎𝑖+1 − 𝑎𝑖 ≥ 2 do đó 𝑏𝑖+1 − 𝑏𝑖 ≥ 3 và 𝑏1 ≥ 1, 𝑏𝑘 ≤ 𝑛. Định nghĩa 𝑓 (𝑥) = 𝑦 = {𝑏1 , 𝑏2 , ..., 𝑏𝑘 },


suy ra 𝑦 ∈ 𝐵. Vậy 𝑓 là một ánh xạ, hơn nữa dễ thấy 𝑓 là một song ánh do đó ta có điều cần
chứng minh.
3. Sử dụng ánh xạ để chứng minh bất đẳng thức tổ hợp
Ví dụ 6. Cho 𝑚, 𝑛 là các số nguyên dương lớn hơn 1, S là một tập gồm 𝑛 phần tử. Giả
sử 𝐴1 , 𝐴2 , ..., 𝐴𝑚 là các tập con của S sao cho với mọi 𝑥, 𝑦 ∈ 𝑆 bao giờ cũng có một tập 𝐴𝑖 để
cho 𝑥 ∈ 𝐴𝑖 và 𝑦 ∈ / 𝐴𝑖 . Chứng minh rằng 𝑛 ≤ 2𝑚 .
/ 𝐴𝑖 hoặc 𝑥 ∈ 𝐴𝑖 và 𝑦 ∈
Lời giải
Gọi T là tập tất cả các xâu nhị phân có 𝑚 ký tự. Xét một tương ứng đi từ S đến T như sau:
với mỗi 𝑥 ∈ 𝑆 đặt 𝑓 (𝑥) = 𝑥1 𝑥2 ...𝑥𝑚 , trong đó 𝑥𝑖 = 1 nếu 𝑥 ∈ 𝐴𝑖 và 𝑥𝑖 = 0 nếu 𝑥 ∈
/ 𝐴𝑖 . Khi đó
hiển nhiên 𝑓 là một đơn ánh nên |𝑆| ≤ |𝑇 | hay 𝑛 ≤ 2𝑚 .

Nguyễn Ngọc Duy - Trường Phổ Thông Năng Khiếu ĐHQG TPHCM 3
Ví dụ 7. Cho X là tập gồm 𝑛 số bất kỳ. Giả sử 𝑎1 , 𝑎2 , ..., 𝑎𝑘 là các số thỏa mãn ∀𝑢, 𝑣 ∈
𝑋, 𝑢 ̸= 𝑣 tồn tại 𝑎𝑖 sao cho (𝑢 − 𝑎𝑖 )(𝑣 − 𝑎𝑖 ) ≤ 0. Chứng minh rằng 𝑛 ≤ 3𝑘 .
Lời giải
Đặt 𝑌 = {(𝑢1 , 𝑢2 , ..., 𝑢𝑘 ) : 𝑢𝑖 = 0, 1, 2}. Với mỗi 𝑢 ∈ 𝑋 đặt 𝑓 (𝑢) = {𝑢1 , 𝑢2 , ..., 𝑢𝑘 } ∈ 𝑌 , trong đó



⎪ 0, 𝑢 < 𝑎𝑖

𝑢𝑖 = 1, 𝑢 = 𝑎𝑖



2, 𝑢 > 𝑎𝑖

với mọi 𝑖 = 1, 2, ..., 𝑘. Ta sẽ chứng minh 𝑓 là đơn ánh. Thật vậy, giả sử tồn tại 𝑢, 𝑣 ∈ 𝑋 mà
𝑢 ̸= 𝑣 và 𝑓 (𝑢) = (𝑢1 , 𝑢2 , ..., 𝑢𝑘 ) = (𝑣1 , 𝑣2 , ..., 𝑣𝑘 ) = 𝑓 (𝑣) thì vì 𝑢 ̸= 𝑣 nên tồn tại 𝑎𝑖 sao cho
(𝑢 − 𝑎𝑖 )(𝑣 − 𝑎𝑖 ) ≤ 0. Để ý rằng 𝑢𝑖 = 𝑣𝑖 , ∀𝑖 = 1, 2..., 𝑘 nên hoặc 𝑢, 𝑣 < 𝑎𝑖 hoặc 𝑢, 𝑣 = 𝑎𝑖 hoặc
𝑢, 𝑣 > 𝑎𝑖 . Cả ba trường hợp này đều không xảy ra nên 𝑓 là đơn ánh.
Từ đó suy ra 𝑛 = |𝑋| ≤ |𝑌 | = 3𝑘 .
Ví dụ 8. Với mỗi số tự nhiên 𝑛 ký hiệu 𝐻𝑛 là tập tất cả các hoán vị (𝑎1 , 𝑎2 , ..., 𝑎𝑛 ) của
[𝑛] = {1, 2, ..., 𝑛}. Xét các tập hợp 𝑆𝑛 = {(𝑎1 , 𝑎2 , ..., 𝑎𝑛 ) ∈ 𝐻𝑛 : 𝑎𝑖 ≥ 𝑖 − 1, ∀𝑖 = 1, 2, ..., 𝑛}, 𝑇𝑛 =
{(𝑎1 , 𝑎2 , ..., 𝑎𝑛 ) ∈ 𝐻𝑛 : 𝑎𝑖 ≤ 𝑖 + 1, ∀𝑖 = 1, 2, ..., 𝑛}. Tìm tất cả các số nguyên dương 𝑛 sao cho
|𝑇𝑛 |
|𝑆𝑛 | > 31 .
Lời giải
Đặt 𝑃𝑛 = {(𝑎1 , 𝑎2 , .., 𝑎𝑛 )𝑖𝑛𝑆𝑛 : 𝑎1 = 1}, 𝑄𝑛 = {(𝑎1 , 𝑎2 , ..., 𝑎𝑛 ) ∈ 𝑆𝑛 : 𝑎𝑛 ̸= 1}. Hiển nhiên
𝑃𝑛 ∩ 𝑄𝑛 = ∅, 𝑃𝑛 ∪ 𝑄𝑛 = 𝑆𝑛 .
Xét ánh xạ 𝑓 đi từ 𝑃𝑛 đế 𝑄𝑛 như sau: với (𝑎1 , 𝑎2 , ..., 𝑎𝑛 ) ∈ 𝑃𝑛 ta cho tương ứng với (𝑎2 , 𝑎1 , ..., 𝑎𝑛 ) ∈
𝑄𝑛 . Dễ thấy 𝑓 là một song ánh nên |𝑃𝑛 | = |𝑄𝑛 | = 21 |𝑆𝑛 |.
Xét ánh xạ 𝑔 đi từ 𝑃𝑛 đến 𝑆𝑛−1 như sau: với (𝑎1 , 𝑎2 , ..., 𝑎𝑛 ) ∈ 𝑃𝑛 ta cho tương ứng với (𝑎2 −1, 𝑎3 −
1, ..., 𝑎𝑛 − 1) ∈ 𝑆𝑛−1 . Dễ thấy 𝑔 là song ánh nên |𝑃𝑛 | = |𝑆𝑛−1 |. Do đó ta có |𝑆𝑛 | = 2|𝑆𝑛−1 |, 𝑛 ≥ 2.
Hơn nữa |𝑆2 | = 2, |𝑆1 | = 1 nên |𝑆𝑛 | = 2𝑛−1 .
Xét ánh xạ ℎ đi từ 𝑇𝑛 đến 𝑇𝑛−1 ∪ 𝑇𝑛−2 xác định như sau: với mỗi (𝑎1 , 𝑎2 , .., 𝑎𝑛 ) ∈ 𝑇𝑛 ta cho
tương ứng với (𝑎2 − 1, 𝑎3 − 1, ..., 𝑎𝑛 − 1) nếu 𝑎1 = 1 và tương ứng với (𝑎3 − 2, 𝑎4 − 2, ..., 𝑎𝑛 − 2)
nếu 𝑎1 = 2. Dẽ thấy ℎ là một song ánh nên |𝑇𝑛 | = |𝑇𝑛−1 | + |𝑇𝑛−2 |, mà |𝑇2 | = 2, |𝑇1 | = 1 do đó
√ √
1 1 + 5 𝑛+1 1 − 5 𝑛+1
|𝑇𝑛 | = √ [( ) −( ) ].
5 2 2
|𝑇𝑛 | 1
Từ đó giải được |𝑆𝑛 | > 3 ⇔ 𝑛 ≤ 6.
4. Bài tập
Bài 1. Cho 𝑋 = {1, 2, .., 𝑛}. Một tập con 𝑆 = {𝑠1 , 𝑠2 , ..., 𝑠𝑘 } của X (𝑠1 < 𝑠2 < ... < 𝑠𝑘 )
được gọi là m- tách được (𝑚 ∈ N) nếu 𝑠𝑖 − 𝑠𝑖−1 ≥ 𝑚; 𝑖 = 1, 2, ..., 𝑘. Có bao nhiêu tập con m-
tách được gồm 𝑘 phần tử của X, trong đó 0 ≤ 𝑘 ≤ 𝑛 − (𝑚 − 1)(𝑘 − 1).
Bài 2. Cho 𝑋 = {1, 2, ..., 𝑛}, với mỗi tập con khác rỗng 𝐴𝑖 = {𝑎1 , 𝑎2 , ..., 𝑎𝑖 } (không
mất tổng quát giả sử 𝑎1 > 𝑎2 > ... > 𝑎𝑖 ) ta định nghĩa tổng hỗn tạp của 𝐴𝑖 là số 𝑚(𝐴𝑖 ) =
∑︀
𝑎1 − 𝑎2 + 𝑎3 − ... ± 𝑎𝑖 . Tính 𝑚(𝐴𝑖 ).
𝐴𝑖 ⊂𝑋

Nguyễn Ngọc Duy - Trường Phổ Thông Năng Khiếu ĐHQG TPHCM 4
Bài 3. Cho tập S gồm tất cả các số nguyên dương trong đoạn [1, 2, ..., 2002]. Gọi T là tập
hợp tất cả các tập con khác rỗng của S. Với mỗi X thuộc T ký hiệu m(X) là trung bình cộng
các phần tử thuộc X. Tính ∑︀
𝑋∈𝑇 𝑚(𝑋)
𝑚= .
|𝑇 |
Bài 4. Với mọi số tự nhiên 𝑛, gọi 𝑝𝑛 (𝑘) là số các hoán vị của tập 𝐴 = {1, 2, ..., 𝑛} mà có
đúng 𝑘 điểm cố định (0 ≤ 𝑘 ≤ 𝑛). Chứng minh rằng
𝑛
∑︁
𝑘.𝑝𝑛 (𝑘) = 𝑛!.
𝑘=0

Bài 5. Cho 𝑛 ≥ 3 là một số nguyên dương. Ký hiệu 𝑇𝑛 = {0, 1, 2, ..., 𝑛 − 1}.


Xét các tập
𝐴𝑛 = {(𝑎, 𝑏, 𝑐) : 𝑎, 𝑏, 𝑐 ∈ 𝑇𝑛 , 𝑎 < 𝑏 < 𝑐, 𝑎 + 𝑏 + 𝑐 ≡ 0(𝑚𝑜𝑑 𝑛)}

𝐵𝑛 = {(𝑎, 𝑏, 𝑐) : 𝑎, 𝑏, 𝑐 ∈ 𝑇𝑛 , 𝑎 ≤ 𝑏 ≤ 𝑐, 𝑎 + 𝑏 + 𝑐 ≡ 0(𝑚𝑜𝑑 𝑛)}

Chứng minh |𝐴𝑛+3 | = 3|𝐵𝑛 |.


Bài 6. Cho số nguyên dương 𝑛 và 𝑑 là một ước dương của 𝑛. Gọi S là tập tất cả những
bộ (𝑥1 , 𝑥2 , ..., 𝑥𝑛 ) nguyên dương thỏa 0 ≤ 𝑥1 ≤ 𝑥2 ≤ ... ≤ 𝑥𝑛 ≤ 𝑛 và 𝑑|𝑥1 + 𝑥2 + ... + 𝑥𝑛 . Chứng
minh rằng có đúng một nửa các phần tử của S có tính chất 𝑥𝑛 = 𝑛.
Bài 7. Mỗi hoán vị (𝑥1 , 𝑥2 , ..., 𝑥2𝑛 ) của tập 𝑆 = {1, 2, ..., 2𝑛} được gọi là có tính chất P
nếu |𝑥𝑖 − 𝑥𝑖+1 | = 𝑛 với ít nhất một giá trị 𝑖 ∈ {1, 2, ..., 2𝑛 − 1}. Chứng minh rằng với mỗi 𝑛
nguyên dương số hoán vị có tính chất P lớn hơn số hoán vị không có tính chất P.

Nguyễn Ngọc Duy - Trường Phổ Thông Năng Khiếu ĐHQG TPHCM 5
BÀI TOÁN CHIA KẸO EULER
Vũ Nguyên Duy

(THPT chuyên Huỳnh Mẫn Đạt, Kiên Giang)

A. BÀI TOÁN CHIA KẸO EULER

Bài toán gốc. Có k chiếc kẹo giống nhau chia cho n (n  k ) em bé sao cho em nào cũng có kẹo.
Hỏi có tất cả bao nhiêu cách chia kẹo?

Bài toán tương đương: Tìm số nghiệm nguyên dương của phương trình:

x1  x2  ...  xn  k
Đáp án: Có tất cả là Ckn11 cách.

Bài toán mở rộng. Có k chiếc kẹo giống nhau chia cho n em bé, không nhất thiết em nào cũng
có kẹo. Hỏi có tất cả bao nhiêu cách chia kẹo?

Bài toán tương đương: Tìm số nghiệm nguyên không âm của phương trình:

x1  x2  ...  xn  k (2)

Lời giải:

Ta có: (2)  ( x1  1)  ( x2  1)  ...  ( xn  1)  n  k

Đặt yi  xi  1, i  1,.., n

Số nghiệm của phương trình (2) bằng số nghiệm nguyên dương của phương trình:

y1  y2  ...  yn  k  n

Đáp án: Có tất cả là Cnnk11 cách.

Ta còn được biết đến bài toán trên với tên gọi là bài “stars and bars” hoặc “ balls and urns”.

Nguyên nhân của tên gọi này là vì để đếm số nghiệm ở trên, người ta thực hiện như sau:

Trước hết, ta đếm số cách chia thành các phần có số lượng phần tử dương:

Xếp k ngôi sao lên một đường thẳng


*****************
Giữa các ngôi sao có đúng k  1 khoảng trống, ta tìm cách đặt các thanh chắn vào giữa chúng để
chia các ngôi sao thành n phần, chẳng hạn
**** | **** | ******** | *

Chọn n  1 khoảng trống trong k  1 khoảng trống, có Ckn11.


Trở về bài toán đếm số nghiệm không âm, ta chỉ cần mượn thêm n ngôi sao nữa rồi cho trước
mỗi phần 1 ngôi sao, khi đó, ta lại đưa về đếm số cách chia thành các phần có số lượng phần tử dương
và tổng phần tử là n  k . Kết quả là Cnnk11 .

Kết quả của bài toán mở rộng chính là số tổ hợp lặp chập k của n phần tử .

Ta có định nghĩa tổ hợp lặp như sau:

Định nghĩa: Một bộ gồm k phần tử lấy ra từ n phần tử phân biệt thỏa mãn 2 điều kiện:

- Có tính thứ tự.

- Một phần tử có thể được chọn nhiều hơn 1 lần.

Chính là một tổ hợp lặp chập k của n phần tử.

Để xác định số tổ hợp lặp này, ta có thể nhận xét rằng: hai tổ hợp khác nhau khi có một số lượng
một phần tử nào đó trong n phần tử đã cho là khác nhau.

B. CÁC ÁP DỤNG CƠ BẢN:


1. Các bài toán về số nghiệm nguyên dương (không âm) của phương trình, bất phương trình:
Ví dụ 1. Tìm số nghiệm nguyên của phương trình a  b  c  d  16 với a  1, b  2, c  3, d  4 .
Lời giải:
Đặt x  a  1, y  b  2, z  c  3, t  d  4 .
Khi đó yêu cầu bài toán trở thành: Tìm số nghiệm nguyên không âm của phương trình
x y  z t  6
Áp dụng bài toán gốc (2) ta có kết quả là C93  84 nghiệm.
 x1  x2  x3  x4  17
Ví dụ 2. Tìm số nghiệm nguyên của hệ:  .
2  xi  5, i  1, 4
Lời giải: Đặt yi  xi  2, i  1,.., 4 , từ giả thiết suy ra 0  yi  3, i  1, 4 . Ta có hệ:
 y1  y2  y3  y4  9 (1)
(I )  .
0  yi  3 i  1, 4
Gọi X là tập tất cả các nghiệm nguyên không âm của (1), khi đó X  C123 .
Gọi A, B, C, D lần lượt là tập tất cả các nghiệm nguyên của bốn hệ:
 y1  y2  y3  y4  9  y1  y2  y3  y4  9  y  y  y3  y4  9
 ; ;...;  1 2 .
 y1  4  y2  4  y4  4
Ta tính được:
A  B  C  D  C83
A B  AC  A D  B C  B  D  C  D  4
A B C  AC  D  B C  D  A B  D  0
A B C  D  0
Theo nguyên lý bù trừ, ta có số nghiệm của hệ (I) bằng:
X  A  B  C  D  X  ...  C123   4C83  4.6   20 .
Vậy có 16 nghiệm thỏa yêu cầu bài toán.
Ví dụ 3. Tìm số các nghiệm nguyên không âm của bất phương trình: x1  x2  x3  x4  11 .
Lời giải:
11
Cách 1: xét 12 phương trình dạng x1  x2  x3  x4  i (i  0,..,11) . Kết quả:  C33i  1365 .
i 0

Cách 2: Số nghiệm cần tìm bằng số nghiệm nguyên không âm của phương trình:
x1  x2  x3  x4  x5  11 . Kết quả: C154  1365 (nghiệm).
 x1  x2  x3  x4  10
Ví dụ 4. Tìm số các nghiệm nguyên không âm của hệ:  .
 x1  3
Lời giải:
Số nghiệm cần tìm bằng số nghiệm nguyên không âm của hệ:
 x1  x2  x3  x4  x5  10

 x1  3

Lần lượt cho x1 nhận các giá trị 0; 1; 2; 3, sử dụng bài toán chia kẹo Euler, ta có số nghiệm cần tìm
bằng: C133  C123  C113  C103  791 (nghiệm).

2. Các bài toán chọn:

Ví dụ 5: Có bao nhiêu số nguyên không âm sao cho biểu diễn thập phân của nó có không quá 2017
chữ số, và các chữ số đó viết theo thứ tự không giảm? (chẳng hạn số 55677 là số chấp nhận được,
còn 54 thì không được).

Lời giải:

Gọi xi là số lần xuất hiện phần tử i với i 0,1,2,3, 4,5,6,7,8,9 .


Ta giải phương trình: x0  x1  x2  ...  x9  2017 (1) với xi không âm.
Số bộ nghiệm ( x0 ; x1 ;...; x9 ) không âm của phương trình trên chính là số các số cần tìm.
9
Vậy ta có C2026 số cần tìm.

Ví dụ 6:

6a. (Uzbekistan, 2012) Gọi S là tập hợp các số tự nhiên có 6 chữ số mà trong biểu diễn thập phân
của nó có chữ số 7. Biết rằng nếu x  S thì số y tạo thành bằng cách hoán đổi các chữ số của x sẽ
không thuộc S. Tính giá trị lớn nhất có thể có của S .

6b. Có bao nhiêu số nguyên dương nhỏ hơn 106 mà tổng các chữ số bằng 23.

Lời giải.

6a. Với tính chất của tập hợp S , không mất tính tổng quát, ta có thể giả sử rằng tất cả các số thuộc
tập hợp S đều bắt đầu bằng số 7. Khi đó, 5 số còn lại là một tổ hợp có tính lặp của tập hợp
0,1, 2,...,9 và để đạt được GTLN, ta sẽ chọn hết các tổ hợp lặp đó.
Gọi xi  i  0,1, 2,...,9  là số lần xuất hiện của số i . Số tổ hợp có tính lặp của tập hợp 0,1, 2,...,9 là
số bộ nghiệm  x0 , x1 ,..., x9  không âm của phương trình x0  x1  x2  ...  x9  5 .
Vậy kết quả cần tìm là C109 51  2002 số.

6b. Số cần tìm có dạng a1a2 a3 a4 a5 a6 với a1  a2  a3  a4  a5  a6  23 (*) và 0  ai  9, i  1, 6 .

Gọi Ai , i  1, 6 là tập hợp các nghiệm không âm của (*) nhưng ai  10.

6 1 5
Trước hết, ta thấy số nghiệm của (*) là C23  6 1  C28 .

6
Ta sẽ tính A
i 1
i với chú ý rằng Ai  A j  Ak  0 với 1  i  j  k  6 vì tổng các số là 23.

Tính A1 : đặt a1  a1  10  0 thì a1  a2  a3  a4  a5  a6  13 , phương trình này có số nghiệm là


C136161  C185 . Tương tự với Ai , i  2, 6.

Tính A1  A2 : đặt a1  a1  10  0, a2  a2  10  0 thì a1  a2  a3  a4  a5  a6  3 , phương trình


này có số nghiệm là C36611  C85 . Tương tự với Ai  A j mà 1  i  j  6 khác.

6 6
Do đó  Ai   Ai 
i 1 i 1

1i  j  6
Ai  Aj  6  C185  C52  C85 .

5
Vậy số lượng các số cần tìm là C28   6  C185  C52  C85   47432.

Ví dụ 7: Tìm số cách chọn ra r số phân biệt từ n số nguyên dương đầu tiên sao cho trong sự lựa
chọn đó không chứa 2 số nguyên liên tiếp.

Lời giải:

Sắp xếp n số nguyên dương đầu tiên thành một hàng theo thứ tự tăng bắt đầu từ 1. Nếu một
số được chọn thì đặt biểu tượng Y dưới số đó, nếu không chọn thì đặt biểu tượng N dưới số đó. Gọi
x1 là số lượng số có biểu tượng N đứng trước biểu tượng Y đầu tiên, x2 là số lượng số có biểu tượng
N giữa biểu tượng Y đầu tiên và biểu tượng Y thứ hai,…, xr là số lượng số có biểu tượng N giữa
biểu tượng Y thứ r  1 và biểu tượng Y thứ r , xr 1 là số lượng số có biểu tượng N đứng sau biểu
tượng Y thứ r .
Khi đó có một tương ứng một-một giữa những sự lựa chọn chấp nhận được với những nghiệm
nguyên của phương trình x1  x2  ...  xr 1  n  r  x  0, x
1 r 1 
 0, xi  1 i  2, r .Vậy có Cnr r 1
cách lựa chọn thỏa đề.
Ví dụ 8:
8a. Có 10 người xếp thành hàng dọc. Có bao nhiêu cách chọn ra 3 người sao cho không có hai người
kề nhau được chọn.
8b. Có 10 người xếp thành một vòng tròn, hỏi có bao nhiêu cách chọn ra 3 người sao cho không có 2
người kề nhau được chọn.
Lời giải:
8a. Thứ tự của 3 người được chọn sẽ hoàn toàn được xác định bởi các tham số sau: x1 là số người
đứng đầu đến sát người được chọn đầu tiên, x2 là số người giữa người 1 và người 2, x3 là số người
giữa người 2 và người 3, x4 là số người sau người 3 đến cuối.
Khi đó có một tương ứng một-một giữa những sự lựa chọn chấp nhận được với những nghiệm
nguyên không âm của phương trình x1  x2  x3  x4  7 , trong đó x2 , x3  1 .
Đặt y2  x2  1, y3  x3  1 . Ta được x1  y2  y3  x4  5 với các biến nguyên không âm. Áp
dụng bài toán chia kẹo Euler ta có đáp số là C83  56 .
8b. Cố định người A trên bàn tròn.
 Xét trường hợp A là người được chọn:
Gọi x1 là số người ngồi bên phải của A cho đến người thứ hai được chọn, x2 là số người ngồi
bên phải của người được chọn thứ 2 đến người được chọn thứ 3, x3 là số người ngồi bên phải của
người được chọn thứ 3 đến A. Trong trường hợp này số cách chọn là số bộ nghiệm nguyên dương
của phương trình x1  x2  x3  7 . Theo bài toán Euler ta có C62 cách chọn.
 Xét trường hợp A không được chọn, bài toán trên tương đương với bài toán 8a với 9 người xếp
thành hàng dọc. Như vậy có C73 cách chọn.

Vậy có C62  C73  50 cách chọn.

Ví dụ 9: Tìm số cách xếp 30 viên bi giống nhau vào 5 hộp sao cho hộp 1 có ít nhất 5 bi, hộp 2 và 3
có không quá 6 bi.
Lời giải: Gọi x1 , x2 , x3 , x4 , x5 lần lượt là số viên bi trong hộp 1,2,3,4 và 5.
Ta có x1  x2  x3  x4  x5  30 (*) và x1  5, 0  x2 , x3  6 và x4 , x5  0 .
Gọi y1  x1  5 thay vào phương trình (*) ta được y1  x2  x3  x4  x5  25 (**)
Như vậy số bộ nghiệm của phương trình (*) với x1  5 và x2 , x3 , x4 , x5  0 cũng chính là số bộ
4
nghiệm của phưởng trình (**) bằng C29 .
Gọi A là tập nghiệm của phương trình (*) với x1  5, x2  7 và x3 , x4 , x5  0 .
Gọi B là tập nghiệm của phương trình (*) với x1  5, x3  7 và x2 , x4 , x5  0 .
Nên A  B là tập nghiệm của phương trình (*) với x1  5, x2 , x3  7 và x4 , x5  0 .
4
Áp dụng bài toán Euler, ta tính được A  B  C22 ; A  B  C154 .
A  B  A  B  A  B  13265.
Vậy số cách xếp thỏa đề là C294  A  B  10486.

Ví dụ 10: Có một dãy gồm 30 quyển vở đặt trên bàn và Tuấn được cô giáo thưởng 4 quyển vở trong
số đó. Tuấn sinh vào ngày 5/5 nhưng lại rất ghét số 5 nên bạn ấy muốn chọn các quyển vở sao cho
không có 2 quyển liên tiếp nào có thứ tự chênh lệch nhau một lượng chia hết cho 5. Biết rằng Tuấn
đã chọn ra hai quyển vở ở 2 đầu của dãy. Hỏi Tuấn có tổng cộng bao nhiêu cách chọn ra thêm 2
quyển vở nữa theo ràng buộc trên ?

Lời giải: Gọi x1 , x2 , x3 lần lượt là số sách bên phải từ quyển vở đầu tiên được chọn đến quyển chọn
thứ hai, số sách bên phải từ quyển chọn thứ hai đến quyển chọn thứ ba và số sách bên phải từ quyển
chọn thứ ba đến quyển chọn bìa.

Ta có x1  x2  x3  26 (*) và xi  5k  4 với i  1,3 .

(*)  y1  y2  y3  29 trong đó yi  xi  1, i  1,3 và yi không chia hết cho 5.

Đặt yi  5qi  ri ; 1  ri  4
r1  r2  r3  4(mod 5)
Ta có  suy ra r1  r2  r3  4 hoặc r1  r2  r3  9 .
3  r1  r2  r3  12

 Xét trường hợp r1  r2  r3  4 có C32 cách chọn bộ  r1 , r2 , r3  .

Mặt khác: 5  q1  q2  q3   25  q1  q2  q3  5 nên có C72 cách chọn bộ  q1 , q2 , q3  .

Số nghiệm phương trình trong trường hợp này là C32 .C72  63 .

 Xét trường hợp r1  r2  r3  9 có 10 cách chọn bộ  r1 , r2 , r3  (hoán vị các trường hợp


1; 4; 4  ,  2;3; 4  ,  3;3;3 ).
Mặt khác: 5  q1  q2  q3   20  q1  q2  q3  4 nên có C62 cách chọn bộ  q1 , q2 , q3  .
Số nghiệm phương trình trong trường hợp này là 10.C62  150.
Vậy số cách chọn sách của Tuấn là 213.

i i
Ví dụ 11: Tung một đồng xu cân đối đồng chất một cách vô tư 10 lần. Gọi P  ( tối giản) là xác
j j
suất để không có hai lần liên tiếp xuất hiện mặt ngửa. Tính i  j .

Lời giải: Số phần tử của không gian mẫu   210.

Gọi k là số lần xuất hiện mặt ngửa 0  k  5, k   (Nếu k  6 thì có hai lần liên tiếp xuất hiện mặt
ngửa). Số khả năng xảy ra k lần mặt ngửa và trong đó không có hai lần liên tiếp xuất hiện mặt ngửa là
số bộ nghiệm của phương trình x1  x2  ...  xk 1  10  k với x1 , xk 1  0, xi  1, i  2, k .

Suy ra số bộ nghiệm là C11k  k .

5
Số khả năng xảy ra là n   C11k  k  144.
k 0

9
Xác suất P  . Vậy i  j  73.
64

C. CÁC ÁP DỤNG NÂNG CAO.

Ví dụ 12. (Ấn Độ, 2012) Cho tam giác ABC và điểm P gọi là tốt nếu tìm được 27 tia chung gốc P cắt
tam giác thành 27 tam giác con có diện tích bằng nhau. Đếm số điểm tốt.

Lời giải.

Trước hết ta thấy rằng PA, PB, PC phải là 3 trong 27 tia nêu trong đề bài (nếu không thì sẽ có một phần
không là tam giác trong các phần của tam giác ABC ).

Giả sử S ABC  27 thì diện tích mỗi tam giác nhỏ đều bằng 1.
Đặt S PBC  x, S PCA  y, S PAB  z thì rõ ràng, các tam giác PBC , PCA, PAB đều phải chứa trong đó một số
nguyên các tam giác có diện tích bằng 1 nên ta có x, y, z    và x  y  z  27 (*) .

z P y

x
B C
   
Chú ý rằng với mỗi P nằm trong tam giác thì PA  S PBC  PB  S PCA  PC  S PAB  0 và ngược lại với mỗi
bộ ( S PBC , S PCA , S PAB ) như thế thì tồn tại duy nhất P thỏa mãn.

31 2
Do đó, số điểm cần tìm chính là số nghiệm của phương trình (*) và là C27 1  C26  325 .

Ví dụ 12. (VMO 2012) Cho một nhóm gồm 5 cô gái, kí hiệu là G1 , G2 , G3 , G4 , G5 và 12 chàng trai. Có
17 chiếc ghế được xếp thành một hàng ngang. Người ta xếp nhóm người đã cho ngồi vào các chiếc ghế
đó sao cho các điều kiện sau được đồng thời thỏa mãn:

1/ Mỗi ghế có đúng một người ngồi;

2/ Thứ tự ngồi của các cô gái, xét từ trái qua phải, là G1 , G2 , G3 , G4 , G5 ;

3/ Giữa G1 , G2 có ít nhất 3 chàng trai;

4/ Giữa G4 , G5 có ít nhất 1 chàng trai và nhiều nhất 4 chàng trai.

Hỏi có tất cả bao nhiêu cách xếp như vậy?

(Hai cách xếp được coi là khác nhau nếu tồn tại một chiếc ghế mà người ngồi ở chiếc ghế đó trong hai
cách xếp là khác nhau)

Lời giải.

Tương tự trên, ta cũng gọi a1 , a2 , a3 , a4 , a5 , a6 là số lượng chàng trai ngồi trước cô gái G1 , giữa cô gái
G1 , G2 , giữa cô gái G2 , G3 , giữa cô gái G3 , G4 , giữa cô gái G4 , G5 và sau cô gái G5 .

Ta có hệ điều kiện sau với các số tự nhiên a1 , a2 , a3 , a4 , a5 , a6 :

a1  a2  a3  a4  a5  a6  12
 .
a2  3,1  a5  4

Ta thấy a5  1, 2,3, 4 và do a2  3 nên có thể đặt b  a2  3  0 , ta đưa về

a1  b  a3  a4  a6  9  a5 .
Từ đây ta tính được số nghiệm của phương trình là C95a15 51  C134  a5 .

Với a5  1, 2,3, 4 thì số nghiệm sẽ là C124  C114  C104  C94  1161 .

Do nữ cố định nhưng các bạn nam có thể thay đổi vị trí nên đáp số của bài toán là 12!  1161 .

Ví dụ 13. (VMO 2014) Cho đa giác đều có 103 cạnh. Tô màu đỏ 79 đỉnh của đa giác và tô màu xanh
các đỉnh còn lại. Gọi A là số cặp đỉnh đỏ kề nhau và B là số cặp đỉnh xanh kề nhau.

a. Tìm tất cả các giá trị có thể nhận được của cặp ( A, B ).

b. Xác định số cách tô màu các đỉnh của đa giác để B  14. Biết rằng hai cách tô màu được xem là như
nhau nếu chúng có thể nhận được nhau qua một phép quay quanh tâm của đường tròn ngoại tiếp đa giác.

Lời giải.

a. Gọi k là số dãy gồm các đỉnh liên tiếp được tô xanh và bị chặn hai đầu bởi đỉnh màu đỏ và h là số
dãy gồm các đỉnh liên tiếp được tô đỏ và bị chặn hai đầu bởi đỉnh màu xanh. Dễ thấy giữa 2 dãy đỏ là 1
dãy xanh và giữa 2 dãy xanh là 1 dãy đỏ nên h  k .

Gọi a1 , a2 ,..., ak là số lượng các đỉnh trong các dãy xanh; b1 , b2 ,..., bk là số lượng đỉnh trong các dãy đỏ
k k
thì  ai  24,  bi  79 .
i 1 i 1

Ngoài ra, rõ ràng nếu một dãy cùng màu có số lượng t thì có t  1 cặp đỉnh liên tiếp cùng màu.
k
Suy ra A   (ai  1)  79  k và tương tự B  24  k .
i 1

Do đó, các cặp ( A, B) sẽ có dạng (79  k , 24  k ) với 0  k  24. Có tổng cộng 25 cặp như thế.

b) Do B  14 nên theo đẳng thức đã xây dựng ở trên thì 14  24  k  k  10 . Như thế, có tổng cộng 10
dãy xanh và 10 dãy đỏ.
10 10
9 9
Số nghiệm nguyên dương của các phương trình sau  ai  24,  bi  79 lần lượt là C23 , C78 .
i 1 i 1

Do xếp trên một vòng tròn nên chỉ xét vị trí tương đối của nhóm với nhau chứ không xét vị trí cố định
9
C23  C789
của từng nhóm. Vậy số cách tô cần tìm là .
10

Ví dụ 14. (Đề kiểm tra Trường Đông 2014) Cho n  2 là một số nguyên dương. Xét tập hợp các
đường đi ngắn nhất trên lưới nguyên từ điểm A(0; 0) đến điểm B (n; n) . Một đường đi như thế sẽ tương
ứng với một dãy gồm n lệnh T (lên trên) và n lệnh P (sang phải). Trong dãy đó, một cặp lệnh (T , P) kề
nhau được gọi là một bước chuyển (lưu ý, cặp ( P, T ) không được gọi là bước chuyển). Ví dụ dãy
PTTPTPPT có 2 bước chuyển. Hãy tìm số các đường đi ngắn nhất từ A đến B sao cho trong đó có đúng:

a) 1 bước chuyển.

b) 2 bước chuyển.

Lời giải.
a) Ta phát biểu lại bài toán là: Cho xâu nhị phân có độ dài 2n với n số 0 và n số 1. Xác định số xâu nhị
phân thỏa mãn: cặp 10 chỉ xuất hiện đúng một lần.

Gọi A là các xâu nhị phân chứa toàn số 1, B là các xâu nhị phân chứa toàn số 0. Rõ ràng chỉ có các xâu
dạng sau thỏa mãn đề bài: AB, ABA, BAB, BABA .

Dạng 1 có 1 xâu.

Dạng 2 có Cn211  n  1 xâu.

Dạng 3 có n  1 xâu.

Dạng 4 có (n  1) 2 xâu.

Vậy tổng cộng có (n  1) 2  2(n  1)  1  n 2 .

b) Tương tự trên, ta có các dạng ABAB, ABABA, BABAB, BABABA .

Dạng 1 có (n  1) 2 xâu.

31 2 1 (n  1) 2 (n  2)
Dạng 2 có C n 1 C
n 1  xâu.
2

(n  1) 2 (n  2)
Dạng 3 có xâu.
2

(n  1)2 (n  2)2
Dạng 4 có Cn311  Cn311  xâu.
4

Vậy số đường đi tổng cộng thỏa mãn là

(n  1)2 (n  2) 2 (n  1) 2 (n  2) (n  1) 2 (n  1) 2 n2
4
 2
2
 (n  1)2 
4
 (n  2)2  4  (n  2)  4  
4
.

D. Các bài tập.


1. Tìm nghiệm tự nhiên của các phương trình, bất phương sau:
a. x1  x2  x3  x4  x5  x6  2017 trong đó các xi (i  1,.., 6) không chia hết cho 3.
b. 1017  x1  x2  x3  x4  x5  2017 .
 x1  x2  x3  x4  17
2. Tìm số nghiệm nguyên của hệ:  .
3  xi  5, i  1, 4
3. Có bao nhiêu bộ nghiệm số nguyên dương  a, b, c  thỏa mãn a.b.c  1200 ?
4. Có bao nhiêu số tự nhiên có 7 chữ số a1a2 a3a4 a5 a6 a7 , trong đó a1  a2  a3  a4  a5  a6  a7 ?
5. (Mở rộng AIME 1986) Cho xâu nhị phân: 001101001111011 có 4 cặp 01, 3 cặp 10, 5 cặp 11 và 2 cặp
00 cạnh nhau. Hỏi với các số tự nhiên a, b, c, d nào thì tồn tại một xâu nhị phân có độ dài dương và có
a cặp 01, b cặp 10, c cặp 11 và d cặp 00 đứng cạnh nhau? Trong trường hợp đó, có tất cả bao nhiêu
xâu nhị phân cùng tính chất như thế?
6. (AIME 2007) Có bao nhiêu cách tô màu 12 ô vuông của bảng ô vuông 6  4 sao cho trên mỗi hàng có
đúng 2 ô được tô và trên mỗi cột có đúng 4 ô được tô?

7. Chia 21 viên bi vào 9 hộp, sao cho mỗi 3 hộp liên tiếp có tổng số bi chia hết cho 3. Tìm số cách chia.

Tài liệu tham khảo

1. artofproblemsolving.com
2. Lê Phúc Lữ- Tổ hợp lặp và Bài toán chia kẹo Euler.
3. Tạp chí Toán học và Tuổi trẻ.
THPT Chuyên Nguyễn Chí Thanh - Đăk Nông GV: Nguyễn Văn Đệ
HÀM LỒI VÀ BẤT ĐẲNG THỨC JENSEN
1. HÀM LỒI VÀ MỘT SỐ TÍNH CHẤT
1.1. Định nghĩa hàm lồi.
Hàm số f được gọi là hàm lồi ( lồi dưới) trên khoảng  a; b  nếu với mọi

x, y   a; b  và với mọi cặp số dương  ,  có tổng     1 , ta đều có:

f  x1   x2    f  x1    f  x2  . (1.1)

Dấu “=” xảy ra khi và chỉ khi x1  x2 , thì ta nói hàm số f lồi thực sự (chặt) trên

 a; b  .
Hàm số f được gọi là hàm lõm (lồi trên) trên khoảng  a; b  nếu với mọi

x, y   a; b  và với mọi cặp số dương  ,  có tổng     1 , ta đều có:

f  x1   x2    f  x1    f  x2  . (1.2)

Dấu “=” xảy ra khi và chỉ khi x1  x2 , thì ta nói hàm số f lõm thực sự (chặt) trên

 a; b  .
1.2. Mô tả hình học.
Hàm số f lồi trên khoảng I khi và chỉ khi mọi điểm M 1 , M 2 thuộc đồ thị (C):

y  f  x  đều có cung M 1M 2 của đồ thị (C) nằm ở phía dưới của đoạn thẳng M 1M 2 .

1.3. Một số tính chất


Tính chất 1. Cho f lồi trên khoảng I, c là hằng số. Khi đó cf là hàm lồi trên I
nếu c  0 và lõm trên I nếu c  0 .
Tính chất 2. Tổng hữu hạn của các hàm lồi trên khoảng I là một hàm lồi trên I.
Tính chất 3.
1) Nếu f là hàm số liên tục và lồi trên khoảng I và g lồi và đồng biến trên tập
giá trị của f thì g(f(x)) là hàm số lồi trên I.
2) Nếu f là hàm số liên tục và lồi trên khoảng I và g lõm và nghịch biến trên tập
giá trị của f thì g(f(x)) là hàm số lõm trên I.
Tính chất 4. Nếu f là hàm liên tục và đơn điệu trên khoảng I và g là hàm ngược
của f thì
1) f lõm, đồng biến  g lồi, đồng biến.
2) f lõm, nghịch biến  g lõm, nghịch biến.

1
THPT Chuyên Nguyễn Chí Thanh - Đăk Nông GV: Nguyễn Văn Đệ
3) f lồi, nghịch biến  g lồi và nghịch biến.
Tính chất 5. Nếu f là hàm khả vi trên khoảng I thì f là hàm lồi trên I khi và chỉ
khi f’ là hàm đơn điệu tăng trên I.
Tính chất 6. Giả sử f có đạo hàm trên khoảng I. Nếu tiếp tuyến tại mỗi điểm
của đường cong (C): y  f  x  nằm phía dưới của (C) thì f lồi trên I.

2. BẤT ĐẲNG THỨC JENSEN


2.1. Các định lí.
Định lí 1. Nếu f lồi và liên tục thì tại mỗi điểm của đường cong y  f  x  có ít

nhất một đường đi qua, đường thẳng này nằm phía dưới đường cong hoặc trên nó.
Định lí 2. Nếu f khả vi bậc hai trên khoảng I thì f lồi (lõm) trên I khi và chỉ khi
f ''  x   0 ( f  x   0 ) trên I.

Định lí 3. (Jensen) Giả sử f liên tục trên [a;b]. Khi đó điều kiện cần và đủ để
hàm số f lồi trên khoảng (a;b) là:
 x  x  f  x1   f  x2 
f  1 2  , x1 , x2   a; b  .
 2  2
2.2. Bất đẳng thức Jensen và một số mở rộng.
Định lí 4.(Bất đẳng thức Jensen). Gi ả sử f là hàm lồi trên khoảng I và
x1 , x2 ,..., xn  I. Khi đó ta có:

 x  ...  xn  f  x1   ...  f  xn 
f 1  (2.1)
 n  n

Với mọi hàm lồi chặt đẳng thức chỉ xảy ra khi x1  x2  ...  xn . Và khi hàm f là
hàm lõm trên khoảng I thì ta có bất đẳng thức đổi chiều.
Định lí 5. (Bất đẳng thức Jensen tổng quát). Cho f là hàm liên tục và lồi trên
khoảng I. Nếu x1 ,..., xn  I và t1 ,..., tn   0;1 sao cho t1  ...  tn  1. Khi đó

f  t1 x1  ...  t n xn   t1 f  x1   ...  tn f  xn  . (2.2)

Với mọi hàm lồi chặt đẳng thức chỉ xảy ra khi x1  x2  ...  xn . Khi hàm f lõm
trên khoảng I thì ta có bất đẳng thức đổi chiều.
Định lí 6. (Bất đẳng thức tiếp tuyến) Cho hàm số y  f  x  liên tục và có đạo

hàm cáp hai trên [a;b].


i) Nếu hàm số lồi trên [a;b] thì f  x   f '  x0  x  x0   f  x0  , x0   a; b  .

2
THPT Chuyên Nguyễn Chí Thanh - Đăk Nông GV: Nguyễn Văn Đệ
ii) Nếu hàm số lỗm trên [a;b] thì f  x   f '  x0  x  x0   f  x0  , x0   a; b  .

Dấu bằng xảy ra khi và chỉ khi x  x0 .


Chú ý: Bất đẳng thức Jensen áp dụng dưới dạng (2.1) và (2.2) có điểm yếu là
hàm số có dạng tổng mà không có dạng tích. Ta đưa ra một kết quả sau để làm mạnh
Jensen.
2.3. Một bất đẳng thức khác.

Cho f : a,b , a 0  . Khi đó bất đẳng thức:


f(x1)+…+f(xn) ≥ nf( n x ...x
i n
) (2.3)

đúng với mọi n ≥ 2 khi và chỉ khi (I) đúng với n = 2


Chứng minh:
* Điều kiện cần: hiển nhiên.
* Điều kiện đủ:
- Giả sử (I) đúng với n = 2. Khi đó (I) đúng với n = 2k, k = 1, 2,…
- Giả sử bất đẳng thức (I) đúng với (n+1) số bất kì x1,… xn, xn+1 tức:
f(x1)+…+f(xn)+f(xn+1) ≥ nf ( n1 x1... x n x n1 )

- Lấy xn+1 = x = n
x1...x n suy ra:

f(x1)+…+f(xn) ≥ nf ( n x1...x n ).
3. MỘT SỐ ỨNG DỤNG CỦA BẤT ĐẲNG THỨC JENSEN
3.1. Chứng minh các bất đẳng thức cổ điển.
Ví dụ 1. Bất đẳng thức Cauchy.
a1  ...  an n
Cho n số thực không âm. Chứng minh rằng:  a1...an .
n
Chứng minh:
Nếu tồn tại ai  0, 1  i  n  . Khi đó bất đẳng thức hiển nhiên đúng.

Nếu ai  0, i  1, n . Khi đó tồn tại các số thực x1 ,..., xn sao cho a1  e x ,..., an  e x . Xét
1 n

hàm số f  x   e x trên R.

Ta có f '  x   e x , f "  x   e x  0, x  R . Vậy f là hàm số lồi trên R.

Theo bất đẳng thức Jensen (2.1) ta có

3
THPT Chuyên Nguyễn Chí Thanh - Đăk Nông GV: Nguyễn Văn Đệ
x1 ... xn
e x1  ...  e xn a  ...  an n
e n
 hay 1  a1...an .
n n
Dấu bằng xảy ra khi và chỉ khi a1  ...  an .
Kết hợp cả hai trường hợp trên ta có điều phải chứng minh.
Nhận xét: Kỹ thuật trên là biến đổi nó về dạng (2.1) và xét hàm số phù hợp.
Ví dụ 2. Bất đẳng thức Bunhiacopxki.
Cho 2n số thực a1 ,..., an và b1 ,..., bn dương. Khi đó ta có:
2
a 2
1  
 ...  an2 b12  ...  bn2   a1b1  ...  anbn  .

Chứng minh:
Xét trường hợp ai  0, bi  0, i  1, n .
2
 a1 2 an 2 
 b b1  ...  b bn 
Bất đẳng thức trở thành:  a12  ...  an2   1 2 n
2

 b1  ...  bn 
 
 

bi2 ai n
Đặt ti  n
2
, xi 
bi
, i  1, n . Khi đó ti  0,  i  1, n và t i  1.
b
j 1
j
i 1

Xét hàm số f  x   x 2 trên R. Khi đó f '  x   2 x, f ''  x   2  0 với mọi x . Do đó hàm

số f lồi trên R. Theo bất đẳng thức Jensen (2.2) ta có:


f  t1 x1  ...  t n xn   t1 f  x1   ...  tn f  xn 
2
Suy ra  t1 x1  ...  t n xn   t1 x12  ...  tn xn2

Hay
2
 
  2 2
 a1b1  ...  anbn   a1  ...  an   a1b1  ...  anbn 2  a12  ...  an2 b12  ...  bn2 .   
 n 2 n
2 
n
2
n

  b j  b j   b j  b 2j
 j 1 j 1  j 1 j 1

a1 a
Dấu bằng xảy ra khi và chỉ khi  ...  n .
b1 bn

Nếu tồn tại ai  0,1  i  n thì


VT  a1  ...  an
2 2
 b
1
2
 ...  bn
2
   a b  ...  a b 
1 1 n n
2
 VP .

Kết hợp cả hai trường hợp ta có đpcm.

4
THPT Chuyên Nguyễn Chí Thanh - Đăk Nông GV: Nguyễn Văn Đệ
Ví dụ 3. Bất đẳng thức Holder
1 1
Cho ai  0, bi  0, i  1, n, p  0, q  0 và   1 . Chứng minh rằng:
p q
1 1
n
 n p  p  n q q

i 1
a b
i i    ai    bi  .
 i 1   i 1 
Chứng minh:
Bất đăng thức có dạng:
p
 n  n
p
  ai bi a
1 p 1
n
 n
 n
  n p
  n n p 1  i
p  q
p p
 aibi    aip  .   biq     ai bi     ai  .   bi    i n1   i 1
n
.
 i 1   i 1   i 1   i 1   i 1   q  q
  bi b
i 1
 i
 i 1  i 1

p
 n ai q  n

  b q 1 bi
p
 a i
Suy ra  i 1 n i   i 1
n
.
 q 
  bi   biq
 i 1  i 1

biq
Đặt ti  n
; xi  ai bi1 q với mọi i  1, n .
q
b
j 1
j

Xét hàm số f  x   x p trên R. Khi đó f '  x   px p 1 , f "  x   p  p  1 x p 2  0 với mọi

x   0;   . Do đó hàm số lồi trên  0;   .

Theo Bất đẳng thức Jensen (2.2) ta có:


f  t1 x1  ...  tn xn   t1 f  x1   ...  tn f  xn  .
p
  p p
  b q a b1 q   
 ...  bnq anbn1 q 
a b  ...  anbn   1 11 a1p  ...  anp
Suy ra  1 1 n n
 n
.
 
  b qj  b q
j b q
j
 j 1  j 1 j 1

Dấu bằng xảy rakhi và chỉ khi x1  ...  xn hay a1b11 q  ...  anbn1 q .
Ví dụ 4. Bất đẳng thức Mincopxki.
Cho hai dãy số a1 ,..., an và b1 ,..., bn thỏa mãn ai  0, bi  0, i  1, n . Chứng minh rằng
n a1...an  n b1...b n  n
 a1  b1  ...  an  bn  .
Chứng minh:
Xét hàm số f  x   ln 1  e x  .
5
THPT Chuyên Nguyễn Chí Thanh - Đăk Nông GV: Nguyễn Văn Đệ
ex ex
Ta có f '  x   suy ra f "  x   2
 0 với mọi x  R .
1 ex 1  e x 
Suy ra f là hàm số lồi trên R.
bi
Áp dụng bất đẳng thức Jensen với xi  ln ta có:
ai

 x1 ... xn
 ln 1  x1   ...  ln 1  xn 
ln  1  e n  
  n

 b1...b n    a  b  ...  a  b  
1 1 n n
Suy ra ln 1  n   ln  n 
 a1...an   a ...a 
 1 n 

b1...b n  a1  b1  ...  an  bn 
Suy ra 1  n  n
a1...an a1...an

Suy ra n a1...an  n b1...bn  n


 a1  b1  ...  an  bn 
b1 b
Dấu bằng xảy ra khi và chỉ khi:  ...  n .
a1 an

3.2. Chứng minh các bất đẳng thức đại số.


- Dựa vào bài toán chọn hàm f(x) thích hợp.
- Chứng minh f(x) là hàm lồi (lõm).
- Sử dụng bất đẳng thức Jensen đưa ra lời giải.
Ví dụ 5: Cho các số thực dương x1 ,..., xn có tổng bằng 1. Tìm giá trị nhỏ nhất của
2 2
 1  1
S   x1    ...   xn   .
 x1   xn 

Giải.
2
 1
Xét hàm số f  x    x   trên  0;1 .
 x
2 6
Ta có f '  x   2 x  3
, f "  x   2  4  0, x   0;1 .
x x
Vậy hàm f lồi trên (0;1).
f  x1   ...  f  xn   x  ...  xn 
Theo bất đẳng thức Jensen ta có:  f 1 
n  n 
2 2
 1 
2
1  x  ...  xn
Suy ra: S   x1    ...   xn    n  1
2
  n   
n2  1
.
x1  xn  n    x  ...  x  
n
     1 n  

6
THPT Chuyên Nguyễn Chí Thanh - Đăk Nông GV: Nguyễn Văn Đệ
1
Dấu bằng xảy ra khi và chỉ khi x1  ...  xn  .
n
Tổng quát: Cho n số thực dương x1 ,..., xn có tổng bằng 1 và a>1. Chứng minh rằng:
a
n  1 (n 2 1) a
  x1   
a 1
.
i 1
 x1  n
Chứng minh:
a
 1
Xét hàm y  f  x    x1 
 x1 
Ta có:
a 1
 1  1 
y ' a1 2  x  
 x  x 
a 1 2 a 2
2 1  1  1
y '' a 3  x    a(a 1)1 2   x   0
x  x  x   x
 y  f  x  là hàm lồi.
1
Chọn  1... n 
n
Áp dụng bất đẳng thức Jensen ta có:

1n 1 a
f (
 1 x1 ) ... f (
an xn )   (xi  )
n i1 xi
 f ( 1x1... n x n)
1 n 
f x 
 n i 1 i 
a a
1 1   n 2 1 
 f ( )   n  = 
n n   n 
a
n  1  (n 2 1) a
   x i    a1
i 1
 xi  n
7
THPT Chuyên Nguyễn Chí Thanh - Đăk Nông GV: Nguyễn Văn Đệ
Ví dụ 6: Với mọi dãy số thực a1,a2,…,an lớn hơn hoặc bằng 1. Chứng minh rằng:
1 1 n
 ...   .
1  a1 1  an 1  a1...an
n

Giải:
Đặt ai  e x , i  1, n .
i

1
Xét hàm số f  x   , x  0.
1 ex

Ta có f '  x   
ex
, f " x  

ex ex 1   0, x  0.
x 2 x 3
1  e  1  e 
Vậy f là hàm lồi trên  0;   .

Áp dụng Bất đẳng thức Jensen ta có:


 x  ...  xn  f  x1   ...  f  xn 
f 1 
 n  n

1 1 1 1 
Hay ln a1 ... ln an
  ln a1
 ...  
n
n 1 e 1  eln an 
1 e

1 1 1 1 
    ...  
1  a1...an n  1  a1
n 1  an 

Dấu bằng xảy ra khi và chỉ khi a1  ...  an .


Nhận xét: Ta có thể sử dụng bất đẳng thức dạng (2.3) để chứng minh như sau:
- Bất đẳng thức trên đúng nếu ta chứng minh được với trường hợp 2 số:

1 1 2
2
 2
 , với mọi a, b ≥ 1
1 a 1b 1 ab
Thật vậy:

1 1 2
 
1 a 2 1b 2 1 ab
2
  a b   ab 1  0 (hiển nhiên đúng).

n
Ví dụ 7: Giả sử các số thực dương x1, x2 ,..., xn có :  x 1 .
i 1 i

Chứng minh rằng:

8
THPT Chuyên Nguyễn Chí Thanh - Đăk Nông GV: Nguyễn Văn Đệ

x1  x 2 ... x n  n
1 x1 1 x 2 1 x n n 1
Giải:

x
Xét hàm: f  x  , x(0,1)
1 x
Ta có:

4 x
f"( x)  7
 0.
2
8(1 x)
 f  x  là hàm lồi trên (0,1)
1
Áp dụng bất đẳng thức Jensen cho  1... n  ta có:
n
x1  x 2 ... x n  f ( x1... x n )
1 x1 1 x 2 1 x n n

1 n
f   =
 n  n 1
Ví dụ 8: Chứng minh rằng: Nếu a1... a n  n,a1,a 2,...a n  0 thì:
k k k k 1 k 1 k 1
a1  a 2 ... a n a1  a 2 ... a n ,k 1
Giải:
* Bổ đề:
  
Nếu a1... a n  n, 1 thì: a1  a 2 ... a n  a1 a 2 ... a n
Thật vậy:
Xét hàm f(x) = x trên (0,+),  > 1
 1
f '( x) x
 f '  x  đồng biến trên (0,+)

 f  x  là hàm lồi
9
THPT Chuyên Nguyễn Chí Thanh - Đăk Nông GV: Nguyễn Văn Đệ
1
Áp dụng bất đẳng thức Jensen cho  1 2 ... n  ta có:
n
1  1  1  1 1 1  1n
a1  a2  ... an  ( 
a1 a 2 ... a n)   a i
n n n n n n n i1
 a1  a 2 ... a n  a1 a 2 ... a n
* Trở lại bài toán:
Ta sử dụng bổ đề hai lần:
k 1 k 1 k 1
+) a1  a 2 ... a n  n
k k k
k 1 k 1 k 1 k 1 k 1 k 1
+) (a1 )  (a 2 ) ... (a n )  a1k 1  a 2 k 1 ... a n k 1
 a1k  a 2 k ... a n k  a1k 1  a 2 k 1 ... a n k 1 (đpcm)
Ví dụ 9. Cho các số thực dương a1 ,..., an . Chứng minh rằng:
a1 ... an
 a  ...  an 
a1a1 ....anan   1  .
 n 
Chứng minh:
1
Xét hàm số f  x   x ln x ta có f '  x   1  ln x, f "  x    0, x   0;   .
x
Suy ra f lồi trên  0;   .

 a1  ...  an  f  a1   ...  f  an 
Áp dụng bất đẳng thức Jen sen dạng (2.1) ta có: f  
 n  n
a1  ...  an a  ...  an a1 ln a1  ...  an ln an
Suy ra .ln 1 
n n n
a1 ... an a1 ... an
 a1  ...  an   a  ...  an 
Hay ln 
 n


 
 ln a1a1 ...anan tức là  1
 n


 a1a1 ...anan .

Dấu bằng xảy ra khi và chỉ khi a1  ...  an .


Ví dụ 10. Cho các số thực dương x1 ,..., xn . Chứng minh rằng:
x1 x2 xn n
 ...   .
2 2 2
x1 x2  x 2 x2 x3  x 3 xn x1  x 1
2

Chứng minh:

10
THPT Chuyên Nguyễn Chí Thanh - Đăk Nông GV: Nguyễn Văn Đệ
x2 x
Đặt a1  ,..., an  1 . Khi đó a1....an  1
x1 xn

Đặt a1  et ,..., an  et
1 n

1
Xét hàm số f  x   trên R ta được
e x  e2 x

f " x  

e 2 x 2e x  4e 2 x  1   0, x  R .
5


4 e e x 2x 2

Vậy hàm số lồi trên R. Theo bất đẳng thức Jensen ta có:
f  t1   ...  f  tn   t  ...  tn 
 f 1 .
n  n 
1 1 n
Hay  ...  
et1  e 2t1 e tn  e 2 tn t1 ... tn
n
t ... tn
21
n
e  ...  e
1 1 n n
Suy ra  ...   
2 2 ln  a1 ....a n  ln  a1 ....a n  2
a1  a 1 an  a n n
2
n
e e
x1 x2 xn n
Hay  ...   .
2 2 2
x1 x2  x 2 x2 x3  x 3 xn x1  x 1
2

Dấu bằng xảy ra khi và chỉ khi x1  ...  xn .


Ví dụ 11. Cho ba số thực dương a, b, c. Chứng minh rằng:
2 2 2
 2a  b  c    2b  c  a    2c  a  b  8.
2 2 2
2a 2   b  c  2b 2   c  a  2c 2   a  b 

Chứng minh:
Bất đẳng thức là thuần nhất nên ta có thể giả sử a  b  c  1 .
2 2 2

Bất đẳng thức trở thành


1  a    b  1   c  1  8.
2 2 2
2a  1  a  2b 2  1  b  2c 2  1  c 
2

Nếu xét hàm số f  x  


1  a  trên (0;1) thì ta thấy hàm số không luôn lồi hoặc
3a 2  2a  1
lõm, muốn áp dụng được Jensen phải chia trường hợp phức tạp.
4  x  1 2 x  1 1 1 8
Ta có: f '  x    2
, f '    4, f   
 3x 2
 2x 1  3 3 3

11
THPT Chuyên Nguyễn Chí Thanh - Đăk Nông GV: Nguyễn Văn Đệ
4
Phương trình tiếp tuyến của đồ thị hàm số y  f  x  là: y  4 x  .
3
2

Xét hàm số g  x   f  x    4 x    
 4  3x  1  4 x  1
 0, x   0;1
3   
3 3x 2  2 x  1 
1
Dấu bằng xảy ra khi và chỉ khi x  .
3
2 2 2
1  a    b  1   c  1   4a  4   8
Áp dụng ta được: 2 2
2a 2  1  a  2b 2  1  b  2c 2  1  c 
2  
sym  3 
(đpcm).

1
Dấu bằng xảy ra khi và chỉ khi a  b  c  .
3
Ví dụ 12. Cho 3 số thực dương x, y, z . Chứng minh rằng:

x2 y2 z2
x5  y 5  z 5  x5  y5  z5 .
yz zx xy

Chứng minh:
Bất đẳng thức thuần nhất bậc nên có thể giả sử xyz  1 .

Bất đẳng thức trở thành: x5  y 5  z 5  x 6 x  y 6 y  z 6 z .

Xét hàm số f  x   x k , k  1 trên R  . Dễ thấy f là hàm lồi. Do đó

xk  y k  z k  x  y  z .
13/2 13/2 13/2
Suy ra x5  y 5  z 5  x  y  z và  x 5  5
 
 y5 5
 
 z5 5  x 5  y 5  z 5 đpcm.

Dấu bằng xảy ra khi và chỉ khi x  y  z .


Ví dụ 13. Cho 3 số thưc dương thỏa mãn x  y  z  3. Chứng minh rằng:
1 1 1 3
2 2
 2 2
 2 2
 .
x  y 2 y z 2 z x 2 4
Giải.
x  1

Gỉa sử x  max  x, y, z   y  z  2 .
 yz  x 2

1
Xem z là tham số và xét hàm f  x   trên  0;   . Khi đó
x  z2  2
2

2 z 2  6 x2  4
f " x   3
 0.
 x2  z2  2 
12
THPT Chuyên Nguyễn Chí Thanh - Đăk Nông GV: Nguyễn Văn Đệ
Vậy hàm số lõm trên  0;   . Áp dụng BĐT Jensen ta có

1 1 1 8
 2 2 2
 2
x  y  2 x  z2  2
2 2
 yz
2
4x   y  z   8
x2    2
 2 
Suy ra
8 1 8 2 8 2
P 2
  2   2 
2
4x   y  z   8 y  z  2 4 x   y  z   8  y  z   4 4 x   3  x   8  3  x 2  4
2 2 2 2 2

8 2
Xét hàm số f  x   2
 2 trên 1;3 .
5 x  6 x  17 x  6 x  13
8 2 3 2
Ta có 2
 2
5 x  6 x  17 x  6 x  13 4

   x  1 15 x 2  78 x  111  0 luôn đúng. Đẳng 
thức xảy ra khi x  1 .
3
Vậy P  . Dấu bằng xảy ra khi x  y  z  1.
4
Ví dụ 14. Cho ba số thực không âm thỏa mãn x  y  z  2 . Tìm GTNN của
5
P  x3  1  y 3  1  z 1 .
2
Hướng dẫn:

Xét hàm số f  x   x3  1 trên  0;   .

x 4  3x
Ta có: f "  x    0 , x   0;   .
x3  1

Do đó f là hàm lồi trên  0;   . Theo Bất đẳng thức Jensen ta có:


3 3
 x y  2 x 
x3  1  y 3  1  2   1  2    1 dấu bằng xảy ra khi và chỉ khi x=y.
 2   2 
3
 2 z  5
Do đó P  2   1  z 1
 2  2
3
 2 z  5
Xét hàm số g  z   2   1  z  1 trên [0;2].
 2  2
2
3 2  z  5 1 5
g ' x        0, z   0; 2
3
2 2  2  z   16 4 z 1 2 4

13
THPT Chuyên Nguyễn Chí Thanh - Đăk Nông GV: Nguyễn Văn Đệ
5 4 2
Hàm số g(x) đồng biến trên [0;2] do đó g  z   g  0   .
2
Đẳng thức xảy ra khi x  y  1, z  0 .

5 4 2
Vậy GTNN của P là khi x  y  1, z  0 .
2
Một ví dụ cho thấy Jensen khá tinh tế.
Ví dụ 15. Cho ba số thực dương thoả mãn abc  1 . Chứng minh rằng
1 1 1
2
 2  2 1 .
a  a 1 b  b 1 c  c 1
Chứng minh:
Đặt a  e x , b  e y , c  e z , x, y, z  R . Khi đó x  y  z  0 .
1
Xét hàm số f  a   2
. Với a  e x khi đó
a  a 1

f " x  

e x 4e3 x  3e 2 x  3e x  1   0, x  0 . Do đó f(x) là hàm lồi trên  0;   .
3
e 2x
 ex 1 
Gỉa sử a=min{a,b,c} khi đó a  1, bc  1 tức là x  0, y  z  0 .
Áp dụng Bất đẳng thức Jensen ta có:
 yz 1 1 1
f  y   f  z   2. f   hay 2 y  2z  2. y z y z
 2  e  e 1 e  ez  1
y
2
2 2
e e 1
1 1 1 a
Suy ra   2.  2. .
b2  b  1 c2  c  1 bc  bc  1 a  a 1
1 a 2
Bất đẳng thức trở thành 2
a  a 1
2
a  a 1
1   
a 1 a a2  a  1  0 . 
Dấu bằng chỉ xảy ra khi a  b  c  1 .
3 2 2
Ta có 2
 2   a  1  0 luôn đúng.
a  a 1 a 1
1 1 1 3
Cần chứng minh 2
 2  2  .
a 1 b 1 c 1 2
Xét trường hợp tồn tại một số nhỏ hơn 1. Gỉa sử a  1 khi đó bc  1 .
Áp dụng BĐT (2.3) ta chỉ cần chứng minh cho trường hợp 2 số là đủ. Thật vậy
1 1 2 2
2
 2    b  c   bc  1  0 (luôn đúng với mọi bc  1 ).
b  1 c  1 bc  1

14
THPT Chuyên Nguyễn Chí Thanh - Đăk Nông GV: Nguyễn Văn Đệ
1 1 1 3 3
Do đó 2
 2  2  3
 .
a  1 b  1 c  1 1  abc 2
Nếu cả ba số đều lớn hơn 1 thì abc  1 vô lí.
Dấu bằng xảy ra khi và chỉ khi a=b=c=1.
yz zx xy x4
Cách 3. Đặt a 
x 2
, b  2 , c  2 . Bất đẳng thức trở thành:
y z
 x 4  x 2 yz  y 2 z 2  1 .
Áp dụng Bất đẳng thức Bunhiacopxki ta có
2
x4 x 2
 y2  z 2 
 x 4  x 2 yz  y 2 z 2  4
 x 2 yz  y 2 z 2
.
x
Ta cần chứng minh:
2
x 2
 y2  z2    x 4

 x 2 yz  y 2 z 2  x 2 y 2  y 2 z 2  z 2 x 2  xyz  x  y  z  luôn đúng.

Nhận xét: Bất đẳng thức trên có tên là Vasc có khá nhiều ứng dụng.
Ví dụ 16. (VMO 2014) Tìm giá trị lớn nhất của biểu thức
x3 y 4 z 3 y 3 z 4 x3 z3 x4 y3
T  
2 3 2 3 2 3
x 4
 y4  xy  z   y 4
 z4  yz  x   z 4
 x4  zx  y 
với x, y, z là các số thực dương.
Hướng dẫn
x y z
Đặt a  , b  , c  khi đó a,b,c là các số thực dương thỏa mãn abc=1.
y z x
1 1 1
Bất đẳng thức trở thành: T  3
 3
 3
a 4

 1 b  c  b 4

1 c  a c 4

1  a  b

2 3 8
Mà b  c  2 bc   b  c   suy ra
a a a

1 1 a a
3
 . 4 .
a 4

1 b  c  8 a 1

1 a a b b c c 
Bất đẳng thức trở thành T   4  4  4 
8  a 1 b 1 c 1 

a a 3 a 1
Lại có  . thật vậy
a4  1 4 a2  a  1
x3 3 x2  1 2
8
 . 4
x 1 4 x  x 1 2 
   a  1 3a 8  6a 7  12a 6  14a 5  16a 4  14a 3  12a 2  6a  3  0 
luôn đúng.
15
THPT Chuyên Nguyễn Chí Thanh - Đăk Nông GV: Nguyễn Văn Đệ
Mặt khác, từ bất đẳng thức Vasc ta suy ra bất đẳng thức:
a 1 b 1 c 1
2
 2  2  2.
a  a 1 b  b 1 c  c 1
3
Vậy T  . Dấu bằng xảy ra khi và chỉ khi a  b  c  1 .
16
Bài tập tự luyện
Bài 1. Cho 0  a  1, 0  b  1 và a  b  1 chứng minh rằng: a a  bb  2 .
Bài 2. Cho các số thực dương thỏa mãn a  b  c  1 . Chứng minh rằng:
a b c 6
   .
1 a 1 b 1 c 2
Bài 3. (ĐH-B2005) Chứng minh rằng với mọi số thực x ta có:
x x x
 12   15   20  x x x
      3 4 5 .
 5  4  3 
Bài 4. Chứng minh rằng với mọi số nguyên dương k và với mọi số không âm x1 ,..., xn
2k
ta đều có:  x1  ...  xn   n 2 k 1  x12 k  ...  xn2 k  .

Bài 5. Cho 3 số thực dương a, b, c thỏa mãn a  b  c  1 . Tìm giá trị nhỏ nhất của
10 10 10
 1  1  1
a    b    c  
 a  b  c

Bài 6. Cho ba số thực dương a, b, c . Chứng minh rằng:


2 2 2
b  c  a   c  a  b   a  b  c  3
 .
2 2 2
 b  c   a 2  c  a   b2  a  b   c 2 5

Bài 7. Cho x,y,z là các số thực dương thỏa x  y  z  1 . Tìm GTNN của

x y 8z 2 x2  2 y 2
P 2
 2
 .
 y  z x  z  z  1 x  y 
Ví dụ 8. Cho a, b, c là các số thực dương thỏa mãn abc  1 . Chứng minh rằng
1 1 1
2
 2  2  3.
a  a 1 b  b 1 c  c 1

16
MỘT SỐ PHƯƠNG PHÁP GIẢI TOÁN
TỔ HỢP

Cao Trần Tứ Hải∗

Tóm tắt nội dung

Trong bài viết này, để có tính hệ thống, trước hết chúng tôi sẽ trình bày một
cách vắn tắt phần lý thuyết cơ bản của tổ hợp, sau đó chúng tôi sẽ tập trung vào
giới thiệu các phương pháp giải bài toán tổ hợp nâng cao như phương pháp sử dụng
công thức bao hàm loại trừ, phương pháp song ánh, phương pháp sử dụng công
thức truy hồi, phương pháp hàm sinh, sử dụng công cụ số phức và cuối cùng là một
số bài toán ứng dụng tổ hợp.

GIỚI THIỆU
Lý thuyết tổ hợp là một ngành toán học rời rạc, nghiên cứu về các cấu hình kết hợp
các phần tử của một tập hữu hạn phần tử. Các cấu hình đó là các hoán vị, chỉnh hợp, tổ
hợp, . . . Các vấn đề liên quan đến lý thuyết tổ hợp là một bộ phận quan trong, hấn dẫn
và lý thú trong toán học nói chung và trong toán rời rạc nói riêng. Bởi vì nó có nội dung
phong phú và được ứng dụng nhiều trong đời sống, đặc biệt từ khi tin học ra đời.
Lý thuyết tổ hợp đóng một vai trò khá quan trọng trong các môn khoa học và đặc
biệt là trong Tin học và Toán ứng dụng. Có thể nói, lý thuyết bài toán tổ hợp với cơ sở
là các bài toán đếm, bài toán tồn tại, bài toán liệt kê, bài toán tối ưu có nhiều ứng dụng
và thường xuất hiện ở hầu khắp các lĩnh vực khoa học như: Xác suất Thống kê, Sinh học
Di truyền, Hóa học Cấu trúc ,. . .

THPT Chuyên Lê Quý Đôn, Ninh Thuận. Email: tuhai.thptlequydon@ninhthuan.edu.vn

1
Nhưng giải một bài toán tổ hợp không hề đơn giản. Khi mới làm quen với bài toán tổ
hợp chúng ta vẫn hay nhầm lẫn như khi nào dùng tổ hợp, chỉnh hợp, chỉnh hợp lặp, tổ
hợp lăp, . . . Khi đã vượt qua những khó khăn ban đầu này, ta lại gặp những bài toán mà
việc áp dụng trực tiếp các quy tắc và các đối tượng tổ hợp cơ bản không đem lại kết quả
mong muốn ngay lập tức. Với những bài toán như vậy, ta cần đến các phương pháp nâng
cao hơn.
Trong nhiều kỳ thi học sinh giỏi quốc gia, thi Olympic toán quốc tế, các bài toán tổ
hợp và các bài toán rời rạc cũng rất hay đề cập và thường thuộc loại rất khó và khi giải
các bài toán tổ hợp người quan tâm sẽ cảm thấy rất hấp dẫn và bổ ích. Tuy nhiên, các tài
liệu về toán tổ hợp như là một chuyên đề chọn lọc phục vụ cho học sinh giỏi toán chưa
được nhiều, chưa được hệ thống theo dạng toán cũng như phương pháp giải.
Các bài toán về một số vấn đề về lý thuyết tổ hợp, đại số tổ hợp là một phần quan
trọng của đại số lớp 10 và lớp 11 trong chương trình chuyên toán THPT hiện hành. Học
sinh thường phải đối mặt với nhiều dạng toán khó liên quan đến vấn đề này và gặp khó
khăn trong vấn đề về định hướng phương pháp thường được sử dụng để giải một bài toán
trong các kì thi học sinh giỏi.
Để đáp ứng cho nhu cầu học toán về chuyên đề tổ hợp, chúng tôi bài báo cáo này
nhằm cung cấp một số kiến thức cơ bản về tổ hợp, các phương pháp thường được sử dụng
trong các kì thi học sinh giỏi, cũng như các ứng dụng của tổ hợp vào một số vấn đề khác
của toán học. Đây cũng bài giảng mà chúng tôi đã dạy cho học sinh đội tuyển thi học
sinh giỏi môn toán của nhà trường, là tài liệu học sinh và đồng nghiệp tham khảo.
Trong thực tế giảng dạy cho đội tuyển thi học sinh giỏi toán ở địa phương, đối với mỗi
phương pháp, chúng tôi giới thiệu về cơ sở lý luận của phương pháp và lựa chọn một số
bài toán phù hợp làm minh họa, một số bài khác hướng dẫn các em tự nghiên cứu cũng
như thảo luận nhóm. Với mong muốn, sau khi lĩnh hội được phương pháp giải, học sinh
có khả năng độc lập sáng tạo ra hướng giải quyết vấn đề từ đó mang lại nhiều hứng thú
trong việc tìm tòi, nghiên cứu các bài toán tổ hợp. Hơn nữa việc lựa chọn phương pháp,
suy nghĩ hướng đi cho mỗi bài toán, sáng tạo ra phương pháp mới là một việc làm cần
thiết, giúp tiết kiệm thời gian trên lớp cũng như phát triển tư duy toán học cho các em.
Tuy đã hết sức cố gắng và suy nghĩ cẩn thận tập hợp kinh nghiệm cùng nhiều phương

2
pháp trong nhiều năm giảng dạy để viết báo cáo này, nhưng chắc chắn sẽ không tránh
khỏi những sai sót do khả năng và trình độ hạn chế. Chúng tôi rất mong nhận được các
ý kiến góp ý của các chuyên gia giáo dục, các thầy cô giáo, các bạn đồng nghiệp và các
em học sinh để bài sáng kiến được hoàn thiện hơn.
Xin chân thành cảm ơn.

1 Các quy tắc cơ bản trong tổ hợp


Quy tắc cộng:
Nếu A, B là các tập hợp không giao nhau thì

|A ∪ B| = |A| + |B| .

Quy tắc cộng còn có thể phát biểu một cách khác như sau:
Nếu một công việc có thể thực hiện bằng một trong hai phương án loại trừ lẫn nhau:
phương án thứ nhất có m cách thực hiện và phương án thứ hai có n cách thực hiện. Khi
đó công việc đó có m + n cách thực hiện.
Quy tắc cộng mở rộng:
Nếu tập hợp hữu hạn C là hợp của n tập đôi một rời nhau C1 , C2 , ..., Cn thì |C| =
|C1 | + |C2 | + ... + |Cn |.
Quy tắc nhân:
Nếu A và B là hai tập hợp hữu hạn thì A × B cũng hữu hạn và ta có |A × B| = |A| . |B|,
trong đó A × B là tập hợp tất cả các cặp thứ tự (a, b) với a ∈ A, b ∈ B (A × B được gọi
là tích Descartes của hai tập hợp A, B ).
Định nghĩa về tích Descartes và quy tắc nhân trên đây có thể mở rộng cho nhiều tập hợp.
Quy tắc nhân có thể phát biểu một cách khác như sau: Nếu một quá trình có thể được
thực hiện qua hai công đọan: công đọan I có n1 cách thực hiện, công đọan II (sau khi
thực hiện I) có n2 cách thực hiện. Khi đó có n1 .n2 cách thực hiện quá trình đó.
Công thức bao hàm và loại trừ:
Với n tập hợp A1 , A2 , , . . . , An bất kỳ ta có công thức
n
X X
|A1 ∪ A2 ∪ · · · ∪ An | = |Ai | − |Ai ∩ Aj | + · · · + (−1)n−1 |A1 ∩ A2 ∩ · · · ∩ An | .
i=1 16i<j6n

3
Chỉnh hợp chập k của n phần tử:
Giả sử E = {a1 , a2 , . . . , an }. Chỉnh hợp chập k của n phần tử là một bộ sắp thứ tự gồm
k phần tử (ai1 , ..., aik ). Số các chỉnh hợp chập k của n phần tử được ký hiệu là Akn . Ta có
n!
Akn = n (n − 1) ... (n − k + 1) =
(n − k)!

Tổ hợp chập k của n phần tử:


Giả sử E = {a1 , a2 , . . . , an }. Tổ hợp chập k của n phần tử là một bộ không sắp thứ tự
gồm k phần tử {ai1 , ..., aik }. Nói cách khác, đó là một tập con gồm k phần tử. Số các tổ
hợp chập k của n phần tử được ký hiệu là Cnk . Ta có
n (n − 1) ... (n − k + 1) n!
Cnk = = .
k! k! (n − k)!
Hoán vị:
Giả sử E = {a1 , a2 , . . . , an }. Một hoán vị của E là một cách xếp các phần tử của E theo
một thứ tự nào đó. Nói cách khác, đó chính là chỉnh hợp chập n của n phần tử. Số các
hoán vị của n phần tử ký hiệu là Pn . Ta có Pn = n!
Chỉnh hợp lặp:
Giả sử E = {a1 , a2 , . . . , an }. Chỉnh hợp lặp chập k của n phần tử là một bộ sắp thứ tự
gồm k phần tử (ai1 , ..., aik ), trong đó cho phép lấy lặp lại. Số các chỉnh hợp lặp chập k
của n, theo quy tắc nhân, bằng nk .
Tổ hợp lặp:
Giả sử E = {a1 , a2 , . . . , an }. Tổ hợp lặp chập k của n phần tử là một bộ không sắp thứ tự
gồm k phần tử {ai1 , . . . , aik }, trong đó cho phép lấy lặp lại. Nói cách khác, đó là một đa
tập hợp gồm k phần tử lấy từ E. Số các tổ hợp lặp chập k của n phần tử được ký hiệu
là Hnk . Ta có Hnk = Cn+k−1
k
.
Hoán vị lặp:
Xét đa tập hợp E (r1 , r2 , . . . , rs ) có n phần tử, trong đó phần tử a1 có r1 phiên bản, phần
tử a2 có r2 phiên bản, . . . , phần tử as có rs phiên bản, r1 + r2 + · · · + rs = n.
Một cách xếp các phần tử của E theo thứ tự nào đó được gọi là một hoán vị lặp của n
phần tử của E. Số hoán vị lặp của đa tập hợp E (r1 , r2 , . . . , rs ) bằng
n!
.
r1 ! . . . rs !

4
Nhị thức Newton:

(x + y)n = Cn0 xn + Cn1 xn−1 y + · · · + Cnn y n .

Khi giải một bài toán tổ hợp nói chung, các quy tắc tổ hợp nói trên thường nảy sinh
một cách tự nhiên như quy tắc cộng, quy tắc nhân, tổ hợp, chỉnh hợp, hoán vị, . . . Tuy
nhiên, với các công cụ cơ sở trên, chúng ta thường chỉ giải được những bài toán ở dạng
đơn giản nhất. Với các bài toán có yêu cầu phức tạp hơn trong các kỳ thi học sinh giỏi,
cần đến các phương pháp nâng cao hơn.
Có nhiều phương pháp giải bài toán tổ hợp dựa trên các nền tảng lý thuyết khác nhau.
Ví dụ phương pháp dùng công thức bao hàm loại trừ dựa vào lý thuyết tập hợp mà cụ
thể là tổng quát hóa của công thức |A ∪ B| = |A| + |B| − |A ∩ B| , phương pháp song
ánh dựa vào lý thuyết tập hợp và ánh xạ, phương pháp quỹ đạo dựa vào một định lý cơ
bản về số đường đi ngắn nhất giữa hai điểm của lưới nguyên, phương pháp thiết lập hệ
thức truy hồi dựa vào ý tưởng quy nạp, phương pháp hàm sinh sử dụng các kiến thức
tổng hợp của đại số và giải tích, . . .
Dưới đây chúng ta sẽ giới thiệu một số phương pháp giải bài toán tổ hợp như vậy.

2 Phương pháp dùng công thức bao hàm loại trừ


Bài toán 1 Có bao nhiêu cách xếp 8 con xe lên bàn cờ quốc tế đã bị gạch đi một đường
chéo chính sao cho không có con nào ăn con nào?

Lời giải
Có 8! cách xếp 8 con xe lên bàn cờ quốc tế sao cho không có con nào ăn con nào. Ta cần
đếm số cách xếp không hợp lệ, tức là số cách xếp có ít nhất một con xe nằm trên đường
chéo.
Gọi Ai là tập hợp các cách xếp có quân xe nằm ở ô (i, i). Ta cần tìm |A1 ∪ · · · ∪ A8 |.
Nhưng dễ dàng thấy rằng

|Ai | = 7!, |Ai ∩ Aj | = 6!, . . . , |A1 ∩ · · · ∩ A8 | = 1

nên từ công thức bao hàm loại trừ ta suy ra


8! 8! 8!
|A1 ∪ ... ∪ A8 | = C81 .7! − C82 .6! + C83 .6! − · · · − C88 .1! = 8! − + − ··· − .
2! 3! 8!
5
Như vậy số cách xếp 8 con xe lên bàn cờ quốc tế đã bị gạch đi một đường chéo chính sao
cho không có con nào ăn con nào là
   
8! 8! 8! 1 1 1
N (8) = 8! − 8! − + − · · · − = 8! − + ... + = 14833.
2! 3! 8! 2! 3! 8!

Bài toán 2 Trong tập hợp S = {1, 2, . . . , 280} có bao nhiêu số không chia hết cho 2,3,5,7?

Lời giải Trước hết ta đếm trong tâp S có bao nhiêu số chia hết cho ít nhất một trong
. . .
các số 2,3,5,7. Kí hiệu A1 = { k ∈ S|k ..2} , A2 = { k ∈ S|k ..3} , A3 = { k ∈ S|k ..5} ,
.
A4 = { k ∈ S|k ..7} .
Khi đó A1 ∪ A2 ∪ A3 ∪ A4 là tập tất cả các số chia hết cho ít nhất một trong các số
2,3,5,7. Ta có
       
280 280 280 280
|A1 | = = 140, |A2 | = = 93, |A3 | = = 56, |A4 | = = 40;
2 3 5 7
     
280 280 280
|A1 ∩ A2 | = = 46, |A1 ∩ A3 | = = 28, |A1 ∩ A4 | = = 20,
6 10 14
     
280 280 280
|A2 ∩ A3 | = = 18, |A2 ∩ A4 | = = 13, |A3 ∩ A4 | = = 8;
15 21 35
   
280 280
|A1 ∩ A2 ∩ A3 | = = 9, |A1 ∩ A2 ∩ A4 | = = 6,
30 42
   
280 280
|A1 ∩ A3 ∩ A4 | = = 4, |A2 ∩ A3 ∩ A4 | = = 2;
70 105
 
280
|A1 ∩ A2 ∩ A3 ∩ A4 | = = 1.
210
Do đó
| A1 ∪ A2 ∪ A3 ∪ A4 | = 216.

Vậy trong tập S có 280 – 216 = 64 số thỏa yêu cầu bài toán.

Bài toán 3 Với mỗi số nguyên dương n > 1, kí hiệu ϕ(n) là các số nguyên dương bé hơn
n và nguyên tố cùng nhau với n. Chứng minh rằng
    
1 1 1
ϕ(n) = n 1 − 1− ... 1 −
p1 p2 pm

trong đó p1 , p2 , . . . , pm là tất cả các ước nguyên tố phân biệt của n.

6
Lời giải
Kí hiệu S = {1, 2, . . . , n}. Ta sẽ đếm trong tập S có bao nhiêu số chia hết cho ít nhất
.
một trong các số p1 , p2 , . . . , pm . Gọi Ai = { k ∈ S|k ..pi } với i = 1, 2, ..., m. Khi đó
A1 ∪ A2 ∪ · · · ∪ Am là tập tất cả các số chia hết cho ít nhất một trong các số p1 , p2 , ..., pm .
Ta có
n
| Ai1 ∩ Ai2 ∩ ... ∩ Aik | = .
pi1 .pi2 ...pik
Theo công thức bao hàm loại trừ ta có
m
X n X n n
|A1 ∪ A2 ∪ · · · ∪ Am | = − − · · · + (−1)m−1 .
p
i=1 i
pp
16i<j6m i j
p1 .p2 . . . pm

Do ϕ(n) là số các số không chia hết cho tất cả số p1 , p2 , ..., pm nên


m
!
X 1 X 1 (−1)m
ϕ(n) = n − |A1 ∪ A2 ∪ ... ∪ Am | = n 1 − + + ... +
p
i=1 i
pp
16i<j6m i j
p1 .p2 ...pm
    
1 1 1
=n 1− 1− ... 1 − .
p1 p2 pm

Chú ý: ϕ(n) được gọi là ϕ-hàm Euler, đặt theo tên nhà toán học Thụy Sỹ Leonhard
Euler. Hàm số này có nhiều ứng dựng trong đại số, số học, giải tích, tổ hợp, . . . vì nó là
kích thước của nhóm nhân các số nguyên modulo n tức là ϕ(n) là số phần tử của một hệ
thặng dư thu gọn theo mod n. Quan trọng hơn ϕ(n) là cấp của nhóm các đơn vị trong
vành có đơn vị Z/nZ. Ngoài ra, ϕ-hàm Euler là một hàm nhân tính, nghĩa là nếu m, n
nguyên tố cùng nhau thì ϕ(mn) = ϕ(m)ϕ(n).

3 Phương pháp song ánh


Phương pháp song ánh dựa vào một ý tưởng rất đơn giản như sau: Nếu tồn tại một song
ánh từ A vào B thì |A| = |B| . Do đó, muốn chứng minh hai tập hợp có cùng số phần tử,
chỉ cần xây dựng một song ánh giữa chúng. Hơn nữa, ta có thể đếm được số phần tử của
một tập hợp A bằng cách xây dựng song ánh từ A vào một tập hợp B mà ta đã biết cách
đếm. Đây là một phương pháp không mới nhưng rất thú vị. Nó có thể đưa ra những lời
giải khá bất ngờ cho những bài toán tổ hợp khó.

7
Bài toán 4 (Đề thi HSG cấp Trường 2013) Có 2013 người xếp thành hàng dọc. Hỏi
có bao nhiêu cách chọn ra 1006 người sao cho không có hai người liên tiếp trên hàng dọc
đó được chọn?

Lời giải
Ta đánh số 2013 người bằng các số thứ tự 1, 2, . . . , 2013. Một cách chọn thích hợp chính
là một bộ số 1 6 a1 < a2 < . . . < a1006 6 2013 thỏa mãn điều kiện ai+1 − ai > 2. Vậy ta
cần tìm số phần tử của

A = (a1 , a2 , . . . , a1006 ) 1≤ a1 < a2 < . . . < a1006 ≤ 2013, ai+1 − ai ≥ 2, i = 1, 1005 .

Xét ánh xạ

f (a1 , a2 , . . . , a1006 ) = (b1 , b2 , . . . , b1006 ) với bi = ai − i + 1

thì rõ ràng ta có

1. b1 = a1 > 1

2. bi+1 − bi = (ai+1 − (i + 1) + 1) − (ai − i + 1) = ai+1 − ai − 1 > 0

3. b1006 = a1006 − 1005 6 1008.

Suy ra (b1 , b2 , . . . , b1006 ) là phần tử của tập hợp

B = {(b1 , b2 , . . . , b1006 ) |1≤ b1 < b2 < . . . < b1006 ≤ 1008} .

Dễ thấy f là một đơn ánh.


Ngoài ra, ánh xạ g (b1 , b2 , . . . , b1006 ) = (a1 , a2 , . . . , a1006 ) với ai = bi + i − 1 là đơn ánh
từ B vào A. Do đó f là song ánh. Vậy

1006
|A| = |B| = C1008 = 507 528.

Bài toán 5 Cho tập A = { 1, 2, 3, . . . , 2n} . Một tập con B của A gọi là tập cân nếu
trong tập đó số các số chẵn bằng số các số lẻ. Tập rỗng cũng là một tập cân . Hỏi A chứa
bao nhiêu tập cân. ( Tài liệu Chuyên toán- Đại số và Giải tích 11 )

8
Lời giải
Ta thấy đây là một bài toán chứng minh lực lượng của hai tập hợp bằng nhau. Một cách
tự nhiên ta nghĩ tới việc thiết lập một song ánh giữa họ các tập cân của A với các tập
con có n phần tử của A. Điều đó gợi ý ta đến hướng giải của bài toán.
Gọi M là họ tất cả các tập cân của A , N là họ các tập con của A có n phần tử. Xét
X = { 2, 4, . . . , 2n} là tập các số chẵn của A, Y = { 1, 3, ..., 2n − 1} là tập các số lẻ của
A. Gọi B là một phần tử thuộc M (tức B là một tập cân). Gọi B1 và B2 theo thứ tự là
tập chỉ gồm các số chẵn và số lẻ của B. Theo định nghĩa tập cân ta có |B1 | = |B2 |. Xét
một ánh xạ f từ M vào N như sau:

f (B) = B1 ∪ ( Y \B2 ) .

Do B1 và Y \B2 là hai tập rời nhau nên ta có

|f (B)| = |B1 | + |Y \B2 | = |B1 | + |Y | − |B2 | = |Y | = n.

Do đó f (B) ∈ N.
Công việc còn lại của ta chỉ là chứng minh f là song ánh.
+ Chứng minh f là đơn ánh:
Giả sử tồn tại mà f (B) = f (C) . Suy ra

B1 ∪ ( Y \B2 ) = C1 ∪ ( Y \C2 ) .

Mặt khác do B1 , C1 là tập các số chẵn còn ( Y \B2 ) , ( Y \C2 ) là tập các số lẻ nên
B1 = C1 và ( Y \B2 ) = ( Y \C2 ) suy ra B1 = C1 và B2 = C2 kéo theo B = C . Vậy f
là đơn ánh.
+ Chứng minh f là toàn ánh:
Với D ∈ N là một tập con của A ( tức D có n phần tử). Ta kí hiệu D1 , D2 theo thứ
tự là tập chỉ gồm các số chẵn và các số lẻ của D. Khi đó đặt B1 = D1 , B2 = Y \D2 và
B = B1 ∪ B2 . Ta có |B1 | = |D1 | và

|B2 | = |Y | − |D2 | = n − |D2 | = |D| − |D2 | = |D1 |

suy ra |B1 | = |B2 |. Do đó B là một tập cân. Vậy f cũng là toàn ánh. Từ đây ta có thể
kết luận được rằng f là song ánh. Vì có một song ánh giữa M và N nên |M| = |N| , hơn
n n
nữa ta lại có |N| = C2n . Vậy A có C2n tập cân.

9
Bài toán 6 Cho X = {1, 2, ..., n}. Một tập con của X được gọi là tập béo nếu mọi phần
tử của của tập con đó đều không nhỏ hơn số phần tử của nó. Chú ý rằng tập rỗng cũng là
một tập béo. Khi đó đặt an là số các tập con béo của X sao cho một tập béo bất kì không
chứa hai số liên tiếp, bn là số các tập con của X mà hai phần tử bất kì hơn kém nhau ít
nhất 3 đơn vị. Chứng minh an = bn .

Lời giải
Cách 1. Ta đi tính lần lượt an , bn .
* Tính an . Với mỗi k , gọi M là họ các tập béo có k phần tử thoả mãn đề bài. Gọi N là
họ các tập con có k phần tử của {k, k + 1, . . . , n − k + 1} .
Xét f : M → N cho bởi quy tắc như sau: Giả sử {m1 , . . . , mk } ∈ M với n > mi > k,
∀i = 1, 2, . . . , k . Không giảm tổng quát ta giả sử mi < mj với i < j. Đặt m1 =
n1 , m2 − 1 = n2 , . . . , mk − k + 1 = nk . Do {m1 , . . . , mk } không chứa hai số liên tiếp nên
m1 < m2 − 1 < . . . < mk − k + 1 . Suy ra k ≤ n1 < n2 < · · · ≤ n − k + 1. Ta định nghĩa

f ({ m1 , m2 , . . . ., mk } ) = { n1 , n2 , . . . , nk } ,

khi đó
{ n1 , n2 , . . . , nk } ∈ N.

Ta dễ dàng chứng minh được f là song ánh. Mặt khác |N| chính là cách chọn k phần
n
k k
P
tử từ tập {k, k + 1, . . . , n − k + 1} và bằng Cn−2k+2 . Vậy an = Cn−2k+2 với quy ước
k=0
Cki = 0 nếu i > k hoặc k < 0 .
* Tính bn .
Ý tưởng của chúng ta cũng là thiết lập một song ánh đi từ họ các tập con của X mà hai
phần tử bất kì hơn kém nhau ít nhất 3 đơn vị sang một tập khác dễ đếm hơn. Lúc này
dựa vào ý tưởng đã hình thành ở chỗ tính an ta cũng mạnh dạn thiết lập một song ánh
tương tự.
Gọi Ck là họ các tập con có k phần tử của X mà hai phần tử bất kì hơn kém nhau ít
nhất 3 đơn vị, Dk là họ các tập con có k phần tử của tập {1, 2, . . . , n − 2k + 2} .
Ta thiết lập một ánh xạ f : Ck → Dk như sau: Giả sử {c1 , c2 , . . . , ck } ∈ Ck , ta có thể
giả thiết c1 < c2 < . . . < ck . Đặt di = ci − 2i + 2 với i = 1, 2, .., k . Vì ci+1 − ci > 3 nên
di+1 − di > 1. Do đó 1 6 d1 < d2 < . . . < dk 6 n − 2k + 2.

10
Ta định nghĩa f ({c1 , c2 , . . . , ck }) = {d1 , d2 , . . . , dk } suy ra {d1 , d2 , . . . , dk } ∈ Dk . Ta
cũng dễ dàng chứng minh được f là song ánh.
Mặt khác |Dk | chính là cách chọn q phần tử từ tập {1, 2, . . . , n − 2k + 2} và bằng
q
Cn−2k+2 . Khi đó
n
X n
X n
X
k
bn = |Ck | = |Dk | = Cn−2k+2
k=0 k=0 k=0

( ta cũng quy ước như trên )


Đến đây suy ra được an = bn . Vậy ta có điều phải chứng minh.
Nhận xét: Rõ ràng hai ánh xạ sử dụng trong tính an và bn có vẻ khá giống nhau: sự
k
xuất hiện mờ ám của Cn−2k+2 . Điều này gợi cho ta liên tưởng tới một song ánh trực tiếp
giữa các họ lần lượt có lực lượng là an và bn . Đây chính là cơ sở hình thành cách giải thứ
hai của bài toán.
Cách 2. Gọi Ak là họ các tập béo có k phần tử thoả mãn đề bài, Bk là họ các tập con
của X có tính chất hai phần tử bất kì hơn kém nhau ít nhất 3 đơn vị.
Ta thiết lập một ánh xạ f : Ak → Bk như sau: Giả sử {a1 , a2 , . . . , ak } ∈ Ak . Ta có thể
giả sử
k ≤ a1 < a2 < a3 < . . . < ak ≤ n.

Đặt
b1 + k − 1 = a1 , b2 + k − 2 = a2 , . . . , bk = ak .

Ta có ai+1 > ai + 2 với mọi i = 1, 2, . . . , k − 1

⇒ ai+1 − ai ≥ 2 ⇒ bi+1 − bi ≥ 3


b1 ≥ 1, bk ≤ n.

Ta định nghĩa

f ({a1 , a2 , . . . , ak }) = { b1 , b2 , ..., bn } = { a1 + 1 − k, a2 + 2 − k, ..., ak } .

Suy ra { b1 , b2 , . . . , bn } ∈ Bk . Do vậy f là một ánh xạ từ Ak vào Bk . Ta dễ dàng chứng


minh được f là song ánh. Vì tồn tại một song ánh giữa Ak và Bk nên |Ak | = |Bk | từ đó
suy ra

11
Nhận xét: Rõ ràng cách thứ hai ngắn gọn và sáng sủa hơn nhiều so với cách thứ nhất ở
trên. Tuy nhiên mỗi cách đều có cái hay riêng của nó. Cách giải thứ nhất tuy dài nhưng
đây chính là cơ sở hình thành tri thức phương pháp cho người mới bắt đầu làm quen với
phương pháp song ánh và cũng là cơ sở giúp ta giải quyết những vấn đề hóc búa hơn sau
này.
Bên cạnh những ứng dụng của phép song ánh thì ta cũng có thể sử dụng ánh xạ đơn
ánh, toàn ánh để giải các bài toán tổ hợp. Cụ thể như sau:
Với A, B là các tập hợp hữu hạn. Xét f là một ánh xạ đi từ A vào B. Khi đó, nếu f là
đơn ánh thì |A| 6 |B|, nếu f là toàn ánh thì |A| > |B|. Hơn nữa, nếu f là đơn ánh và
không toàn ánh thì |A| < |B|, nếu f là toàn ánh và không đơn ánh thì |A| > |B| .

Bài toán 7 ( IMO - 1989) Một hoán vị (a1 , a2 , . . . , a2n ) của { 1, 2, . . . 2n} , n ∈ N
được gọi là có tính chất P nếu tồn tại ít nhất một giá trị i ∈ {1, 2, . . . , 2n − 1} sao cho
|ai − ai+1 | = n. Chứng minh rằng với mọi giá trị của n thì số hoán vị có tính chất P luôn
lớn hơn số hoán vị không có tính chất ấy.

Lời giải
Chúng ta sẽ bắt đầu từ những trường hợp nhỏ trước . Dễ dàng kiểm chứng khẳng định
đúng với n = 1.
Giả sử n > 2. Gọi A, B tương ứng là họ các tập không có và có tính chất P. Ta cần chứng
minh |A| < |B| . Bây giờ ta sẽ chứng minh dựa trên ý tưởng thiêt lập một ánh xạ f từ A
vào B sao cho f là đơn ánh nhưng không là toàn ánh.
Đặt T = (a1 , a2 , . . . , a2n ) là một phần tử của A . Do T không có tính chất P nên hai
phần tử liên tiếp nào cũng hơn kém nhau một khoảng khác n. Do đó phần tử cách a1
khoảng cách n là ar trong đó 2 < r < 2n + 1. Đặt

T0 = f (T ) = (a2 , a3 , . . . , ar−1 , a1 , ar , ar+1 , . . . , a2n ) .

Khi đó { a1 , ar } là cặp liền kề có khoảng cách n . Vì vậy T0 ∈ B nên f là một ánh xạ từ


A vào B .
* Chứng minh f là đơn ánh:
Gọi α = (x1 , x2 , . . . , x2n ) và β = (y1 , y2 , . . . , y2n ) là các hoán vị trong họ A. Gọi thành

12
phần cách x1 , y1 khoảng cách n đơn vị lần lượt trong α và β là xr và ys khi đó

3 ≤ xr ≤ 2n, 3 ≤ ys ≤ 2n.

Giả sử

f (α) = f (β) ⇔ (x2 , x3 , . . . , xr−1 , x1 , xr , xr+1 , . . . , x2n ) = (y2 , y3 , . . . , ys−1 , y1 , ys , ys+1 , . . . , y2n )

trong đó ( x1 , xr ) và ( y1 , ys ) là các cặp cách nhau n duy nhất tương ứng trong f (α) và
f (β) . Do đó ta suy ra r = s. Thật vậy, nếu r 6= s thì ta sẽ tìm được một cặp số ( xs , xs+1 )
thoả mãn |xs − xs+1 | = n , trái giả thiết α ∈ A.
Từ đó suy ra xi = yi , ∀i = 1, 2, ..., 2n hay α = β. Vậy f là đơn ánh.
* Chứng minh f không là toàn ánh.
Để ý rằng f (A) chứa tất cả các hoán vị của S chỉ gồm 1 cặp liền kề cách nhau n và B
gồm các hoán vị chứa ít nhất một cặp liền kề cách nhau n nên f (A) họ con thực sự của
B. Chẳng hạn, ta thấy (1, n + 1, 2, n + 2, 3, 4, ..., n, n + 3, ..., 2n) thuộc B nhưng không
thuộc f (A). Suy ra f không là toàn ánh. Vậy ta có điều phải chứng minh.
Nhận xét: Chứng minh trên có thể mở rộng ra với khoảng cách k bất kì chứ không nhất
thiết phải là n (k ∈ {1, 2, . . . , n}). Phương pháp sử dụng song ánh là một phương pháp
giải rất hay, rất đẹp, thể hiện ở sự tinh tế, ngẫu hứng của nó. Để tìm ra một lời giải cho
một bài toán bằng phương pháp này quả thực không hề đơn giản, nhưng khi đã tìm ra
thì có cảm giác vô cùng tuyệt vời. Hi vọng các bạn ít nhiều đã tìm ra cho mình nhưng
kinh nghiệm để giải quyết những bài toán tổ hợp hóc búa hơn sau này.

4 Phương pháp thiết lập hệ thức truy hồi


Phương pháp thiết lập hệ thức truy hồi là phương pháp giải bài toán với n đối tượng
thông qua việc giải bài toán tương tự với số đối tượng ít hơn bằng cách xây dựng các
quan hệ nào đó, gọi là quan hệ đệ quy. Sử dụng quan hệ này, ta có thể tính được đại
lượng cần tìm nếu chú ý rằng với n nhỏ, bài toán quy về xác định công thức số hạng tổng
quát của dãy số cho bằng công thức truy hồi. Trước hết xin nhắc lại công thức số hạng
tổng quát của dãy sai phân tuyến tính rất hay sử dụng trong phương pháp này. Cho dãy

13
số {un } thoả mãn điều kiện

u1 = α, u2 = β, a un+2 + b un+1 + c.un = 0, n ∈ N ∗ .

Phương trình trên được gọi là phương trình sai phân tuyến tính cấp hai thuần nhất. Khi
đó xét nghiệm của phương trình đặc trưng a.λ2 + b.λ + c = 0.
+ Nếu λ1 , λ2 là hai nghiệm thực khác nhau thì un = A.λn1 + B.λn2 , trong đó A và B được
xác định khi biết u1 , u2 .
+ Nếu λ1 = λ2 = λ là nghiệm kép thì un = (A + Bn) .λn , trong đó A và B được xác
định khi biết u1 , u2 .
+ Nếu phương trình đặc trưng có hai nghiệm phức liên hợp λ(cosϕ ± isinϕ) thì un =
λn (A cos nϕ + B sin nϕ) , trong đó A và B được xác định khi biết u1 , u2 .
Nhận xét: Công thức số hạng tổng quát của dãy số sai phân tuyến tính thuần nhất trên
có thể mở rộng cho dãy sai phân tuyến tính cấp cao hơn và không thuần nhất (xem [2]).
Ta minh họa phương pháp này thông qua một số bài toán sau đây.

Bài toán 8 (Bài toán chia kẹo của Euler) Cho k, n là các số nguyên dương. Tìm số
nghiệm nguyên không âm của phương trình x1 + x2 + ... + xn = k .

Lời giải
Gọi số nghiệm nguyên không âm của phương trình trên là S (n, k) . Dễ dàng thấy rằng
S (1, k) = 1 . Để tính S (n, k) , ta chú ý rằng phương trình đã cho đã cho tương đương với
x1 + x2 + ... + xn−1 = k − xn . Suy ra với xn cố định thì số nghiệm của phương trình này
là S (n − 1, k − xn ) . Từ đó ta được công thức

S (n, k) = S (n − 1, k) + S (n − 1, k − 1) + · · · + S (n − 1, 0) .

Đây có thể coi là công thức truy hồi tính S (n, k) . Tuy nhiên, công thức này chưa thật
tiện lợi. Viết công thức trên cho (n, k − 1) ta được

S (n, k − 1) = S (n − 1, k − 1) + S (n − 1, k − 2) + ... + S (n − 1, 0) .

Từ đây, trừ các đẳng thức trên vế theo vế, ta được

S (n, k) − S (n, k − 1) = S (n − 1, k) hay S (n, k) = S (n, k − 1) + S (n − 1, k) .

14
Từ công thức này, bằng quy nạp ta có thể chứng minh được rằng

n−1
S (n, k) = Ck+n−1 .

Nhận xét: Bài toán chia kẹo của Euler có thể giải bằng phương pháp song ánh bằng
cách đưa về dãy nhị phân như sau: Gọi A là họ các bộ {x1 , x2 , . . . , xk } thoả mãn phương
trình đã cho, B là họ các dãy nhị phân có độ dài n + k − 1 gồm k − 1 chữ số 0. Xét ánh xạ
f cho bởi quy tắc: Với mỗi bộ {x1 , x2 , . . . , xk } ta thực hiện viết liên tiếp từ trái qua phải
x1 chữ số 1, rồi đến 1 chữ số 0, rồi lại đến x2 chữ số 1, cứ như thế đến hết xn . Như vậy
ứng với mỗi bộ {x1 , x2 , . . . , xk } ta xây dựng được một dãy nhị phân có độ dài n + k − 1
gồm k − 1 chữ số 0 và n chữ số 1. Dễ dàng chứng minh được f là một song ánh. Từ đó
n−1
suy ra số nghiệm là Ck+n−1 .

Bài toán 9 Có bao nhiêu xâu nhị phân độ dài n trong đó không có hai chữ số 1 đứng
cạnh nhau?

Lời giải
Gọi cn là số xâu nhị phân độ dài n thỏa mãn điều kiện đầu bài. Ta có c1 = 2, c2 = 3. Để
tìm công thức truy hồi, ta xây dựng xâu nhị phân độ dài n thỏa mãn điều kiện đầu bài
có dạng an an−1 an−2 ......a2 a1 . Ta xét hai trường hợp sau.
• Trường hợp 1: an = 1 . Khi đó an−1 = 0 và an−2 ......a2 a1 có thể chọn là một xâu bất kỳ
độ dài n − 2 thỏa điều kiện. Có cn−2 xâu như vậy, suy ra trường hợp này có cn−2 xâu.
•Trường hợp 2: an 6= 1 . Khi đó an−1 an−2 . . . a2 a1 có thể chọn là một xâu bất kỳ độ dài
n − 1 thỏa điều kiện. Có cn−1 xâu như vậy, suy ra trường hợp này có cn−1 xâu. Vậy tổng
cộng xây dựng được cn−1 + cn−2 xâu, nghĩa là ta có hệ thức truy hồi cn = cn−1 + cn−2 .
Phương trình đặc trưng X 2 − X − 1 = 0 có hai nghiệm phân biệt
√ √
1+ 5 1− 5
X= ,X = và c1 = 2, c2 = 3
2 2

nên ta có được công thức tổng quát


√ !n+2 √ !n+2
1 1+ 5 1 1− 5
cn = √ −√ .
5 2 5 2

15
Bài toán 10 Có bao nhiêu cách lát đường đi kích thước 3 × 2n bằng các viên gạch kích
thước 1 × 2 ?

Lời giải
Gọi cn là số cách lát đường đi kích thước 3 × 2n . Dễ thấy c1 = 3 . Để tính cn , ta chia
các cách lát đường đi kích thước 3 × 2n thành n lọai, trong đó lọai thứ k là các cách lát
mà phần đường đi 3 × 2k đầu tiên được phủ kín hoàn toàn, nhưng không tồn tại i < k
sao cho phần đường đi 3 × 2i đầu tiên được phủ kín hoàn toàn. Gọi Ak là tập hợp các
cách lát lọai k thì rõ ràng

cn = |A1 | + |A2 | + . . . + |An | .

Dễ dàng nhận thấy |A1 | = 3cn−1 (phần đường đi 3 × 2 được lát kín bằng 3 cách, phần
còn lại được lát bằng cn−1 cách). Tiếp theo, có thể chứng minh dễ dàng rằng, chỉ có hai
cách phủ phần đường đi 3 × 2k cho các cách phủ thuộc Ak với k = 2, 3, . . . , n, chính là
cách phủ và cách phủ thu được bằng cách lấy đối xứng.
Từ đó suy ra |Ak | = 2cn−k . Như vậy, ta có

cn = 3cn−1 + 2cn−2 + . . . + 2.

Đây là dạng công thức truy hồi bậc vô hạn. Để thu được một công thức truy hồi bậc hữu
hạn, ta thay n bằng n + 1 suy ra

cn+1 = 3cn + 2cn−1 + 2cn−2 + . . . + 2.

Từ đó, trừ hai đẳng thức cuối cùng vế theo vế, ta được cn+1 − cn = 3cn − cn−1 và cuối
cùng là cn+1 = 4cn − cn−1 . Phương trình đặc trưng X 2 − 4X + 1 = 0 có hai nghiệm phân
biệt
√ √
X =2− 3, X = 2 + 3 và c1 = 3, c2 = 11

nên ta có được công thức tổng quát


√ √
3 + 1 √ n 3 − 1 √ n
cn = √ 2+ 3 + √ 2− 3 .
2 3 2 3

16
Bài toán 11 (Dự tuyển IMO-1996) Cho bảng ô vuông n×n(n > 1). Hỏi có bao nhiêu
cách đánh dấu các ô vuông trong bảng sao cho trong mỗi hình vuông 2 × 2 có đúng hai ô
vuông được đánh dấu?

Lời giải
Gọi Sn là số cách đánh dấu trong bảng n × n . Xét tập T gồm các ô vuông nằm trong
cột cuối cùng tính từ phải sang và hàng cuối cùng tính từ trên xuống dưới, ta gọi An là
các cách đánh dấu mà có hai ô vuông kề nhau trong T cùng được đánh dấu hoặc không
cùng được đánh dấu và Bn là các cách đánh dấu mà các ô vuông trong T được đánh dấu
xen kẽ.
Ta thấy với mỗi cách đánh dấu trong bảng n × n, bỏ đánh dấu các ô tên T sẽ được một
cách đánh dấu trong bảng (n − 1) × (n − 1) . Do đó mỗi cách đánh dấu trong Bn sẽ ứng
với một cách đánh dấu thuộc Bn−1 , còn mỗi cách đánh dấu trong An sẽ ứng với một cách
đánh dấu thuôc An−1 và một cách đánh dấu thuộc Bn−1 . Từ đó

|Bn | = |Bn−1 | , |An | = |An−1 | + |Bn−1 | (n > 2).

Hơn nữa Sn = |An | + |Bn | , ∀n > 1 nên Sn = 2Sn−1 − Sn−2 , ∀n > 3


Bằng cách tính trực tiếp ta được S2 = 6, S3 = 14 và phương trình đặc trưng cho dãy
số tuyến tính cấp hai có nghiệm kép X = 1 nên Sn = 2n − 10, ∀n > 1.

Bài toán 12 (VMO 2003) Với mỗi số nguyên dương n > 2 gọi sn là số các hoán vị
(a1 , a2 , ..., an ) của tập hợp En = {1, 2, ..., n} , mà mỗi hoán vị có tính chất 1 6 |ai − i| 6 2
với mọi i = 1, 2, . . . , n . Chứng minh rằng 1.75sn−1 < sn < 2sn−1 với n > 6 .

Lời giải
Trước hết ta sẽ xây dựng một hệ thức truy hồi cho sn . Với mỗi n tự nhiên khác không,
ký hiệu Sn là tập hợp tất cả các hoán vị A = (a1 , a2 , ..., an ) thỏa mãn yêu cầu đầu bài.
Giả sử n > 4 . Xét hoán vị A = (a1 , a2 , . . . , an , an+1 ) bất kỳ thuộc Sn . Từ giả thiết suy
ra an−1 = n + 1 hoặc an = n + 1 .
Trường hợp 1: an−1 = n + 1 , khi đó phải có an = n − 1 hoặc an = n − 2 .
• Nếu an = n − 1 thì phải có an+1 = n. Do đó trong trường hợp này A phải có dạng
(a1 , a2 , ..., an−2 ) ∈ Sn−2 .

17
• Nếu an = n − 2 thì phải có an+1 = n − 1 hoặc an+1 = n.
+ Nếu an+1 = n − 1 thì phải có an−2 = n. Do đó trong trường hợp này A phải có dạng
A2 = (a1 , a2 , . . . , an−3 , n, n + 1, n − 2, n − 1) với (a1 , a2 , . . . , an−3 ) ∈ Sn−3 .
+ Nếu an+1 = n thì A phải có dạng A3 = (a1 , a2 , . . . , an−2 , n + 1, n − 2, n) với

{a1 , a2 , . . . , an−2 } = {1, 2, 3, . . . , n} \ {n − 2, n} .

Trường hợp 2: an = n + 1, khi đó phải có an+1 = n hoặc an+1 = n − 1.


• Nếu an+1 = n thì A phải có dạng A4 = (a1 , a2 , ..., an−1 , n + 1, n) với (a1 , a2 , ..., an−1 ) ∈
Sn−1 .
• Nếu an+1 = n − 1 thì A phải có dạng A5 = (a1 , a2 , ..., an−1 , n + 1, n − 1) với

{a1 , a2 , ..., an−1 } = {1, 2, 3, ..., n} \ {n − 1} .

Từ các cách xác định A1 , A2 , A3 , A4 , A5 suy ra các phép đặc tương ứng f thỏa

f : A1 ∈ Sn+1 7→ (a1 , a2 , ..., an−2 ) ∈ Sn−2 ,

f : A2 ∈ Sn+1 7→ (a1 , a2 , ..., an−3 ) ∈ Sn−3 ,

f : A3 ∈ Sn+1 7→ (a1 , a2 , ..., an−2 , n, n − 2) ∈ Sn ,

f : A4 ∈ Sn+1 7→ (a1 , a2 , ..., an−1 ) ∈ Sn−1 ,

f : A5 ∈ Sn+1 7→ (a1 , a2 , ..., an−1 , n − 1) ∈ Sn .

Xác lập một đơn ánh


f : Sn+1 → Sn ∪ Sn−1 ∪ Sn−2 ∪ Sn−3 .

Ta thấy
B∈
/ f (Sn+1 ) ⇔ B = (b1 , b2 , ..., bn−1 , n − 2) ∈ Sn với bn−1 6= n

⇔ B = (b1 , b2 , ..., bn−4 , n − 1, n, n − 3, n − 2) ∈ Sn với (b1 , b2 , ..., bn−4 ) ∈ Sn−4 .

Từ đó suy ra f là một song ánh từ Sn+1 đến

(Sn ∪ Sn−1 ∪ Sn−2 ∪ Sn−3 ) \ (Sn−4 ∪ { n − 1, n, n − 3, n − 3}) .

Vì thế ta có công thức truy hồi sn+1 = sn + sn−1 + sn−2 + sn−3 − sn−4 .

18
Suy ra sn+1 − sn = sn−1 + sn−2 + sn−3 − sn−4 , ∀n > 4.
Do đó sn+2 − sn+1 = sn + sn−1 + sn−2 − sn−3 , ∀n > 4.
Suy ra sn+2 − 2sn+1 = sn−4 − 2sn−3 , ∀n > 4,
dễ thấy sn−3 > sn−4 , ∀n > 4 kéo theo sn−4 − 2sn−3 < 0, ∀n > 4, nên sn < 2sn−1 .
Bằng cách đếm trực tiếp, ta có s1 = 0, s2 = 1, s3 = 2, s4 = 4 suy ra s5 = 6, s6 =
13, s7 = 24, s8 = 49. Nên bài toán cũng đúng khi n = 7, 8.
Hơn nữa
sn > sn−1 + sn−2 + sn−3 , ∀n > 5 nên

sn−1 sn−2 sn−1 sn−1


sn > sn−1 + + > sn−1 + + = 1, 75.sn−1 , ∀n > 8.
2 2 2 4

5 Phương pháp hàm sinh


Phương pháp hàm sinh là một phương pháp hiện đại, sử dụng các kiến thức về chuỗi,
chuỗi hàm (đặc biệt là khai triển Newton suy rộng, công thức Taylor). Đây là phương
pháp rất mạnh để giải bài toán giải tích tổ hợp.
Cho dãy số a0 , a1 , a2 , ..., an , . . . Chuỗi hình thức

A (x) = a0 + a1 x + a2 x2 + ... + an xn + . . .

được gọi là hàm sinh của dãy {an }.


Ý tưởng cơ bản của phương pháp hàm sinh như sau: Giả sử ta cần tìm công thức tổng
quát của một dãy số {an } nào đó. Từ công thức truy hồi hoặc những lý luận tổ hợp trực
tiếp, ta tìm được hàm sinh

A (x) = a0 + a1 x + a2 x2 + · · · + an xn + . . . .

Khai triển A (x) thành chuỗi và tìm hệ số của xn trong khai triển đó ta tìm được an . Sau
đây là một số công thức khai triển thường sử dụng.
* Công thức nhị thức Newton suy rộng

α (α − 1) x2 α (α − 1) ... (α − n + 1) n
(1 + x)α = 1 + αx + + ··· + x + ...
2 n!

19
Đặc biệt
1
= 1 + x + x2 + · · · + xn + . . .
1 − x
1
= 1 − x + x2 − · · · + (−1)n xn + . . .
1 + x
1 1
 1  1
 n
√ 1 x2 − 1 − 2 ... − n + 1 x
1 + x=1+ x− + ··· + 2 2 2 2
+ ...
2 8 n!
* Công thức khai triển Taylor một số hàm quen thuộc

x2 xn
ex = 1 + x + + ··· + + ...
2! n!
x3 x2n+1
sin x = x − + · · · + (−1)n + ...
3! (2n + 1)!
x2 x2n
cos x = 1 − + · · · + (−1)n + ...
2! (2n)!
Nhận xét: Những công thức trên là trường hợp riêng của khai triển Taylor dạng tổng
quát
f 00 (0) 2 f (n) (0) n
f (x) = f (0) + f 0 (0)x + x + ··· + x + ...,
2! n!
trong đó kí hiệu f (n) (0) là đạo hàm cấp n của hàm f (x) tại x = 0
(Đây là kiến thức trong phần nhập môn Giải tích I trong chương trình đại học năm thứ
nhất).

Bài toán 13 Tìm số hạng tổng quát của dãy số Fibonacci được xác đinh bởi f0 = 1, f1 =
1, fn+1 = fn + fn−1 .

Lời giải
Xét hàm sinh

F (x) = f0 + f1 x + f2 x2 + · · · + fn xn + . . .

= f0 + f1 x + (f0 + f1 ) x2 + · · · + (fn−1 + fn−2 ) xn + . . .

= f0 + f1 x + x2 (f0 + f1 x + . . .) + x (f1 x + . . .)

= f0 + f1 x + x2 F (x) + x (F (x) − f0 ) .

Từ đó suy ra
1
F (x) = .
1 − x − x2

20
Tiếp theo, ta khai triển F (x) thành chuỗi. Ta có

1 1
,
α β
là nghiệm của phương trình x2 − x − 1 = 0,
với √ √
1+ 5 1− 5
α= ,β =
2 2
Viết F (x) dưới dạng
A B
F (x) = +
1 − αx 1 − βx
với A, B là hằng số. Bằng cách đồng nhất hai vế, ta được
√ √
1+ 5 1− 5 α β
A = √ ,B = − √ hay A = √ , B = − √
2 5 2 5 5 5
.

Từ đó, sử dụng công thức

1
= 1 + x + x2 + ... + xn + . . .
1−x
ta có

F (x) = A (1 + αx + · · · + αn xn + . . .) + B (1 + βx + · · · + β n xn + . . .)

= A + B + (Aα + Bβ)x + · · · + (Aαn + Bβ n )xn + . . .

suy ra
fn = Aαn + Bβ n

do đó √ !n+1 √ !n+1
1 1+ 5 1 1− 5
fn = √ −√ .
5 2 5 2

Nhận xét: Bài toán tìm công thức số hạng tổng quát của dãy Fibonacci có thể giải bằng
phương pháp sử dụng phương trình sai phân tuyến tính cấp hai khá gọn. Tuy nhiên,
chúng tôi muốn sử dụng bài toán này làm ví dụ minh họa phương pháp hàm sinh (một
phương pháp khá mới và “xa lạ” cần trang bị kiến thức về toán cao cấp như chuỗi lũy
thừa và khai triển Taylor) để tạo ra sự “gần gũi” của phương pháp hàm sinh.

21
Bài toán 14 Tìm số hạng tổng quát của dãy a0 = 1, an a0 + an−1 a1 + ... + a0 an = 1.

Lời giải
Xét hàm sinh
A (x) = a0 + a1 x + a2 x2 + · · · + an xn + . . .

Biểu thức truy hồi gợi chúng ta đến hệ số của hai đa thức

A (x) .A (x) = a0 + (a0 a1 + a1 a0 ) x + ... + (an a0 + an−1 a1 + ... + a0 an ) xn + ...


.
2 n −1
= 1 + x + x + ... + x = (1 − x) .

Từ đó suy ra

A (x) = (1 − x)−1/2

= 1 + (1/2) x + (1/2) (3/2) x2 /2 + ... + (1/2) (3/2) ... (n − 1/2) xn /n! + ...

và như vậy
n
1.3.5... (2n − 1) C2n
an = = .
2n .n! 4n

Bài toán 15 Có 2n điểm trên đường tròn. Hãy tìm số cách nối 2n điểm này bằng n dây
cung không cắt nhau.

Lời giải
Gọi cn là đáp số bài toán. Ta có c1 = 1, c2 = 2 và quy ước c0 = 1. Khi đó ta dễ dàng xây
dựng được công thức truy hồi

cn = cn−1 c0 + cn−2 c1 + · · · + c0 cn−1 .

Xét hàm sinh


A (x) = c0 + c1 x + c2 x2 + ... + cn xn + . . .

Từ công thức truy hồi trên, ta có

(A (x))2 = c0 + (c0 c1 + a1 a0 ) x + ... + (cn c0 + cn−1 c1 + ... + c0 cn ) xn + ...

= c1 + c2 x + ... + cn+1 xn + ...

Từ đó suy ra
x(A (x))2 = A (x) − 1

22
⇔ x2 (A (x))2 = xA (x) − x
 2
1 1 1 − 4x
⇔ xA (x) − = −x + = .
2 4 4
nên 1 1
1 + (1 − 4x) 2 1 − (1 − 4x) 2
xA (x) = hoặc xA (x) = .
2 2
Do xA (x) không có số hạng không chứa x, tức là nó triệt tiệu tại x = 0 nên
1
1 − (1 − 4x) 2
xA (x) =
2
∞ n
1 1 X 12 21 − 1 12 − 2 ... 12 − n + 1 (−4x)
  
⇔ xA (x) = −
4 4 n=0 n!

1 X 12 12 32 ... 2n−3 (4x)n
  
2
⇔ xA (x) =
2 n=1 n!

X 1.3 . . . (2n − 3)2n−1 xn
⇔ xA (x) =
n=1
n!

X 1.3 . . . (2n − 3)2n−1 xn−1
⇔ A (x) =
n=1
n!

X 1.3 . . . (2n − 1)2n xn
⇔ A (x) = .
n=0
(n + 1)!

Do đó
1.3 . . . (2n − 1)2n (2n)! Cn
cn = = = 2n .
(n + 1)! n!(n + 1)! n+1
Chú ý: Người ta định nghĩa các số Catalan được cho bằng công thức truy hồi c0 = 1,
cn = cn−1 c0 + cn−2 c1 + · · · + c0 cn−1 . Khi đó theo bài toán trên ta xác định được các số
n
C2n
Catalan cn = n+1
. Ngoài ra ta có thể xác định các số Catalan bằng phương pháp quỹ
đạo và nguyên lý đối xứng gương như sau: Giả sử A(a; α), B(b; β) là các điểm có toạ độ
nguyên, hơn nữa b > a > 0, β > α > 0 và A0 (a; −α) là điểm đối xứng với A qua trục Ox.
Gọi quỹ đạo từ A đến B là đường đi từ A đến B chỉ qua các điểm nguyên và chỉ đi theo
hướng lên trên và qua phải. Khi đó số các quỹ đạo từ A đến B cắt trục Ox hoặc có điểm
chung với Ox bằng số các quỹ đạo từ A0 đến B. Từ đó ta thu được công thức tính tất cả
các số Catalan.

23
6 Dùng công cụ số phức
Số phức có rất nhiều ứng dụng trong nhiều ngành toán học khác nhau như hình học, đại
số, số học, . . . Trong mục này sẽ trình bày một số ứng dụng của số phức trong giải bài
toán tổ hợp. Để tiện theo dõi chúng tôi trình bày một số kết quả cơ bản về số phức nhưng
không chứng minh. Chi tiết các chứng minh xin xem [3].
Cho p là một số nguyên tố lẻ. Khi đó phương trình xp = 1 có p nghiệm phức phân biệt
2kπ 2kπ
xk = cos + i sin , với k = 0, 1, 2, . . . , p − 1.
p p
Từ đẳng thức
xp − 1 = (xp − 1) xp−1 + xp−2 + · · · + 1 và x0 = 1


ta suy ra phương trình xp−1 + xp−2 + · · · + 1 = 0 có p − 1 nghiệm phức phân biệt


2kπ 2kπ
xk = cos + i sin với k = 1, 2, . . . , p − 1.
p p
Gọi α là một nghiệm bất kỳ của phương trình xp−1 + xp−2 + · · · + 1 = 0 , trong đó p
là một số nguyên tố lẻ. Khi đó vế trái của phương trình trên là đa thức bất khả quy
trên trường số thực và tập nghiệm phức của phương trình là {α, α2 , . . . , αp−1 }. Hơn nữa
 n 2n
α , α , . . . , α(p−1)n là một hoán vị của {α, α2 , . . . , αp−1 } , trong đó n nguyên tố cùng
nhau với p.
n
ak xk là một đa thức với hệ số thực và m là một số nguyên dương, đặt
P
Giả sử f (x) =
k=0
cos 2π
m
+ i sin 2π
m
. Khi đó
m−1
X m−1
XX n n
X m−1
X
j jk
f (z ) = ak z = ak z jk .
j=0 j=0 k=0 k=0 j=0

Gọi A là tập con của {1, 2, . . . , n} gồm những phần tử chia hết cho m. Từ 1 + z k + z 2k +
· · · + z (m−1)k bằng m khi k chia hết cho m và bằng không khi k không chia hết cho m.
Suy ra !
m−1
X m−1
XX n X
f (z j ) = ak z jk = m ak .
j=0 j=0 k=0 k∈A

Do đó
m−1
X 1 X
ak = f (z j ).
k∈A
m j=0

24
Bài toán 16 (Bay Area Math Circle 1999) Cho m, n là các số nguyên dương. Một hình
chữ nhật được tạo thành bằng cách xếp một số hình chữ nhật 1 × m và n × 1. Chứng minh
ta có thể chỉ sử dụng một loại 1 × m hoặc n × 1 để tạo thành hình chữ nhật đã cho.

Lời giải
Giả sử hình chữ nhật có dạng a × b, rõ ràng a, b là các số nguyên dương. Ta cần chứng
minh a chia hết cho n hoặc b chia hết cho m. Gọi
2π 2π 2π 2π
α = cos + i sin , β = cos + i sin .
m m n n
Chia hình chữ nhật a × b thành a.b hình chữ nhật đơn vị. Ứng với hình chữ nhật đơn vị
ở cột x và hàng y ta đặt số phức αx β y . Tổng các số phức ở mỗi hình chữ nhật n × 1 bắt
đầu là ô vuông cột x, hàng y nào đó:
βn − 1
αx β y (1 + β + · · · + β n−1 ) = αx β y = 0.
β−1
Tổng các số phức ở mỗi hình chữ nhật 1 × m bắt đầu là ô vuông cột x, hàng y nào đó:
m
x y m−1 x yα −1
α β (1 + α + · · · + α )=α β = 0.
α−1
Trong khi tổng tất cả các số phức trên hình chữ nhật là
αa − 1 β b − 1
(α + α2 + ... + αa )(β + β 2 + ... + β b ) = αβ = 0.
α−1 β−1
Vì vậy αa = 1 hoặc β b = 1 , do đó a chia hết cho n hoặc b chia hết cho m. Vậy ta có được
điều phải chứng minh.

Bài toán 17 Cho p là một số nguyên tố lẻ và số nguyên dương k không vượt quá p − 1.
Tìm số tập con X của tập {1, 2, . . . , p} biết rằng X chứa đúng k phần tử và tổng tất cả
các phần tử của X chia hết cho p.

Lời giải
Đặt
A = {X ⊂ {1, 2, . . . , p} : |X| = k} , Aj = {X ∈ A : S(X) ≡ j(modp)}

với j = 0, 1, . . . , p − 1. Khi đó |A| = Cpk . Vì A = A0 ∪ A1 ∪ · · · ∪ Ap−1 và Ai ∩ Aj là tập


rỗng với i 6= j nên
|A| = |A0 | + |A1 | + · · · + |Ap−1 | .

25
Xét đa thức P (x) = xp−1 +xp−2 +· · · +x +1 , đa thức này có p −1 nghiệm phức phân biệt.
Giả sử α là một nghiệm bất kỳ của P (x) thì tập nghiệm của P (x) là {α, α2 , ..., αp−1 } và
αp = 1. Do phương trình xp = 1 có p nghiệm phức phân biệt α, α2 , ..., αp nên ta có thể viết
xp − 1 = (x − α)(x − α2 )...(x − αp ), so sánh hệ số của xp−k ta được (−1)k
P S(X)
α = 0.
X∈A
Chú ý rằng αS(X) = αj nếu X ∈ Aj . Do đó
p−1
X
|Aj | αj = 0.
j=0

Vì vậy α là nghiệm của đa thức


p−1
X
Q(x) = |Aj | xj .
j=0

Nên ta được
|A0 | = |A1 | = ... = |Ap−1 |

suy ra
|A0 | + |A1 | + ... + |Ap−1 | |A|
|A0 | = = .
p p
Do đó
Cpk
|A0 | = .
p
Vậy số tập con của X chứa đúng k phần tử và tổng tất cả các phần tử của X chia hết
cho p là
Cpk
.
p

Bài toán 18 Cho ba số nguyên dương m, n, p trong đó n + 1 chia hết cho m. Tìm số các
bộ (x1 , x2 , . . . , xp ) gồm p số nguyên dương sao cho tổng x1 + x2 + ... + xp chia hết cho m,
trong đó mỗi số x1 , x2 , ..., xp đều không lớn hơn n.

Lời giải
Xét đa thức
p
P (x) = xn−1 + xn−2 + · · · + x + 1 ,

26
bằng cách khai triển đa thức này, ta có thể viết
m
X
P (x) = ak x k ,
k=0

trong đó ak là số các bộ (x1 , x2 , . . . , xp ) với xj ∈ {1, 2, . . . , n} , j = 1, 2, . . . , p sao cho


x1 +x2 +· · ·+xp = k. Vì vậy số các bộ (x1 , x2 , . . . , xp ) với xj ∈ {1, 2, . . . , n} , j = 1, 2, . . . , p
sao cho x1 + x2 + · · · + xp chia hết cho m bằng tông các số ak với k chia hết cho m . Ta
biết rằng tổng của các số ak với k chia hết cho m bằng
m−1
1 X 2π 2π
f (αj ), với α = cos + i sin .
m j=0 m m

Khi m = 1 thì tổng này bằng P (1) = np còn khi m > 1 thì αj 6= 1 với j = 1, 2, .., m − 1 .
Để ý rằng khi x 6= 1 thì ta có được
p
xn+1 − 1

P (x) = −1 .
x−1

Do vậy p
α(n+1)j − 1

j
P (α ) = −1
αj − 1
với j = 1, 2, . . . , m − 1. Vìn + 1 chia hết cho m nên α(n+1)j = 1 với j = 1, 2, . . . , m − 1. Do
đó P (αj ) = (−1)p với j = 1, 2, . . . , m − 1 . Vậy số các bộ (x1 , x2 , . . . , xp ) gồm p số nguyên
dương sao cho tổng x1 + x2 + · · · + xp chia hết cho m , trong đó mỗi số x1 , x2 , . . . , xp đều
không lớn hơn n bằng

np + (m − 1)(−1)p np − (−1)p
= + (−1)p .
m m

7 Ứng dụng của tổ hợp


Lý thuyết tổ hợp không chỉ giải quyết các bài toán được đặt ra trong chính lý thuyết
này mà còn nhiều ứng dụng thú vị trong các ngành toán học khác, ví dụ như trong đại
số, số học, hình học tổ hợp, lý thuyết xác suất, . . .
Các hệ số nhị thức thường được nảy sinh một cách tự nhiên trong số học modular, trong
đại số giao hoán, trong lý thuyết đại số Lie modular, vì vậy, những đẳng thức liên quan
đến hệ số nhị thức đóng một vai trò đặc biệt quan trọng.

27
Dưới đây, chúng ta xét một số ví dụ liên quan đến ứng dụng của tổ hợp trong các lĩnh
vực khác nhau của toán học.

Bài toán 19 Cho p là một số nguyên tố. Đường tròn được chia thành p cung bằng nhau.
Hỏi có bao nhiêu cách tô p cung bằng a màu khác nhau (Hai cách tô màu thu được bằng
một phép quay được coi là giống nhau)?

Lời giải
Mỗi mội cung có a cách tô màu, như vậy có ap cách tô màu p cung (với quy ước cố định
vị trí). Trong số này có a cách tô màu bằng chỉ một màu. Với mỗi cách tô màu dùng 2
màu trở lên, ta có thể dùng phép quay để tạo ra p cách tô màu khác được tính trong ap
cách tô màu trên nhưng không được tính theo cách tính đề bài. Như vậy số cách tô màu
thoả mãn điều kiện đề bài là
ap − a
+ a.
p
Nhận xét: Ta có thể chứng minh định lý nhỏ Fermat, “Cho p là số nguyên tố và a là số
nguyên, khi đó ap − a chia hết cho p” bằng cách áp dụng kết quả của Bài toán trên.
Lý thuyết tổ hợp còn có thể được áp dụng để chứng minh cách đẳng thức tổ hợp một
cách vô cùng hiệu quả. Ý tưởng cơ bản là: Nếu ta đếm một tập hợp bằng hai cách khác
nhau thì các kết quả thu được phải bằng nhau, cho dù, với các cách đếm khác nhau ta
có thể ra các biểu thức rất khác nhau. Đây được gọi là phương pháp đếm bằng hai cách
và cũng thường được sử dụng trong tổ hợp.

Bài toán 20 Chứng minh hệ thức

n
2
X
k(Cnk ) = nC2n−1
n−1
.
k=1

Lời giải
Ta đặt ra bài toán sau: Có n học sinh chuyên toán và n học sinh chuyên văn. Hỏi có bao
nhiêu cách chọn ra một đội xung kích gồm n học sinh, trong đó có 1 học sinh chuyên toán
làm đội trưởng. Sau đây ta giải quyết bài toán sau bằng hai cách.
Cách thứ nhất: Chọn đội trưởng là học sinh chuyên toán , sau đó chọn n − 1 thành viên
n−1
còn lại từ 2n − 1 học sinh còn lại. Có nC2n−1 cách.

28
Cách cách thứ hai: Chọn k học sinh chuyên toán, chọn đội trưởng là một học sinh chuyên
toán trong k học sinh chuyên toán đã chọn sau đó chọn n − k học sinh chuyên văn với
k = 1, 2, . . . , n . Số cách chọn là
n n
2
X X
Cnk .k.Cnn−k = k(Cnk ) .
k=1 k=1

Từ đây ta có được đẳng thức cần chứng minh.

Bài toán 21 Chứng minh rằng từ 2n − 1 số nguyên bất kỳ luôn tìm được n số có tổng
chia hết cho n.

Lời giải
Ta gọi mệnh đề ở đề bài là A (n). Trước hết ta thấy rằng nếu A (m) , A (n) đúng thì
A (mn) cũng đúng . Từ đây, bài toán quy về việc chứng minh A (p) đúng với p là số
nguyên tố. Xét E = {a1 , a2 , . . . , a2p−1 }. Giả sử ngược lại rằng với mọi bộ ai1 , ..., aip lấy từ
ta có ai1 + ... + aip không chia hết cho p. Khi đó, theo định lý nhỏ Fermat

p−1
ai1 + ... + aip ≡ 1 (mod p) .

Từ đó suy ra
X p−1 p
ai1 + ... + aip ≡ C2p−1 (mod p)

trong đó tổng tính theo tất cả các tập con p phần tử của E. Mặt khác, ta đếm số lần xuất
p−k
hiện của đơn thức aaj11 ...aajkk với α1 + ... + αk = p − 1 trong tổng ở vế trái. Có C2p−1
k
C2p−k−1
tổng dạng ai1 + · · · + aip có chứa ai1 , . . . , aip . Trong mỗi tổng này, đơn thức aaj11 . . . aajkk xuất
hiện với hệ số
(p − 1)!
.
a1 ! . . . ak !
Như vậy, đơn thức aaj11 . . . aajkk sẽ xuất hiện trong tổng vế trái với hệ số

k p−k (p − 1)! (2p − 1)! (p − 1)!


C2p−1 C2p−k−1 = .
a1 ! . . . ak ! k! (p − k)! (p − 1)! a1 ! . . . ak !
Do 1 6 k 6 p − 1 nên hệ số này luôn chia hết cho p, suy ra tổng vế trái chia hết cho p.
Mặt khác
p (2p − 1)! (p + 1) ... (2p − 1)
C2p−1 = =
p! (p − 1)! (p − 1)!
không chia hết cho p. Đến đây ta gặp điều mâu thuẫn. Vậy bài toán chứng minh xong.

29
Bài toán 22 Cho a là số thực dương và n là số nguyên dương cho trước. Tìm giá trị lớn
nhất của biểu thức
x1 x2 ...xn
,
(1 + x1 ) (x1 + x2 ) ... (xn−1 + xn ) (xn + an+1 )
trong đó x1 , x2 , ..., xn là các số dương tuỳ ý.

Lời giải
Đặt
x2 an+1
u0 = x1 , u1 = , ..., un =
x1 xn
thì u0 u1 ...un = an+1 và ta cần tìm giá trị nhỏ nhất của

(1 + u0 ) (1 + u1 ) ... (1 + un ) .

Ta có
X X X
(1 + u0 ) (1 + u1 ) ... (1 + un ) = 1 + ui + ui1 ui2 + ... + ui1 ...uik + ... + u0 u1 ...un ,

k
số hạng, tích của chúng sẽ là một biểu thức bậc kCnk .
P
trong đó tổng ui1 ...uik có Cn+1
Do tính đối xứng, mỗi một số hạng sẽ đúng góp bậc là
k
kCn+1
.
n+1
Suy ra tích của tất cả các số hạng này bằng
k k
(an+1 )(kCn+1 /(n+1)) = akCn+1 .

Áp dụng bất đẳng thức Cauchy, ta có


X
k
ui1 ...uik >Cn+1 ak .

Do đó
X X X
1+ ui + ui1 ui2 + ... + ui1 ...uik + ... + u0 u1 ...un

≥ 1 + (n + 1) a + C 2 n+1 a2 + ... + C k n+1 ak + ... + an+1 = (1 + a)n+1 .

Dấu bằng xảy ra khi và chỉ khi u0 = u1 = ... = un = a tức là khi

x1 = a, x2 = a2 , ..., xn = an .

30
Bài toán 23 (Định lý Lucas) Cho p là số nguyên tố và hai số nguyên dương m, n thỏa
mãn m 6 n được biểu diễn dưới dạng p-phân như sau: m = as as−1 ...a0 |p , n = bs bs−1 ...b0 |p
với bs 6= 0. Chứng minh rằng

a
Cnm ≡ Cbass .Cbs−1
s−1
....Cba00 (modp) .

Lời giải
Do Cpk ≡ 0 (modp) với mọi k = 1, 2, ..., p − 1 nên (a + b)p ≡ ap + bp (modp). Từ đó bằng
k
quy nạp toán học, ta dễ dàng chứng minh được (a + b)p ≡ ap + bp (modp) với mọi a, b
k k

nguyên. Hơn nữa

s s−1
(1 + x)n = (1 + x)bs p (1 + x)bs−1 p . . . (1 + x)b0

và lấy theo modulo p ta được

s s−1
(1 + x)n ≡ (1 + xp )bs (1 + xp )bs−1 ...(1 + x)b0 (modp) .

Hệ số của xm ở vế trái là Cnm . Do biểu diễn của n dưới dạng p-phân là duy nhất nên hệ
a
số của xm ở vế phải là Cbass .Cbs−1
s−1
....Cba00 nên ta suy ra được điều phải chứng minh.

Bài toán 24 (TST VN2010) Gọi Sn là tổng bình phương các hệ số trong khai triển
của (1 + x)n . Chứng minh rằng S2n + 1 không chia hết cho 3.

Lời giải
Ta có ! !
2n
X 2n
X
(1 + x)4n = (1 + x)2n (1 + x)2n = k k
C2n x k k
C2n x .
k=0 k=0
2n
So sánh hệ số của x ở hai vế ta có
2n
X
k
2 2n
C2n = C4n
k=0

suy ra
2n
X
k
2 2n
S2n + 1 = C2n + 1 = C4n + 1.
k=0
m
ak .3k với ak ∈ {0, 1, 2}.
P
Giả sử 2n có biểu diễn dưới dạng tam phân là 2n =
k=0
m
2ak .3k .
P
+ Nếu ak ∈ {0, 1} , ∀k = 1, . . . , m. Khi đó 4n có biểu diễn tam phân là 4n =
k=0

31
m
P m
P
Ta thấy ak là số chẵn và gọi 2p = ak . Theo định lý Lucas ta có
k=0 k=0
2n a1 a2 am
C4n ≡ C2a1
.C2a2
....C2am
(mod3) .


a1 a2 am
C2a1
.C2a2
....C2am
= 2a1 .2a2 ....2am = 22p = 4p

nên S2n + 1 = 4p + 1 ≡ 1 (mod3) hay S2n + 1 không chia hết cho 3. Đây là điều phải
chứng minh.
+ Nếu tồn tại ai = 2. Không mất tính tổng quát, giả sử i là số nhỏ nhất sao ai = 2. Khi
đó hệ số tương ứng của 4n trong hệ tam phân là 1. Do C12 = 0 (vì số chập lớn hơn số
2n
phần tử) nên lại áp dụng định lý Lucas suy ra C4n ≡ 0(mod3) hay S2n + 1 không chia
hết cho 3. Bài toán đã giải quyết hoàn toàn.
Nhận xét: Cách tiếp cận bài toán này, yêu cầu chứng minh S2n + 1 không chia hết cho
3 có hai mục đích. Thứ nhất, tính được S2n bằng cách so sách hệ số khi khai triển nhị
2n
thức Newton. Thứ hai, đưa ra bản chất tổ hợp của C4n , từ đó gợi ý đến cách khai triển
theo modulo.

Tài liệu
[1] NGUYỄN VĂN MẬU (chủ biên), Các chuyên đề chọn lọc tổ hợp và rời rạc, NXB GD,
2008.

[2] NGUYỄN VĂN MẬU (chủ biên), Dãy số, NXB GD, 2008.

[3] NGUYỄN VĂN MẬU (chủ biên), Số phức và áp dụng, NXB GD, 2008.

[4] TRẦN NAM DŨNG, Bài giảng tổ hợp, Tập huấn GV Chuyên Toán tại Đà Nẵng 2013.

[5] TITU ANDREESCU, ZUMING FENG, Combinatorial problems, from the training
the USA IMO Team.

[6] PABLO SOBERON BRAVO, ZUMING FENG, Problem solving method in combina-
torics, On appoach to Olympiad problems, Berkhauser.

[7] Tài liệu Internet, đặc biệt là các website: www.mathscope.org, www.diendantoanhoc.net.

32
TƯ DUY CỰC HẠN TRONG CÁC BÀI TOÁN TỔ HỢP (Chuyên đề tập huấn giáo viên 2017)

TƯ DUY CỰC HẠN TRONG CÁC BÀI TOÁN TỔ HỢP

TÓM TẮT: Nguyên lý cực hạn khẳng định một tập hữu hạn A ⊂ R luôn có phần tử nhỏ nhất và phần
tử lớn nhất. Trong nhiều bài toán Olympic, việc xem xét các phần tử cực biên, đối tượng cực biên sẽ
đưa đến một lời giải đẹp, đôi khi còn là hướng tiếp cận duy nhất cho bài toán. Trong chuyên đề này,
tôi không trình bày lại cơ sở lý thuyết cũng như các ví dụ dẫn nhập cho nguyên lý này, mà tập trung
vào phân tích một số bài toán, cùng các ý tưởng, tư tưởng giải toán. Do đó trong chuyên đề sẽ không
có những ví dụ dễ, phù hợp với việc ôn tập phần tổ hợp cho các em học sinh chuẩn bị thi học sinh giỏi
cấp Quốc gia. Các lời giải trong các ví dụ sẽ tương đối dài, vì bên cạnh lời giải là những phân tích
có lý cho các đề toán. Khi hiểu nội dung lời giải, thì trình bày lại có thể rút ngắn lời giải rất nhiều.
Chuyên đề được viết trên cơ sở đúc rút kinh nghiệm của bản thân trong quá trình ôn tập các đội tuyển.
Mong được sự chia sẻ của quý đồng nghiệp về nội dung của chuyên đề.

VÍ DỤ 1 (ITALY 2000). Cho X là một tập hợp hữu hạn với |X| = n, và đặt A1 , A2 , . . . , Am là các tập
T
con chứa √3 phần tử của X thỏa mãn |A i A j | ≤ 1 với mọi i 6= j. Chứng tỏ tồn tại tập con A của X chứa
ít nhất [ 2n] phần tử mà không nhận bất cứ tập Ai (i = 1, 2, . . . , m) nào là tập con của nó.

Giải. 1. Để tập A không chứa bất kỳ tập Ai nào làm tập con, thì lẽ tự nhiên tập A chứa càng ít phần
tử càng tốt.

2. Tuy nhiên đề bài lại yêu cầu |A| ≥ 2n, tức một chặn dưới cho |A|. Nghĩa là đề bài yêu cầu tồn
tại tập A có số lượng phần tử "tương đối" nhiều. Để làm việc với những tập tương đối nhiều
phần tử, thông thường ta làm trên tập có nhiều phần tử nhất. Giả sử A là một tập con của X mà
không chứa bất kỳ một tập Ai nào, với số phần tử lớn nhất. Đặt k = |A|.

• Ý nghĩa như sau: bất kỳ tập con nào của X có số phần tử > k, sẽ đều chứa một tập con Ai
nào đó.
• Do đó ta sẽ khảo cứu những tập vi phạm sinh ra từ A, những tập đó bằng tập A thêm một
phần tử ngoài A, tức thêm vào A một phần tử của tập X\A.

3. Cách 1. Vì |A| chứa k phần tử nên X\A có n − k phần tử.

4. Cách 2. Xét x là một phần tử của X nhưng không thuộc A, x ∈ X\A. Theo tính tối đại của tập A, thì
S
A {x} sẽ không thỏa mãn điều kiện bài toán. Nghĩa là sẽ tồn tại một chỉ số i(x) ∈ {1, 2, . . . , m}
sao cho [
Ai(x) ⊆ A {x}.

Ai(x) x b

A
X\A
Ai(y)y
b

Ths Trần Minh Hiền(0989.541.123) - Trường THPT chuyên Quang Trung 1


TƯ DUY CỰC HẠN TRONG CÁC BÀI TOÁN TỔ HỢP (Chuyên đề tập huấn giáo viên 2017)

S
Vì Ai(x) 6⊂ A và Ai(x) ⊆ A {x} nên x ∈ Ai(x) . Từ đó suy ra

Ai(x) \{x} ⊂ A.

Vì mỗi tập Ai có ba phần tử nên Ai(x) \{x} chỉ có hai phần tử, mà nó lại là tập con của A, dẫn đến
tập \
Lx = A Ai(x)
T
có đúng hai phần tử. Lại theo giả thiết bài toán |Ai A j | ≤ 1 với mọi i 6= j nên các tập Lx đều là
các tập phân biệt (theo nghĩa, hai tập hợp Lx và Ly , ở đây x 6= y thuộc X\A thì Lx 6= Ly . Vì nếu
T
Lx = Ly , giả sử Lx = Ly = {a, b}. Khi đó {a, b} ⊂ Ax , {a, b} ⊂ Ay , dẫn đến Ax Ay = {a, b}, mâu
thuẫn).

5. Từ đó chúng ta đã xác lập được một đơn ánh từ tập X\A đến tập hợp chứa các tập hai phần tử
của A. Do đó theo tính chất đơn ánh thì
  √
k k2 − k 2 −1 + 1 + 8n √
n−k ≤ = =⇒ k + k ≥ 2n =⇒ k ≥ > 2n − 1.
2 2 2
√ √
Vì k nguyên và k > 2n − 1 nên k ≥ [ 2n].

Ví dụ trên cho thấy vai trò của tập chứa số lượng phần tử nhiều nhất, thì ví dụ dưới đây lại cho thấy
vai trò của một phần tử thuộc nhiều tập hợp nhất.

VÍ DỤ 2. Cho 2005 tập hợp, mỗi tập hợp có có 44 phần tử. Biết rằng hai tập hợp bất kỳ đều có đúng
một phần tử chung. Chứng minh rằng tồn tại một phần tử thuộc tất cả 2005 tập hợp đã cho.

Ta cần khẳng định tồn tại một phần tử thuộc tất cả 2005 tập hợp đã cho. Do đó ta sẽ làm việc với
một phần tử x thuộc nhiều tập hợp nhất (vì chỉ có phần tử này mới có cơi hội nhiều nhất thỏa đề bài).
Để làm được điều này, ta phải biết được x thuộc bao nhiêu tập hợp. Nếu x thuộc ít hơn 2005 tập hợp,
bằng lập luận dẫn đến vô lý.
Giải. Ta có 2005 tập hợp, mỗi tập có 44 phần tử nên có tối đa là 2005 × 44 phần tử. Với x là một phần
tử bất kỳ trong chúng, đặt

nx = {số tập hợp trong 2005 tập hợp mà chứa phần tử x} ∈ N.

Vì {nx }x ⊂ N là hữu hạn nên tồn tại phần tử lớn nhất trong chúng, mà ta giả sử luôn là x. Tức x là
phần tử thuộc nhiều tập hợp nhất, gọi các tập hợp này là A1 , A2 , . . . , Ak . Nếu k = 2005 thì bài toán
được chứng minh. Giả sử k < 2005, tức tồn tại một tập hợp B, trong số 2005 tập hợp, không chứa x.
Theo giả thiết bài toán
x1 = B ∩ A1 6= x,
x2 = B ∩ A2 6= x,
dễ thấy x1 6= x2 vì nếu không thì x1 = x2 ∈ A1 ∩ A2 , trong khi đó theo điều kiện bài toán thì A1 ∩ A2 = x,
duy nhất. Tương tự ta có B sẽ chứa các phần tử x1 , x2, . . . , xk . Tuy nhiên B chỉ chứa đúng 44 phần tử nên

Ths Trần Minh Hiền(0989.541.123) - Trường THPT chuyên Quang Trung 2


TƯ DUY CỰC HẠN TRONG CÁC BÀI TOÁN TỔ HỢP (Chuyên đề tập huấn giáo viên 2017)

k ≤ 44. Vì mỗi phần tử của B không thuộc quá 44 tập hợp (vì phần tử x thuộc nhiều tập hợp nhất là k
tập hợp mà k ≤ 44) nên B có giao khác rỗng với nhiều nhất là

442 = 1936 < 2004

tập hợp, mâu thuẫn với giả thiết B có giao khác rỗng với 2004 tập hợp còn lại.

Vậy điều giả sử là sai, tức k = 2005. Bài toán được giải xong.
Ví dụ dưới đây cho thấy tư tưởng, để xây dựng một đối tượng bị chặn dưới, thì ta cần lấy các đối
tượng thỏa mãn càng nhiều tính chất càng tốt.

VÍ DỤ 3. Cho 2014 thùng trái cây, mỗi giỏ trái cây có cả ba loại trái cây Táo, Xoài, Cam. Chứng minh
rằng có thể chọn ra được 1008 thùng trái cây, sao cho tổng số Xoài, tổng số Táo, tổng số Cam trong
1008 thùng trái cây này đều lớn hơn một nửa tổng số Xoài, tổng số Táo, tổng số Cam của 2014 thùng
trái cây ban đầu.

Chứng minh. Đánh số A1 , A2, · · · , A2014 là 2014 thùng trái cây đó. Gọi (ai , bi , ci )lần lượt là số táo, xoài,
cam trong thùng Ai . Chọn ra 2 thùng Ai , A j có số táo và số xoài lớn nhất. Nếu i = j thì ta chọn thùng
Ai và một thùng bất kỳ. Gọi A và B lần lượt là số táo và số xoài lớn nhất trong 2012 thùng còn lại. Ta
sẽ chứng minh trong 2012 thùng còn lại, có thể chia vào 2 nhóm I, II (mỗi nhóm có 1006 thùng) sao
cho 
 ∑ ai − ∑ ai ≤ A


i∈I i∈II

 ∑ bi − ∑ bi ≤ B


i∈I i∈II

Giả sử a1 ≥ a2 ≥ . . . ≥ a2012 , ta xét các cặp Xi = (a2i−1 , a2i ), (i = 1, 2, . . . , 1006). Với mỗi cặp
(
a2i−1 ∈ Xi
a2i ∈ Xi

ta sẽ cho cặp đó vào 2 nhóm nhau. Khi đó với mọi cách chia thì

∑ ai − ∑ ai ≤ a1 + a3 + · · · + a2011 − a2 − a4 − · · · − a2012
i∈I i∈II
= a1 + (a3 − a2 ) + · · · + (a2011 − a2010 ) − a2012 ≤ a1 ;
∑ ai − ∑ ai ≥ a2 + a4 + · · · + a2012 − a1 − a3 − · · · − a2011
i∈I i∈II
= −a1 + (a2 − a3 ) + · · · + (a2010 − a2011) + a2012 ≥ −a1 .

Nên với mọi cách chia thì

∑ ai − ∑ ai ≤ A.
i∈I i∈II

Gọi T là một cách chia như thế. Nếu

∑ bi − ∑ bi ≤B
i∈I i∈II

Ths Trần Minh Hiền(0989.541.123) - Trường THPT chuyên Quang Trung 3


TƯ DUY CỰC HẠN TRONG CÁC BÀI TOÁN TỔ HỢP (Chuyên đề tập huấn giáo viên 2017)

thì chọn cách này. Nếu

∑ bi − ∑ bi >B
i∈I i∈II

không mất tính tổng quát, giả sử

∑ bi > ∑ bi ⇒ ∑ bi − ∑ bi > B.
i∈I i∈II i∈I i∈II

Khi đó tồn tại j ∈ I, k ∈ II sao cho b j > bk . Khi đó ta chỉ việc đổi chỗ (a j , b j ) từ nhóm I sang nhóm II
và (ak , bk ) từ nhóm II sang nhóm I (cách đổi chỗ này không ảnh hưởng đến điều kiện (1)). Khi đó ∑ bi
i∈I
sẽ giảm đi ít nhất 1, còn ∑ bi sẽ tăng lên ít nhất 1. Do đó
i∈I

∑ bi − ∑ bi
i∈I i∈II

sẽ giảm đi ít nhất là 2. Nếu sau khi đổi chỗ mà thỏa điều kiện (2) thì dừng, nếu chưa thỏa thì tiếp tục
làm như trên sẽ đến lúc thỏa vì hiệu trên giảm ngặt trên tập số nguyên dương. Vậy ta luôn chia được
thành 2 nhóm sao cho

∑ ai − ∑ ai ≤A
i∈I i∈II

∑ bi − ∑ bi ≤ B.
i∈I i∈II

Trong hai nhóm I và II, thì tổng số cam trong mỗi nhóm đã được xác định, nên ta sẽ chọn được nhóm
nào số số cam nhiều hơn. Giả sử nhóm I. Khi đó thêm 2 thùng đã lấy ra cho vào nhóm I, thì số cam lấy
ra lớn hơn 1 nửa và
−A ≤ ∑ ai − ∑ ai ≤ A ⇒ ∑ ai ≤ ∑ ai + A,
i∈I i∈II i∈II i∈I

tức số Táo thỏa mãn điều kiện. Tương tự kiểm tra cho số Xoài.
Ví dụ tiếp theo tiếp tục củng cố cho quan điểm: càng nhiều càng tốt.

VÍ DỤ 4. Cho X là một tập hợp hữu hạn, |X| = n và A1 , A2 , . . . , Am là các tập con của X thỏa mãn

|Ai | ≥ 2, ∀i = 1, 2, . . . , n và |Ai ∩ A j | =
6 1, ∀1 ≤ i < j ≤ m.

Chứng minh rằng có thể tô các phần tử của X bằng hai màu, mỗi phần tử tô một màu, sao cho mọi tập
Ai đều chứa các phần tử được tô cả hai màu.

Chứng minh. 1. Giả sử X = {x1 , x2 , . . . , xn }. Giả sử kết luận bài toán sai. Trong tất cả các cách
tô màu mỗi phần tử của tập X, chọn cách tô được một tập con cực đại của X, giả sử Y =
{x1, x2 , . . . , xk }(k < n) thỏa mãn bài toán theo nghĩa: nếu thêm phần tử xk+1 vào Y thì không thể
tô màu cho xk+1 được để thỏa mãn bài toán.

2. Điều đó dẫn đến sẽ có hai tập, giả sử là Ar , As chứa xk+1 và Ar , As ⊆ {x1 , x2 , . . . , xk , xk+1 } sao cho

Ths Trần Minh Hiền(0989.541.123) - Trường THPT chuyên Quang Trung 4


TƯ DUY CỰC HẠN TRONG CÁC BÀI TOÁN TỔ HỢP (Chuyên đề tập huấn giáo viên 2017)

• Nếu xk+1 được tô màu xanh thì Ar ∩Y gồm toàn các phần tử màu xanh;
• Còn nếu xk+1 được tô màu đỏ thì As ∩Y gồm toàn các phần tử màu đỏ.

Nhưng khi đó thì Ar ∩ As = {xk+1 }, mâu thuẫn với giải thiết bài toán. Vậy điều giả sử là sai. Bài
toán được chứng minh.

Bây giờ ta qua các ví dụ để giải bài toán, chúng ta phải mã hóa các bài toán thành các tập hợp
A, B,C, . . .. Rồi đi đếm các tính chất nội bộ trong A, và đếm các tính chất có thể có từ A đến B.

VÍ DỤ 5. Một quốc gia có 210 thành phố. Ban đầu giữa các thành phố chưa có đường đi. Người ta một
xây dựng một số con đường một chiều nỗi giữa các thành phố sao cho: Nếu có đường đi từ A đến B và
có đường đi từ B đến C thì không có đường đi từ A đến C. Hỏi có thể xây dựng được nhiều nhất bao
nhiêu đường đi trong thành phố.

Giải. 1. Do đề bài đang cần tìm số lượng đường đi nhiều nhất. Do đó ta sẽ tập trung vào thành
phố nào có nhiều đường đi nhất. Gọi A là thành phố có nhiều đường đi nhất (gồm cả đường đi
xuất phát từ A và đường đi đến A). Khi đó các thành phố còn lại sẽ chia làm ba loại:

• Loại I: Gồm các thành phố có đường đi xuất phát từ A;


• Loại II: Gồm các thành phố có đường đi đến A;
• Loại III: Gồm các thành phố không có đường đi xuất phát từ A cũng như không có đường
đi đến A.

I
B b
E
C
b b

F
K
b

b
b

b II
b
b
L
b
b
b

b b

III
b

X
b
b

2. Bây giờ ta đi đếm số đường đi nhiều nhất có thể có giữa các loại thành phố này. Đặt m = |I|, n =
|II|, p = |III|. Khi đó m + n + p = 209

• Số đường đi liên quan đến A (xuất phát từ A và đến A) là m + n;


• Các thành phố trong I không có đường đi nào cả (giả sử có đường đi nối B → C, thì vi phạm
vì có A → B, B → C mà có luôn A → C);

Ths Trần Minh Hiền(0989.541.123) - Trường THPT chuyên Quang Trung 5


TƯ DUY CỰC HẠN TRONG CÁC BÀI TOÁN TỔ HỢP (Chuyên đề tập huấn giáo viên 2017)

• Các thành phố trong II cũng không có đường đi nào cả (giả sử có đường đi nối E → F, thì
vi phạm vì có A → E, E → F mà có luôn F → A);
• Các đường đi liên quan đến III, một thành phố X trong III sẽ gồm các đường xuất phát,
và điểm đến giữa nội bộ trong III, và gồm các đường đi giữa I và III, giữa II và III. Tuy
nhiên A là thành phố có nhiều đường đi nhất là m + n. Do đó mỗi thành phố X trong III sẽ
có đường đi (đến và xuất phát) không vượt quá A, tức không vượt quá m + n. Do đó tổng số
đường đi của các thành phố X trong III là ≤ p(m + n).
• Giữa các thành phố I và II có thể xây dựng đường một chiều từ I đến II (như đường K → L).
Do đó số đường đi này không vượt quá m.n.
Vậy tổng số đường đi giữa các thành phố không vượt quá

(m + n + p + 1)2 2102
p(m + n) + m + n + mn = m(p + 1) + mn + n(p + 1) ≤ = .
3 3
Dấu bằng xảy ra với đồ thị 3 phe, mỗi phe có 70 thành phố, thành phố phe I có đường đi đến thành phố
phe II, thành phố phe II có đường đi đến thành phố phe III, thành phố phe III có đường đi đến thành
phố phe I.
Một ví dụ tương tự ví dụ trên, với việc thiết lập đếm các đối tượng giữa các tập hợp.

VÍ DỤ 6. Có 3 trường học, mỗi trường học đều có n học sinh. Biết rằng với mỗi học sinh bất kỳ đều
quen biết với n + 1 học sinh khác trường. Chứng minh rằng luôn có thể tìm được ba học sinh, mỗi học
sinh một trường, mà quen biết với nhau từng đôi một.

Giải. Có tổng cộng 3n học sinh trong ba trường học. Đặt tên ba trường học là X,Y, Z. Với một học
sinh S bất kỳ của trường X, thì S quen biết với n + 1 học sinh thuộc vào hai trường Y và Z. Tương tự
cho mối quan hệ của các học sinh của trường Y và Z.

Gọi A là học sinh có nhiều bạn nhất ở một trường khác. Gọi số bạn quen biết với A là k. Thì rõ
ràng k ≤ n. Không mất tính tổng quát, có thể giả sử A thuộc trường X, và A quen biết với k bạn học
sinh trường Y . Khi đó số học sinh ở trường Z mà quen với A là:

n + 1 − k ≥ 1.

Điều này có nghĩa là A quen biết với ít nhất một học sinh, chẳng hạn học sinh C, ở trường Z.

Học sinh C của trường Z này sẽ quen biết với một số học sinh của trường Y . Nếu có học sinh B của
trường Y quen biết với C và cũng quen với A thì ba học sinh A, B,C thuộc ba trường X,Y, Z đôi một
quen nhau, thỏa mãn yêu cầu bài toán.

Còn trong tình huống ngược lại, tức là trong số học sinh quen biết với C ở trường Y , không có học
sinh nào quen biết với A. Thì vì A quen với k học sinh của trường Y , do đó số học sinh của trường Y
mà quen biết với C và không quen với A lớn nhất là n − k. Do đó số học sinh trong trường X mà quen
biết với C ít nhất là n + 1 − (n − k) = k + 1 > k. Mâu thuẫn với tính chất k là số học sinh lớn nhất của
một trường quen biết với A. Vậy chứng tỏ luôn có một học sinh trường Y quen biết cả C và A. Ta có
điều phải chứng minh.

Ths Trần Minh Hiền(0989.541.123) - Trường THPT chuyên Quang Trung 6


TƯ DUY CỰC HẠN TRONG CÁC BÀI TOÁN TỔ HỢP (Chuyên đề tập huấn giáo viên 2017)

b
Y1 n−k
b

b
b

X b

b
b
b b
Y
b

Yk
b

b
b
A b
k
k+1 b
b

C b

D
Z

Ta có thể minh họa chứng minh trên bằng hình ảnh trên. Nếu dùng công thức PIE ta có thể trình bày
khác đi một chút như sau:
Gọi A là học sinh có nhiều bạn nhất ở một trường khác. Gọi số bạn nhiều nhất này là k. Giả sử A ở
trường X và tập những bạn quen A là M = {Y1 ,Y2, . . . ,Yk } ở trường Y . Theo giả thiết có ít nhất một học
sinh C ở trường Z quen với A. Vì C quen không quá k học sinh ở trường X nên theo giả thiết C quen
với ít nhất n + 1 − k học sinh ở trường Y , Đặt N = {Y1′ ,Y2′ , . . . ,Ym′ } là những người quen với C ở trường
Y thì m ≥ n + 1 − k. Vì M, N đều là tập con của tập chứa n phần tử và

|M| + |N| ≥ k + n + 1 − k = n + 1
T T
nên ta có M / Khi đó chọn học sinh B ∈ M
N 6= 0. N thì ba học sinh A, B,C là ba học sinh cần chọn.

Ví dụ dưới đây tiếp tục cho thấy tính hiệu quả của việc đếm các quan hệ giữa các tập với nhau và
bản thân nội bộ các tập.

VÍ DỤ 7. Trong quốc hội Mỹ, mỗi một nghị sĩ có không quá 3 kẻ thù. Chứng minh rằng có thể chia
quốc hội thành 2 viện sao cho trong mỗi viện, mỗi một nghị sĩ có không quá một kẻ thù.

Giải. Ta chia quốc hội ra thành hai viện A, B một cách bất kỳ. Với mỗi viện A, B, ta gọi s(A), s(B) là
tổng của tổng số các kẻ thù của mỗi thành viên tính trong viện đó. Vì số cách chia là hữu hạn nên phải
tồn tại cách chi (A0, B0 ) sao cho s(A0 ) + s(B0) nhỏ nhất. Ta chứng minh cách chia này thỏa mãn yêu
cầu bài toán.

Giả sử cách chia này vẫn chưa thỏa mãn yêu cầu, tức là vẫn có một nghị sĩ nào đó có nhiều hơn 1
kẻ thù trong viện của mình. Không mất tính tổng quát, giả sử nghị sĩ X thuộc A0 có ít nhất 2 kẻ thù
trong A0 . Khi đó ta thực hiện phép biến đổi sau: chuyển x từ A0 sang B0 để được cách chia mới là
[
A′ = A0 \{X}, B′ = B0 {X}.

Vì X có ít nhất 2 kẻ thù trong A0 và A′ không còn chứa X nên ta có

s(A′ ) ≤ s(A0 ) − 4

Ths Trần Minh Hiền(0989.541.123) - Trường THPT chuyên Quang Trung 7


TƯ DUY CỰC HẠN TRONG CÁC BÀI TOÁN TỔ HỢP (Chuyên đề tập huấn giáo viên 2017)

(trong tổng mất đi ít nhất 2 của s(X) và 2 của các kẻ thù của x trong A0 ).

Vì x có không quá 3 kẻ thù và có ít nhất 2 kẻ thù trong A0 nên x có nhiều nhất 1 kẻ thù trong B0
(hay trong B′ ), cho nên
s(B′ ) ≤ s(B0 ) + 2.
Từ đó
s(A′ ) + s(B′ ) ≤ s(A0 ) + S(B0) − 2.
Mâu thuẫn với tính nhỏ nhất của s(A0) + s(B0 ). Vậy điều giả sử là sai, tức là cách chia (A0 , B0 ) thỏa
mãn yêu cầu bài toán.
Các ví dụ tiếp theo minh họa cho ý tưởng: thay vì xem xét toàn bộ các phần tử trong tập hữu hạn
A, hãy chú ý đến phần tử nhỏ nhất và phần tử lớn nhất.

VÍ DỤ 8. Đặt các số nguyên 1, 2, . . . , n2 vào một bàn cờ kích thước n × n (n ≥ 2) một cách ngẫu nhiên,
mỗi số đúng một lần, mỗi ô một số. Chứng minh rằng tồn tại hai ô vuông kề nhau (là hai ô vuông
chung nhau một cạnh hoặc chung nhau một đỉnh) mà hai số trên đó là a > b thì a − b ≥ n + 1.

Chứng minh. Nhận xét: Với 2 ô bất kỳ, luôn tồn tại một dãy không quá n ô vuông kề nhau nối chúng
(dĩ nhiên tính luôn cả hai ô vuông đó). Giả sử phản chứng, mọi ô vuông kề nhau có hiệu ≤ n. Xét hai
ô vuông chứa số 1 và n2 (Đây là hai phần tử cực biên của dãy).

• Vì có một dãy không quá n ô vuông kề nhau nối hai ô 1 và n2 , mà hai ô kề nhau sai khác ≤ n.
Do đó tổng sự sai khác của dãy này

≤ |n + n +
{z· · · + n} = n(n − 1).
n−1 lần

• Nhưng hai ô 1 và n2 sai khác là n2 − 1.

Từ hai điều trên suy ra n2 − 1 ≤ n(n − 1) ⇒ n ≤ 1, mâu thuẫn. Vậy điều giả sử là sai. Bài toán được
chứng minh.
Nếu yêu cầu hai ô vuông kề nhau chỉ được chung nhau một cạnh, thì ta cũng có một kết quả tương
tự, nhưng yếu hơn.

VÍ DỤ 9 (IMO SHORTLIST 1988). Các số 1, 2, . . . , n2 được viết trên các ô vuông trong bảng ô
vuông n × n, mỗi số được viết đúng một lần. Chứng minh rằng có hai ô vuông kề nhau mà hai số viết
trên đó có hiệu ≥ n.

Chứng minh. Ta chứng minh bằng phản chứng, tồn tại một cách đánh số sao cho mọi số ở hai ô kề
nhau (là hai ô vuông chung cạnh) đều có hiệu ≤ n − 1, Với mỗi số nguyên dương k ≥ 1, đặt

Sk = {1, 2, . . . , k},
Nk = {k + 1, k + 2, . . . , k + n − 1},
Tk = {k + n, k + n + 1, . . ., n2 }.

Khi đó

Ths Trần Minh Hiền(0989.541.123) - Trường THPT chuyên Quang Trung 8


TƯ DUY CỰC HẠN TRONG CÁC BÀI TOÁN TỔ HỢP (Chuyên đề tập huấn giáo viên 2017)

• Mỗi số trong Nk đều có thể kề (hai số này nằm ở hai ô vuông kề nhau trong bảng ô vuông) với
một số trong Sk và không có số nào trong Sk được phép kề với một số trong Tk , vì hiệu của
hai số bất kỳ, một số trong Sk , một số trong Tk đều có hiệu ≥ n, mâu thuẫn. Khi đó Sk , Tk là
hai nhóm như đã trình bày ở trên.

• Với mỗi k, thì |Nk | = n − 1, mà bảng ô vuông gồm n dòng, n cột. Do đó tồn tại một dòng và tồn
tại một cột mà trong dòng đó, cột đó không chứa bất kỳ phần tử nào của Nk . Hợp của dòng
đó và cột đó ta ký hiệu là một chữ thập +k . Lưu ý các phần tử trong +k chỉ chứa hoặc là toàn
các phần tử của Sk , hoặc là toàn chứa các phần tử của Tk , vì nếu chứa cả hai loại số trong hai
tập, thì sẽ vi phạm với ý trên.

• Với k = 1, khi đó chữ thập +1 (chứa 2n − 1 phần tử) không thể là các phần tử trong tập S1 (vì S1
chỉ có một phần tử). Do đó với k = 1 thì chữ thập +1 gồm các phần tử thuộc vào T1 .

• Tương tự, với k = n2 − n, thì chữ thập +n2 −n gồm các phần tử thuộc vào tập Sn2 −n .

• Với mỗi k = 1, 2, . . . , n2 − n, thì mỗi phần tử trong chữ thập +k hoặc thuộc vào loại Sk hoặc Nk . Vì
chữ thập +1 thuộc vào T1 , và chữ thập +n2 −n thuộc vào Sn2 −n . Do đó sẽ tồn tại j : 1 ≤ j < n2 − n
sao cho: + j gồm các phần tử thuộc T j , và + j+1 sẽ thuộc vào một tập S j+1 . Nhưng hai chữ
thập + j và + j+1 luôn giao nhau tại hai ô vuông, dẫn đến hai số trên hai ô vuông này vừa
thuộc vào T j vừa thuộc vào S j+1 . Điều này vô lý vì

T j ∩ S j+1 = 0.
/

Từ đó giả thiết phản chứng sai, bài toán được chứng minh.
Tiếp theo ta sẽ nghiên cứu hai ví dụ, cho thấy được việc nghiên cứu các phần tử biên sẽ giúp định
hình cấu trúc của tập hợp.

VÍ DỤ 10. Giả sử a1 , a2 , . . . , an là các số nguyên với ước chung lớn nhất của chúng bằng 1. Giả sử S là
tập các số nguyên thỏa mãn:

1. Với mỗi i = 1, 2, . . . , n thì ai ∈ S;

2. Với i, j = 1, 2, . . . , n (không nhất thiết phân biệt) thì ai − a j ∈ S;

Ths Trần Minh Hiền(0989.541.123) - Trường THPT chuyên Quang Trung 9


TƯ DUY CỰC HẠN TRONG CÁC BÀI TOÁN TỔ HỢP (Chuyên đề tập huấn giáo viên 2017)

3. Với hai số nguyên x, y ∈ S tùy ý, nếu x + y ∈ S thì x − y ∈ S.


Chứng minh S = Z.
Chứng minh. 1. Với các số nguyên a1 , a2 , . . . , an ∈ Z với ước chung lớn nhất của chúng bằng 1, ta
nói S sinh bởi a1 , a2 , . . . , an nếu các điều kiện 1, 2, 3 đều thỏa mãn.

2. 0 = a1 − a1 ∈ S theo điều kiện 2;

3. −s = 0 − s, với mọi s ∈ S bởi hai điều kiện 2 và kết quả trên.

4. Nếu S sinh bởi a1 , a2 , . . . , an thì S cũng được sinh bởi a1 , a2 − a1 , . . . , an − a1 .

• Điều kiện 3 không liên quan đến tập sinh. Do đó ta kiểm tra hai điều kiện 1 và 2. Ta có
as − a1 ∈ S với mọi s > 1 bởi điều kiện 2;
• Hiệu a1 − (as − a1 ) thuộc S với mọi s > 1 (vì a1 ∈ S, as − a1 ∈ S mà a1 + (as − a1 ) = as ∈ S),
do đó theo điều kiện 3 ta có. Các hiệu khác đều có dạng

(as − a1 ) − (at − a1 ) = as − at

cũng nằm trong S với mọi s,t > 1.

5. Nếu S sinh bởi a1 , a2 , . . . , an thì S cũng sinh bởi −a1 , a2 , . . . , an (kiểm tra tương tự).

6. Bây giờ giả sử S sinh bởi a1 , a2 , . . . , an (không mất tính tổng quát, giả sử tất cả các ai đều khác 0.
Vì số 0 luôn được tạo thành từ hiệu của hai số trong S). Theo nhận xét trên, thì có thể giả sử tất
cả các ai > 0, với mọi i. Cho các số nguyên b1 , b2 , . . . , bk > 0, với ước chung lớn nhất bằng 1,
sinh ra tập S và b1 + b2 + · · · + bk nhỏ nhất. Chú ý các b1 , b2 , . . . , bk phải phân biệt (nếu không,
chúng ta có thể bỏ đi phần tử trùng nhau ra khỏi tập sinh), và không mất tính tổng quát, ta có thể
giả sử b1 là số nhỏ nhất trong tất cả các số trên.

7. Giả sử k > 1, đặt c1 = b1 , và cs = bs −b1, với s = 2, . . . , k. Khi đó (c1 , c2, . . . , ck ) = (b1 , . . . , bk ) = 1


bà S sinh bởi c1 , c2 , . . . , ck . Tuy nhiên

c1 + c2 + · · · + ck = (b1 + b2 + · · · + bk ) − (k − 1)b1 < b1 + b2 + · · · + bk ,

mâu thuẫn với tính nhỏ nhất của b1 + · · · + bk . Vậy k = 1 và b1 = 1 (vì nếu b1 > 1 thì ước chung
lớn nhất của nó sẽ là b1 > 1).

8. Vấn đề còn lại là kiểm tra S sinh bởi 1, thì S = Z. Ta chứng tỏ 0, 1, 2, . . . , k thuộc S với mọi số
nguyên dương k bằng quy nạp. Chú ý −1, 0, 1 ∈ S. Khi đó k = 1 đúng, giả sử 0, 1, 2, . . . , k ∈ S,
khi đó
k + 1 = k − (−1) ∈ S.
Vậy S chứa tất cả các số nguyên dương. Khi đó S cũng chứa luôn tất cả các các nguyên âm.

Tiếp theo là một ví dụ cho thấy vai trò của phần tử nhỏ nhất trong việc định hình cấu trúc tập hợp.
VÍ DỤ 11.√Tìm tất cả các tập con khác rỗng A của tập {2, 3, . . .} sao cho với mọi n ∈ A, thì cả hai số
n2 + 4 và [ n] + 1 đều thuộc vào A.

Ths Trần Minh Hiền(0989.541.123) - Trường THPT chuyên Quang Trung 10


TƯ DUY CỰC HẠN TRONG CÁC BÀI TOÁN TỔ HỢP (Chuyên đề tập huấn giáo viên 2017)

Phân tích ý tưởng giải. 1. Làm việc trên những kiểu bài toán này, hãy chú ý phần tử nhỏ nhất trong
A? Phần tử nhỏ √ nhất trong A bằng bao nhiêu? Gọi m là phần tử nhỏ nhất trong A. Khi đó theo
giả thiết thì [ m] + 1 ∈ A. Vì m là phần tử nhỏ nhất trong A nên
√ √
m ≤ [ m] + 1 ≤ m + 1 ⇒ m = 2.

2. Liên hệ gữa hàm phần nguyên với số n2 + 4? Ta có [ n2 + 4] = n, ∀n ≥ 2 (vì n2 ≤ n2 + 4 <
(n + 1)2). Đến đây ta có, nếu n ∈ A thì
p
2
n + 4 ∈ A ⇒ [ n2 + 4] + 1 ∈ A hay n + 1 ∈ A.

Do đó theo giả thiết ta có A = {2, 3, . . .}.

Và cuối cùng là khi đánh giá các bất đẳng thức với yếu tố rời rạc, thì việc dựa vào các phần tử biên
sẽ cho ta nhiều thông tin để ước lượng nhất.

VÍ DỤ 12. Một chiếc xe đua cần 150 lít xăng để hoàn thành một chuyến đi vòng quanh hồ Balaton.
Người ta đặt 5 can xăng, mỗi can chứa 30 lít ở 5 vị trí nào đó trên đường. Chứng minh rằng ta có thể
chọn một vị trí trên đường đua để nếu xuất phát từ vị trí đó, chiếc xe đua ban đầu chưa có xăng, sẽ có
thể đổ xăng (từ các can xăng nói trên) và hoàn thành chuyến đi vòng quanh hồ (Dĩ nhiên, xe sẽ bắt đầu
từ 1 trong 5 vị trí đặt can xăng).
a
b

b
e

b
b

c d
Chứng minh.
b

Giả sử vị trí của các trạm xăng được mô tả như hình vẽ, trong đó a, b, c, d, e là lượng xăng cần sử
dụng để di chuyển giữa hai trạm xăng tương ứng. Việc tìm được vị trí trạm xăng nào thì phải thỏa bốn
bất đẳng thức. Ta có a + b + c + d + e = 150. Không mất tính tổng quát, ta giả sử a là số nhỏ nhất. Khi
đó 5a ≤ a + b + c + d + e = 150 ⇒ a ≤ 30. Ta xét 2 trường hợp

1. Nếu a + b > 60 và a + e > 60 thì

a + b + e = (a + b) + (a + e) − a > 60 + 60 − 30 = 90.

Suy ra
c + d = 150 − (a + b + e) < 150 − 90 = 60.
Đến đây, nếu c + d + b > 90 hoặc c + d + e > 90 thì vô lí vì

a + b + c + d + e = (c + d + b) + (a + e) > 150, a + b + c + d + e = (c + d + e) + (a + b) > 150.

Ths Trần Minh Hiền(0989.541.123) - Trường THPT chuyên Quang Trung 11


TƯ DUY CỰC HẠN TRONG CÁC BÀI TOÁN TỔ HỢP (Chuyên đề tập huấn giáo viên 2017)

Do đó c + d + b ≤ 90 (trường hợp c + d + e ≤ 90 chứng minh tương tự) thì do a ≤ 30 nên ta có


a + b + c + d ≤ 120. Lại vì c + d < 60 nên phải có một số trong hai số c hoặc d nhỏ hơn 30. Giả
sử là c. Trong trường hợp này ta chọn được đường đi với lượng xăng cần dùng như sau:

cdbae, c < 30, c + d < 60, c + d + b ≤ 90, c + d + b + a ≤ 120.

2. Nếu a + b ≤ 60 (trường hợp a + e ≤ 60 chứng minh tương tự), ta cũng có 2 trường hợp

(a) Nếu a + b + c > 90 và a + b + e > 90 thì c + d < 60, d + e < 60. Ta có a + e ≤ d + e < 60 và
a + e + d < 30 + 60 = 90 và a + e + c + d < 120 nên trong trường hợp này, ta có thể chọn
được vị trí đi

aedcb, a ≤ 30, a + e < 60, a + e + d < 90, a + e + c + d < 120.

(b) Nếu a + b + c ≤ 90 (trường hợp a + b + e ≤ 90 chứng minh tương tự) thì rõ ràng phải có
a + b + c + d ≤ 120 hoặc a + b + c + e ≤ 120 vì nếu không, cả 2 bất đẳng thức này đều sai
thì
a + b + c + d + e = (a + b + c + d) + (a + b + c + e) − (a + b + c) > 150,
mâu thuẫn. Do đó, trường hợp này cũng có thể chọn được vị trí thỏa mãn. Ví dụ

abcde, a ≤ 30, a + b < 60, a + b + c ≤ 90, a + b + c + d ≤ 120.

LUYỆN TẬP

Dưới đây là các bài tập luyện tập với các tư tưởng giống hoàn toàn các ví dụ ở trên.

BÀI TẬP 1 (CROATIAN TST 2011). Có n người tham gia một buổi tiệc, trong đó có một số người
quen biết nhau (nếu X quen Y thì Y cũng quen X). Biết rằng với 4 người tùy ý, luôn có 3 người hoặc
quen biết với nhau đôi một hoặc không quen biết với nhau đôi một. Chứng minh rằng có thể chia n
người này vào hai nhóm A, B sao cho mọi người trong nhóm A đều quen biết với những người còn lại
trong nhóm, còn mọi người trong nhóm B đều không quen biết với mọi người còn lại trong nhóm.

BÀI TẬP 2 (APMO 2008). Các học sinh trong một lớp học được chia thành các nhóm, mỗi nhóm có
3 học sinh và hai nhóm phân biệt có chung nhau nhiều nhất 1 học sinh. Chứng minh rằng nếu lớp học
đó có 46 học sinh, thì tồn tại một tập gồm 10 học sinh mà trong tập này không chứa một nhóm nào.

BÀI TẬP 3. Chứng minh rằng trong một grapg G có V là tập n đỉnh đều có thể chia thành hai tập đỉnh
V1 ,V2 sao cho với mỗi đỉnh trong V1 đều có số cạnh nối đến tập V2 nhiều hơn số cạnh nối đỉnh đó trong
V1 , và tương tự với mỗi đỉnh trong V2 đều có số cạnh nối đến tập V1 nhiều hơn số cạnh nối đỉnh đó
trong V2.

BÀI TẬP 4 (INDIAN TST 2003). Cho n là số nguyên dương. Phân hoạch tập {1, 2, . . . , 3n} thành ba
tập A, B,C với |A| = |B| = |C| = n. Chứng minh rằng tồn tại x ∈ A, y ∈ B, z ∈ C để một trong ba số x, y, z
là tổng của hai số còn lại.

Ths Trần Minh Hiền(0989.541.123) - Trường THPT chuyên Quang Trung 12


TƯ DUY CỰC HẠN TRONG CÁC BÀI TOÁN TỔ HỢP (Chuyên đề tập huấn giáo viên 2017)

BÀI TẬP 5 (FRANCE 1997). Tại mỗi đỉnh của đa giác đều 1997 cạnh người ta viết một số nguyên
sao cho tổng của tất cả các số nguyên đó là số dương. Bắt đầu từ một đỉnh tùy ý, chúng ta lần lượt viết
xuống bảng các số trên các đỉnh theo chiều ngược kim đồng hồ. Hỏi có thể chọn được một đỉnh nào đó,
sao cho tổng của k số nguyên dương được viết xuống bảng là số nguyên dương với k = 1, 2, . . . , 1997
hay không?

BÀI TẬP 6. Cho a1 , a2 , . . . , là dãy các số nguyên gồm vô hạn các số nguyên dương và và vô hạn các
số nguyên âm. Với mỗi số nguyên dương n, phần dư của các số a1 , a2 , . . . , an khi chia cho n đều khác
nhau. Chứng minh rằng mỗi số nguyên dương xuất hiện đúng một lần trong dãy.

BÀI TẬP 7 (CHINA TST 2010). Cho trước số nguyên a1 ≥ 2 và với mọi n ≥ 2 thì an là số nguyên
dương nhỏ nhất không nguyên tố cùng nhau với an−1 và không bằng với a1 , a2 , . . . , an−1 . Chứng minh
rằng ngoại trừ số 1, tất cả các số nguyên đều xuất hiện trong dãy (an ).

BÀI TẬP 8 (CHINA 2009). Cho M là tập con của R xác định bởi tập R bỏ đi một số hữu hạn số.
Chứng minh rằng với mỗi số nguyên dương n bất kỳ, tồn tại một đa thức P(x) bậc n sao cho tất cả các
hệ số và có n nghiệm thực đều nằm trong M.

BÀI TẬP 9 (BULGARIA 2002). Với mỗi số nguyên dương n ≥ 3, cho a1 , a2 , . . . , an là n số thực
phân biệt có tổng là số dương. Một hoán vị (b1 , b2 , . . . , bn ) của n số này được gọi là "tốt" nếu với mọi
k = 1, 2, . . . , n thì b1 + b2 + · · · + bk > 0. Hỏi số hoán vị tốt nhỏ nhất bằng bao nhiêu?

BÀI TẬP 10. Cho n là số nguyên dương và x1 , x2 , . . . , xn là các số không âm có tổng x1 + x2 + · · ·+ xn =


1. Tìm giá trị nhỏ nhất của
F = ∑ xi x j (xi + x j ).
1≤i< j≤n

Ths Trần Minh Hiền(0989.541.123) - Trường THPT chuyên Quang Trung 13


Phương pháp giải toán dựa trên nguyên lý cực hạn
qua một số ví dụ chọn lọc
Nguyễn Văn Phương
Trường THPT chuyên Trần Đại Nghĩa, TP.HCM
nguyenvanphuong88@gmail.com

Nguyên lý cực hạn dựa trên các kết quả cơ bản sau:
- Một tập hợp hữu hạn khác tập rỗng các số thực luôn có phần tử lớn nhất và phần tử
nhỏ nhất.
- Một tập hợp khác tập rỗng các số nguyên không âm luôn có phần tử nhỏ nhất.
Chuyên đề này bàn về cách vận dụng nguyên lý cực hạn trong hai nhóm bài toán chứng minh
sự tồn tại/ không tồn tại và chứng minh bất đẳng thức.
Xét bài toán cần chứng minh sự tồn tại/ không tồn tại của một đối tượng thỏa một tính chất cụ
thể nào đó. Nguyên lý cực hạn gợi ý chúng ta quan tâm các đối tượng có một đặc trưng ở
trạng thái “nhỏ nhất” hoặc “lớn nhất” theo một nghĩa nào đó, ta gọi là phần tử cực biên. Dựa
trên sự tồn tại của phần tử cực biên này, chúng ta lập luận để khẳng định/ bác bỏ sự tồn tại
của đối tượng được yêu cầu. Nguyên lý cực hạn thường được vận dụng như là một bước khởi
đầu và có thể xem như là một bước xử lý kỹ thuật “tinh tế” khi sử dụng phương pháp phản
chứng hoặc phương pháp lùi vô hạn.
Chúng ta xét ba ví dụ mở đầu.
Ví dụ 1. Trong một giải đấu cờ có n ( n  3 ) kì thủ tham gia, mỗi cặp kì thù đấu với nhau một
ván và không có trận nào kết thúc với kết quả hòa. Chứng minh rằng có một kì thủ A mà với
kì thủ B bất kì thì hoặc A đánh bại B, hoặc A đánh bại kì thủ C nào đó và C thì đánh bại B.
Lời giải. Gọi A là kì thủ có số trận thắng nhiều nhất trong tất cả các kì thủ (nguyên lý cực
hạn). Ta sẽ chứng minh A là kì thủ thỏa yêu cầu bài toán. Thật vậy, nếu A không thỏa yêu
cầu thì tồn tại một kì thủ B đánh thắng A và tất cả các kì thủ C đã thua A, suy ra B có số trận
thắng nhiều hơn A, điều này mâu thuẫn với cách chọn A là kì thủ có số trận thắng nhiều nhất.

Ví dụ 2. Chứng minh rằng bốn hình tròn có các đường kính là các cạnh của một tứ giác lồi
thì phủ kín tứ giác đó.

1
Lời giải. Lấy một điểm M bất kì thuộc miền trong của tứ giác lồi ABCD, ta cần chứng minh
M thuộc ít nhất một trong bốn hình tròn có đường kính là các đoạn thẳng AB, BC , CD, DA .
Không mất tính tổng quát, giả sử  AMB là góc có số đo lớn nhất trong các góc
   
AMB, BMC , CMD, DMA (nguyên lý cực hạn). Do    CMD
AMB  BMC   DMA
  360

nên 
AMB  90 , suy ra M thuộc hình tròn đường kính AB. ■

Ví dụ 3. Chứng minh rằng phương trình x 3  2 y 3  4 z 3 không có nghiệm nguyên nào khác
nghiệm x  y  z  0 .

Lời giải. Giả sử phương trình có nghiệm nguyên  x, y , z    0, 0, 0 . Trong các nghiệm này ta
gọi  x0 , y0 , z0  là nghiệm mà x0  y0  z0 có giá trị nhỏ nhất (nguyên lý cực hạn).

Ta có x0 3  2 y0 3  4 z0 3 suy ra x0 chia hết cho 2. Đặt x0  2 x1 .

Ta được 8 x13  2 y0 3  4 z0 3 tức 4 x13  y0 3  2 z0 3 , suy ra y0 chia hết cho 2. Đặt y0  2 y1 .

Ta được 4 x13  8 y13  2 z0 3 , tức 2 x13  4 y13  z0 3 , suy ra z0 chia hết cho 2. Đặt z0  2 z1 .

Ta được 2 x13  4 y13  8 z13 , tức x13  2 y13  4 z13 , suy ra  x1 , y1 , z1  cũng là nghiệm của
1
phương trình đã cho. Nhưng do x1  y1  z1 
2
 x0  y0  z0   x0  y0  z0 , mâu thuẫn
với cách chọn  x0 , y0 , z0  . Vậy phương trình không có nghiệm nguyên khác x  y  z  0 . ■

Trong các ví dụ trên, chúng ta vận dụng nguyên lý cực hạn để vừa có thêm thông tin, vừa
giúp lập luận ngắn gọn hơn. Nguyên lý cực hạn được vận dụng trong nhiều bài toán khác
nhau, đặc biệt trong các bài toán yêu cầu chứng minh sự tồn tại/ không tồn tại và một số bài
toán chứng minh bất đẳng thức.
Nguyên lý cực hạn và bài toán chứng minh sự tồn tại/ không tồn tại
Đối với các bài toán đã chỉ rõ các đối tượng cần quan tâm và yêu cầu chứng minh trong số
này tồn tại một đối tượng sở hữu một tính chất cụ thể nào đó, ta nên chú ý đến các đối tượng
được xây dựng từ các phần tử cực biên của giả thiết. Các phần tử cực biên có thể là đoạn
thẳng ngắn nhất/ dài nhất, góc nhỏ nhất/ lớn nhất, hình có diện tích nhỏ nhất/ lớn nhất,
đường gấp khúc có độ dài ngắn nhất/ dài nhất, điểm ở vị trí gần nhất/ xa nhất, ước (nguyên
tố) nhỏ nhất/ lớn nhất, phương án tốt nhất/ xấu nhất,..
Ví dụ 4. Trên mặt phẳng cho n ( n  3 ) điểm không cùng nằm trên một đường thẳng. Chứng
minh rằng tồn tại một đường tròn đi qua ba trong số n điểm sao cho miền trong của đường
tròn này không chứa bất kì điểm nào trong số những điểm còn lại.
Phân tích. Đối tượng của bài toán là các đường tròn đi qua ba điểm trong số n điểm đã cho.
Số đường tròn này là hữu hạn do số điểm là hữu hạn. Ta sẽ xác định đường tròn thỏa yêu cầu
dựa trên nhận xét rằng nếu một điểm X thuộc miền trong của đường tròn đi qua ba điểm A, B,
C thì 
AXB  min  ACB , 180  
 ACB . 

2
Lời giải. Gọi AB là đoạn thẳng có độ dài ngắn nhất trong tất cả các đoạn thẳng tạo bởi n
điểm đã cho và C là điểm mà ACB là góc lớn nhất trong tất cả các góc 
AXB với X là một
trong n  2 điểm khác A và B (nguyên lý cực hạn). Ta sẽ chứng minh đường tròn ngoại tiếp
tam giác ABC thỏa yêu cầu bài toán.

Gọi  là số đo cung AB nhỏ của đường tròn ABC thì   180 . Tam giác ABC có cạnh AB là

cạnh nhỏ nhất nên suy ra 
ACB  60 , suy ra 
ACB  (1).
2
Giả sử tồn tại điểm D thuộc miền trong của đường tròn ABC. Do AB là đoạn thẳng có độ dài
ngắn nhất nên D không thuộc đoạn thẳng Nếu D nằm cùng phía với C so với AB, gọi E là

giao của AD và đường tròn ABC thì  ADB  AEB  ; nếu D nằm khác phía với C so với AB
2
 
thì 
ADB  180  ACB  180   (2).
2 2

Từ (1) và (2) suy ra mâu thuẫn với cách chọn ACB là góc lớn nhất. Vậy miền trong của
đường tròn ABC không chứa bất kì điểm nào trong số những điểm còn lại. ■
Ví dụ 5 (Hàn Quốc 1995). Cho n ( n  3 ) điểm trên mặt phẳng sao cho bất kì ba điểm nào
cũng tạo thành một tam giác có diện tích không vượt quá 1. Chứng minh rằng tất cả n điểm
đã cho nằm bên trong hoặc trên cạnh của một tam giác có diện tích không vượt quá 4.
Phân tích. Do số tam giác là hữu hạn nên tồn tại tam giác có diện tích lớn nhất. Lời giải dưới
đây dựa trên nhận xét rằng khi cho một tam giác, ta dựng được một tam giác có diện tích gấp
4 lần diện tích tam giác ban đầu bằng cách kẻ các đường thẳng đi qua đỉnh và song song với
cạnh đối diện.
Lời giải. Gọi ABC là tam giác có diện tích lớn nhất trong tất cả các tam giác tạo thành từ ba 3
trong n điểm đã cho (nguyên lý cực hạn). Qua các đỉnh của tam giác ABC, kẻ các đường
thẳng song song với cạnh đối diện. Các đường thẳng này giao nhau tạo thành tam giác DEF
như hình vẽ. Do các tam giác ABC, DCB, ECA và FBA bằng nhau nên diện tích tam giác
DEF gấp 4 lần diện tích tam giác ABC, suy ra diện tích tam giác DEF không vượt quá 4.

3
Giả sử tồn tại điểm M trong n điểm đã cho nằm bên ngoài tam giác DEF. Không mất tính
tổng quát, giả sử M nằm khác phía với B, C so với đường thẳng EF. Khi đó khoảng cách từ M
đến BC lớn hơn khoảng cách từ A (thuộc EF) đến BC, suy ra diện tích tam giác MBC lớn hơn
diện tích tam giác ABC, mâu thuẫn với cách chọn tam giác ABC. Vậy tất cả n điểm đã cho
nằm bên trong hoặc trên cạnh của một tam giác có diện tích không vượt quá 4. ■
Trong nhiều bài toán, đối tượng cần đánh giá bị ẩn đi, đòi hỏi nhiều phân tích để tìm ra và
vận dụng nguyên lý cực hạn. Trong ví dụ tiếp theo, việc chọn phần tử cực biên giúp cho lập
luận mạch lạc hơn.
Ví dụ 6. Chứng minh rằng trong 16 số nguyên dương từ 2 đến 2017 và đôi một nguyên tố
cùng nhau luôn có ít nhất một số nguyên tố.
Phân tích. Do 16 số đôi một nguyên tố cùng nhau nên các ước nguyên tố của hai số bất kỳ
trong 16 số đều hoàn toàn khác nhau. Trong trường hợp “xấu” nhất, tức mỗi số chỉ có một
ước nguyên tố thì cũng cần có 16 số nguyên tố khác nhau. Chú ý rằng số nguyên tố thứ 16 là
53 và 53 2  2017 . Từ đây chúng ta thấy có thể vận dụng nguyên lý cực hạn để lập luận như
trong lời giải bên dưới.

Lời giải. Gọi ai là các số nguyên dương từ 2 đến 2017, i  1,..., 16 . Gọi pi là ước nguyên tố
nhỏ nhất của ai , và p là số lớn nhất trong các pi (nguyên lý cực hạn). Do các số ai đôi một
nguyên tố cùng nhau nên pi khác nhau đôi một, suy ra p  53 (53 là số nguyên tố thứ 16).
Do 53 2  2017 nên số ai mà nhận p làm ước nguyên tố nhỏ nhất không thể là hợp số mà
phải là số nguyên tố. ■
Ví dụ tiếp theo là một minh họa khi nguyên lý cực hạn được vận dụng cùng với phương pháp
lùi vô hạn (phương pháp xuống thang).
Ví dụ 7. Chứng minh rằng không thể chia một khối lập phương thành một số hữu hạn các
khối lập phương mà độ dài cạnh đôi một khác nhau.
Phân tích. Do số khối lập phương nhỏ là hữu hạn, nên tồn tại khối lập phương có cạnh nhỏ
nhất. Tuy vậy, vì các khối lập phương xếp không theo một trật tự nào nên rất khó để lập luận
mà không xét đến vị trí của khối lập phương có cạnh nhỏ nhất này. Để đơn giản, ta xét mặt
đáy của khối lập phương ban đầu, vốn sẽ bị phân hoạch thành hữu hạn các hình vuông có
kích thước khác nhau. Một cách tự nhiên, ta xét xem hình vuông nhỏ nhất có đặc điểm gì đặc
biệt. Nếu hình vuông nhỏ nhất nằm sát một cạnh của mặt đáy (như trong hình vẽ) thì không
thể cùng lúc có các hình vuông lớn hơn và nằm sát cạnh hình vuông nhỏ nhất. Tình huống
tương tự cho trường hợp hình vuông nhỏ nhất nằm ở một góc của mặt đáy. Như vậy hình
4
vuông nhỏ nhất phải nằm hoàn toàn bên trong mặt đáy. Dựa trên nhận xét này, chúng ta xây
dựng một cách lập luận như trong lời giải bên dưới.
Lời giải. Giả sử tồn tại cách chia một khối lập phương thành một số hữu hạn các khối lập
phương mà độ dài cạnh đôi một khác nhau.
Xét mặt đáy của khối lập phương ban đầu. Mặt đáy được chia thành một số hữu hạn các hình
vuông có cạnh đôi một khác nhau. Chọn hình vuông có độ dài cạnh nhỏ nhất (nguyên lý cực
hạn). Hình vuông này không thể chạm vào các cạnh của mặt đáy (vì không thể có các hình
vuông khác lớn hơn nó và nằm về cả 2 hoặc 3 phía của hình vuông nhỏ nhất này), tức nó nằm
hoàn toàn bên trong mặt đáy.

Khối lập phương nhận hình vuông nhỏ nhất này làm mặt đáy bị bao quanh bởi các khối lập
phương có cạnh lớn hơn nên mặt trên của nó phải bị chia thành các hình vuông bởi các khối
lập phương nằm trọn vẹn trên nó. Chọn hình vuông nhỏ nhất trong số các hình vuông này và
lặp lại các lập luận ở trên.
Quá trình này cứ tiếp diễn vô hạn lần vì các khối lập phương nằm trọn vẹn trên một khối lập
phương không thể cùng chạm mặt trên cùng của khối lập phương ban đầu. Điều này mâu
thuẫn với điều kiện số khối lập phương là hữu hạn. Vậy không thể chia một khối lập phương
thành một số hữu hạn các khối lập phương mà độ dài cạnh đôi một khác nhau. ■
Nguyên lý cực hạn giúp ích nhiều trong chứng minh các bài toán tổ hợp, đặc biệt khi trong
giả thiết đã xuất hiện yếu tố cực trị. Ví dụ dưới đây là một minh họa.

Ví dụ 8 (Ý TST 1999). Cho X là một tập có n phần tử và A1 , A2 ,... Am là các tập con của X
sao cho:

(i) Ai  3 với i  1, 2,..., m;

(ii) Ai  A j  1 với mọi cặp i , j phân biệt.

Chứng minh rằng tồn tại tập con của X với ít nhất  2 n  phần tử mà không chứa bất kì tập
Ai nào.

Phân tích. Bài toán yêu cầu chứng minh một so sánh giữa lực lượng của tập con S của X mà
không chứa bất kì tập Ai nào lớn hơn hoặc bằng biểu thức phụ thuộc n, do đó việc chọn X là
tập không chứa Ai nào và có nhiều phần tử nhất là khá tự nhiên.

5
Lời giải. Gọi S là tập con có nhiều phần tử nhất của X mà không chứa bất kì tập Ai nào
(nguyên lý cực hạn). Giả sử S  k . Ta sẽ chứng minh k   2 n  .

Nhận xét rằng với mỗi x thuộc X \ S , tồn tại một cặp phần tử  y, z thuộc S sao cho
 x, y, z  Ai
với i nào đó. Thật vậy, nếu không tồn tại cặp  y, z  như vậy thì ta có thể thêm
x vào tập S, mâu thuẫn với định nghĩa của tập S.
Xét ánh xạ f từ tập X \ S vào tập hợp các cặp phần tử của S sao cho phần tử của X \ S cùng
với cặp phần tử được ánh xạ đến tạo thành một trong các tập Ai . Từ điều kiện (ii) ta suy ra f
đơn ánh.

Số phần tử của tập X \ S là n  k , số các cặp phần tử của S là Ck2 nên n  k  Ck2 . Từ đây
1
suy ra k   2 n mà k nguyên nên k   2 n  . ■
2
Vận dụng nguyên lý cực hạn giúp chúng ta nắm bắt được các đối tượng trong giả thiết một
cách cụ thể, xác định rõ hơn cấu trúc của các đối tượng và quan hệ giữa chúng. Ba ví dụ tiếp
theo minh họa điều này.
Ví dụ 9 (Pháp 1997). Mỗi đỉnh của một đa giác 1997 đỉnh được gắn nhãn là một số nguyên
sao cho tổng tất cả các số nguyên là 1. Bắt đầu từ một đỉnh nào đó, ta liệt kê các nhãn của các
đỉnh theo chiều ngược chiều kim đồng hồ xung quanh đa giác. Có thể chọn được hay không
một đỉnh nào đó để bắt đầu sao cho tổng của k số nguyên được liệt kê đầu tiên là một số
dương với mọi k  1; 2;...; 1997 ?

Lời giải. Xuất phát từ V1 , kí hiệu tổng của k nhãn đầu tiên theo chiều ngược chiều kim đồng
hồ bởi bk . Gọi m là giá trị nhỏ nhất của các bk (nguyên lý cực hạn). Chọn k sao cho bk  m
(nếu có nhiều k như vậy thì chọn k có giá trị lớn nhất). Khi đó Vk 1 thỏa mãn yêu cầu bài
toán.
Kí hiệu là giá trị nhãn tại đỉnh .
Do định nghĩa của Vk 1 nên ak 1  0 .

Xét các tổng của nhãn từ Vk 1 đến V j .

Nếu j  k  1 , do a1  ...ak  ak 1  ...  a j  m mà


a1  ...  ak  m nên ak  1  ...  a j  0 .

Nếu j  k  1 , do
ak  1  ...  a1997  a1  ...  a j  ak  1  ...  a1997  a1  ...  ak
mà ak  1  ...  a1997  a1  ...  ak  1 nên ak 1  ...  a j  1 .

Vậy ta luôn có ak 1  ...  a j  0 với mọi j, tức đỉnh Vk 1 thỏa mãn yêu cầu bài toán. ■

6
Ví dụ 10 (Ấn Độ TST 2003). Với n là số nguyên dương và  A, B, C  là một phân hoạch của
tập 1; 2; 3;...; 3n sao cho A  B  C  n . Chứng minh rằng tồn tại các phần tử
x  A, y  B, z C sao cho có một trong ba số này bằng tổng của hai số còn lại.

Lời giải. Giả sử có một phân hoạch A, B, C của 1; 2; 3;...; 3n mà không tồn tại
x  A, y  B, z  C sao cho có một trong ba số này bằng tổng của hai số còn lại. Không mất
tính tổng quát, giả sử 1  A .
Gọi k là số nhỏ nhất mà không thuộc A (nguyên lý cực hạn), không mất tính tổng quát giả sử
k  B . Khi đó 1, 2,...k  1  A . Do đó:

(i) Không có hai phần tử nào thuộc A và C mà hiệu của chúng bằng k;
(ii) Không có cặp phần tử nào thuộc B và C mà hiệu của chúng nhỏ hơn k.
Gọi m là một phần tử bất kì thuộc C.
Do (ii) nên m  1 không thuộc B.
Nếu m  1 thuộc C thì do m  k không thuộc A (theo (i)) và m  k cũng không thuộc B (do
 m  1   m  k   k  1 và (ii)) nên m  k thuộc C. Ngoài ra, m  k  1 không thuộc A (do (
 m  1   m  k  1  k và (i)) và m  k  1 cũng không thuộc B (do  m  k    m  k  1  1
và (ii)) nên m  k  1 phải thuộc C. Như vậy nếu có môt cặp số nguyên liên tiếp  m; m  1
thuộc C thì kéo theo một cặp số nguyên liên tiếp nhỏ hơn  m  k ; m  k  1 cũng thuộc C.
Do quá trình này lặp lại vô hạn lần nên mâu thuẫn với giả thiết hữu hạn phần tử ban đầu.
Do đó, nếu m thuộc C thì m  1 phải thuộc A. Từ đây ta có một đơn ánh từ tập C đến tập A.
Lại có A  C nên suy ra tương ứng này phải là song ánh. Do 1 thuộc A nên suy ra 2 thuộc
C, điều này mâu thuẫn với giả sử phần tử nhỏ nhất mà không thuộc A thì phải thuộc B.
Vậy không tồn tại phân hoạch như giả sử, tức với mọi phân hoạch thỏa giả thiết thì tồn tại các
phần tử x  A, y  B, z  C sao cho có một trong ba số này bằng tổng của hai số còn lại. ■

Ví dụ 11 (IMO 1977). Cho hàm số f : *   * sao cho f  n  1  f  f  n   với mọi


n  * . Chứng minh rằng f  n  n với mọi n  * .

Lời giải. Đặt k  min  f  n : n  * (nguyên lý cực hạn). Giả sử f  m   k . Nếu m  1 thì
từ điều kiện của giả thiết suy ra k  f  m   f  f  m  1  , mâu thuẫn với tính chất nhỏ nhất
của k. Vậy m  1 , suy ra f  1  f  2 .

Đặt l  min  f  n : n  2 (nguyên lý cực hạn). Giả sử f  p   l . Từ điều kiện của giả thiết
suy ra l  f  p   f  f  p  1  , suy ra f  f  p  1   1 , kéo theo f  p  1  1 , suy ra
p  1  1 , tức p  2 . Suy ra f  2  f  3 .

Lập luận tương tự ta được f  1  f  2  ...  f  n  f  n  1  ... , suy ra f  n  n với mọi


n  * .
7
Mặt khác, f là hàm tăng ngặt trên  * nên f  n  1  f  f  n   kéo theo n  1  f  n , tức
f  n  n .

Vậy f  n  n với mọi n  * . Thử lại ta thấy yêu cầu bài toán thỏa mãn. ■

Nguyên lý cực hạn và bài toán chứng minh bất đẳng thức
1 1
Ví dụ 12. Biết rằng các số a1 , a2 ,...an và b1 , b2 ,..., bn là hai hoán vị của các số 1, ,..., .
2 n
4
Chứng minh rằng nếu a1  b1  a2  b2  ...  an  bn thì am  bm  với mọi m  1, 2,..., n .
m

Lời giải. Với mỗi m  1, 2,..., n , xét m cặp  ak , bk  với k  1, 2,..., m . Không mất tính tổng
m
quát, giả sử bk  ak với ít nhất cặp  ak , bk  . Gọi bt là số nhỏ nhất trong các số bk
2
2 4 4
(nguyên lý cực hạn) thì bt  . Khi đó at  bt  2bt  . Do t  m nên am  bm  at  bt 
m m m
.■
Hai ví dụ tiếp theo sử dụng kết quả sau:

Nếu f  x  là hàm số liên tục trên khoảng đóng  a; b  thì tồn tại các điểm x1 , x2  a; b  sao
cho f  x1   f  x  f  x2  với mọi x  a; b  . Trong trường hợp f  x  là hàm tuyến tính thì
f  a   f  x   f  b với mọi x  a; b  hoặc f  b  f  x   f  a  với mọi x  a; b  .

Ví dụ 13. Cho x, y , z  0; 2  , chứng minh rằng 2  x  y  z   xy  yz  zx  4 .

Lời giải. Xét hàm số f  x    2  y  z  x  2 y  2 z  yz  4 với x  0; 2  .

Ta có:

f  0  2 y  2 z  yz  4    2  y  2  z   0 do 0  y , z  2 .

f  2   2  y  z  2  2 y  2 z  yz  4   yz  0 do 0  y , z  2 .

Do f là hàm tuyến tính trên đoạn  0; 2 nên với mọi x  0; 2  , ta có


f  x  max  f  0 , f  2  0 , suy ra 2  x  y  z   xy  yz  zx  4 .

Vậy 2  x  y  z   xy  yz  zx  4 với x, y , z  0; 2  . ■

Ví dụ 14. Tìm giá trị lớn nhất của biểu thức Sn  a1 1  a2   a2 1  a3   ...  an 1  a1  với
1
 ai  1 với mọi i  1, 2,..., n .
2

8
Lời giải. Biểu thức S n là một hàm tuyến tính đối với mỗi biến và các biến xác định trên đoạn
1  1 
 2 ; 1 nên giá trị lớn nhất đạt được tại những điểm đầu mút của mỗi biến, tức ai   2 ; 1
với i  1, 2,..., n .

1  1
Xét giá trị của S n tại ai   ; 1 với i  1, 2,..., n . Mỗi số hạng của S n có thể bằng 0,
2  4
1
hoặc . Nếu có đúng m ( 0  m  n ) biến ai chọn bằng 1, thì m số hạng của S n bằng 0,
2
1 1
nhiều nhất m số hạng bằng và các số hạng còn lại bằng . Do đó giá trị lớn nhất của S n
2 4
1 1 n n
không vượt quá m.0  m.   n  2 m .  . Nếu m  0 thì S n  .
2 4 4 4
n
Vậy giá trị lớn nhất của S n là .■
4
Bài tập
Bài 1. Trong một đường tròn tâm O đã cho, xét tập hợp C gồm một số hữu hạn các dây cung
thỏa mãn tính chất sau: mỗi dây cung thuộc tập C đi qua trung điểm của một dây cung khác
cũng thuộc C. Chứng minh rằng tất cả các dây cung này đều là đường kính.
Bài 2. Trên một đường tròn có 2017 số thực, mỗi số bằng giá trị tuyệt đối của hiệu hai số
đứng ngay sau nó theo chiều kim đồng hồ. Biết rằng tổng của tất cả các số bằng 1, hãy tìm
các số trên đường tròn.
Bài 3. Cho 2n  2 ( n  1 ) điểm trên mặt phẳng trong đó không có ba điểm nào thẳng hàng.
Chứng minh rằng tồn tại hai điểm trong chúng sao cho đường thẳng đi qua hai điểm này chia
mặt phẳng thành hai miền mà mỗi miền chứa đúng n điểm đã cho.
Bài 4. Cho 2n  3 ( n  1 ) điểm trên mặt phẳng trong đó không có ba điểm nào thẳng hàng và
không có bốn điểm nào cùng thuộc một đường tròn. Chứng minh rằng tồn tại ba điểm trong
chúng sao cho đường tròn đi qua ba điểm này chia mặt phẳng thành hai miền (miền trong và
miền ngoài dường tròn) mà mỗi miền chứa đúng n điểm đã cho.
Bài 5 (Định lí Gallai). Xét một tập hữu hạn các điểm trên mặt phẳng sao cho tất cả những
điểm này không cùng nằm trên một đường thẳng. Chứng minh rằng tồn tại một đường thẳng
đi qua đúng hai trong số những điểm đã cho.
1 1 1
Bài 6. Chứng minh với mọi số tự nhiên n  1 , tổng 1    ...  không phải là số
2 3 n
nguyên.
Bài 7 (Rumania 1986). Cho f :    là hàm toàn ánh và g :    là hàm đơn ánh sao
cho với mọi n  thỏa mãn f  n  g  n . Chứng minh rằng f  n  g  n với mọi n  .

Bài 8. Xét một số hữu hạn các số thực không âm sao cho tổng của chúng bằng 3, và tổng của
bình phương của chúng lớn hơn 1. Chứng minh rằng có thể chọn được ba số trong các số này
có tổng lớn hơn 1.
9
Bài 9. Cho xi  1; 1 với i  1, 2,..., n ( n  2 ). Tìm giá trị nhỏ nhất của biểu thức
S   xi x j
i j

Bài 10. Cho các số không âm a, b, c, A, B, C và k thỏa mãn a  A  b  B  c  C  1 . Chứng


minh rằng aA  bB  cC  1 .
Kết luận
Vận dụng nguyên lý cực hạn giúp cho chúng ta hiểu rõ các đối tượng trong bài toán và mối
quan hệ giữa chúng. Trong nhiều trường hợp, vận dụng nguyên lý cực hạn một cách khéo léo
là chìa khóa để xây dựng được lời giải cho các bài toán khó.
Tài liệu tham khảo
1. Problem Solving Strategies, Arthur Engel, Springer, 1998
2. Giải toán bằng phương pháp đại lượng cực biên, Nguyễn Hữu Điển, Nxb Giáo dục,
2005
3. The Art and Craft of Problem Solving, Paul Zeitz, John Wiley and Sons, 2006

10
NGHIỆM CỦA ĐA THỨC VÀ ỨNG DỤNG

NGHIỆM CỦA ĐA THỨC VÀ ỨNG DỤNG


Giáo viên: NGUYỄN THỊ QUYÊN

Đơn vị: Trường THPT Chuyên Lý Tự Trọng

Email: ntquyen.cltt@cantho.edu.vn

Điện thoại: 0986436908

PHẦN 1: TÓM TẮT


Đa thức là một kiến thức quen thuộc đã được giới thiệu cho học sinh từ ở chương trình
THCS. Như đã biết đây cũng là một dạng toán thường gặp trong kì thi học sinh giỏi các cấp.
Kiến thức về đa thức là tương đối rộng và phong phú. Trong bài viết này, tôi chỉ đề cấp đến
một mảng kiến thức nhỏ về đa thức nhưng cũng có khá nhiều ứng dụng đó là “Nghiệm của đa
thức “.

Ở chương 1, bài viết sẽ giới thiệu về một số định nghĩa , định lí và tính chất có liên quan
về nghiệm của đa thức và chọn lọc một số bài tập thích hợp để làm rõ ứng dụng của định lí và
tích chất khi nêu ra. Tiếp theo ở chương 2, bài viết sẽ nêu một số ứng dụng về nghiệm của đa
thức như:

 Xác định đa thức

 Bài toán chia hết

 Chứng minh đa thức khả quy, bất khả quy

 Giải hệ phương trình

PHẦN 2: NỘI DUNG

CHƯƠNG 1: NGHIỆM CỦA ĐA THỨC


I.ĐỊNH NGHĨA NGHIỆM ĐA THỨC

Cho đa thức P  x    x và số thực  , số  được gọi là nghiệm của đa thức P  x  nếu P    0.

II. ĐỊNH LÝ BEZOUT


NGUYỄN THỊ QUYÊN Trang 1
NGHIỆM CỦA ĐA THỨC VÀ ỨNG DỤNG

Cho đa thức P  x    x và số thực  . Khi đó,  là nghiệm của P  x  khi và chỉ khi P  x  x    .

Điều này có nghĩa là: tồn tại đa thức Q  x    x  sao cho P ( x )  ( x   ).Q ( x ), x  . .

Hơn nữa, nếu đa thức P  x  là đa thức mônic bậc n và có n nghiệm thực 1 ,  2 ,...,  n thì

P ( x )  ( x  1 )( x   2 )...( x   n ), x  .

Ví dụ 1: Cho đa thức P  x  với hệ số nguyên. Chứng minh đa thức không có nghiệm nguyên

nếu P (0) và P (1) là các số lẻ.

Giải.

Ta sử dụng phương pháp phản chứng để giải quyết bài toán này

Giả sử x0 là nghiệm nguyên của P ( x ) . Khi đó, P( x)  ( x  x0 )Q( x), x   , trong đó Q ( x ) là đa


thức với hệ số nguyên.

Thay x  0 vào P ( x ) ta được P (0)   x0Q(0). Do P (0) là số lẻ nên x0 cũng là số lẻ.

Thay x  1 vào P ( x ) ta được P (1)  (1  x0 )Q(1). Do P (1) là số lẻ nên (1  x0 ) cũng là số lẻ hay x0


là số chẵn ( điều này vô lí)

Vậy, đa thức P ( x ) không có nghiệm nguyên.

 Đây là một bài toán quen thuộc về nghiệm của đa thức. Ta có thể sử dụng hướng giải này để
giải quyết các bài toán tương tự sau:

Ví dụ 2: Cho đa thức P  x  với hệ số nguyên thỏa mãn P(a)  P(b)  P(c)  1 , với a, b, c là ba

số nguyên phân biệt. Chứng minh rằng đa thức P ( x ) không có nghiệm nguyên.

Giải.

Giả sử x0 là nghiệm nguyên của P ( x ) . Khi đó, P( x)  ( x  x0 )Q( x), x   , trong đó Q ( x ) là đa


thức với hệ số nguyên.

Thay x  a vào P ( x ) ta được P(a)  (a  x0 )Q(a). Do P(a)  a  x0 . Q(a)  1 nên a  x0  1

Tương tự ta có b  x0  1; c  x0  1 .

NGUYỄN THỊ QUYÊN Trang 2


NGHIỆM CỦA ĐA THỨC VÀ ỨNG DỤNG

Suy ra, các số a  x0 , b  x0 , c  x0 nhận một trong hai giá trị 1 hoặc -1. Theo nguyên lý Dirichlet
ta có hai trong ba số a  x0 , b  x0 , c  x0 bằng nhau. Do đó, hai trong ba số a, b, c bằng nhau ( vô
lí)

Vậy, đa thức P ( x ) không có nghiệm nguyên.

Ví dụ 3: Cho đa thức P  x  với hệ số nguyên thỏa mãn P(2017 k ).P(2018k )  2019k , với k là một

số nguyên dương. Chứng minh rằng đa thức P ( x ) không có nghiệm nguyên.

Giải.

Giả sử x0 là nghiệm nguyên của P ( x ) . Khi đó, P( x)  ( x  x0 )Q( x), x   , trong đó Q ( x ) là đa


thức với hệ số nguyên.

Thay x  2017 k vào P ( x ) ta được P (2017 k )  (2017 k  x0 )Q (2017 k ).

Tương tự ta có P (2018k )  (2018k  x0 )Q (2018k ).

Theo giả thiết ta có

P(2017 k ).P(2018k )  2019k  (2017 k  x0 )Q (2017 k ).(2018k  x0 )Q (2018k )  2019 k (1)

Ta thấy hai số 2017 k  x0 , 2018k  x0 là một số chẵn và một số lẻ nên VT (1) là một số chẵn, mà
VP (1) là một số lẻ ( vô lí)

Vậy, đa thức P ( x ) không có nghiệm nguyên.

III. NGHIỆM BỘI

Cho đa thức P( x)   x  và   , k   * . Số  được gọi là nghiệm bội k của P  x  nếu P  x  chia

hết cho ( x   )k nhưng không chia hết cho ( x   )k 1.

Điều này có nghĩa là: tồn tại đa thức Q  x    x  sao cho P( x)  ( x   )k Q  x  , x   và


Q ( )  0.

 Hơn nữa, nếu P( x)    x  có nghiệm 1 ,  2 ,...,  m với bội tương ứng k1 , k 2 ,.., km thì tồn tại đa
k1 k2 km
thức Q( x)    x  sao cho P  x    x  1  .  x   2   x   m  .Q  x  , x  .

NGUYỄN THỊ QUYÊN Trang 3


NGHIỆM CỦA ĐA THỨC VÀ ỨNG DỤNG

Ví dụ 4.(Trung Quốc 1996) Cho đa thức P ( x ) bậc 5 có 5 nghiệm thực phân biệt. Tìm số các hệ
số khác không bé nhất của đa thức.

Giải.

TH1: P( x)  ax 5 , a  0. Đa thức P ( x ) có nghiệm bội (loại)

TH2: P( x)  ax 5  bx n (a, b  0, n  {2,3, 4}). Đa thức P ( x ) có nghiệm bội (loại)

b
TH3: P ( x)  ax 5  bx  ax  x 4   , (a, b  0). Đa thức P ( x ) có tối đa 3 nghiệm (loại)
 a

TH4: P( x)  ax 5  b (a, b  0). Đa thức P ( x ) có duy nhất một nghiệm (loại)

TH5: P( x)  ax 5  bx 4  cx3  dx 2  ex  f trong đó a  0, hai trong năm hệ số b, c, d , e, f khác 0

Chọn đa thức P( x)  x5  10 x3  9 x  x( x 2  1)( x 2  9) có 5 nghiệm phân biệt

Vậy, số các hệ số khác không bé nhất là 3.

IV. NGHIỆM NGUYÊN, NGHIỆM HỮU TỈ

Cho đa thức P  x    x có bậc n là P ( x )  an x n  an 1 x n 1  ...  a1 x  a0 .

p
Nếu P  x  có nghiện hữu tỷ là x  với  p, q   1 thì p | a0 , q | an .
q

Ví dụ 5. Tìm các nghiệm hữu tỉ của phương trình 3x 4  5 x3  x 2  5 x  2  0 (1) .

Giải.

p
Ta xét các số hữu tỉ x  với p  2, q 3
q

1 2
Do đó, x  1,  , 2,  là các số hữu tỉ có thể là nghiệm của phương trình (1)
3 3

1
Bằng cách thế trực tiếp phương trình (1) nhận được hai nghiệm hữu tỉ là 2, .
3

Ví dụ 6. Chứng minh số 2  3 3 là số vô tỷ.

Giải.

NGUYỄN THỊ QUYÊN Trang 4


NGHIỆM CỦA ĐA THỨC VÀ ỨNG DỤNG

Đặt x  2  3 3  x 3  3 2 x 2  6 x  2 2  3  x 3  6 x  3  2(3 x 2  2)

 x 6  6 x 4  6 x3  12 x 2  36 x  1  0 (1)

Suy ra x  2  3 3 là một nghiệm của (1).

Mà nghiệm hữu tỉ có thể có của (1) là 1 .

Vậy, số 2  3 3 là số vô tỷ.

Ví dụ 7.(IMO 1997) Tồn tại hay không đa thức P ( x ) hệ số nguyên thỏa mãn

P ( 3 3  3 9)  3  3 3.

Giải.

Giả sử tồn tại đa thức P ( x ) thỏa mãn đề bài

Đặt   3 3  3 9   3  9  12  Q ( x)  x 3  9 x  12 nhận  làm nghiệm.

Trước hết ta có nhận xét u 3 3  v 3 9  t , (u , s, t  )  u  v  0

Thật vậy, u 3 3  v 3 9  t  t 2v  u 2v 3 9  3v 3 3 3  6uv 2  t 2 v  6uv 2  u 2 v 3 9  3v 3 3 3 (1)

Mặt khác u 2t  u 3 3 3  u 2v 3 9 (2)

Từ (1) và (2) ta có t 2v  6uv 2  u 2t  3 3(3v 3  u 3 )  u 3  3v 3  u  v  0 .

Quay lại bài toán

TH1: P ( x )  ax  b(a  0)

Ta có P ( 3 3  3 9)  ( 3 3  3 9)a  b  3  3 3  (a  1) 3 3  a 3 9  3  b  a  1  a  0 : vô nghiệm

TH2: P( x)  ax 2  bx  c(a  0)

2
Ta có P( x)  a  3
339  b  3

3  3 9  c  3  3 3  (3a  b  1) 3 3  (a  b) 3 9  3  6a  c

1 1
Suy ra 3a  b  1  a  b  0  a  , b   , c  0. (loại)
2 2

TH3: Xét P ( x ) có bậc không nhỏ hơn 3

Lấy P ( x ) chia cho Q ( x ) ta được P ( x )  Q ( x ).H ( x )  R ( x ), deg R ( x )  3


NGUYỄN THỊ QUYÊN Trang 5
NGHIỆM CỦA ĐA THỨC VÀ ỨNG DỤNG

Khi đó, P ( )  R ( ) . Theo TH1, TH2 suy ra không tồn tại đa thức P ( x ).

Vậy, không tồn tại đa thức P ( x ) hệ số nguyên thỏa mãn P ( 3 3  3 9)  3  3 3.

V. QUAN HỆ GIỮA BẬC VÀ SỐ NGHIỆM CỦA ĐA THỨC

 Một đa thức bậc n, n   * không thể có hơn n nghiệm khác nhau.

 Một đa thức bậc n có ít nhất n +1 nghiệm thì đa thức đó đồng nhất với 0.

 Một đa thức bậc n nhận giá trị C không đổi tại ít nhất n + 1 điểm khác nhau của biến thì đa
thức đó là đa thức hằng f  C.

 Hai đa thức có bậc n và nhận n + 1 giá trị như nhau tại n +1 điểm khác nhau của biến thì hai
đa thức đó đồng nhất.

Ví dụ 8. Cho đa thức P ( x ) có bậc 6 và thỏa mãn P ( k )  P (  k ), k  1, 2,3. Chứng minh rằng hai đa
thức P ( x ) và P (  x ) đồng nhất nhau.

Giải.

Xét đa thức Q ( x )  P ( x)  P (  x ), x   . Vì deg P ( x ) = 6 nên deg Q ( x )  6 .

Theo giả thiết P ( k )  P (  k ), k  1, 2,3  P ( k )  P ( k ), k  1, 2, 3

Do đó, đa thức Q ( x ) có các nghiệm x  0, 1, 2, 3

Mà deg Q ( x )  6 nên Q ( x )  0, x   hay P ( x )  P (  x ), x   .

Vậy, hai đa thức P ( x ) và P (  x ) đồng nhất nhau.

VI. ĐỊNH LÍ VIET

 ĐỊNH LÝ VIETE THUẬN

Cho P  x   an x n  an1 x n1  ...  a1 x  a0 trong đó ai   và a0  0 .

Nếu P  x  có n nghiệm x1 , x2 ,...xn phân biệt hay trùng nhau thì

n
an1
S1   xi  
i 1 an

NGUYỄN THỊ QUYÊN Trang 6


NGHIỆM CỦA ĐA THỨC VÀ ỨNG DỤNG

an 2
S2  
1i  j  n
xi x j 
an

....................................................

an  k
Sk   x1 x2 ...xk  (1) k .
1 i1  i2 ... ik  n an

với S k là tổng các tích chập k của n số xi .

 ĐỊNH LÝ VIETE ĐẢO

Nếu n số x1 , x 2 , x3 ,..., xn có các tổng của các tích chập k từ n số đó là S k thì x1 , x 2 , x3 ,..., xn là
n
nghiệm nếu có của phương trình X n  S1 X n 1  S2 X n  2  ...   1 .Sn X n  n  0

Ví dụ 9. (Áo, 1983) Hãy tìm tất cả các giá trị a để 3 nghiệm x1 , x2 , x3 của đa thức

x 3  6 x 2  ax  a thỏa mãn ( x1  3)3  ( x2  3)3  ( x3  3)3  0 .

Giải.

Đặt y  x  3 .

Ta xét đa thức P( y )  ( y  3)3  6( y  3)3  a( y  3)  a  y 3  3 y 2  (a  9) y  4a  27

Khi đó, y1  x1  3, y2  x2  3, y3  x3  3 là ba nghiệm của đa thức P ( y )

Theo định lí Viete ta có

 S1  y1  y2  y3  3

 S2  y1 y2  y2 y3  y3 y1  a  9
 S  y y y  27  4a
 3 1 2 3

Theo giả thiết ta lại có ( x1  3)3  ( x2  3)3  ( x3  3)3  0

 y13  y23  y33  0  S13  3S1S 2  3S3  0  a  9

Vậy, với a  9 là giá trị cần tìm.

Ví dụ 10. Giả sử đa thức P( x)  x3  ax 2  bx  c có ba nghiệm phân biệt. Chứng minh rằng đa


1 ab  c
thức Q ( x)  x 3  ax 2  (a 2  b) x  cũng có ba nghiệm phân biệt.
4 8

NGUYỄN THỊ QUYÊN Trang 7


NGHIỆM CỦA ĐA THỨC VÀ ỨNG DỤNG

Giải.

Gọi x1 , x2 , x3 là ba nghiệm phân biệt của đa thức P ( x ) .

x1  x2 x x x x
Xét y1  , y2  1 3 , y3  3 1
2 2 2


 y1  y2  y3  x1  x2  x3   a

 1 1

Ta có  y1 y2  y2 y3  y3 y1  ( x1  x2  x3 ) 2  x1 x2  x2 x3  x3 x1   a 2  b
4 4


 1 1
 y1 y2 y3  8  ( x1  x2  x3 )( x1 x2  x2 x3  x3 x1 )  x1 x2 x3   8 (ab  c)

Theo định lí Viete đảo ta có y1 , y2 , y3 là ba nghiệm phân biệt của đa thức


1 ab  c
Q ( x)  x 3  ax 2  ( a 2  b) x  .
4 8

Ví dụ 11. Cho a, b là hai số thực và n là số nguyên dương. Giả sử đa thức


f ( x)  x3  ax 2  bx  a có ba nghiệm thực đều lớn hơn 1. Tìm tất cả giá trị của a , b sao cho
b n  3n
biểu thức P  đạt giá trị nhỏ nhất. Tính giá trị nhỏ nhất đó.
an

Giải.

Gọi x1 , x2 , x3 là các nghiệm của đa thức f ( x )

 x1  x2  x3  a

Theo định lý Viete ta có  x1 x2  x2 x3  x3 x1  b
x x x  a
 1 2 3

Theo bất đẳng thức AM- GM ta được:

x1  x2  x3  3 3 x1 x2 x3 hay a  3 3 a  a  3 3  
2
Theo bất đẳng thức  x  y  z   3  xy  yz  xz  ; x, y, z  

2
Ta có b 2   x1 x2  x2 x3  x3 x1   3x1 x2 x3  x1  x2  x3  hay b 2  3a 2  b  3a

b n  3n 3n a n  3n 3n 3n
Suy ra P    3n
  3n
 n
, do  
an an an 3 3  
NGUYỄN THỊ QUYÊN Trang 8
NGHIỆM CỦA ĐA THỨC VÀ ỨNG DỤNG

3n  1
Do đó ta có P 
3n

Đẳng thức xảy ra khi và chỉ khi a  3 3; b  3a  9 . Khi đó phương trình có ba nghiệm trùng
nhau và đều bằng 3 .

3n  1
Vậy, giá trị nhỏ nhất của P là khi a  3 3 và b  9 .
3n

CHƯƠNG 2: MỘT SỐ ỨNG DỤNG CỦA NGHIỆM ĐA THỨC

I.XÁC ĐỊNH ĐA THỨC

 Trong dạng toán xác định đa thức, định lý Bezout đóng một vai trò quan trọng. Chúng ta
sẽ xét một vài ví dụ cụ thể để thấy rõ vai trò đó:

Ví dụ 12. (VMO-B2003)Tìm tất cả các đa thức P(x) với hệ số thực thỏa mãn đẳng thức

( x 3  3 x 2  3x  2) P ( x  1)  ( x3  3x 2  3x  2) P ( x) với mọi số thực x.

Giải.

Ta có ( x3  3x 2  3x  2) P( x  1)  ( x3  3x2  3x  2) P( x), x   (1)

 ( x  2)( x2  x  1) P( x 1)  ( x  2)( x2  x  1) P( x), x  (2)

Thay x  2 vào (2) ta được P ( 2)  0

Thay x  1 vào (2) ta được P (1)  0

Thay x  0 vào (2) ta được P (0)  0

Thay x  1 vào (2) ta được P (1)  0

Từ đó ta suy ra P ( x )  ( x  1) x ( x  1)( x  2)Q( x ), x  , (trong đó Q(x) là một đa thức với hệ số


thực của biến x).

Do đó, P ( x  1)  ( x  2)( x  1) x ( x  1)Q( x  1), x  

Thay P ( x ), P ( x  1) vào (1) ta được

NGUYỄN THỊ QUYÊN Trang 9


NGHIỆM CỦA ĐA THỨC VÀ ỨNG DỤNG

( x 2  x  1)Q( x  1)  ( x 2  x  1)Q( x)(x, x  0, x  1, x  2)


Q( x  1) Q ( x)
 2
 2 , (x, x  0, x  1, x  2)
( x  1)  ( x  1)  1 x  x  1

Q( x)
G ( x)  2
, (x , x  0, x  1, x  2)
Đặt x  x 1

Suy ra G ( x  1)  G ( x ), x, x  0, x  1, x  2  G ( x )  C (C : const )

Hay Q( x)  C ( x 2  x  1) (C : const )

Do đó, P( x)  Cx( x  1)( x  1)( x  2)( x 2  x  1), x  .

Phép thử trực tiếp cho thấy các đa thức P(x) vừa tìm được ở trên thỏa mãn hệ thức đề bài.

Vậy, P( x)  Cx( x  1)( x  1)( x  2)( x 2  x  1), x   là tất cả các đa thức cần tìm.

 Nếu chỉ sử dụng nghiệm thực của đa thức thì trong một số bài toán sẽ bị hạn chế vì không
đủ số nghiệm. Do đó, người ta sẽ xét đến nghiệm phức của đa thức.

Định lý D’ALEMBERT: Mọi đa thức bậc n hệ số phức ( thực) đều có đủ n nghiệm phức phân
biệt hay trùng nhau.

Ví dụ 13.Tìm tất cả các đa thức P(x) khác đa thức hằng sao cho

P ( x) P( x  1)  P ( x 2  x  1); x  . (1)

Giải.

Giả sử x 0 là nghiệm của P( x )  0. Thay x0 vào (1) ta được P ( x0 2  x0  1)  0.

Suy ra x0 2  x0  1 cũng là nghiệm của P(x) .

Thay x bởi x  1 trong (1) ta được P( x  1) P( x)  P( x 2  x  1) .

Vì P( x0 )  0 nên P ( x0 2  x0  1)  0 . Do đó, x0 2  x0  1  0 cũng là nghiệm của P(x) .

Chọn  là nghiệm có module lớn nhất (nếu tồn tại vài nghiệm với module lớn nhất, ta chọn
một trong số các nghiệm đó)

Từ cách chọn  suy ra: |  2    1 ||  | và |  2    1||  | (2)

Ta có   0 (vì   0 thì mẫu thuẫn với (2) )


NGUYỄN THỊ QUYÊN Trang 10
NGHIỆM CỦA ĐA THỨC VÀ ỨNG DỤNG

Và 2 |  || ( 2    1)  ( 2    1) ||  2    1|  |  2    1| |  |  |  | 2 |  |

Vì  là module lớn nhất nên ta có |  2    1 ||  2    1 ||  |

Suy ra  2    1  ( 2    1)   2  1  0    i

Như vậy ta có thể viết: P ( x)  ( x 2  1) m Q( x) ; m   * . Trong đó Q(x) là đa thức không chia hết
cho x 2  1 .

Thế ngược P( x) trở lại vào (1) ta thấy Q(x) thỏa mãn

Q( x)Q( x  1)  Q ( x 2  x  1), x   (3)

Nếu phương trình Q ( x )  0 lại có nghiệm thì lập luận như trên ta suy ra nghiệm có modun lớn
nhất của nó phải là  i . Điều này không thể xảy ra vì Q(x) không chia hết x 2  1 .

Do đó, Q(x) phải là một hằng số. Giả sử Q ( x )  C _ const , x  

Thay Q( x) vào (3) ta được C  1 .

Phép thử trực tiếp cho thấy các đa thức P(x) vừa tìm được ở trên thỏa mãn hệ thức đề bài.

Vậy, P ( x)  ( x 2  1) m , n   * là tất cả các đa thức cần tìm.

Ví dụ 14. Tìm tất cả các đa thức P(x) khác đa thức hằng thỏa mãn P( x) P( x  1)  P( x 2 ), x  .
(1)

Giải.

Giả sử  là nghiệm của P ( x )  0 . Khi đó từ (1) ta suy ra  2 ,  4 ,  8 ,... cũng là nghiệm của
P ( x )  0 . Từ đây suy ra |  | 0 hoặc |  | 1 , vì nếu ngược lại ta sẽ thu được dãy vô hạn các

nghiệm phân biệt của P(x) .

Tương tự   1 là nghiệm của P(x) và lập luận tương tự, ta cũng được |   1 | 0 hoặc
|   1 | 1 .

Giả sử rằng |  | 1 và |   1| 1 (2)

Đặt   cos   i sin  ;   [0; 2 ]

NGUYỄN THỊ QUYÊN Trang 11


NGHIỆM CỦA ĐA THỨC VÀ ỨNG DỤNG

1  5
Từ (2) ta có cos     hoặc   .
2 3 3


Giả sử   , xét  2 cũng là nghiệm của P(x) , như vậy  2  1 cũng là nghiệm của P(x) và
3
2
2  2  2
|   1 |  cos  1  sin 2  3 mâu thuẫn vì mọi nghiệm của P(x) đều có modun bằng 0
 3  3
hoặc 1 .

5
Tương tự trên với trường hợp   . Như vậy có thể kết luận   1 hoặc   1  1 . Từ đây
3
P (x) có dạng P( x)  cx m (1  x) n , c là hằng số và m, n   , thay vào phương trình đã cho ta dễ

dàng kiểm tra được c  1 và m  n . Vậy các đa thức thỏa mãn: P( x)  x m (1  x) m , m  .

II. BÀI TOÁN CHIA HẾT

 Ta biết rằng, nếu đa thức P(x) chia hết cho đa thức Q(x) thì mọi nghiệm của Q(x) đều là
nghiệm của P(x) . Tính chất đơn giản này là chìa khóa để giải nghiệm bài toán về sự chia hết
của đa thức.

 Cho hai số phức 1 ,  2 liên hợp với nhau. Nếu 1 là nghiệm của đa thức P( x) thì  2 cũng là
nghiệm của P( x).

Ví dụ 15. Tìm tất cả giá trị của n để đa thức P( x)  x 2 n  x n  1 chia hết cho đa thức x 2  x  1.

Giải.

1 3  2   2  2
Ta có w1,2    i  cos     i sin   3  là nghiệm của đa thức Q ( x )  x  x  1.
2 2  3   

Đa thức P( x)  x 2 n  x n  1 chia hết cho Q(x) khi và chỉ khi P(w1 )  0 , điều này tương đương
với

 4n   4n   2n   2n 


cos    i sin    cos    i sin   1  0
 3   3   3   3 
 4 n 2 n
 cos  cos 1  0
3 3 2n
  2 cos 1  0
sin 4 n 2 n 3
 sin 0
 3 3

NGUYỄN THỊ QUYÊN Trang 12


NGHIỆM CỦA ĐA THỨC VÀ ỨNG DỤNG

2n  1 2 n  1  3k
 cos   cos  ( k  )
3 2 3 n  3k  2

Vậy với n  3k  1 hoặc n  3k  2 , ( k  ) thì P(x) chia hết cho Q(x) .

Ví dụ 16.(VMO-2015) Cho dãy đa thức f n ( x) xác định bởi

 f0 ( x)  2

 f1 ( x)  3x

 f n ( x)  3xf n1 ( x)  (1  x  2 x ) f n2 ( x),( n  , n  2)
2

Tìm tất cả các giá trị n để đa thức f n ( x) chia hết cho đa thức x 3  x 2  x .

Giải.

 Xác định đa thức f n ( x)

u0  2

Xét dãy số u1  3x , với x là tham số thực.

un  3xun1  (1  x  2 x )un2 ,( n  , n  2)
2

Phương trình đặc trưng t 2  3 xt  2 x 2  x  1  0  t  x  1  t  2 x  1

Suy ra un  A( x  1) n  B(2 x  1) n

u 0  2 A  B  2
Theo giả thiết    A  B 1
u1  3 x  A( x  1)  B (2 x  1)  3 x

Do đó, f n ( x )  ( x  1) n  (2 x  1) n

 Chứng minh đa thức f n ( x) chia hết cho đa thức x 3  x 2  x

Ta có x3  x 2  x  x( x 2  x  1) . Gọi  là nghiệm phức của đa thức ( x 2  x  1).

Suy ra  2    1  0 và  3  1

Vì f n ( x) chia hết cho ( x 2  x  1) nên f n (0)  f n ( )  0 . Do đó,

f n (0)  0  1n  ( 1) n  0  n là số lẻ  n  2k  1, k  

 (  1)(3 )k  (2  1)(3)k  0  2  1  (  1)( )k  0

NGUYỄN THỊ QUYÊN Trang 13


NGHIỆM CỦA ĐA THỨC VÀ ỨNG DỤNG

 Với k  3t : 2  1  (  1)( )3t  0  3  0 : vô lí. Suy ra k  3t không thỏa

 Với k  3t  1 : 2  1  (  1)( )3t 1  0   2    1  0 : đúng. Suy ra k  3t  1 thỏa mãn

 Với k  3t  1 : 2  1  (  1)( )3t  2  0   3   2  2  1  0    3 : vô lí. Suy ra k  3t  2


không thỏa

Vậy, với n  6t  3, t   thì đa thức f n ( x) chia hết cho đa thức x 3  x 2  x .

III. CHỨNG MINH ĐA THỨC KHẢ QUY, BẤT KHẢ QUY

Ví dụ 17. Chứng minh rằng đa thức P ( x)  x n  4, n  , n  2 khả quy trên [x ] khi và chỉ khi n
chia hết cho 4.

Giải.

Giả sử P ( x)  x n  4 khả quy

Gọi 1 ,  2 ,...,  n là các nghiệm của đa thức P ( x ).

Khi đó, ta có P( x)  ( x  1 )( x   2 )...( x   n )

n
Với mỗi i thì  in  4  0   i  4   i  n 4, i  1, n (1)

Giả sử P ( x )  Q ( x ).S ( x ) , với Q( x)  ( x  1 )...( x   m ), S ( x)  ( x   m1 )...( x   n ) là các đa thức


không hằng với hệ số nguyên.

Suy ra 1... m  

Do đó, 1... m  (2)

2m
Từ (1) và (2) ta có 1... m  n 4m  2 n   và 1... m  4

2m 2m 2m
Suy ra 0   2 và    1  n  2m, m   *
n n n

Giả sử m là số lẻ. Khi đó, đa thức Q(x) có bậc lẻ nên đa thức Q(x) có nghiệm thực. Điều này
vô lí vì đa thức P( x)  x n  4  x 2 m  4 không có nghiệm thực.

Do đó, m là số chẵn. Vậy, n chia hết cho 4.

NGUYỄN THỊ QUYÊN Trang 14


NGHIỆM CỦA ĐA THỨC VÀ ỨNG DỤNG

Giả sử n chia hết cho 4.

Khi đó, n = 4m, m   * .

P( x)  x n  4  x 4 m  4 x 2 m  4  4 x 2 m  ( x 2 m  2 x m  2)( x 2 m  2 x m  2)

Suy ra P(x) khả quy.

Ví dụ 18.(IMO 1993) Chứng minh rằng đa thức P( x)  x n  5 x n 1  3, n   * bất khả quy trên
[x].

Giải.

Giả sử 1 ,  2 ,...,  n là tất cả các nghiệm của đa thức P( x) . Khi đó,

P( x)  ( x  1 )( x   2 )...( x   n )

n 1
Với mỗi i ta có  in  5 in 1  3  0   i  5 .  i  3, i  1, n (1)

Giả sử P( x) khả quy và P ( x )  Q ( x ).S ( x ) , với Q( x)  ( x  1 )...( x   m ), S ( x)  ( x   m1 )...( x   n )


là các đa thức không hằng với hệ số nguyên.

Thay x = 0 vào P( x) ta suy ra 1... m .  m1... n  3 và 1... m ,  m 1... n  

Không mất tính tổng quát, giả sử 1... m  3 và  m1... n  1

n 1 n 1
Từ (1) ta có 3m  (1  5)...( m  5) . 1... m  Q ( 5) 1... m  Q ( 5) .3n 1 , Q ( 5)   *

Do đó, m  n  1 . Suy ra S(x) là đa thức bậc nhất. Khi đó, S(x) sẽ có nghiệm nguyên hay P(x)
có nghiệm nguyên (2)

Mà nghiệm nguyên của P(x) chỉ có thể là -3; 3; -1; 1. Ta kiểm tra lại thì tất cả đều không là
nghiệm của P( x) . Mẫu thuẫn với (2).

Vậy, P( x) bất khả quy.

IV. GIẢI HỆ PHƯƠNG TRÌNH

NGUYỄN THỊ QUYÊN Trang 15


NGHIỆM CỦA ĐA THỨC VÀ ỨNG DỤNG


x  y  z  6

Ví dụ 19. Giải hệ phương trình  x 2  y 2  z 2  14 .
 1 1 1 11
   
 x y z 6

Giải.

Ta có ( x  y  z ) 2  x 2  y 2  z 2  2 xy  2 yz  2 zx  36  xy  yz  zx  11

1 1 1 11 x  y  z 11
Ta lại có       xyz  6
x y z 6 xyz 6

Theo định lí Viete đảo, ba số x, y, z là nghiệm của phương trình X 3  6 X 2  11X  6  0

 X  1 X  2  X  3

Vậy, nghiệm của hệ là {x; y; z}  {1; 2;3}.

x  y  z  0

Ví dụ 20.(Olympic SV 2009) Cho x, y, z là các số thực thỏa mãn  x 2  y 2  z 2  2.
 x3  y 3  z 3  0

Chứng minh rằng với mọi số tự nhiên n ta luôn có x 2 n 1  y 2 n1  z 2 n 1  0.

Giải.

Đặt u  x  y  z, v  xy  yz  zx, t  xyz

x  y  z  0 u  0 u  0
  2 
Theo giả thiết ta có  x 2  y 2  z 2  2. Suy ra u  2v  2   v  1
 x3  y 3  z 3  0 u 3  3uv  3t  0 
  t  0

Theo định lí Viete đảo, ba số x, y, z là nghiệm của phương trình X 3  X  0  X  0  X  1

Vậy, x 2 n1  y 2 n 1  z 2 n 1  02 n1  12 n1  (1) 2 n 1  0.

TÀI LIỆU THAM KHẢO


[1]Lê Hoành Phò, Chuyên Khảo Đa Thức, NXBĐHQG Hà Nội, 2013.

[2] Titu Andresscu, Mathematics Olympic Treasure.

NGUYỄN THỊ QUYÊN Trang 16


NGHIỆM CỦA ĐA THỨC VÀ ỨNG DỤNG

[3] mathscope.org.

[4] diendantoanhoc.net

NGUYỄN THỊ QUYÊN Trang 17


Chuyên đề :
SỬ DỤNG TÍNH LỒI CỦA TẬP HỢP ĐỂ GIẢI CÁC BÀI TOÁN VỀ HÌNH
HỌC TỔ HỢP
Giáo viên thực hiện: Nguyễn Mạnh Quyền
Trường THPT chuyên Nguyễn Chí Thanh – Đăk Nông
Báo cáo đang còn hoàn thiện nên còn một số sai sót tác giả sẽ gửi bản hoàn thiện
trong thời gian sớm nhất! xin gửi trước bản thảo để ban tổ chức duyệt trước nội
dung. Xin cám ơn!
Trước hết, xin nhắc lại một số kiến thức cơ bản về tập hợp lồi sẽ dung đến trong
chương này.
Giả sử Ω là một tập hợp cho trước (trên đường thẳng, mặt phẳng hoặc trong không
gian). Tập hợp Ω được gọi là tập hợp lồi nếu như với bất kì hai điểm A, B ϵ Ω, thì
cả đoạn thẳng AB (với hai đầu mút A và B) nằm trọn trong Ω.

Ω B
A

Thí dụ các đa giác lồi, đa diện lồi là những ví dụ cơ bản nhất về tập hợp lồi. Tập
hợp sau không phải tập hợp lồi.

B D

A
(như đã biết hình trên được gọi là một tứ giác lõm).
Dễ dàng thấy được các tính chất đơn giản sau đây về tập hợp lồi:
Nếu A, B là hai tập lồi
thì A ∩ B cũng là tập hợp lồi
A B

A B
Bằng quy nạp, có thể chứng minh được : Nếu , ,…, là các tập hợp lồi thì
∩ ∩ …∩ cũng là tập hợp lồi.
Chú ý rằng hợp của hai tập lồi A và B
A B
chưa chắc là tập hợp lồi.
( Xem hình vẽ minh họa bên).
Xét một ví dụ hay sau đây về tập hợp lồi.
Thí dụ: Trên mặt phẳng, cho hai đa giác lồi (F) và (G). Kí hiệu (H) là tập hợp
trung điểm của các đoạn thẳng có một đầu thuộc (F), còn đầu kia thuộc (G). Chứng
minh rằng (H) là một tập hợp lồi.

C1
C*

C2
A K M
A
(F) E B
M N
(G) B D2
D1 D*
Giả sử A và B là hai điểm bất kì thuộc (H). Vì A ∈ (H) nên A là trung điểm của
đoạn thẳng . Tương tự, B là trung điểm của đoạn thẳng . Ở đây, , ∈
( ), còn , ∈ ( ).
Gọi M, N tương ứng là trung điểm của , thì AMBN là hình bình hành.
Vì , ∈ ( ), mà ( ) là đa giác lồi nên ∈ ( ). Tương tự, ∈ ( ).
Có thể thấy rằng toàn bộ hình bình hành AMBN chính là quỹ tích các trung điểm
của đoạn thẳng có các đầu mút trên các đoạn thẳng , .
Theo định nghĩa, AB ∈ ( ). Vậy, ( ) là tập hợp lồi.
Chú ý:
1. Có thể làm trực tiếp đơn giản như sau:

Lấy E tùy ý trên AB, từ E kẻ đường thẳng song song với MB, đường này cắt AM
tại K. ∩ = ∗ và ta có K là trung điểm của ∗
. Bây giờ ∗ ∩
= ∗ , và cũng có E là trung điểm ∗ ∗ ( do KE // và K là trung điểm

của ). Như vậy, E ∈ ( ). Điều đó có nghĩa là ( ) là tập hợp lồi. Đó là đpcm.

2. Kết quả vẫn đúng nếu thay (F,) (G) thành hai tập lồi tùy ý (không cần thiết
(F,) (G) phải là các đa giác lồi).

Cho A là một tập hợp được cho trước và giả sử nó không phải là tập hợp lồi. Khi
đó, luôn luôn tồn tại một tập hợp lồi nhỏ nhất chứa A. Tập hợp này được gọi là bao
lồi của tập hợp A.

Ta thường dùng kết quả sau đây:

Cho hệ n điểm không cùng nằm trên một đường thẳng. Khi đó bao lồi của hệ điểm
này là một đa giác có k đỉnh ( ≤ ), mà các đỉnh của nó đều là những điểm thuộc
hệ đã cho. Các điểm còn lại ( − điểm), hoặc nằm trên biên, hoặc nằm bên trong
đa giác bao lồi.

Xem ví dụ hình ảnh bao lồi sau đây:

là bao lồi của hệ 10 điểm { , , ,…, , }.

A2 A3

A8
A1 A9 A10
A7 A4
A6
A5
§1. CÁC BÀI TOÁN SỬ DỤNG ĐỊNH LÍ HELLY

Định lí Helly là một trong các định lí quan trọng của hình học tổ hợp. Định lí này
cho ta một điều kiện đủ hữu hiệu để nhận biết rằng khi nào một họ các hình lồi có
giao khác rỗng.

I. ĐỊNH LÍ HELLY TRONG KHÔNG GIAN HAI CHIỀU ℝ :

Trong mặt phẳng cho n hình lồi ( ≥ 4). Biết rằng giao của ba hình lồi bất kì trong
chúng khác rỗng. Khi đó, giao của n hình lồi cũng khác rỗng.

Chứng minh:

Ta chứng minh bằng quy nạp theo số n các hình lồi.

1. Xét khi = 4.

Gọi , , , là 4 hình lồi có tính chất giao của 3 hình bất kì trong chúng là
khác rỗng. Vì ∩ ∩ ≠ ∅ nên tồn tại ∈ ∩ ∩ .

Tương tự, tồn tại:

∈ ∩ ∩ ; ∈ ∩ ∩ ; ∈ ∩ ∩ .

Chỉ có hai khả năng sau xảy ra:

a) Nếu 4 điểm , , , không hoàn toàn khác nhau. Khi đó, không giảm

tính tổng quát, ta cho ≡ . Từ đó suy ra ∈ ∩ ∩ ∩ .

Nên ∩ ∩ ∩ ≠ ∅.

Vậy kết luận của Định lí Helly đúng trong trường hợp khi = 4.

b) Nếu 4 điểm , , , là 4 điểm phân biệt, lúc này lại có hai khả năng

xảy ra:

b1) Bao lồi của , , , chính là tứ giác lồi .

Giả sử O là giao điểm hai đường chéo , .


A3

A1
O A2

A4

Do ∈ ∩ ∩ nên ∈ , ∈ ∩ ∩ nên ∈ .

Vì lồi mà ∈ , ∈ nên [ , ]∈ . Nói riêng, ∈ .

Lập luận hoàn toàn tương tự, suy ra ∈ , ∈ , ∈ . Điều đó có nghĩa là:

∈⋂ . Do đó, ⋂ ≠ ∅.

b2) Bao lồi của chúng là một tam giác chứa một điểm còn lại bên trong. Không
giảm tính tổng quát, ta có thể cho là tam giác chứa .

A2

A1 A4

A3

Vì , , đều thuộc , mà lồi nên toàn bộ miển trong tam giác


thuộc .
Mặt khác, ∈ ∩ ∩ ⇒ ∈⋂ .

Từ đó suy ra ⋂ ≠ ∅. Vậy, định lí Helly đúng khi = 4.

2. Giả sử kết luận của định lí Helly đúng đến ≥ 4.


3. Xét trường hợp khi có + 1 hình lồi, tức là ta có + 1 hình lồi

, ,…, , với giả thiết bất kì 3 hình lồi nào trong chúng đều có giao
nhau khác rỗng.

Xét các hình sau:

= ∩ .

Rõ ràng là lồi với mọi = 1, − 1 (vì = ), còn cũng là lồi vì nó


là giao của hai hình lồi à .

Xét ba hình lồi bất kì , , trong n hình lồi , ,…, .

Nếu trong chúng không có thì theo giả thiết

∩ ∩ = ∩ ∩ ≠ ∅.

Nếu trong chúng có = ∩ . Khi đó, có thể cho là = .

Từ đó, ∩ ∩ = ∩ ∩ ∩ .

Vì giao của ba hình lồi trong các hình lồi , , , là khác rỗng (giả
thiết) nên theo trường hợp = 4 ta có ∩ ∩ ∩ ≠ ∅.

Vậy, với n hình lồi , , … , thỏa mãn điều kiện giao của ba hình lồi bất
kì trong chúng khác rỗng, nên theo giả thiết quy nạp, suy ra ∩ ∩ …∩
≠ ∅. Điều đó có nghĩa là ∩ ∩ …∩ ∩ ≠ ∅.
Định lí Helly đúng trong trường hợp có + 1 hình lồi. Theo nguyên lí quy nạp
suy ra định lí Helly đúng với mọi ≥ 4. Định lí Helly được chứng minh trong
ℝ .

Chú ý: Ta thấy rằng điều kiện ≥ 4 là cần thiết. Thật vậy, hãy xét mệnh đề
tương tự với = 3.

“Cho một họ n hình lồi ( ≥ 3) trong mặt phẳng. Biết rằng giao của hai hình lồi
bất kì trong chúng khác rỗng. Khi đó, giao của n hình lồi cũng khác rỗng”.

Rõ ràng mệnh đề này không chắc đúng. Thật vậy, xét với = 3. Xét ba hình lồi:
đoạn thẳng AB, đoạn thẳng BC, đoạn thẳng CA.

Rõ ràng giao của hai hình lồi bất kì trong chúng khác rỗng.

Nhưng ∩ ∩ = ∅.

B C

II. ĐỊNH LÍ HELLY TRONG KHÔNG GIAN MỘT CHIỀU ℝ

Trên đường thẳng cho n hình lồi ( ≥ 3) . Biết rằng giao của hai hình lồi bất kì
trong chúng khác rỗng. Khi đó, giao của cả n hình lồi cũng khác rỗng.

Chứng minh:

Ta biết rằng hình lồi trên đường thẳng chỉ có thể là đoạn thẳng [ ; ], khoảng
( ; ), hay [ ; ), ( ; ] (ở đây, a có thể là −∞, còn b có thể là +∞). Ta chỉ xét
với các hình lồi là các đoạn thẳng, các trường hợp còn lại chứng minh hoàn toàn
tương tự.
Giả sử có n đoạn thẳng [ ; ], = 1, có tính chất sau: Bất kì giao của hai đoạn
thẳng nào trong chúng cũng khác rỗng, tức là [ ; ] ∩ ; ≠ ∅ với mọi ≠ .
Ta sẽ chứng minh ⋂ [ ; ] ≠ ∅.

Chú ý rằng [ ; ] ∩ ; ≠∅⇔ , ≥ , .

Thật vậy, giả sử [ ; ] ∩ ; ≠ ∅, khi đó tồn tại ∈[ ; ]∩ ; .

≤ ≤

≤ ≤

Hay , ≤ ≤ , .

Đảo lại, giả sử , ≤ , . Khi đó, rõ ràng ta có thể chọn c sao cho
, ≤ ≤ , (1).

Từ (1) suy ra ≤ ≤ ⇒ ∈ [ ; ]; ≤ ≤ ⇒ ∈ ; .

Điều đó có nghĩa là [ ; ] ∩ ; ≠ ∅. Nhận xét được chứng minh.

Từ đó suy ra ≥ . (2).

Từ (2) suy ra tồn tại c sao cho ≥ ≥ . (3).

Bất đẳng thức (3) chứng tỏ rằng ∈ [ ; ] với mọi = 1, .

Nói cách khác, ⋂ [ ; ] ≠ ∅.

Định lí Helly trong ℝ được chứng minh hoàn toàn.

Bây giờ ta sẽ áp dụng định lí Helly để giải hàng loạt các bài toán của hình học tổ
hợp liên qian đến tính giao khác rỗng của các hình lồi.

BÀI 1: Cho bốn nửa mặt phẳng lấp đầy mặt phẳng. Chứng minh rằng tồn tại ba
nửa mặt phẳng trong bốn nửa mặt phẳng ấy, sao cho chỉ riêng ba nửa mặt phẳng
này cũng lấp đầy mặt phẳng.

Giải:
Gọi , , , là bốn nửa mặt phẳng. Từ giả thiết ta có:

∪ ∪ ∪ =ℝ (1)

Rõ ràng là lồi với mọi = 1,4.

Từ (1) suy ra ∪ ∪ ∪ = ∅. (2)

( ở đây ̅ dùng để chỉ phần bù của tập hợp A).

Theo quy tắc Demorgan từ (2) có ∩ ∩ ∩ = ∅. (3)

Vì lồi nên cũng lồi với mọi = 1,4.

Giả thiết phản chứng không tồn tại ba nửa mặt phẳng nào trong số các ( = 1,4),
mà ba nửa mặt phẳng này lấp đầy mặt phẳng. Điều đó có nghĩa là với mọi i, j, k
phân biệt, mà i, j, k∈ {1, 2, 3, 4 } thì ∪ ∪ ⊂ ℝ .

Nói cách khác, ∪ ∪ ≠ ∅. (4)

Theo quy tắc Demorgan thì từ (4) có ∩ ∩ ≠ ∅. (5)

Từ (5) và áp dụng định lí Helly suy ra ∩ ∩ ∩ ≠ ∅. (6)

Bây giờ từ (3) và (4) suy ra mâu thuẫn, tức là phản chứng sai. Từ đó ta có điều phải
chứng minh.

Chú ý: Giả sử ℝ là cả mặt phẳng. Cho A là một hình phẳng trong ℝ . Khi đó kí
hiệu ̅ = { ∈ ℝ : ∉ }.
̅ gọi là phần bù của tập hợp A trong ℝ . Ta dễ dàng chứng minh quy tắc sau:

(gọi là quy tắc Demorgan của phép lấy phần bù).

∪ = ̅∩ ; ∩ = ̅∪ .

Bằng quy nạp, có thể mở rộng quy tắc Demorgan cho n tập hợp ( thí dụ

∪ ∪ …∪ = ∩ ∩ …∩ ).
BÀI 2: Cho n hình tròn ( ≥ 4) trên mặt phẳng. Biết rằng cứ với mỗi ba hình
tròn tùy ý, luôn tồn tại một hình tròn bán kính R chứa cả ba hình tròn. Chứng
minh rằng tồn tại một hình tròn bán kính R chứa cả n hình tròn đã cho.

Giải:

Gọi Ω là các hình tròn tâm , bán kính , Ω =( ; ), = 1, .

Gọi là các hình tròn có tâm là , bán kính − .

=( ; − ), = 1, .

Ω2
Ω1
( Oi,j,k ; R )
Ω3

Lấy i, j, k tùy ý (1 ≤ < < ≤ ), ta sẽ chứng minh ∩ ∩ ≠ ∅.

Thật vậy, theo giả thiết thì tồn tại hình tròn , , ; phủ Ω , Ω , Ω , tức là:

Ω ∪Ω ∪ Ω ⊂ , , ; .
Oi
Ωi ( Oi,j,k )

Nói riêng ta có Ω ⊂ , , ; .

Từ đó, , , ≤ − .

Hay ( ; − ) chứa , , .

Như vậy , , ∈ .

Lập luận tương tự ta có , , ∈ , , , ∈ .

Nói khác đi, ∩ ∩ ≠ ∅.

Giả sử ∗ ∈ ∩ ∩ …∩ .

Xét hình tròn ( ∗ ; ). Do ∗ ∈⋂ suy ra ∗ ∈ , với mọi = 1, .

Vì vậy, ( ∗ ; ) chứa ( ; ). Nói cách khác, ( ∗ ; ) ⊃ ⋃ Ω .

Đó là đpcm.

BÀI 3: Cho n hình tròn ( ≥ 4) trên mặt phẳng. Biết rằng cứ với mỗi ba hình
tròn tùy ý, luôn luôn tồn tại một hình tròn bán kính R nằm trong cả ba hình tròn
này. Chứng minh rằng tồn tại một hình tròn bán kính R nằm trong cả n hình tròn
đã cho.

Giải:
Ωj

Oj
Ωi Oi O
R
Ok

Ωk

R
O
O*
F1 Oi
Ri - R Ri

Ωi

Gọi Ω là các hình tròn tâm , bán kính ,

Ω =( ; ), = 1, .

Gọi là các hình tròn có tâm là , bán kính − .

=( ; − ), = 1, .
Lấy i, j, k tùy ý (1 ≤ < < ≤ ). Rõ ràng, theo giả thiết thì tồn tại hình tròn
Ω với tâm là O, bán kính R sao cho:

Ω⊂Ω ∩ Ω ∩Ω . (1)

Ta sẽ chứng minh ∩ ∩ ≠ ∅. (2)

(xem ở cuối bài).

Từ (2), theo định lí Helly suy ra ⋂ ≠ ∅.


∗ ∗
Giả sử ∈⋂ , tức là ∈ , với mọi = 1, .

Rõ ràng hình tròn tâm , bán kính R nằm trọn trong Ω .

Vậy ( ; ) nằm trong cả n hình tròn đã cho. Đó là đpcm.

Bây giờ ta chỉ còn chứng minh (2).

Thật vậy, do hình tròn Ω nằm trọn trong Ω nên tâm O ∈ .

Tương tự, ∈ , ∈ . Vì thế, ∈ ∩ ∩ .

Điề đó có nghĩa là ∩ ∩ ≠ ∅. Vậy (2) được chứng minh.

Chú ý: Xét bài toán tương tự sau:

Trên mặt phẳng cho n điểm, biết rằng ba điểm bất kì trong số đó có thể được phủ
bởi hình tròn bán kính 1. Chứng minh rằng khi đó, tất cả n điểm có thể được phủ
bằng hình tròn bán kính 1.

Xét điểm bất kì trong số n điểm. Gọi Ω là hinhg tròn tâm , bán kính 1. Ta
nhận thấy nếu có một hình tròn bán kính 1 phủ ( tức là chứa ) thì tâm của hình
tròn này phải thuộc Ω .

Theo giả thiết, với mọi i, j, k thì Ω ∩ Ω ∩ Ω ≠ ∅.

Theo định lí Helly, ta có Ω ∩ Ω ∩ … ∩ Ω ≠ ∅.



Gọi ∈⋂ Ω.

Hình tròn tâm ,bán kính 1 sẽ phủ n điểm đã cho. Đó là đpcm.
BÀI 4: Trên mặt phẳng cho n hình tròn ( ≥ 4). Giả sử cứ với mỗi ba hình tròn
tùy ý, đều có một hình tròn bán kính r cắt ba hình tròn này. Chứng minh rằng tồn
tại một hình tròn bán kính r cắt cả n hình tròn đã cho.

Giải:

O ri r
Ai

Ωi

Gọi là hình tròn tâm , bán kính , ( = 1, ),

=( ; ).

Gọi Ω là hình tròn tâm , bán kính + , ( = 1, ),

Ω =( ; + ).

Như vậy, tâm của tất cả các hình tròn có bán kính r mà cắt đều nằm trong Ω .

Xét n tập hợp lồi Ω , Ω , … , Ω .

Với i, j, k tùy ý mà i, j, k ∈ {1,2,3, … , }.

Theo giả thiết, tồn tại hình tròn , , ; cắt cả , , , tức là

, , ∈ Ω ∩ Ω ∩ Ω . Điều đó chứng tỏ rằng Ω ∩ Ω ∩ Ω ≠ ∅ ,với mọi

i, j, k ∈ {1,2,3, … , }. Theo định lí Helly suy ra ⋂ Ω ≠ ∅.


Vậy tồn tại ∗ ∈⋂ Ω.

Xét hình tròn tâm ∗ và bán kính r, ( ∗ ; ). Hình tròn này rõ ràng cắt với mọi
= 1, . Đó là đpcm.

BÀI 5: Cho một số hữu hạn ≥ 4 các đường thẳng. Biết rằng với ba đường
thẳng tùy ý luôn tồn tại hình tròn có bán kính R cắt cả ba đường thẳng. Chứng
minh rằng tồn tại một hình tròn có bán kính R cắt cả n đường thẳng đó.

Giải:

R
R Fi
di R

Giả sử , ,… , là họ hữu hạn các đường thẳng ( ≥ 4 ).

Với mỗi đường thẳng ta xét là hình cho bởi hai đường thẳng song song với
và cách một khoảng bằng R. Tâm các đường tròn bán kính R mà cắt thì phải
nằm trong . Rõ ràng là hình lồi với mọi = 1, . Như thế, ta có một họ hữu
hạn các tập hợp lồi , , … , . Theo giả thiết, với mọi , , ∈ {1,2, … , }, ta
luôn có ∩ ∩ ≠ ∅ ( vì với mọi , , luôn tồn tại một hình tròn cắt cả
, , ). Theo định lí Helly, suy ra

∩ ∩ …∩ ≠ ∅.

Lấy ∗ ∈ ∩ ∩ … ∩ . Hình tròn tâm ∗ , bán kính R, ( ∗


; ) sẽ cắt tất cả
các đường thẳng , , … , . Đó là đpcm.

BÀI 6: Cho một họ n các da giác lồi ( ≥ 3) đôi một cắt nhau. Chứng minh rằng
tồn tại một đường thẳng cắt tất cả các đa giác này.
Giải:

14

12

10

Pj
6 Pi

5
aj ai 5 bj bi10 15 20 25

14

12

10

5 5 10 15 20 25

x =α
2

Xét tất cả các đa giác , = 1, , đã cho trong hệ trục tọa độ Đề-các (Descartes)
vuông góc. Với mỗi đa giác , ta chiếu nó lên trục hoành và ta được đoạn
[ ; ]. Như vậy, ta có tương ứng 1-1.
⇔[ ; ], = 1, .

Theo giả thiết, với mọi ≠ , ( , ) ∈ {1, 2, … , } . Điều đó có nghĩa là

[ ; ]∩ ; ≠ ∅.

Theo định lí Helly, ⋂ [ ; ] ≠ ∅. Như vậy, tồn tại ∈⋂ [ ; ].

Do đó, đường thẳng = sẽ cắt tất cả n đa giác , = 1, . Đó là đpcm.

BÀI 7: Trên mặt phẳng cho n các da giác lồi ( ≥ 3) đôi một cắt nhau và một
phương l cho trước. Chứng minh rằng luôn kẻ được một đường thẳng song song
với phương l đã cho và cắt tất cả các đa giác.

Giải:

Pi

x
O
ai bi

Xét một hệ tọa độ aphin có một trục song song với phương l đã cho. Chiếu đa giác
theo phương l xuống trục kia, ta sẽ thấy được hình chiếu là [ ; ]. Và cách
giải bài toán này hoàn toàn tương tự như cách giải bài toán 6.

BÀI 8: Trên mặt phẳng có một họ hữu hạn các hình chữ nhật có các cạnh tương
ứng song song với hai trục tọa độ. Chứng minh rằng nếu hai hình bất kì trong
chúng có giao khác rỗng thì cả họ có giao khác rỗng.

Giải:

Lấy hệ tọa độ có trục song song với các cạnh của hình chữ nhật. Chiếu các hình
này lên Ox và Oy. Ta có sự tương ứng 1-1 sau đây:

[ ; ]⊂

[ ; ]⊂

Như vậy, ta có :

Họ các đoạn thẳng [ ; ]⊂ , và họ các đoạn thẳng [ ; ]⊂ , ∀ = 1, .

Do ∩ ≠ ∅ với mọi ≠ , ( , ) ∈ {1, 2, … , } , cho nên


[ ; ]∩ ; ≠ ∅, ( , ) ∈ {1, 2, … , } .

Từ đó, theo định lí Helly thì ⋂ [ ; ] ≠ ∅.

di

Fi

ci Fj

O ai bi

∗ [ ].
Vì thế, ta đã chứng minh được sự tồn tại của ∈⋂ ;
∗ [ ; ].
Tương tự, ta cũng chứng minh được sự tồn tại của ∈⋂
∗ ∗)
Điều đó chứng tỏ rằng ( ; ∈⋂ . Đó là đpcm.

BÀI 9: Trong mặt phẳng cho n điểm và khoảng cách giữa hai điểm bất kì trong
chúng không vượt quá 1. Chứng minh rằng có thể phủ chúng bằng một hình tròn
có bán kính = .

Giải:

R =1/√3
Mi
A B
C

Trước hết, ta có nhận xét rằng không có 3 điểm nào thẳng hàng, vì nếu có thifloaij
bỏ một điểm C (chú ý rằng khoảng cách giữa hai điểm bất kì trong chúng nhỏ hơn
hoặc bằng 1).

Giả sử các điểm đã cho là , = 1, ⇒ ; ≤ 1, ∀ ≠ .

Xét các hình tròn = ; , = 1, .


Lấy 3 điểm tùy ý, ta giả sử là , , .

Chỉ có các trường hợp sau có thể xảy ra:

1. lập thành tam giác không tù. Xét đường tròn ngoại tiếp tam giác

có tâm là O và R là bán kính (O nằm trong tam giác, do tam giác không
tù) lại có thể cho là ≥ 120°.
M1

O
M3
M2

1 1
. 1
= ≤2 ≤ 2 =
sin 60° √3 √3
2
(do ≤ 1).

⇒ = = ≤ ⇒ ∈ ∩ ∩

⇒ ∩ ∩ ≠ ∅.

2. lập thành tam giác tù.

M1

M3
M2 I

Giả sử góc tù. Khi đó đường tròn đườn kính phủ tam giác .

Gọi I là trung điểm của (cạnh lớn nhất), ta có:

= > ; = ≤ < .

⇒ ∈ ∩ ∩ ⇒ ∩ ∩ ≠ ∅.

Vậy ta có với i, j, k tùy ý thì ∩ ∩ ≠ ∅.

Theo định lí Helly suy ra ⋂ ≠ ∅.

Giả sử ∗ ∈⋂ . Xét hình tròn tâm ∗ và bán kính bằng .


O*

Fi Mi

Vì ∗ ∈ nên ∗ ≤ . Từ đó suy ra hình tròn tâm ∗ và bán kính chứa .


√ √

Vậy hình tròn tâm ∗ phủ n điểm , , = 1, . Đó là đpcm.

§2. CÁC BÀI TOÁN SỬ DỤNG PHƯƠNG PHÁP LẤY BAO LỒI.
BÀI 1: Trên mặt phẳng cho một số hữu hạn điểm. Chứng minh rằng luôn luôn
tìm được một điểm sao cho gần nó nhất có không quá ba điểm đã cho.

Giải:
Giả sử , ,…, là n điểm đã cho.
Theo nguyên lí cực hạn thì tồn tại = min , , , , .

Đưa vào xét tập hợp Ω như sau: Ω = |∃ ≠ , = .

Giả sử Ω = , ,…, . Dễ dàng thấy rằng Ω ≠ ∅ ( vì tồn tại khoảng cách


ngắn nhất d ). Xét bao lồi của tập hợp Ω. Chỉ có hai khả năng xảy ra:
1. Nếu bao lồi của Ω là một đoạn thẳng AB.
A B
Khi đó gần đỉnh đầu mút của nó chỉ có không quá một điểm của hệ. Thật vậy,mọi
điểm cách A một đoạn bằng d là các điểm của tập hợp Ω, và do đó, dixnhieen nó
thuộc bao lồi của Ω, tức là thuộc AB. Như vậy, có tối đamột điểm gần A nhất.
2. Nếu bao lồi của Ω là một đa giác lồi. ta chọn A là một đỉnh của bao lồi của Ω

A
B1
B2 d d
A B3
Bi B
B4

Giả sử gần A nhất có quá ba điểm có khoảng cách bằng d tới A. Theo định nghĩa
của d , thì với mọi ≠ , ≥ (ở đây, , , , , … là các điểm có khoảng
cách tới A đều là d ). Xét tam giác có = = , còn ≥ , từ đó
suy ra ≥ 60°, nên:

+ + + ⋯ ≥ 180°
Do vậy, ≥ + + + ⋯ ≥ 180° ( là góc của đa giác bao lồi)
Rõ ràng ≤ 180°, mâu thuẫn này chứng tỏ giả thiết phản chứng là sai, suy ra
đpcm.
BÀI 2: Trong mặt phẳng cho sáu điểm phân biệt. Với mỗi hai điểm trong chúng
ta nối thành một đoạn thẳng. Chứng minh rằng tỉ số giữa đoạn thẳng dài nhất với
đoạn thẳng ngắn nhất lớn hơn hoặc bằng √3.
Giải:
Giả sử ={ , , , , , } là tập hợp sáu điểm đã cho.
Đặt : = max , = min .
, , , ,

Ta phải chứng minh rằng : ≥ √3.


Xét các trường hợp sau:
1. Nếu tồn tại ba điểm thẳng hàng.Không giảm tính tổng quát có thể cho là ba
điểm , , và ≤ .

A1 A2 A3
Rõ ràng ta có ≥ 2 . Theo định nghĩa d thì ≥ ,
suy ra ≥2 .

Lại theo định nghĩa , thì ≤ suy ra : ≥ 2 > √3.


Điều khẳng định của bài toán đúng trong trường hợp này.
2. Tồn tại ba điểm tạo ra một tam giác có góc lớn nhất lớn hơn hoặc bằng 120°

A2
α
A3
A1
Giả sử ba điểm đó là , , và 120° ≤ < 180°.
Theo định lí hàm số Côsin thì :
= + − 2. . . cos
⇒ ≥ + + 2. .
Vì ≥ , còn ≤ , ≤ nên suy ra:

≥ + + =3 ⇒ ≥ √3.
Điều khẳng định của bài toán đúng trong trường hợp này.
3. Xét trường hợp tổng quát không có ba điểm nào thẳng hàng.
Gọi Ω là bao lồi của sáu điểm đã cho. Khi đó chỉ xảy ra các khả năng sau:
3.1. Bao lồi là tam giác:
Giả sử đó là tam giác .Vì không có ba điểm nào trong số sáu điểm đã cho
thẳng hàng, nên ba điểm còn lại , , nằm hẳn trong tam giác .

A1

A4

A2 A3

Xét chẳng hạn điểm . Khi đó, vì + + = 360° nên


phải tồn tại, chẳng hạn ≥ 120°.
Khi đó, áp dụng trường hợp 2, suy ra đpcm.
3.2. Bao lồi là tứ giác, giả sử đó là tứ giác .
Vì không có ba điểm nào trong số sáu điểm đã cho thẳng hàng, nên suy ra hai điểm
còn lại , phải nằm hẳn trong tam giác hoặc tam giác .
Giả sử thuộc phần trong tam giác .

A2
A3

A1 A5

A4
Lập luận như trên trong ba góc , , phải có một góc lớn
hơn hoặc bằng 120°.
Giả sử đó là ≥ 120°. Khi đó lại áp dụng trường hợp 2. Suy ra đpcm.
3.3. Bao lồi là ngũ giác, giả sử đó là ngũ giác .
Từ giả thiết suy ra không thuộc các cạnh của ngũ giác, cũng như không thuộc
các đường chéo , . Vì thế, phải thuộc vào phần trong một trong ba
tam giác , , .
Có thể cho là thuộc vào phần trong của tam giác .

A1

A2

A5
A6
A3
A4
Lại lập luận như trên có thể cho là ≥ 120°, ta lại quay về trường hợp 2.
3.4. Bao lồi là lục giác. Giả sử là lục giác .

A2 A3

A4
A1

A6 A5

Vì + + + + + = 720°
Suy ra max , , , , , ≥ 120°.
Từ đó giả sử ≥ 120°. Khi đó, xét tam giác và ta quay về trường hợp 2.
Bài toán được chứng minh hoàn toàn.
BÀI 3: Trên mặt phẳng cho 2001 điểm phân biệt sao cho không có ba điểm nào
thẳng hàng. Chứng mnh rằng có thể chọn được trong số này 1000 cặp điểm
( ; ) (1 ≤ ≤ 1000), sao cho 1000 đoạn thẳng này cắt nhau ít nhất tại
500 điểm phân biệt.
Giải:
Trước hết ta phải chứng minh bổ đề phụ trợ sau đây:
Bổ đề:
Cho năm điểm trên mặt phẳng sao cho không có ba điểm nào thẳng hàng. Khi đó
trong năm điểm này có thể chọn ra bốn điểm A, B, C, D sao cho ABCD là tứ giác
lồi ( với AC và BD cắt nhau).
Chứng minh bổ đề.
Xét bao lồi của năm điểm A, B, C, D, E đã cho. Vì không có ba điểm nào thẳng
hàng nên dĩ nhiên bao lồi của chúng cũng không thể nào là một đoạn thẳng. Chỉ có
các khả năng sau có thể xảy ra.
1. Bao lồi là tứ giác (có thể cho là tứ giác ABCD). Khi đó tứ giác lồi ABCD
thỏa mãn yêu cầu đề bài.

B C

E
A

D
2. Bao lồi là ngũ giác ABCDE. Khi đó, có thể chọn tứ giác lồi cần tìm là một
trong các tứ giác ABCD, BCDE, CDEA, DEAB, hoặc EABC.
C
B

D
A
E
3. Bao lồi là tam giác ( mà có thể cho là tam giác ABC ). Khi đó hai điểm D, E
nằm hẳn bên trong tam giác ABC, ngoài ra E không nằm trên các đường thẳng AD,
BD, CD.
Vì thế giả sử ∩ = , ∩ =

E H
K D

B C

Thì K, H không trùng với các đỉnh của tam giác ABC.
Bây giờ tứ giác cần chọn có thể là tứ giác BDEC. Bổ đề đã được chứng minh.
Bây giờ trở lại bài toán trên.
Do số điểm đã cho là hữu hạn nên tồn tại đường thẳng d không song song với bất
kì một đoạn thẳng nào nối hai điểm trong 2001 điểm đã cho, vì thế có thể giả thiết
thêm là cả 2001 điểm nằm hoàn toàn về một phía bên phải của d.
Dịch chuyển d song song với chính nó. Khi đó, d lần lượt gặp các diểmđã cho (
mỗi lần gặp một điểm). Đường thẳng song song với d khi gặp điểm đầu tiên sẽ kí
hiệu là , gặp điểm thứ hai là , …, và là đường thẳng qua điểm cuối cùng.
Như vậy ở bên trái các đường , , ,…, , chứa đúng 5, 9, 13,…,
1997, 2001 điểm đã cho. Các đường thẳng này chia mặt phẳng thành 500 dải, mỗi
dải chứa năm điểm. Trong dải 1 có năm điểm, theo bổ đề có thể chọn ra bốn điểm
(mà sẽ gọi là , , , ) sao cho ∩ = .
Ngoài ra, trong dải còn một điểm “tự do”.

B1
A2
C1
C2

A1 B2
dải 1 dải 2
dải 3

Xét bộ năm điểm trong đó có một điểm “tự do” còn lại và bốn điểm trong dải 2.
Lại theo bổ đề, ta có thể chọn ra bốn điểm (mà sẽ gọi là , , , ) sao cho
∩ = .
, , , , dĩ nhiên nằm trong dải 1, còn nằm trong dải 2 (vì ít nhất một
trong hai đoạn , nằm trọn trong dải 2). Lúc này vần còn lại một điểm
“tự do”.
Kết hợp bốn điểm ở dải 3, với điểm “tự do” còn lại sau hai bước làm như trên,lại
theo bổ đề trên, tồn tại bốn điểm (mà sẽ gọi là , , , ) sao cho ∩
= . Lập luận như trên thuộc dải 3 và còn lại một điểm “tự do”.
Cứ làm như vậy cho đến dải 500, ta thu được 500 cặp.
∩ =
∩ =

∩ =
Đó là đpcm.
BÀI 4: Chứng minh rằng trong chín điểm bất kì trên mặt phẳng mà ba trong
chúng không thẳng hàng, luôn tồn tại năm điểm lập thành một ngũ giác lồi.
Giải:
Trước hết, ta chứng minh bổ đề sau.
Bổ đề:
Nếu một tập hợp hữu hạn điểm trên mặt phẳng mà trong đó không có ba điểm nào
thẳng hàng, và không có năm điểm nào trong tập hợp đó lại là các đỉnh của một
ngũ giác lồi, thì khi đó tập đã cho gồm không quá tám điểm.
Chứng minh bổ đề.
Kí hiệu S là tập hợp hữu hạn điểm đã cho. Kí hiệu K là tập đỉnh của bao lồi của tập
hợp S. Kí hiệu L là tập đỉnh của bao lồi của tập S \ K, và kí hiệu = ( \ ) \ .
( Thí dụ minh họa : xem hình vẽ dưới đây :
A3
A2
A5 A7
A8
A1
A6
A4
={ , , , , , , , }
= tứ giác .
\ ={ , , , }
= tam giác
= ).
Chỉ có thể xảy ra các trường hợp sau đây:
1. = ∅.
2. Tập hợp M chỉ gồm duy nhất một điểm.
3. Tập hợp M chứa ít nhất hai điểm trở lên.
Xét lần lượt các khả năng trên:
1. Nếu = ∅. Khi ấy tập hợp S chỉ chứa các đỉnh của hai đa giác lối K và L.
Từ giả thiết suy ra K và L chỉ có thể là tam giác hoặc tứ giác. Vậy S có không quá
tám điểm. Kết luận của bố đề đúng trong trường hợp này.
2. Tập M chứa một điểm duy nhất, và ta sẽ kí hiệu điểm đó là P. Lại có hai khả
năng xảy ra.
2.1. Nếu L là tứ giác. Vì P phải nằm hẳn trong tứ giác ABCD ( A, B, C, D là
bốn đỉnh của bao lồi L), và lại không có ba điểm nào của S thẳng hàng, nên P
phải nằm hẳn bên trong một trong hai tam giác : chẳng hạn tam giác ABC.
D
B

A
P

C
2.2. Nếu L là tam giác chẳng hạn, khi đó P phải nằm trong tam giác ABC.

Q
B

P
A C
Như thế trong trường hợp tập hợp M chứa một điểm duy nhất, thì luôn tồn tại
ba điểm A,B,C của S sao cho P nằm hẳn bên trong tam giác ABC.
Kẻ các tia PA, PB, PC. Ba tia này chia mặt phẳng thành ba góc APB, BPC
và APC. Dễ thấy rằng trong mỗi một góc này, có chứa nhiều nhất là một điểm của
K (chú ý là K là tập các đỉnh của bao lồi các điểm của S). Thật vậy, nếu không phải
như thế giả sử trong góc APB có chứa ít nhất hai đỉnh R, Q của đa giác lồi K (chú ý
rằng A, B, P ∉ K , nên chúng khác R, Q). Ta thu được ngũ giác lồi RQBPA ,với
năm đỉnh là các điểm nằm trong tập hợp S. Điều này trái với giả thiết về S. Vì lẽ đó
K có nhiều nhất là ba điểm (vì trong mỗi góc ta xét có chứa nhiều nhất một điểm
của K).
Theo giả thiết đa giác bao lồi L có nhiều nhất bốn đỉnh, K có ba điểm, M có
duy nhất một điểm. Vậy S chứa không quá tám điểm.
Bổ đề là đúng trong trường hợp này.
3. Bây giờ xét khi M có chứa ít nhất từ hai điểm trở lên.
Kí hiệu E, F là hai điểm bất kì của tập = ( ∖ )∖ .
Do đường thẳng nối E, F không đi qua các đỉnh của đa giác L (vì không có ba
điểm nào của S mà lại thẳng hàng), nên EF phải cắt hai cạnh của đa giác L.

C B

F
D E
A
Lại có ba khả năng xảy ra:
3.1. Nếu EF cắt hai cạnh kề nhau của tứ giác bao lồi L (xem hình vẽ)
Khi ấy ba đỉnh B, C, D của L cùng với E, F tạo thành một ngũ giác lồi. Đó là
điều vô lí. Vậy khả năng này không xảy ra.
3.2. Nếu EF cắt hai cạnh đối diện nhau của tứ giác bao lồi L (xem hình vẽ)

N
A
R C

T F
E
B

D
Kẻ các tia FA, FB, EC, ED. Các tia này cùng với đoạn thẳng EF chia mặt phẳng
tọa độ thành bốn phần.
a) Nếu bên trong góc AFB (một trong bốn phần trên) có chứa hai đỉnh của đa
giác bao lồi K, chẳng hạn hai đỉnh R, T. Khi đó các điểm A, R, T, B, F lập thành
một ngũ giác lồi. Đó là điều vô lí.
Vậy bên trong góc AFB có nhiều nhất là một điểm của đa giác K. Tương tự, bên
trong góc CEB có nhiều nhất là một điểm của đa giác K.
b) Bên trong hai phần còn lại không có điểm nào của K.
(Thí dụ có điểm N, thì năm điểm N, A, F, E, C lại là năm đỉnh của một ngũ giác
lồi ).
Như thế K có nhiều nhất là hai điểm (mỗi góc AFB và CED chứa một điểm).
Nhưng đó là điều vô lí vì bao lồi K phải là tam giác hoặc tứ giác.
Vậy khả năng này không xảy ra.
3.3. Bây giờ ta xét khả năng khi bao lồi L là tam giác.

F E
B C
Giả sử bao lồi là tam giác ABC. Trong các đỉnh của tam giác ấy có một đỉnh
nằm
về hẳn một phía của đường thẳng EF, còn hai đình kia nằm về phía bên kia của EF
(xem hình vẽ).
Lập luận giống như trên ta thấy rằng trong các góc AEC, AFB chứa nhiều
nhất là một đỉnh của đa giác bao lồi K. Trong phần tạo thành bởi FB, EC và đoạn
thẳng EF không thể có điểm nào của bao lồi K. Điều này dẫn đến K có tối đa hai
điểm là vô lí, vì K phải là tam giác hoặc tứ giác.
Vậy, qua lập luận trên suy ra trường hợp tập M chứa ít nhất hai điểm trở lên
là không thể xảy ra. Bổ đề được chứng minh.
Bây giờ quay trở lại bài toán của ta.
Giả sử kết luận của bài toán không đúng, tức là trong tập chín điểm (mà
không có điểm nào thẳng hàng) không thể chọn được năm điểm nào đều là đỉnh
của một ngũ giác lồi.
Khi đó theo bổ đề ta dẫn đén ngay mâu thuẫn ( vì nếu như vậy số điểm đã
cho phải nhỏ hơn hoặc bằng tám). Vậy giả thiết phản chứng là sai. Bài toán được
giải hoàn toàn.
Chú ý:
Bổ đề trong bài 3 và bài 4 nói trên chỉ là trường hợp riêng của bài toán sau đây do
nhà toán học người Hung-ga-ri P.Ezdos đề xướng vào khoảng những năm 30 của
thế kỉ XX.
Bài toán P.Ezdos (giả thuyết P.Ezdos).
Trên mặt phẳng cho trước một tập hợp gồm 2 + 1 điểm ( ≥ 3) có tính chất là
trong tập điểm đó không có ba điểm nào thẳng hàng. Khi ấy liệu chăng luôn luôn
tồn tại n điểm của tập hợp điểm đó mà chúng là các đỉnh của một đa giác lồi n
cạnh.
Nhận xét:
 Với = 3, kết quả là hiển nhiên.
 Với = 4, đó chính là kết quả của bổ đề trong bài 3.
 Với = 5, đó chính là kết quả của bài 4.
Bài toán này cho đến nay vẫn chưa giải quyết được cho trường hợp tổng quát.
BÀI 5: Cho n điểm trong mặt phẳng, với > 4 và không có ba điểm nào thẳng
( )( )
hàng. Chứng minh rằng có ít nhất tứ giác lồi có đỉnh nằm trong số n
điểm đã cho.
Giải:
 Trước tiên xét với = 5. Theo bổ đề của bài 3 có ít nhất một tứ giác lồi mà
bốn đỉnh của nó chọn trong năm điểm đã cho. Lại để ý rằng khi = 5 thì:
(5 − 3)(5 − 4)
=1
2
Vậy kết luận của bài toán đúng khi = 5.
 Xét với > 5. Vì không có ba điểm nào thẳng hàng nên số cách chọn năm
điểm trong n điểm là:
( )( )( )( )
= .

Mỗi cách chọn này, theo trên ( trường hợp = 5)cho ta ít nhất một tứ giác lồi. Tuy
nhiên bất kì tứ giác lồi nào trong số đó cũng có thể được lập từ ( − 4) tập hợp
khác nhau gồm năm điểm nói trên.
Do đó có ít nhất
( )( )( )
( )
=

số tất cả các tứ giác lồi được thành lập từ n điểm đã cho.


Bây giờ ta chỉ còn chứng minh bất đẳng thức sau (với ≥ 5).
( )( )( ) ( )( )
≥ (1)

Rõ ràng
(1) ⇔ ( − 1)( − 2) ≥ 60( − 4)
⇔ ( − 1)( − 2) − 60( − 4) ≥ 0.
⇔ ( − 5)( − 6)( + 8) ≥ 0.
⇔ ( − 5)( − 6) ≥ 0. (2)
Do ≥ 5 và n nguyên nên (2) hiển nhiên đúng.
Đó chính là đpcm.
BÀI 6: Trên mặt bàn đặt n hình vuông bằng cát tông và n hình vuông bằng nhựa
sao cho các hình vuông bằng cát tông (cũng như các hình vuông bằng nhựa)
không có điểm chung (kể cả điểm ở trên biên).Sau khi xếp, người ta nhận thấy
rằng tập hợp các đỉnh hình vuông bằng nhựa hoàn toàn trùng với tập hợp đỉnh
hình vuông bằng cát tông.
Chứng minh rằng các hình vuông bằng cát tông và các hình vuông bằng nhựa
được sắp xếp hoàn toàn trùng nhau.
Giải:
Giả thiết phản chứng kết luận của bài toán không đúng, tức là các hình
vuông bằng cát tông và các hình vuông bằng nhựa được sắp xếp không hoàn toàn
trùng nhau.
Loại bỏ đi các hình vuông hoàn toàn đặt trùng khít nhau. Khi đó, từ giả thiết
phản chứng suy ra còn lại các hình vuông bằng nhựa và cát tông, trong đó không
có hình vuông bằng nhựa và cát tông nào lại trùng khít nhau.
Xét tập hợp các đỉnh hình vuông bằng nhựa còn lại. Rõ ràng, từ giả thiết suy
ra đó cũng chính là tập hợp các đỉnh hình vuông bằng cát tông. Xét bao lồi của tập
hợp các đỉnh này.
Xét một đỉnh A tùy ý của bao lồi vừa xét. Vì mọi hình vuông cát tông và
nhựa còn lại đều nằm hẳn trong bao lồi này, nên nói riêng tại đỉnh A có hai hình
vuông khác loại và nằm trọn vẹn trong góc tại đỉnh (rõ ràng góc này nhỏ hơn 180°)

A
B

Trong hai hình vuông này (dĩ nhiên chúng không trùng khít nhau), sẽ tồn tại
một đỉnh B của hình vuông này nằm hẳn trong hình vuông kia. (Giả sử B là đỉnh
hình vuông cát tông).Theo giả thiết thì qua đỉnh B phải có một hình vuông nhựa
nhận B làm đỉnh. Điều này dẫn đến mâu thuẫn với việc các hình vuông nhựa không
có điểm chung.
Vậy giả thiết phản chứng là sai, tức là các hình vuông cát tông và nhựa hoàn
toàn trùng khít nhau.
Đólà đpcm.
BÀI 7: Trên mặt phẳng cho 2005 điểm , trong số đó không có ba điểm nào thẳng
hàng và không có bốn điểm nào cùng nằm trên một đường tròn. Chứng minh
rằng từ các điểm đó có thể chọn ra được ba điểm sao cho có đúng 1001 điểm
trong số còn lại nằm trong đường tròn đi qua ba điểm đã chọn, còn 1001 điểm
kia thì nằm ngoài.
Giải:
Lấy bao lồi của 2005 điểm. Như đã biết bao lồi này là một đa giác. Do không có ba
điểm nào thẳng hàng, nên mỗi cạnh của đa giác bao lồi chỉ chứa đúng hai điểm là
hai đỉnh của mỗi cạnh. Lấy một cạnh bất kì của đa giác bao lồi và giả sử cạnh đó là
AB.

B
A

C1002
Rõ ràng đường thẳng qua A, B đặt 2003 điểm còn lại về một phía của đường
thẳng này.
Đánh số tất cả 2003 điểm còn lại là , , …, sao cho:
< <⋯<
(chú ý rằng điều này là được vì không có bốn điểm nào cùng nằm trên một đường
tròn).
Xét đường tròn ngoại tiếp tam giác .
Dễ thấy các điểm , , …, nằm ngoài đường tròn ngoại tiếp tam giác
, còn các điểm , , …, nằm trong đường tròn này.
Đó chính là đpcm.
(Xem hình vẽ: ở đây 1 ≤ ≤ 2003).
BÀI 8: Trên mặt phẳng cho n – giác đều. Chứng minh rằng bao lồi của nó là một
đa giác có không ít hơn n đỉnh.
Giải:
Rõ ràng bao lồi của nó là một đa giác lồi mà các đỉnh của nó nằm trong tập hợp các
đỉnh của n – giác đều đã cho. Gọi m là số đỉnh của đa giác bao lồi. Tổng các góc
trong của đa giác bao lồi này là ( − 2) .
Số đo của mỗi góc trong n – giác đều là:
( − 2)

Chú ý rằng bao lồi của n – giác đều phải chứa cả n – giác đều ở bên trong. Vì thế
( )
góc ở mỗi đỉnh của m – giác bao lồi đều phải lớn hơn hoặc bằng .
Gọi là góc nhỏ nhất trong m góc của đa giác bao lồi. Khi đó, hiển nhiên ta có :
( )
≤ . (1)

( )
Mặt khác, ≥ . (2)

Vì thế, (1) và (2) suy ra:


( − 2) ( − 2)

2 2
⇔ 1− ≥1−

1 1
⇔ ≥

⇔ ≥
Vậy số cạnh của đa giác bao lồi không ít hơn n.
Đó là đpcm.
BÀI 9: Trong mặt phẳng cho 100 điểm, trong đó không có ba điểm nào thẳng
hàng. Xét tất cả các khả năng tạo thành tam giác có đỉnh từ những điểm này.
Chứng minh rằng có nhiều nhất 70% những tam giác nói trên là những tam giác
nhọn.
Giải:
Ta có nhận xét sau:
Nếu cho năm điểm, trong đó không có ba điểm nào thẳng hàng, thì tồn tại ít nhất
ba tam giác không nhọn có đỉnh là những điểm này.
(Nhận xét được chứng minh ở cuối bài).
Bây giờ trở lại bài toán.
Từ 100 điểm, ta có cách chọn bộ năm điểm. Theo bổ đề có thể chọn
được 3 tam giác không nhọn (chúng có thể trùng nhau). Tuy nhiên cùng một
tam giác có thể tạo bởi những bộ năm điểm khác nhau; nghĩa là nó nằm trong
bộ năm điểm.

3 5100
Như vậy có thể chọn ít nhất 2 tam giác không nhọn.
97

Mặt khác, số tất cả các tam giác chọn từ 100 điểm đã cho là .
Vậy số lượng tam giác nhọn nhiều nhất có thể là :
5
3
− 100 2
97

Gọi k là tỉ số giữa số tam giác nhọn và tất cả các tam giác. Từ lập luận trên suy ra :

≤ (1)

3 5100 3.100.99.98.97.96.1.2.3.1.2 3 7
VP (1) = 1 − 2 3 =1− =1− = .
1.2.3.4.5.97.96.100.99.98 10 10
97 100
7
Từ (1) đi đến ≤ .
10
Nói cách khác số tam giác nhọn nhiều nhất chỉ chiếm 70% tổng số tất cả các
tam giác. Đó là đpcm.
Bây giờ ta chứng minh nhận xét đã nêu ở trên như sau:
Lấy bao lồi của năm điểm A, B, C, D, E. Chỉ có thể có các trường hợp sau đây:
1. Nếu bao lồi là tam giác (giả sử là tam giác ABC).
Vì không có ba điểm nào thẳng hàng nên D, E nằm trong tam giác ABC.
Xét chẳng hạn điểm E. Ta có:
+ + = 360°.
Mặt khác 0 < < 180° ; 0 < < 180° ; 0 < < 180°, nên
trong ba góc , , có ít nhất hai góc lớn hơn hoặc bằng 90°.
(Thật vậy, nếu trái lại chỉ có một góc, chẳng hạn góc ≥ 90°. Khi đó,
< 90° ; < 90°
⇒ + < 180° ⇒ > 180° là điều vô lí).
Vì thế trong ba tam giác AEC, BEC, AEB có ít nhất hai tam giác không
nhọn. Tương tự, trong ba tam giác DAB, DBC, DCA cũng có ít nhất hai tam giác
không nhọn. Vậy số tam giác không nhọn chọn được từ năm điểm trong trường
hợp này ít nhất là bốn.
2. Nếu bao lồi là tứ giác (giả sử đó là tứ giác ABCD).
Để ý rằng :
+ + + = 360°.
Do đó tồn tại ít nhất một trong bốn góc , , , không phải
là góc nhọn. Như vậy ít nhất một trong bốn tam giác BAD, ADC, DCB, CBA
không phải là tam giác nhọn. Điểm thứ năm E phải nằm hẳn trong tứ giác ABCD
(do không có ba điểm nào trong số năm điểm đã cho thẳng hàng). Vì thế có thể cho
là E nằm hẳn bên trong tam giác ABC. Lập luận tương tự như trên, ít nhất hai
trong ba tam giác EBC, ECA, EAB không phải là tam giác nhọn.
Vậy số tam giác không nhọn chọn được từ năm điểm trong trường hợp này ít nhất
là ba.
3. Nếu bao lồi là ngũ giác ABCDE.
Tổng các góc trong của ngũ giác là (5 − 2). 180° = 540°.
Vì vậy suy ra có ít nhất hai góc của ngũ giác là không nhọn. Lại có hai khả năng
xảy ra:
3.1. Hai đỉnh của hai góc không nhọn là kề nhau (giả sử đó là các đỉnh A, B)
Khi đó ta thấy ngay EAB, ABC, là các tam giác không nhọn.
Xét tiếp tứ giác AEDC. Rõ ràng ít nhất một trong bốn góc , , ,
là không nhọn ( giả sử ≥ 90° ).
Khi ấy EDC là tam giác không nhọn.
Tóm lại, trong trường hợp này, từ năm điểm đã cho có ít nhất ba tam giác
không nhọn.
3.2. Hai đỉnh của hai góc không nhọn không kề nhau
(giả sử đó là các đỉnh A,C ).
Khi đó dĩ nhiên các tam giác EAB, BCD là các tam giác không nhọn.
Xét tứ giác ACDE.
Vì ít nhất một trong bốn góc , , , là không nhọn, nên ít nhất
một tronh bốn tam giác CAE, AED, EDC, DCA không phải là tam giác nhọn.
Điều đó có nghĩa là trong trường hợp này ta có ít nhất ba tam giác không
nhọn.
Nhận xét được chứng minh, và vì thế bài toán được giải hoàn toàn.
Tập huấn GV THPT chuyên toán Bài viết chủ đề đại số 1

SỬ DỤNG TÍNH DUY NHẤT CỦA ĐIỂM BẤT ĐỘNG


ĐỂ GIẢI MỘT SỐ PHƯƠNG TRÌNH HÀM

Tóm tắt. Từ hai bài toán phương trình hàm của đề thi IMO năm 1983 và đề thi
IMO năm 1994, tác giả thu được và phát triển một kĩ thuật sử dụng tính duy nhất
của điểm bất động của hàm số để giải phương trình hàm.

Phần đầu của bài viết trình bày một số bài toán và lời giải của nó mà trong những
lời giải đó có sử dụng tính duy nhất của điểm bất động nhằm minh hoạ kĩ thuật nói
trên. Phần tiếp theo của bài viết giới thiệu một vài bài tập.

Nội dung bài viết.


Định nghĩa. Cho một tập hợp X và một hàm số f : X → X, x0 ∈ X được gọi là
điểm bất động của f nếu và chỉ nếu f (x0 ) = x0 .

Xét bài toán 1 và bài toán 2 được trích từ đề thi IMO năm 1983 và đề thi IMO năm
1994.
Bài toán 1. Tìm tất cả các hàm số f : R+ → R+ thỏa

f (xf (y)) = yf (x) với mọi x, y ∈ R+ . (1)

và f (x) → 0 khi x → +∞.

(IMO 1983)

Lời giải.

Trong (1), thay x = 1, y = 1, ta được f (f (1)) = f (1).


Trong (1), thay y = f (1), x = 1, ta được (f (1))2 = f (f (f (1))) = f (f (1)) = f (1).
Suy ra f (1) = 1, tức là 1 là điểm bất động của f .

Trong (1), thay y = x, ta được f (xf (x)) = xf (x), ∀x ∈ R+ .


Suy ra xf (x), ∀x ∈ R+ là điểm bất động của f .
Bây giờ ta chứng minh nếu a là một điểm bất động của f thì a = 1.

Thật vậy, giả sử a > 1, ta có af (a) = a2 là điểm bất động của f


Mặt khác, ta có f (a4 ) = f (a2 f (a2 )) = a2 f (a2 ) = a4 .

Huỳnh Vĩnh Sang THPT chuyên Nguyễn Đình Chiểu - Đồng Tháp
Tập huấn GV THPT chuyên toán Bài viết chủ đề đại số 2

Suy ra a4 cũng là điểm bất động của f .


Bằng cách chứng minh quy nạp, ta chứng minh được f (a2n ) = a2n , ∀n ∈ N∗ .

Điều này đẫn đến mâu thuẫn vì a2n → +∞ khi n → +∞.


       
1 1 1 1 1
Giả sử 0 < a < 1, ta có 1 = f a =f f (a) = af , suy ra f = .
a a a a a
1
Suy ra là điểm bất động của f .
a
1
Vì 0 < a < 1 nên > 1, dẫn đến mâu thuẫn.
a
Như vậy điểm bất động duy nhất của f là 1.
1
Do đó xf (x) = 1, ∀x ∈ R+ , tức là f (x) = , ∀x ∈ R+ .
x
1 +
Thử lại, ta thấy f (x) = , ∀x ∈ R thoả các điều kiện của đề bài.
x
Bài toán 2. Gọi S là tập hợp tất cả các số thực lớn hơn −1. Tìm tất cả các hàm
số f : S → S thoả

f (x + f (y) + xf (y)) = y + f (x) + yf (x) với mọi x, y ∈ S. (2)

f (x)
và là hàm tăng nghiêm ngặt với −1 < x < 0 và x > 0.
x
(IMO 1996)

Lời giải.

Trong (2), thay x = 0, y = 0, ta được f (f (0)) = f (0).


Trong (2), thay x = 0, y = f (0), ta được

f (f (f (0))) = 2f (0) + (f (0))2 ⇔ f (0) = 2f (0) + (f (0))2 ⇔ (f (0))2 + f (0) = 0.

Vì f (0) > −1 nên f (0) = 0, suy ra 0 là điểm bất động của f .


Trong (2), thay y = x, ta đượcf (x + f (x) + xf (x)) = x + f (x) + xf (x), ∀x ∈ S.

Suy ra x + f (x) + xf (x), ∀x ∈ S là điểm bất động của f .


Ta chứng minh 0 là điểm bất động duy nhất của f .
Để chứng minh 0 là điểm bất động duy nhất của f ta chứng minh f không có điểm

bất động nào thuộc S \ {0}.


Giả sử tồn tại u ∈ S \ {0} là điểm bất động của f .
Trong (2), thay x = u, ta được f (u2 + 2u) = u2 + 2u.

Huỳnh Vĩnh Sang THPT chuyên Nguyễn Đình Chiểu - Đồng Tháp
Tập huấn GV THPT chuyên toán Bài viết chủ đề đại số 3

Suy ra u2 + 2u cũng là điểm bất động của f .


f (x)
Vì là hàm tăng nghiêm ngặt với −1 < x < 0 và x > 0 nên nếu phương trình
x
f (x) = x có nghiệm thì đó là nghiệm duy nhất.
Do đó u2 + 2u = u, tức là u2 + u = 0, điều này là vô lí vì u ∈ S \ {0}.
Vì vậy 0 là điểm bất động duy nhất của f .
x
Suy ra x + f (x) + xf (x) = 0, ∀x ∈ S, tức là f (x) = − , ∀x ∈ S.
x+1
x
Thử lại, ta thấy f (x) = − , ∀x ∈ S thoả các điều kiện của đề bài.
x+1
Nhận xét. Lời giải của hai bài toán trên đã sử dụng tính duy nhất của điểm bất

động của nghiệm hàm. Mặc dù không phải hàm số nào cũng có một điểm bất động
duy nhất, chúng ta vẫn có thể vận dụng linh hoạt tính duy nhất này để giải một số
phương trình hàm. Chúng ta xét các bài toán tiếp theo để thấy được điều đó.

Bài toán 3. Tìm tất cả các hàm số f : Z → Z thoả

f (x3 + f (y)) = y + (f (x))3 với mọi x, y ∈ Z. (3)

Lời giải.
Trong (3), thay x = 0, ta được f (f (y)) = y + (f (0))3 .
Đặt b = f (0), hệ thức trên trở thành

f (f (y)) = y + b3 , ∀y ∈ Z. (4)

Trong (4), thay y = −b3 , ta được f (f (−b3 )) = 0.

Đặt a = f (−b3 ), suy ra f (a) = 0.


Từ đó suy ra b = f (f (a)) = a + b3 .
Trong (3), thay x = −b, y = a, ta được f (−b3 ) = a + f 3 (−b), suy ra f (−b) = 0.

Dễ dàng chứng minh được f là đơn ánh, suy ra a = −b.


Từ đây ta thu được b3 − 2b = 0 ⇔ b = 0, tức là f (0) = 0.
Suy ra 0 là điểm bất động của f .

Trong (3), thay y = 0, ta được f (x3 ) = f 3 (x), ∀x ∈ Z.


Trong (3), thay y = x3 , ta được f (x3 + f 3 (x)) = x3 + f 3 (x), ∀x ∈ Z.
Suy ra x3 + f 3 (x), ∀x ∈ Z là điểm bất động của f .

Huỳnh Vĩnh Sang THPT chuyên Nguyễn Đình Chiểu - Đồng Tháp
Tập huấn GV THPT chuyên toán Bài viết chủ đề đại số 4

Nếu 0 là điểm bất động duy nhất của f thì ta được x3 + f 3 (x) = 0, ∀x ∈ Z.
Ta thu được f (x) = −x, ∀x ∈ Z.

Ta xét tiếp trường hợp 0 không phải là điểm bất động duy nhất của f .
Ta có f (1) = f 3 (1) và f (−1) = f 3 (−1).
Suy ra f (1) = ±1 hoặc f (1) = 0 và f (−1) = ±1 hoặc f (−1) = 0.

Vì f là đơn ánh nên f (1) = 1 và f (−1) = −1.


Gọi A là tập hợp tất cả các điểm bất động của f .
Khi đó, ta có 0 ∈ A và ±1 ∈ A.

Ngoài ra, nếu x0 ∈ A thì x0 +1 ∈ A vì f (x0 +1) = f (13 +f (x0 )) = x0 +f 3 (1) = x0 +1.
Từ đây ta suy ra A = Z.
Do đó f (x) = x, ∀x ∈ Z.

Thử lại, ta thấy f (x) = x, ∀x ∈ Z và f (x) = −x, ∀x ∈ Z thoả đề bài.


Nhận xét. Lời giải của bài toán 3 thể hiện sự vận dụng tính duy nhất của điểm
bất động của hàm số một cách hợp lí.

Sau khi tính được f (0), chúng ta có thể xử lí theo hướng sau:
Ta tính f (1) và thu được f (1) = 1 hoặc f (1) = −1.
Trong (3), thay x bởi f (x), thay y bởi −x3 , ta được

f ((f (x))3 + (f (−x))3 ) = −x3 + x3 = 0, ∀x ∈ Z.

Từ đó suy ra f (−x) = −f (x), ∀x ∈ Z.

Xét trường hợp f (1) = 1, trong (3), thay x = 1, thay y bởi f (y), ta được

f (1 + y) = f (y) + 1, ∀y ∈ Z.

Bằng cách chứng minh quy nạp ta chứng minh được f (x) = x, ∀x ∈ Z.
Xét trường hợp f (1) = −1, ta có f (y + 1) = f (y) − 1, ∀y ∈ Z.
Bằng cách chứng minh quy nạp ta chứng minh được f (x) = −x, ∀x ∈ Z.

Nếu so sánh lời giải vừa trình bày và lời giải sử dụng tính duy nhất của điểm bất
động thì ta thấy lời giải trên đơn giản hơn. Tuy nhiên, tác giả không có ý định làm

Huỳnh Vĩnh Sang THPT chuyên Nguyễn Đình Chiểu - Đồng Tháp
Tập huấn GV THPT chuyên toán Bài viết chủ đề đại số 5

cho lời giải trở nên phức tạp mà mà chỉ cố gắng áp dụng tính chất của điểm bất động
vào lời giải để làm nổi bật vai trò của kĩ thuật đã đề cập.

Bài toán 4. Tìm tất cả các hàm số f : N → N thoả

f (xf (x) + f (y)) = y + f 2 (x) với mọi x, y ∈ N. (5)

Lời giải.
Trong (5), thay x = 0, ta được f (f (y)) = y + f 2 (0), ∀y ∈ N.

Đặt a = f (0), suy ra f (y + a2 ) = f (f (f (y))) = f (y) + a2 , ∀y ∈ N.


Bằng cách chứng minh quy nạp theo x, ta chứng minh được

f (y + xa2 ) = f (y) + xa2 , ∀x, y ∈ N.

Trong (3), thay x bởi f (x), ta được

f (f (x)(x + a2 ) + f (y)) = y + (x + a2 )2 , ∀x, y ∈ N

⇔ f (xf (x) + f (y) + a2 f (y)) = y + (x + a2 )2 , ∀x, y ∈ N

⇔ f (xf (x) + f (y)) + a2 f (y) = y + (x + a2 )2 , ∀x, y ∈ N

⇔ y + f 2 (x) + a2 f (x) = y + (x + a2 )2 , ∀x, y ∈ N

⇔ f 2 (x) + a2 f (x) = (x + a2 )2 , ∀x ∈ N. (6)

Trong (6), thay x = 0, ta được a2 + a3 = a4 ⇔ a = 0.


Trong (5), thay x = 1, y = 0, ta được f (f (1)) = f 2 (1) ⇔ f 2 (1) = 1 ⇔ f (1) = 1.

Gọi A là tập hợp các điểm bất động của f .


Ta có 0 ∈ A và 1 ∈ A.
Ngoài ra, dễ dàng chứng minh được nếu x0 ∈ A thì x0 + 1 ∈ A.

Suy ra A = N, tức là f (x) = x, ∀x ∈ N.


Thử lại, ta thấy f (x) = x, ∀x ∈ N thoả đề bài.
Nhận xét. Đối với bài toán 4, sau khi tìm ra hệ thức (6), chúng ta có thể tìm được

f (x) = x, ∀x ∈ N mà không cần làm như trên. Tuy nhiên, việc trình bày như trên
nhằm minh hoạ cách chứng minh tập hợp các điểm bất động bằng với tập nguồn như
đã thực hiện ở bài toán 3.

Huỳnh Vĩnh Sang THPT chuyên Nguyễn Đình Chiểu - Đồng Tháp
Tập huấn GV THPT chuyên toán Bài viết chủ đề đại số 6

Bài toán 5. Tìm tất cả các hàm số f : R → R thoả

f (x + f (y)) = f (x) + y với mọi x, y ∈ R. (7)

Lời giải.
Trong (7), thay y = 0, ta được f (x + f (0)) = f (x), ∀x ∈ R.

Trong (7), thay y bởi y + f (0), ta được

f (x + f (y + f (0))) = f (x) + y + f (0), ∀x, y ∈ R

⇔ f (x + f (y)) = f (x) + y + f (0), ∀x, y ∈ R.

Từ đây suy ra f (0) = 0, suy ra 0 là điểm bất động của f .


Trong (7), thay y bởi x, ta được f (x + f (x)) = x + f (x), ∀x ∈ R.

Suy ra x + f (x), ∀x ∈ R là điểm bất động của f .


Nếu 0 là điểm bất động duy nhất của f thì x + f (x) = 0, ∀x ∈ R.
Suy ra f (x) = −x, ∀x ∈ R.

Xét trường hợp 0 không phải là điểm bất động duy nhất của f .
Dễ dàng chứng minh được f là một toàn ánh.
Khi đó hàm số g : R → R với g(x) = x + f (x), ∀x ∈ R cũng là một toàn ánh.

Gọi A là tập hợp các điểm bất động của f .


Với mọi x ∈ R tồn tại x0 ∈ R sao cho x = x0 + f (x0 ), suy ra x ∈ A.
Do đó A = R, tức là f (x) = x, ∀x ∈ R.

Thử lại, ta thấy f (x) = −x, ∀x ∈ R và f (x) = x, ∀x ∈ R thoả đề bài.


Bài toán 6. Tìm tất cả các hàm số f : R → R thoả

f (x + y + f (y)) = f (x) + 2y với mọi x, y ∈ R. (8)

Lời giải.
Với mọi x1 , x2 ∈ R mà f (x1 ) = f (x2 ), ta có

f (x1 ) + 2x2 = f (x1 + x2 + f (x2 ))

và f (x1 ) + 2x1 = f (x2 ) + 2x1 = f (x2 + x1 + f (x1 )) = f (x1 + x2 + f (x2 )).

Huỳnh Vĩnh Sang THPT chuyên Nguyễn Đình Chiểu - Đồng Tháp
Tập huấn GV THPT chuyên toán Bài viết chủ đề đại số 7

Suy ra x1 = x2 , vì vậy f là đơn ánh.


Trong (8), thay y = 0, ta được f (x + f (0)) = f (x), ∀x ∈ R.

Suy ra f (0) = 0, tức là 0 là điểm bất động của f .


Trong (8), thay y bởi x, ta được f (2x + f (x)) = 2x + f (x), ∀x ∈ R.
Suy ra 2x + f (x), ∀x ∈ R là điểm bất động của f .

Nếu 0 là điểm bất động duy nhất của f thì suy ra f (x) = −2x, ∀x ∈ R.
Xét trường hợp 0 không phải là điểm bất động duy nhất của f .
Dễ dàng chứng minh được f là toàn ánh.

Suy ra hàm g : R → R với g(x) = f (x) + 2x, ∀x ∈ R cũng là một toàn ánh.
Gọi A là tập hợp các điểm bất động của f .
Với mọi x ∈ R tồn tại x0 ∈ R sao cho x = f (x0 ) + 2x0 , suy ra x ∈ A.

Do đó A = R, tức là f (x) = x, ∀x ∈ R.
Thử lại, ta thấy f (x) = −2x, ∀x ∈ R và f (x) = x, ∀x ∈ R thoả đề bài.
Lưu ý. Trong lời giải của bài toán 5 và bài toán 6, chúng ta không thể chứng minh

tập nguồn bằng tập hợp các điểm bất động bằng cách như đã làm ở bài toán 3 và bài
toán 4. Cách chứng minh đó có thể dùng được nếu tập nguồn là N hoặc Z nhưng không
dùng được nếu tập nguồn là R.

Để kết thúc bài viết, xin giới thiệu bài toán 7. Trong bài toán 7, một lần nữa ý
tưởng về tính duy nhất của điểm bất động của hàm số lại được áp dụng.
Bài toán 7. Tìm tất cả các hàm số f : R → R thoả

f (x + f (x + y)) + f (xy) = x + f (x + y) + yf (x), ∀x, y ∈ R. (9)

(IMO Shortlist 2015)

Lời giải.
Trong (9), thay x = 0, y = 1, ta được f (f (1)) = f (1).

Trong (9), thay x = 0, y = f (1), ta được

f (f (f 1)) + f (0) = f (f (1)) + f (0)f (1) ⇔ f (0)f (1) − f (0) = 0.

Suy ra f (1) = 1 hoặc f (0) = 0.

Huỳnh Vĩnh Sang THPT chuyên Nguyễn Đình Chiểu - Đồng Tháp
Tập huấn GV THPT chuyên toán Bài viết chủ đề đại số 8

Ta chứng minh được nếu f (0) thì f (1) = 1, vì vậy ta chỉ xét trường hợp f (1) = 1.
Trong (9), thay y = 1, ta được f (x + f (x + 1)) = x + f (x + 1), ∀x ∈ R.

Suy ra x + f (x + 1), ∀x ∈ R là điểm bất động của f .


Ta có 1 cũng là điểm bất động của f .
Do đó nếu 1 là điểm bất động duy nhất của f thì ta được x + f (x + 1) = 1, ∀x ∈ R.

Từ đó suy ra f (x + 1) = 1 − x, ∀x ∈ R, tức là f (x) = 2 − x, ∀x ∈ R.


Xét khả năng 1 không phải là điểm bất động duy nhất của f .
Trong (9), thay y = 0 và thay x bởi x + 1, ta được

f (x + f (x + 1) + 1) = x + f (x + 1) + 1, ∀x ∈ R.

Suy ra x + 1 + f (x + 1), ∀x ∈ R là điểm bất động của f .


Trong (9), thay x = 1, ta được

f (1 + f (y + 1)) + f (y) = 1 + f (y + 1) + y, ∀y ∈ R. (10)

Trong (9), thay x = −1, y = 1, ta được f (−1) = −1.


Từ (10) ta suy ra nếu y0 và y0 + 1 là hai điểm bất động của f thì y0 + 2 cũng là

điểm bất động của f .


Do đó x + f (x + 1) + 2, ∀x ∈ R là điểm bất động của f .
Tức là

f (x + f (x + 1) + 2) = x + f (x + 1) + 2, ∀x ∈ R. (11)

Trong (11), thay x bởi x − 2, ta được f (x + f (x − 1)) = x + f (x − 1), ∀x ∈ R.


Trong (9), thay y = −1, ta được

f (x + f (x − 1)) = x + f (x − 1) − f (x) − f (−x), ∀x ∈ R.

Suy ra f (−x) = −f (x), ∀x ∈ R.


Trong (9), thay x = −1, thay y bởi −y, ta được

f (−1 + f (−1 − y)) + f (y) = −1 + f (−1 − y) + y, ∀y ∈ R

⇔ −f (1 + f (1 + y)) + f (y) = −1 − f (1 + y) + y, ∀y ∈ R. (12)

Huỳnh Vĩnh Sang THPT chuyên Nguyễn Đình Chiểu - Đồng Tháp
Tập huấn GV THPT chuyên toán Bài viết chủ đề đại số 9

Kết hợp (10) và (12), ta thu được f (y) = y, ∀y ∈ R.


Thử lại, ta thấy f (x) = 2 − x, ∀x ∈ R và f (x) = x, ∀x ∈ R thoả đề bài.

Một số bài tập đề nghị.


Bài 1. (Bulgaria ’2003) Tìm tất cả các hàm số f : R → R thoả

f (x2 + y + f (y)) = 2y + (f (x))2 , với mọi x, y ∈ R.

Bài 2. Tìm tất cả các hàm số f : N → N thoả

f ((f (x))2 + (f (y))2 ) = x2 + y 2 , với mọi x, y ∈ N.

Bài 3. Tìm tất cả các hàm số f : N → N thoả mãn

f (x2 + f (y)) = (f (x))2 + y, với mọi x, y ∈ N.

Bài 4. Tìm tất cả các hàm số f : Z → Z thoả mãn

f ((f (x))2 + y) = x2 + f (y), với mọi x, y ∈ Z.

Bài 5. Tìm tất cả các hàm số f : Z → Z thoả mãn

f (x + y + f (y)) = f (x) + 2y với mọi x, y ∈ Z.

Bài 6. Tìm tất cả các hàm số f : R → R thoả

f (xn + 2f (y)) = (f (x))n + y + f (y), với mọi x, y ∈ R, n ∈ N, n > 2.

Bài 7. (IMO Shortlish 1988) Tìm tất cả các hàm số f : N∗ → N∗ thoả

f (f (x) + f (y)) = x + y, với mọi x, y ∈ N∗ .

Huỳnh Vĩnh Sang THPT chuyên Nguyễn Đình Chiểu - Đồng Tháp
Tập huấn GV THPT chuyên toán Bài viết chủ đề đại số 10

Tài liệu tham khảo.


[1] Titu Andreescu, Iurie Borecio, Functional Equations, Electronic Edition 2007;

[2]Titu Andreescu, Iurie Borecio, Functional Equations, AwesomeMath LLC, 2007;


[3] Kin Y.Li, Functional Equations, Mathematical Excalibur-Vol8 - Number 1, 2003;
[4] www.imomath.com;

[5] http://math-olympiad.blogsky.com/;
[6] https://artofproblemsolving.com/.

Huỳnh Vĩnh Sang THPT chuyên Nguyễn Đình Chiểu - Đồng Tháp
Tổ hợp

SỬ DỤNG PHƯƠNG PHÁP QUY NẠP ĐỂ


GIẢI MỘT SỐ BÀI TOÁN TỔ HỢP

1 Mở đầu

2 Phương pháp quy nạp toán học


Quy nạp toán học là một phương pháp mạnh để chứng minh các phát biểu phụ thuộc vào
một số tự nhiên.
Cho (P (n))n≥0 là một dãy các mệnh đề. Phương pháp quy nạp toán học được sử dụng để
chứng minh P (n) đúng với mọi n ≥ n0 với n0 là một số tự nhiên.
Phương pháp quy nạp toán học (dạng yếu): Giả sử

• P ( n 0 ) đúng.

• Với mọi k ≥ n 0 và P ( k) đúng thì P ( k + 1) đúng.

Khi đó P (n) đúng với mọi n ≥ n0 .


Phương pháp quy nạp toán học (bước nhảy s): Cho s là số nguyên dương. Giả sử

• P ( n 0 ) , P ( n 0 + 1) , ..., P ( n 0 + s − 1) đúng.

• Với mọi k ≥ n 0 , P ( k) đúng kéo theo P ( k + s) đúng .

Khi đó P (n) đúng với mọi n ≥ n0 .


Phương pháp quy nạp toán học (Dạng mạnh): Giả sử

• P ( n 0 ) đúng

• Với mọi k ≥ n 0 , P ( m) đúng với mọi m mà n 0 ≤ m ≤ k kéo theo P ( k + 1) đúng.

Khi đó P (n) đúng với mọi n ≥ n0 .

3 Một số ví dụ minh họa

3.1 Các bài toán chứng minh tồn tại


Ví dụ 1. Chứng minh rằng với mỗi số tự nhiên n ≥ 2, luôn tồn tại một tập hợp S có n số
nguyên dương sao cho (a − b)2 là ước của a · b với mọi a, b ∈ S.

Lời giải. Với n = 2 ta chọn S2 = {0, 1}.


Giả sử tồn tại tập S n = {a 1 , a 2 , · · · , a n } thỏa yêu cầu bài toán. Gọi x là bội chung nhỏ nhất
của các số khác 0 có dạng (a − b)2 với a, b ∈ S n . Đặt

S n+1 = { x + a |a ∈ S n } ∪ {0} .

Khi đó |S n+1 | = n + 1 và với a, b ∈ S n+1 ta có :

Nguyễn Tất Thu-GV Trường Chuyên Lương Thế Vinh 1


3.1 Các bài toán chứng minh tồn tại Tổ hợp

• Nếu ab = 0 thì (a − b)2 |ab .

• Nếu ab 6= 0, khi đó tồn tại i, j ∈ {1, 2, · · · , n} ; i 6= j sao cho a = x + a i , b = x + a j . Khi đó

( x + a i ) x + a j = x2 + x a i + a j + a i a j ≡ a i a j ≡ 0 mod (a i − a j )2 ,
¡ ¢ ¡ ¢ ¡ ¢

hay
¢¤2 ¡ ¢2 ¯
(a − b)2 = ( x + a i ) − x + a j
£ ¡ ¡ ¢
= a i − a j ¯( x + a i ) x + a j = ab.

Vậy bài toán được chứng minh.

Ví dụ 2. Trên đường tròn người ta điền n (n ≥ 3) số sao cho với mỗi số bất kì thì tổng của
hai số kề bên chia hết cho số đó. Chứng minh rằng tổng các thương không vượt quá 3 n.

Lời giải. Ta chứng minh bài toán bằng phương pháp quy nạp.

• n = 3 . Gọi ba số đó a, b, c , khi đó ta có

 a + b = xc


b + c = ya .

 c + a = zb

Giả sử a = max {a, b, c} ta có

ya = b + c ≤ 2a ⇒ y ∈ {1, 2} .

+) y = 2, ta có a = b = c nên x + y + z = 6.
+) y = 1, ta có b + c = a. Giả sử b ≥ c, khi đó ta có

zb = c + a = 2 c + b ⇒ 2 c = ( z − 1) b ≥ b

nên c ≤ b ≤ 2 c.
Suy ra
xc = a + bc + 2 b ≤ 5 c ⇒ x ≤ 5
zb = c + a = 2 c + b ≤ 3 b ⇒ z ≤ 3.
Do đó x + y + z ≤ 9. Do đó, bài toán đúng với n = 3.

• Giả sử bài toán đúng với n ≥ 3, ta chứng minh bài toán cũng đúng với n + 1.
+) Nếu n + 1 số đã cho bằng nhau thì ta có đpcm.
+) Xét trường hợp ngược lại, tồn tại a, b, c sao cho b, c xếp cạnh a và a > max {b, c} và
b nằm giữa a và d , c nằm giữa a và e. Ta có

 b + c = ma


a + d = nb .

a + e = pc

Nguyễn Tất Thu-GV Trường Chuyên Lương Thế Vinh 2


3.1 Các bài toán chứng minh tồn tại Tổ hợp

Do b + c < 2a nên m = 1 hay b + c = a. Suy ra


(
c + d = ( n − 1) b
.
b + e = ( p − 1) c

Do vậy, khi ta bỏ a khỏi vòng tròn thì n số còn lại vẫn thỏa yêu cầu đề bài, theo giả
thiết quy nạp thì tổng các thương của n số này không vượt quá 3n, do đó tổng các
thương của n + 1 số không vượt quá 3n + 3 = 3 (n + 1).

Vậy bài toán được chứng minh.

Ví dụ 3. Cho tập X = 0, 1, 2, ..., 32015 − 1 . Chứng minh rằng tồn tại một tập con A của X
© ª

sao cho | A | = 22015 và trong A không có ba số nào lập thành một cấp số cộng.

Lời giải. Ta chứng minh bài toán tổng quát như sau: Luôn tồn tại một tập con A của
tập X k = 0, 1, 2, ..., 3k − 1 sao cho | A | = 2k và trong A không có ba số nào lập thành cấp số
© ª

cộng.
Ta chứng minh bài toán bằng phương pháp quy nạp.
• Với k = 2, ta chọn A = {0, 1, 3, 7}.
• Giả sử tồn tại một tập con A của tập X k sao cho | A | = 2k và trong A không có ba số nào
lập thành cấp số cộng. Xét tập X k+1 = {0, 1, ..., t} với t = 3k+1 − 1.
© ª
Giả sử A = a 1 , a 2 , ..., a 2k , ta xét tập
© ª
A 0 = a 1 , a 2 , ..., a 2k , t − a 1 , t − a 2 , ..., t − a 2k .

Ta có
a i 6= a j ⇒ t − a i 6= t − a j

a i + a j ≤ 3k − 1 + 3k − 2 < 3k+1 − 1
nên t − a i 6= a j .
Do đó | A 0 | = 2 | A | = 2k+1 .
Giả sử tồn tại các số m, n, p sao cho
¡ ¢
am + an = 2 t − a p (1)
a m + a n = 2a p (2)
¡ ¢
(t − a m) + (t − a n) = 2 t − a p (3)
( t − a m ) + ( t − a n ) = 2a p (4)
( t − a m ) + a n = 2a p (5)

Từ (1) ta suy ra
2t = a k + a m + a n. (∗)
Ta có
a m + a n + a p ≤ 3k − 1 + 3k − 2 + 3k − 3 = 3k+1 − 6 < 2 t
nên (*) không thể xảy ra,hay (1) không xảy ra. Tương tự, các trường hợp khác cũng không
xảy ra. Do đó, trong A 0 không chứa ba số nào lập thành cấp số cộng.
Vậy bài toán được chứng minh.

Nguyễn Tất Thu-GV Trường Chuyên Lương Thế Vinh 3


3.1 Các bài toán chứng minh tồn tại Tổ hợp

Ví dụ 4. (THTT T10/468) Một người thợ luôn phải lát những căn phòng có kích thước
2n × 2n ô vuông bằng các mảnh đá lát hình đô-mi-nô kích thước 1 × 3 ô vuông và các mảnh
đá lát hình thước thợ 3 ô vuông. Anh ta phải lát kín cả căn phòng chỉ trừ lại một ô vuông
duy nhất để trống khôn lát đá để trang trí đặc biệt. Do giá thành của một viên đá lát
hình thước thợ dắt hơn nhiều giá thành của một viên đá lát hình đo-mi-nô, cho nên anh
ta muốn sử dụng càng ít viên đá lát hình thước thợ càng tốt. Anh ta nhận thấy rằng dù
vị trí ô đặc biệt ở đâu đi chăng nữa, chưa lần nào anh ta phải dùng nhiều hơn n viên
gạch lát hình thước thợ để hoàn thành công việc của mình. Hãy chứng minh rằng khẳng
định của người thợ luôn đúng với mọi số nguyên dương n.

Thước thợ Đô-mi-nô

Lời giải. Trước hết ta chứng minh các bổ đề sau:


Bổ đề 1. Mọi hình chữ nhật có kích thước một cạnh chia hết cho 3 thì lát đượcbởi các
viên đá hình đô-mi-nô 1 × 3.
Bổ đề này là hiển nhiên.
Bổ đề 2. Mọi hình thước thợ tạo bởi ba hình vuông kích thức 2n × 2n đều lát được bởi một
số viên đá hình đô-mi-nô và 1 viên đá hình thước thợ.
Chứng minh:
Kết luận hiển nhiên đúng khi n = 1.
Giả sử kết luận đúng cho n = k − 1 thì ta chia một hình thước thợ tạo bởi 3 hình vuông
kích thước 2k × 2k thành các hình chữ nhật có một cạnh chia hết cho 3 và một thước thợ
tạo bởi 3 hình vuông kích thước 2k−1 × 2k−1 (như trong hình 1).

Hình 1 Hình 2

Dễ thấy các hình chữ nhật thu được đều có đồ dài một cạnh chia hết cho 3 nên chúng
được lát bởi các viên đá đô-mi-nô theo Bổ đề 1.
Hình thước thợ gồm 3 ô vuông 2k−1 × 2k−1 theo giả thiết quy nạp lạt được bởi các viên đá
đo-mi-nô và một viên đá hình thức thợ.
Trở lại bài toán.
Với n = 1 thì bài toán hiển nhiên đúng.
Giả sử kết luận của bài toán đúng với n = k − 1 ≥ 1. Xét một căn phòng kích thước 2k × 2k
ô vuông. Chia hình vuông này thành 4 hình vuông bằng nhau thì ô trống sẽ rời vào một

Nguyễn Tất Thu-GV Trường Chuyên Lương Thế Vinh 4


3.2 Các bài toán cực trị Tổ hợp

trong 4 hình vuông này, chẳng hạn rời vào góc phần tư thứ nhất (hình 2). Theo giả thiết
quy nạp thì cần không quá k − 1 viên đá lát hình thước thợ để lát hình vuông ở góc phần
tư thứ nhất. Để lát hình thước thợ gồm 3 hình vuông kích thức 2k−1 × 2k−1 , thi theo Bổ đề
2 ta cần một viên đá hình thước thợ. Do đó, ta cần không qua (k − 1) + 1 = k viên đá hình
thước thợ để lát căn phòng kích thước 2k × 2k .
Vậy bài toán được chứng minh.

3.2 Các bài toán cực trị


Ví dụ 5. (THTT) Cho T là một tập có n phần tử. Gọi X là tập các tập con của T gồm 3
phần tử và hai tập con bất kì có giao khác rỗng. Tìm số phần tử lớn nhất của tập X .

Lời giải. Không mất tính tổng quát ta giả sử T = {1, 2, 3, · · · , n} và gọi k(n) = max | X |.
Với 3 ≤ n ≤ 5 , khi đó hai tập con bất kì gồm ba phần tử của T luôn có giao khác rỗng. Do
đó k(n) = C 3n .
Xét n ≥ 6, xét A là tập con gồm hai phần tử của tập {2, 3, · · · , n}. Khi đó các tập A ∪ {1} có 3
phần tử và hai tập bất kì có giao khác rỗng. Do vậy k(n) ≥ C 2n−1 .
Ta chứng minh
k ( n) ≤ C 2n−1 (1).

Giả sử k(m) ≤ C 2m−1 với mọi m = 6, 7, · · · , n . Ta chứng minh (1) đúng với n + 1 .
Xét tập A ∈ X , không mất tính tổng quát ta giả sử A = {1, 2, 3}.
Nếu mọi B đều chứa hai phần tử của tập A thì | X | = 1 + 3 (n − 3) < C 2n .
Giả sử có ít nhất một tập B chỉ chứa một phần tử của tập A , giả sử B = {1, 4, 5}. Nếu các
tập con của X đều chứa 1 thì | X | = C 2n−1 .
Nếu có tập C ∈ X không chứa 1. Do A ∩ C 6= ∅, B ∩ C 6= ∅ nên ta giả sử C = {2, 4, 6}.
( n − 1) ( n − 2) ( n − 2) ( n − 3)
Vì − = n − 2 nên nếu có không quá n − 2 tập chứa 7 thì do giả
2 2
thiết quy nạp ta có khẳng định đúng.
Trái lại giả sử có ít nhất n − 1 tập chứa số 7. Với mỗi tập như thế phải chứa 1, 2 hoặc 3.
Nếu chứa 1 thì phải chứa 2, 4 hoặc 6. Điều này xảy ra nếu tập đó là {1, 2, 7}, {1, 4, 7} hoặc
{1, 6, 7}.
Nếu nó chứa 2 thì phải chứa 1, 4 hoặc 5. Như vậy có thêm hai khả năng là {2, 4, 7} hoặc
{2, 5, 7}.
Nếu nó chứa 3 thì nó phải chứa 1, 4, 5 nên chỉ có một khả năng {3, 4, 7}.
Như vậy có 6 khả năng cho tập chứa 7. Vì thế nếu n − 2 ≥ 6 thì có không quá n − 2 tập
chứa 7. Xét n = 7 , ta có T = {1, 2, · · · , 7} ta lí luận tương tự và đi đến kết luận có không quá
n − 2 tập chứa
 6.
 0 nếu 1≤n<3


Vậy k(n) = C 3n nếu 3≤n≤5 .

C 2 nếu

n≥6
n−1

Ví dụ 6. (Chọn đội tuyển Nghệ an năm 2015-2016, ngày 1)


Một tập hợp con A có ít nhất ba phần tử của tập các số nguyên dương được gọi là tập
“thô” nếu với 3 phần tử phân biệt a, b, c của A thì a không là bội của (b, c), với ( x, y) là kí

Nguyễn Tất Thu-GV Trường Chuyên Lương Thế Vinh 5


3.2 Các bài toán cực trị Tổ hợp

hiệu ước chung lớn nhất của hai số nguyên x và y. Chứng minh rằng nếu A là tập “thô”
có n phần tử và n ≥ 4 thì max A ≥ 4 n2 − n − 1 .
¡ ¢

Lời giải.
Bổ đề 3. Từ 2n − 2 số tự nhiên đầu tiên ta chọn ra n số, khi đó luôn có hai trong n số đã
chọn là bội của nhau.
Thật vậy:
Xét n số 2k i m i với k i là các số tự nhiên và m i là các số nguyên dương lẻ.
Vì trong tập {1, 2, ..., 2n − 2} có đúng n − 1 số lẻ, suy ra trong n số {m 1 , m 2 , ..., m n } có hai số
.
bằng nhau, chẳng hạn m t = m l ( t > l ). Khi đó 2k t m t ..2k l m l (đpcm).
Nhận xét: Từ bổ đề trên ta thấy:
Nếu lấy n số mà trong đó không có hai số nào là bội của nhau thì max của n số đó không
nhỏ hơn 2n − 1.
Bổ đề 4. Nếu A = {a, b, c} là tập “thô” thì max {a, b, c} ≥ 15.
Thật vậy:
Vì {a, b, c} là tập thô nên (a, b) , (b, c), ( c, a) là ba số mà không có hai số nào là bội của nhau.
Theo bổ đề 1, ta có
max {(a, b) , ( b, c) , ( c, a)} ≥ 5.
a b
Hơn nữa 1 < 6= nên
(a, b) (a, b)
a b
½ ¾
max , ≥ 3.
(a, b) (a, b)
Từ đó, suy ra
max {a, b, c} ≥ 3 max {(a, b) , ( b, c) , ( c, a)} ≥ 15.

Trở lại bài toán.


Xét n = 4, đặt A = {a, b, c, d }. Gọi x1 , x2 , · · · , x6 là UCLN của hai trong 4 số thuộc tập A và
x = max { x1 , x2 , · · · , x6 }. Ta có x ≥ 11.
Dễ thấy x ∉ A .

+) Nếu kx ∈ A với k ≥ 4 thì max A ≥ 4.11 = 44.

+) Nếu 2 x ∈ A, 3 x ∈ A . Xét y ∈ A \ {2 x, 3 x}. Nếu ( y, 3) = 1 thì ( y, 3 x) = ( y, x) |2 y (vô lí). Do đó


3 | y , hay A = {2 x, 3 x, 3 y, 3 z}. Do A là tập thô nên B = { x, y, z} cũng là tập thô, suy ra

max A ≥ 3 max { x, y, z} ≥ 3.15 = 45 > 44.

Suy ra bài toán đúng với n = 4.


Giả sử bài toán đúng đến n − 1. Ta chứng minh bài toán đúng đến n.
Xét A là tập thô có n phần tử, khi đó có C 2n UCLN của hai phần tử bất kì thuộc A.
Đặt X = { x1 , x2 , · · · , xk } , k = C 2n là tập các UCLN và x = max X .
Theo chứng minh tương tự như trường hợp n = 4 ta chỉ xét 2 x, 3 x ∈ A. Khi đó

A = {2 x, 3 x, 3a 1 , 3a 2 , · · · , 3a n−2 }

Nguyễn Tất Thu-GV Trường Chuyên Lương Thế Vinh 6


3.2 Các bài toán cực trị Tổ hợp

Và tập Y = { x, a 1 , a 2 , · · · , a n−2 } là tập thô, nên

max Y ≥ 4 ( n − 1)2 − ( n − 1) − 1 .
¡ ¢

Suy ra
max A ≥ 12 n2 − 3 n + 1 ≥ 4 n2 − n + 1 ∀ n ≥ 4.
¡ ¢ ¡ ¢

Vậy bài toán được chứng minh.

Ví dụ 7. (Chọn đội tuyển Nghệ an năm 2015-2016, ngày 2) Cho n ≥ 2 con chim đậu
trên đường tròn (O ), mỗi điểm có nhiều nhất một con chim đậu. Hai con chim đậu tại
0
điểm P i , P j được gọi là trông thấy nhau nếu P
à i OP j ≤ 120 . Tìm theo n số nhỏ nhất các cặp
con chim trông thấy nhau.

Lời giải. Gọi f (n) là số nhỏ nhất các cặp con chim trông thấy nhau.
( n − 1)2
· ¸
Ta chứng minh f (n) = .
4
Ta có f (2) = 0,· f (3) = 1¸nên bài toán đúng với n = 2, n = 3. ·
( n − 1)2 n2
¸
Giả sử f (n) = , n ≥ 2. Ta chứng minh f ( n + 1) = .
4 4
Theo nguyên tắc cực hạn thì trong n + 1 con chim có một con chim trông thấy nhiều con
chim nhất, giả sử đó là con chim đậu ở điểm A (ta gọi tắt là A ) và m là số con chim mà A
nhìn thấy .
Trên đường tròn lấy hai điểm X , Y sao cho

AOX
ƒ=X OY = Y
ƒ ƒ O A = 1200 .
_
Vì A trông thấy m con chim nên M con chim này nằm trên cung X AY tính cả hai điểm
_
mút. Do đó, trên cung nhỏ X Y có n + 1 − (m + 1) = n − m con chim, và mỗi con chim này
trông thấy n − m − 1 con chim còn lại.
Suy ra số cặp chim trông thấy, trong đó có A là m và do tính chất của A nên ta có
n−1 hni
m ≥ n−m−1 ⇒ m ≥ ≥ .
2 2
Số nhỏ nhất các cặp chim thấy nhau của n con chim còn lại (trừ A ) là f (n) và rõ ràng các
cặp chim trông thấy nhau này khác với m cặp chim trông thấy nhau ở trên. Do đó, ta có

( n − 1)2 n
· ¸ h i
f ( n + 1) = f ( n) + m ≥ + .
4 2
Ta chứng minh
( n − 1)2
¸ h i · 2¸
n n
·
+ = . (1)
4 2 4
+) n = 2k thì
· 2¸
1 n
· ¸
2
V T (1) = k ( k − 1) + + [ k] = k ( k − 1) + k = k = = V P (1).
4 4
+) n = 2k + 1 thì · 2¸
1 n
·
¸
2 2
V T (1) = k + k + = k +k= = V P (1).
2 4

Nguyễn Tất Thu-GV Trường Chuyên Lương Thế Vinh 7


Tổ hợp

( n − 1)2
· ¸
Do đó, ta chứng minh được f (n) ≥ . Ta chỉ ra trường hợp có dấu “=”.
4
_ hni
Trên đường tròn lấy hai điểm X , Y sao cho cung sđ X Y = 300 . Cho con chim đậu trên
2
_ hni n+1 _
· ¸
cung nhỏ X Y và n − = con chim còn lại đầu trên cung nhỏ X 0 Y 0 đối xứng với
2 2
_ _ _
cung X Y qua tâm O . Khi đó, các con chim nằm trên hai cung X Y và X 0 Y 0 trông thấy
_ _
nhau , đồng thời hai con chim bất kì nằm trên hai cung X Y và X 0 Y 0 đều không trông
thấy nhau. Do đó, số cặp chim trông thấy nhau là
n+1 n+1
h n i ³h n i ´ · ¸ µ· ¸ ¶
−1 −1
( n − 1)2
· ¸
2 2 2 2
+ = .
2 2 4

4 Bài tập
Bài tập 1. Chứng minh rằng với mọi số nguyên n ≥ 2, sẽ tồn tại các số nguyên dương
a 1 , a 2 , ..., a n sao cho a j + a i chia hết cho a j − a i với 1 ≤ i < j ≤ n.

Bài tập 2. Chứng minh rằng n dây cung, cắt nhau tại m (n > m) điểm trong một hình
tròn thì sẽ chia hình tròn này thành n + m + 1 phần.

Bài tập 3. Trong một giải đấu cờ vua, có n ≥ 5 đấu thủ tham gia (hai đấu thủ bất kỳ đấu
với nhau tối đa một trận). Chứng minh rằng nếu số trận đấu đã diễn ra không nhỏ hơn
n2
· ¸
+ 2 thì luôn tồn tại 5 đấu thủ A , B, C , D , E mà A đã đấu với cả 4 đấu thủ còn lại,
4
ngoài ra B đã đấu với C và D đã ·đấu¸với E . Khẳng định của bài toán còn đúng không nếu
n2
số trận đấu đã diễn ra nhỏ hơn + 2?
4

Lời giải. Coi mỗi đấu thủ là một đỉnh của đồ thị. Hai đỉnh kề nhau nếu hai đấu thủ đã
đấu với nhau.
n2
· ¸
Nếu số trận đã đấu là không vượt quá + 1 thì ta xét phản ví dụ sau:
4
hni ·
n+1
¸
Chia n đỉnh thành 2 nhóm, một nhóm đỉnh và một nhóm đỉnh. Nối mỗi đỉnh
2 2
thuộc nhóm 1 với tất cả các đỉnh thuộc nhóm 2, ngoài ra chọn 2 đỉnh tùy ý trong nhóm 1
và nối chúng với nhau. Như vậy số cạnh là
h ni· n +1¸ n2
·¸
+1 = +1
2 2 4

và rõ ràng không tồn tại 5 đỉnh thỏa mãn yêu cầu bài toán.
n2
· ¸
Ta chứng minh nếu số cạnh là không nhỏ hơn + 2 thì tồn tại 5 đỉnh thỏa mãn yêu
4
cầu (1).
25
· ¸
Với n = 5: Số ván đã đấu là + 2 = 8. Nếu mỗi đỉnh có bậc lớn nhất là 3 thì số cạnh tối
4

Nguyễn Tất Thu-GV Trường Chuyên Lương Thế Vinh 8


Tổ hợp

5.3
đa là < 8 (vô lý). Do đó tồn tại ít nhất một đỉnh có bậc ít nhất là 4, giả sử là a. Gọi
2
b, c, d, e là 4 đỉnh kề với a. Trong đồ thị con 4 đỉnh này luôn tồn tại 4 cạnh, suy ra phải
tồn tại 2 cạnh không chung đỉnh, giả sử là bc, de. Bài toán được khẳng định.
k2
· ¸
Giả sử (1) đã đúng đến n = k − 1 (k ≥ 6). Xét đồ thị k đỉnh với đúng + 2 cạnh.
4
k
· ¸
Nếu tồn tại 1 đỉnh có bậc không vượt quá thì bỏ đỉnh này đi và xét đồ thị con gồm
2
k − 1 đỉnh còn lại. Khi đó tổng số cạnh của đồ thị con k − 1 đỉnh này ít nhất là
( k − 1)2
· 2¸
k k k k+1 k k−1
· ¸ · ¸ µ· ¸ ¶ · ¸· ¸ · ¸
+2− = 2+ −1 = 2+ = 2+
4 2 2 2 2 2 4
Như vậy khẳng định đúng theo giả thiết ·quy¸ nạp.
k
Nếu tất cả các đỉnh đều có bậc ít nhất là + 1.
2
Nếu k = 2 l (l ≥ 3) thì tổng số cạnh ít nhất là
· 2¸
2 l ( l + 1) 2 k
= l ( l + 1) > l + 2 = + 2.
2 4
Nếu k = 2l + 1 (l ≥ 3) thì tổng số cạnh ít nhất là
· 2¸
(2 l + 1) ( l + 1) k
≥ l ( l + 1) + 2 = + 2.
2 4
Dấu bằng phải xảy ra nên l = 3, k = 7 và tất cả các đỉnh đều có bậc 4.
Bài tập 4. Quan sát một con thạch sùng bắt muỗi, ta thấy quy luật sau:
• Con thạch sùng nghỉ ngơi sau mỗi lần bắt muỗi;

• Con muỗi đầu tiên bị bắt sau thời gian nghỉ 1 phút;

• Thời gian nghỉ trước khi bắt con muỗi thứ 2 m bằng thời gian nghỉ trước khi bắt con
muỗi thứ m và kém 1 phút so với thời gian nghỉ trước khi bắt con muỗi thứ 2m + 1;

• Khi thạch sùng hết nghỉ thì nó bắt muỗi ngay lập tức.

Coi thời gian thực hiện động tác bắt muỗi của thạch sùng bằng 0. Hỏi sau bao nhiêu
phút thì thạch sùng bắt được con muỗi thứ 82?
Lời giải. Gọi f (n) là thời gian nghỉ (tính bằng phút) của con thạch sùng trước khi nó
bắt con muỗi thứ n. Khi đó, tổng thời gian để con thạch sùng bắt được con muỗi thứ n
chính là
f (1) + f (2) + f (3) + · · · + f ( n).
Dựa theo giả thiết, ta thấy rằng



 f (1) = 1

f (2 m) = f ( m) , m≥0



 f (2 m + 1) = f (2 m) + 1

Ta sẽ chứng minh rằng f (m) chính là số các số 1 có trong biểu diễn nhị phân của m. (*)
Thật vậy:

Nguyễn Tất Thu-GV Trường Chuyên Lương Thế Vinh 9


Tổ hợp

• Với m = 1 thì m = 12 nên (*) đúng.

• Giả sử (*) đúng với m = n, ta có hai trường hợp:

+ Nếu n· là số chẵn, đặt n = 2 k thì f (n + 1) = f (2 k + 1) = f (2 k) + 1 = f (k) + 1. Do 2k + 1


2k + 1
¸
lẻ và = k nên số các số 1 trong biểu diễn nhị phân của 2 k + 1 bằng số các
2
số 1 trong biểu diễn nhị phân của k cộng với 1 và (*) cũng đúng với m = n + 1.
+ Nếu n là số lẻ, đặt n = 2k + 1 thì f (n + 1) = f (2 k + 2) = f (k + 1). Do 2k + 2 chẵn và
2k + 2
ta cũng có = k + 1 nên số các số 1 trong biểu diễn nhị phân của 2 k + 2 và
2
k + 1 là bằng nhau. Suy ra (*) cũng đúng với m = n + 1.
Do đó, (*) cũng đúng với m = n + 1.

Theo nguyên lí quy nạp, (*) được chứng minh.


Tiếp theo, ta cần tính f (1) + f (2) + f (3) + · · · + f (82).
Trước hết, ta tính tổng f (0) + f (1) + f (2) + f (3) + · · · + f (63), mỗi giá trị f là tổng các chữ số
1 của 64 số có dạng X X X X X X (6 vị trí này có thể nhận giá trị 0 hoặc 1).
Tại mỗi vị trí X trong số trên, dễ thấy rằng có đúng 32 số 1 và 32 số 0 nên suy ra tổng
tất các các chữ số 1 nói trên là 32 × 6 = 192.
Tiếp theo, ta lại tính tổng f (64) + f (65) + f (66) + · · · + f (79), mỗi giá trị f là tổng các chữ số
1 của 16 số có dạng 100 X X X X (4 vị trí này có thể nhận giá trị 0 hoặc 1).
Tương tự trên, ta tính được tổng này là 16 + 8 × 4 = 48.
Cuối cùng, dễ thấy f (80) = 2, f (81) = 3, f (82) = 3.
Do đó, kết quả cuối cùng cần tìm là 192 + 48 + 2 + 3 + 3 = 248.

Bài tập 5. Cho các số 1, 2, 3, · · · , n. Chúng ta thực hiện việc xóa hai số bất kì trên bảng
và thay bằng số mới bằng 2 lần tổng của hai số đó. Cứ tiếp tục quá trình như vậy cho
đến khi trên bảng chỉ còn lại một số. Số cuối cùng trên bảng chính là số kẹo mà học sinh
4 n3
nhận được. Chứng minh rằng số kẹo đó luôn lớn hơn với mọi số tự nhiên n ≥ 2.
9
p p p
Lời giải. Với a, b là các số dương, ta có a + b ≤ 2(a + b) nên bằng việc thay hai số bởi
2 lần tổng của hai số đó thì tổng tất cả các căn bậc hai của các số trên bảng không giảm.
p
Do đó, số nhận được cuối cùng là S thì S sẽ không nhỏ hơn tổng các căn bậc hai của các
số ban đầu. Như thế, ta có
³p p p p ´2
S ≥ 1+ 2+ 3+···+ n .

Ta sẽ chứng minh bất đẳng thức sau bằng quy nạp


³p p p p ´2 4 n3
1 + 2 + 3 + ... + n ≥ . (∗)
9
Với n = 2, ta có
³p p ´2 p 32 4 · 8
1+ 2 = 3+2 2 > 5 > =
9 9

Nguyễn Tất Thu-GV Trường Chuyên Lương Thế Vinh 10


Tổ hợp

nên (*) đúng.


Giả sử (*) đúng đến n = k ≥ 2 thì
³p p p p ´2 4 k3
1 + 2 + 3 + ... + k ≥ .
9
Ta có
s 2 s
3 3
³p p p p p ´2 4 k p 4 k 4 k3 ( k + 1)
1 + 2 + 3 + ... + k + k + 1 ≥  + k + 1 = +k+1+2 .
9 9 9

Ta cần chứng minh


s
4k 3
4 k3 ( k + 1) 4( k + 1)3 p
+k+1+2 > ⇔ 144 k3 ( k + 1) > 12 k2 + 3 k − 5.
9 9 9

Khai triển ra ta được 72k3 + 111 k2 + 30k > 25, đúng nên (*) cũng đúng với n = k + 1.
Theo nguyên lí quy nạp, (*) được chứng minh.
4 n3
Vậy số nhận được cuối cùng trên bảng luôn không nhỏ hơn .
9
Bài tập 6. (Azerbaijan TST 2015) Tập A = {a 1 , a 2 , ..., a n } gồmn > 2 số nguyên dương
phân biệt được gọi là tập “ tốt” nếu a2016
i
luôn chia hết cho tích a 1 a 2 ...a i−1 a i+1 ...a n với mọi
i = 1, 2016. Tìm số nguyên dương n lớn nhất để tồn tại một tập “tốt” có đúng n phần tử.

Lời giải. Giả sử A = {a 1 , a 2 , ..., a n } là tập “tốt” với a 1 < a 2 < ... < a n . Đặt P1 = a 2 a 3 ...a n , theo
đề bài ta có P1 ¯a2016 nên P1 ≤ a2016 . Mặt khác P1 > a 1n−1 nên
¯
1 1

n − 1 < 2016 ⇒ n < 2017 ⇒ n ≤ 2016.

Ta chứng min với mọi 3 ≤ n ≤ 2016 luôn tồn tại một tập “tốt” có n phần tử.
Với n = 3 ta chọn A = {6, 18, 36}.
Giả sử A = {a 1 , a 2 , ..., a k } là tập “tốt” với k < 2016.
Đặt P = a 1 a 2 ...a k và
b 0 = P, b 1 = a 1 P, b 2 = a 2 P, ..., b k = a k P.

Khi đó
b 1 b 2 ...b k = P k a 1 a 2 ...a k ¯P 2016 = b2016
¯
0


b 0 b 1 ...b i−1 b i+1 ...b k = P k a 1 ..a i−1 a i+1 ...a k ¯ b2016
¯
i .

Do đó tập B = {b0 , b1 , ..., b k } là tập “tốt” có k + 1 phần tử.


Theo nguyên lí quy nạp ta có đpcm.

Bài tập 7. Cho một bảng hình vuông n × n. Hỏi phải tô màu đỏ ít nhất bao nhiêu ô vuông
đơn vị để ta luôn chọn được một hình vuông 2 × 2 có chứa ít nhất ba ô được tô màu đỏ.

Nguyễn Tất Thu-GV Trường Chuyên Lương Thế Vinh 11


Tổ hợp

Bài tập 8. (VN TST 2006) Trong không gian cho 2016 điểm mà trong đó không có 4
điểm nào đồng phẳng. Người ta nối tất cả các điểm đó lại bởi các đoạn thẳng. Số tự nhiên
m gọi là số "tốt" nếu ta có thể gán cho mỗi đoạn thẳng trong các đoạn thẳng đã nối bởi
một số tự nhiên không vượt quá m sao cho mỗi tam giác tạo bởi ba điểm bất kì trong số
các điểm đó đều có hai cạnh được gán bởi hai số bằng nhau và cạnh còn lại gán bởi số lớn
hơn hai số đó. Tìm số tốt có giá trị nhỏ nhất.

Bài tập 9. (VN TST 2007) Cho hai tập hợp A , B là tập hợp các số nguyên dương thỏa
mãn | A | = |B| = n (với n là số nguyên dương) và có tổng các phần tử bằng nhau. Xét bảng
ô vuông n × n. Chứng minh rằng ta có thể điền vào mỗi ô vuông của bảng một số nguyên
không âm thỏa mãn đồng thời các điều kiện:

i) Tập hợp tổng các số ở mỗi hàng là tập A .

ii) Tập hợp tổng các số ở mỗi cột là tập B.

iii) Có ít nhất (n − 1)2 + k số 0 trong bảng với k là số các phần tử chung của A và B.

Lời giải. Trước hết, ta thấy rằng nếu một giá trị k sao cho tồn tại 2 phần tử bằng nhau
ở mỗi tập là a k = b k = t thì ta điền số t vào ô vuông nằm ở hàng thứ k và cột thứ k, các
ô còn lại của hàng thứ k và cột thứ k đều điền vào số 0; như thế thì tổng các số ở hàng
và cột này thỏa mãn đề bài và không ảnh hưởng đến các hàng và cột khác. Do đó, không
mất tính tổng quát, ta xét trường hợp A ∩ B = ∅(trường hợp có các phần tử chung thì điền
thêm vào các hàng và cột theo cách tương tự như trên), tức là số phần tử chung của hai
tập là k = 0 .
Ta sẽ chứng minh bài toán này bằng quy nạp.
Gọi T là tập hợp các điều kiện i), ii), iii) như trên (điều kiện iii) tương ứng với trường hợp
xét số nguyên dương n). Với n = 1, bài toán hiển nhiên đúng.
Giả sử bài toán đúng với mọi số tập hợp có n − 1 phần tử. Ta sẽ chứng minh rằng với hai
tập A , B có n phần tử, ta cũng có thể xây dựng một bảng n × n thỏa mãn điều kiện T .
Thật vậy, ta xét hai tập

A = {a 1 , a 2 , ..., a n } , B = { b 1 , b 2 , ..., b n } ,

trong đó a 1 < a 2 < ... < a n , b1 < b2 < ... < b n và hai tập này không có phần tử chung.
Giả sử a 1 < b i . Vì tổng các phần tử bằng nhau nên tồn tại chỉ số i sao cho

a i > b 1 > b 1 − a 1 ⇒ a i − ( b 1 − a 1 ) > 0.

Xét tập A 0 , B0 như sau

A 0 = {a 2 , a 3 , ..., a i−1 , a i − b 1 + a 1 , ..., a n } , B0 = { b 2 , ..., b n } .

Hai tập hợp này có cùng số phần tử là n–1 nên theo giả thiết quy nạp, tồn tại một bảng
có kích thước (n − 1) × (n − 1) thỏa mãn điều kiện T (trong bảng này có ít nhất (n − 2)2 số 0).
Ta thêm vào bên trái bảng một cột và bên trên bảng một hàng nữa như hình vẽ. Ở ô góc

Nguyễn Tất Thu-GV Trường Chuyên Lương Thế Vinh 12


Tổ hợp

bên trái và phía trên, ta điền số a 1 , ở hàng thứ i của bảng ban đầu (hàng có tổng các
phần tử bằng a i − b1 + a 1 ), ta điền số b1 − a 1 ; còn tất cả các ô còn lại của hàng và cột vừa
thêm vào, ta điền vào các số 0. Khi đó, bảng này có tổng các phần tử ở mỗi hàng là tập A
và tổng các phần tử ở mỗi cột là tập B, số các số 0 ở bảng vừa lập được không nhỏ hơn

( n − 2)2 + 2( n − 1) − 1 = ( n − 1)2

và do đó nó thỏa mãn điều kiện T .


Do đó, bài toán cũng đúng với mọi tập hợp có n phần tử. Theo nguyên lí quy tạp, bài toán
này đúng với mọi số nguyên dương n.
Vậy ta có đpcm.

Bài tập 10. Có n + 1 quả cân lần lượt có các khối lượng 30 , 31 , · · · , 3k , · · · , 3n . Chứng minh
rằng bằng cân đĩa với n + 1 quả cân đã cho, ta có thể cân mọi vật có khối lượng là số
nguyên x nằm trong đoạn [1; S n ] với S n = 30 + 31 + · · · 3n .

Lời giải. Giả sử cân có đĩa A và đĩa B. Ta cân vật có khối lượng x bằng cách đặt vật lên
đĩa A, sau đó đặt một số quả cân lên cả hai đĩa A và B sao cho cân thăng bằng. Nếu phép
cân được thực hiện thì

x = ( tổng khố lượng các quả cân đĩa B) − (tổng các quả cân ở đĩa A),

hay
x = a 0 30 + a 1 31 + · · · + a k 3k + · · · + a n 3n . (1)

trong đó a k chỉ nhận các giá trị 1, 0, −1 tùy thuộc vào quả cân 3k được đặt lên đĩa B, đĩa
A hay không được đặt lên đĩa nào.
Bằng quy nạp ta chứng minh với mọi số nguyên x ∈ [1; S n ] luôn tồn tại các số a i ∈ {−1, 0, 1}
mà x thỏa mãn (1).

• Với n = 0 thì bài toán hiển nhiên đúng.

• Với n = 1 ta có S 1 = 4 và

1 = 30 , 2 = (−1) · 30 + 21 , 3 = 31 , 4 = 30 + 31 .

• Giả sử bài toán đúng đến n − 1, tức là với mọi số nguyên y mà 1 ≤ y ≤ S n−1 đều tồn
tại bộ số a i ∈ {−1, 0, 1}, i = 1, n − 1 sao cho

y = a 0 .30 + a 1 .31 + · · · + a n−1. 3n−1 .

Ta xét x > S n−1 .

+ Nếu S n−1 < x < 3n , đặt y = 3n − x ta có y ∈ [1; S n−1 ]. Theo giả thiết quy nạp ta có
nX
−1 nX
−1
y= a i · 3 i , hay x = − a i · 3 i + 3n .
i =0 i =1

+ Nếu x = 3n , hiển nhiên đúng.

Nguyễn Tất Thu-GV Trường Chuyên Lương Thế Vinh 13


Tổ hợp

+ Nếu 3n < x ≤ S n , đặt y = x − 3n , ta có y ∈ [1; S n−1 ]. Theo giả thiết quy nạp ta có
nX
−1 nX
−1
y= a i · 3 i , hay x = a i · 3 i + 3n .
i =0 i =1

Vậy bài toán được chứng minh.

Bài tập 11. Cho đa giác lồi n cạnh (n ≥ 4). Kẻ kn + 1 đường thẳng chéo khác nhau (k ≥ 0
n( n − 3)
nguyên, kn + 1 ≤ ). Chứng minh rằng từ các đường chéo đã kẻ đó có thể chọn ra
2
được 2k + 1 đường tạo thành một đường gấp khúc mà không có hai cạnh nào cắt nhau.

Lời giải. Ta chứng minh bài toán bằng phương pháp quy nạp theo k: rằng tồn tại đường
gấp khúc về cả hai hướng thỏa mãn đầu bài.

Hình 1

• Với k = 0, có 1 đường chéo được kẻ và ta chọn đường chéo đó.

• Giả sử khẳng định đúng đến k = m, ta chứng minh khẳng định đúng với k = m + 1.
Giả sử ta đã kẻ được (m + 1)n + 1 đường chéo. Tại mỗi đỉnh mà ở đó ta kẻ ít nhất
một đường chéo, sắp xếp các đường chéo theo thứ tự ngược chiều kim đồng hồ, rồi
tô đỏ đường chéo đầu tiên. Ta đã tô đỏ nhiều nhất n đường chéo, như vậy có ít nhất
m.n + 1 đường chéo không được tô màu. Ta chọn ra đúng mn + 1 đường trong số này.
Theo quy nạp ta có thể chọn theo cả hai hướng, mỗi hướng một đường gấp khúc có
độ dài 2m + 1 cạnh thỏa mãn bài toán.
Chọn một trong hia đường gấp khúc đó, chẳng hạn ta chọn đường gấp khúc (P k )
như hình 2a. Khi đó ta có thể bổ sung vào đường gấp khúc (P k ) này hai đường chéo
đã tô đỏ vào hai đầu mút của đường khúc (P k ). Ta có

(a) Tại mỗi đầu mút của (P k ) có một đường chéo đỏ;
(b) Hai đường chéo tô đỏ này không thuộc đường gấp khúc (P k );
(c) Do các chọn hai đường chéo tô đỏ này không thể cắt các cạnh của đường gấp
khúc (P k ).

Đường gấp khúc (P k+1 ) mới là được gấp khúc (P k ) được mở rộng với hai đường chéo
tô đỏ, có số cạnh là 2( m + 1) + 1 và thỏa mãn yêu cầu đề bài.
Tương tự cho trường hợp ta chọn đường gấp khúc hình 2 b.

Vậy bài toán được chứng minh.

Nguyễn Tất Thu-GV Trường Chuyên Lương Thế Vinh 14


VẺ ĐẸP CỦA CÁC BÀI TOÁN TỔ HỢP ĐẶC SẮC TRÊN LƯỚI VUÔNG

Trong mỗi bài toán tổ hợp ra các kỳ thi học sinh giỏi, ta nhìn thấy ở đó có những đột phá và sự
thông minh ở từng cách giải. Mỗi bài toán như một bông hoa “kiêu kì” trong vườn hoa toán học.
Chúng ta hãy cùng nhau khám phá vẻ đẹp trong từng cách giải của các bài tổ hợp trên lưới vuông
là đại diện cho một số phương pháp thường sử dụng trong toán tổ hợp như: phương pháp truy hồi,
phương pháp quy nạp, phương pháp song ánh, phương pháp phản chứng, phương pháp tô màu, bất biến,
nguyên lí cực hạn, nguyên lí Dirichlet,….

1. Một số bài toán đếm thông thường.


Bài 1: Trong một bảng ô vuông kích thước 999  999 , mỗi ô được tô bởi một trong hai màu trắng hoặc
đỏ. Gọi T là số bộ  C1 , C2 , C3  các ô mà hai ô đầu trong cùng một hàng và hai ô cuối trong cùng một cột,
với C1 và C3 màu trắng, C2 màu đỏ. Tìm giá trị lớn nhất của T.

Phân tích: Ta cần chọn ra ô đặc biệt nhất trong ba ô  C1 , C2 , C3  và thực hiện thao tác đếm số bộ ba ô
 C1 , C2 , C3  “trắng – đỏ – trắng” trên hàng thứ i và cột thứ j.
Nhận xét đánh giá, tìm ra sự đặc biệt để đánh giá.
HD:
4n 4
Ta chứng minh trong một bảng vuông n  n có nhiều nhất bộ như vậy. Giả sử hàng i và cột j
27
chứa ai và b j ô trắng tương ứng, R là tập hợp các ô đỏ. Với mỗi ô đỏ  i; j  có ai .b j bộ  C1 , C2 , C3  chấp
nhận được với C2   i; j  vì vậy T   ai b j
 i , j R

Áp dụng bất đẳng thức 2ab  a 2  b2 ta có


1 1  1 1 1 n 1 n
T  ai b j    ai2  b 2j     ai2   b 2j    ai2   b 2j    n  ai  ai2    n  b j  b 2j
 i , j R 2 i , j R 2  i , j R  2 i , j R
 i , j R  2 i , j R 2 i 1 2 j 1
Do có  n  ai  ô đỏ tại hàng i,  n  bi  ô đỏ tại cột j,
Áp dụng bất đẳng thức AM – GM ta có
3
1 2 1  2n  4n3
 n  x  x   2n  2 x  .x.x  .    .
2 2  3  27
2n
Đẳng thức xảy ra khi 2n  2 x  x  x  .
3
1 n 4n 3 1 n 4n3 1 4n 3 1 4n 3 4n 4
Suy ra T      .n.  .n.  27 .
2 i 1 27 2 j 1 27 2 27 2 27
2n
(Max T đạt được khi số ô màu trắng ở mỗi hàng và mỗi cột cùng bẳng ).
3
2n 2.999
Nếu n  999 thì tô màu tuỳ ý bảng ô vuông với x    666 ô trắng trong mỗi hàng và cột.
3 3
Cách tô: Chẳng hạn tô màu ô  i; j  trắng nếu i  j  1, 2,..., 666  mod 999  , và các ô khác màu đỏ.
4.9994
Vậy giá trị lớn nhất của T có thể đạt được là .
27
Có thể minh hoạ cách tô trên trong bảng 6  6 như sau

Lời bình: Ngoài cách tô trên ta có thể ti6 theo nhiều cách khác. 
Bài 2: Cho lưới ô vuông đơn vị kích thước (n 2  n  1)  (n 2  n  1) . Trên mỗi ô ta ghi vào số 0 hoặc 1,
sao cho không có bốn số 1 nào là đỉnh của một hình chữ nhật. CMR tổng các số có trong lưới không vượt
quá (n  1)(n2  n  1).

Phân tích: Việc tìm tổng các số có trong bảng cũng chính là việc đếm xem trong bảng có bao nhiêu
con số 1.
Với giả thiết số chữ số 1 ở mỗi hàng có biến động nên việc gọi ẩn là điều chúng ta có thể suy nghĩ đến.

HD:
1 0
1 1 0
0 1 1 0
0 1 1 0
0 1 1 0
0 1 1 0
0 1 1
Đặt N  n2  n  1
Gọi xi là số số 1 ở hàng thứ i i  1, N .
N
Đặt S   xi . Ta cần chứng minh S   n  1 .N
1

Xét tập M gồm các cặp  k ; l  với 1  k  l  N .

Ta có M  C 2

n 2
 n  n 2  n  1
N
2
Với mỗi i, i  1, N . Đặt M i là tập gồm các cặp  k ; l  với 1  k  l  N , sao cho hai cột k và l của lưới đều
có số 1 ở hàng thứ i. Ta có
Cx2i xi  2 x  x  1
Mi    i i .
0 0  xi  2 2
Từ giả thiết không có bốn số 1 nào ở đỉnh của một hình chữ nhật nên M i  M j   nếu i  j.
N N
Rõ ràng  M i  M nên
i 1
M
1
i  M ,
xi  xi  1  n  n  n  n  1
N 2 2 N N
    xi2   xi   n 2  n  n 2  n  1  0. (1)
i 1 2 2 i 1 i 1
2
 N  N N
S2 N 2
Mà   xi   N  xi   n  n  1  xi 
2 2 2
  xi (Bất đẳng thức BCS)
 i 1  i 1 i 1 N i 1
Thế vào (1) suy ra
S2 2
2
 S   n 2  n  n 2  n  1  0  S 2   n 2  n  1 S   n 2  n  n 2  n  1  0
n  n 1
n 2  n  1   2n  1  n 2  n  1
S   n  1  n 2  n  1 . 
2
2. Phương pháp quy nạp.
Bài 3: Giả sử mỗi ô của bảng  2n  1;2n  1 có chứa số thuộc đoạn  1;1 và tổng 4 số trong các bảng
con 2  2 luôn bằng 0. Với mỗi n cố định hãy tính giá trị lớn nhất tổng các số có mặt trên bảng.

HD:
Ta thử bắt đầu với một vài trường họp nhỏ, gọi S là tổng các số có mặt trên bảng.
TH1: Với n  1
a11 a12 a13

a21 a22 a23

a31 a32 a33

Ta có S1  a11  a22  a33  a11  a22  a33  3.


TH2: Với n  2
a11 a12 a13 a14 a15

a21 a11 a12 a24 a25

a31 a21 a33 a34 a35


a41 a42 a43 a44 a45

a51 a52 a53 a54 a55


S 2  S1  S12  a33  ( a11  a 22  a33 )  ( a33  a 44  a55 )  a33  a11  a 22  a33  a44  a55  5
TH3: Với n  3
a11

a11
a33

a44
a55

a66

a77
S 3  S 2  S13  a55  a11  a 22  a33  a 44  a55  a66  a77  7
Tương tự bằng quy nạp ta luôn có thể tính được cho trường hợp n là số bất kì
S  Sn1  S1n  a2 n 1,2 n1

S n 1

a2 n 1,2 n 1

S1n

Với Sn1  a11  a22  a33  ...  a2 n1,2 n 1 , S1n  a2 n 1,2 n1  a2 n ,2 n  a2 n 1,2 n 1
2 n 1
i 1
Suy ra S  a11  a22  a33  ...  a2 n1,2 n1  a2 n,2 n  a2 n 1,2 n 1    1
i 1
.aii  2n  1. 

Bài 4: Người ta lát kín một nền nhà hình vuông cạnh dài n  2t , (t là một số nguyên lớn hơn 1) khuyết
một ô thoát nước tại vị trí ( x; y ) bằng những viên gạch màu hình thước thợ (chữ L) gồm 3 ô đơn vị). Hai
viên gạch kề nhau (dù chỉ một đơn vị dài) phải khác màu. Các ô trong nền nhà được đánh số theo dòng
x  1, 2,...n tính từ trên xuống và theo cột y  1, 2,...n tính từ trái qua phải. Chứng minh rằng ta luôn có
thể thực hiện được công việc với không quá 3 màu gạch và vị trí ô thoát nước là tuỳ ý.
HD:
TH1: Với n  2 thì ta chỉ cần dùng 1 màu để lát.

(Ô không tô là lỗ thoát nước)


TH2: Với n  2 ta sẽ chứng minh bằng cách chỉ ra các lát với 3 màu.
Xét một hình vuông kích thướt n  2t bất kì có lỗ thoát nước tại ô (n; n) . Ta bắt đầu lát từ hình
vuông 2  2 góc trên bên trái như hình a). Lấy đối xứng theo trục đứng và trục ngang, đồng thời đổi hình
số i thành hình số 4  i , tịnh tiến theo đường chéo hình vuông 2  2 ban đầu, qua thao tác này ta thu
được hình b), cứ làm liên tục thao tác để hình thành hình b) gồm lấy đối xứng theo trục đứng và trục
ngang, đồng thời đổi hình số i thành hình số 4  i , tịnh tiến theo đường chéo hình vuông 2t 1  2t 1 sau
nhiều lần ta thu được hình c) hình này gồm các viên gạch lát kín hình vuông 2t  2t với ô thoát nước ở vị
trí (2t ; 2t ).

Bây giờ chuyển sang bước 2 ta sẽ di chuyển ô thoát nước vị trí (2t ; 2t ) đến vị trí ô ( a; b) bất kì
của bảng.
Bổ đề 1: Trong hình c) thì ta luôn đưa được ô (2t ; 2t ) về vị trí 1 trong 4 ô trung tâm (2t 1; 2t 1 ),
(2t 1  1; 2t 1 ),(2t 1 ; 2t 1  1),(2t 1  1; 2t 1  1).
Chứng minh: Bằng cách cho ô (2t ; 2t ) di chuyển trên đường chéo ta luôn đưa được ô này về vị trí
(2t 1  1; 2t 1  1) . Từ đó bằng cách xoay hình lần lượt các góc 900 ,1800 , 2700 quanh tâm I của hình vuông
ban đầu ta có thể di chuyển ô thoát nước từ vị trí đến các vị trí (2t 1; 2t 1 ), (2t 1  1; 2t 1 ), (2t 1 ; 2t 1  1). Bổ
đề đã được chứng minh.
Bổ đề 2: Với hai ô (2t ; 2t ) (ô thoát nước) và ô (a; b) bất kì ta luôn có thể di chuyển hai ô này về
cùng 1 mảnh phần tư của một hình vuông của hình vuông 2t 1  2t 1 và ô thoát nước luôn nằm tại góc của
hình vuông 2t 1  2t 1 .
Chứng minh: Ta tạm gọi 4 mảnh trong hình vuông bất kì theo tên gọi là A,B,C,D như hình vẽ.
A B
C D
Ô thoát nước (2t ; 2t ) luôn nằm trong mảnh D.
TH1: Nếu ô (a; b) cũng nằm trong mảnh D thì bổ đề 2 luôn đúng.
TH2: Nếu ô (a; b) không nằm trong mảnh D thì với bổ đề 1 ta luôn có thể đưa ô thoát nước đến
một trong các ô (2t 1; 2t 1 ), (2t 1  1; 2t 1 ),(2t 1 ; 2t 1  1) các ô này sẽ có một ô nằm cùng mảnh phần tư với ô
(a; b).
Bổ đề 2 đã chứng minh xong.
Bây giờ ta trở lại với bài toán chính ban đầu, với việc sử dụng bổ đề 2 sau một số lần ta sẽ thu
được trạng thái là ô thoát nước và ô (a; b) nằm cùng trong một hình vuông 2  2 và đến đây thì việc đổi
chổ ô thoát nước và ô (a; b) ta chỉ cần thực hiện sau nhiều nhất 3 phép quay mà thôi.
Đến đây bài toán hoàn toàn được giải quyết. 

Bài 5: Một người thợ luôn phải cắt lát những căn phòng có kích thước 2 n  2n ô vuông bằng các viên đá
lát hình đô-mi-nô kích thước 1  3 ô vuông và các viên đá hình thước thợ kích thước 3 ô vuông. Anh ta
phải lát kín căn phòng, chỉ trừ lại 1 ô vuông để trống không lát để trang trí đặc biệt. Do giá thành 1 viên
đá lát hình thước thợ đắt hơn nhiều giá thành 1 viên đá hình đô-mi-nô cho nên anh ta sử dụng càng ít viên
đá hình thước thợ càng tốt. Anh ta nhận thấy rằng dù ô vuông đặc biệt ở vị trí nào đi nữa chưa lần nào anh
ta dùng hơn n viên đá hình thước thợ để hoàn thành công việc của mình. Hãy chứng minh rằng khẳng
định của người thợ luôn đúng với mọi số nguyên dương n.

HD:
Ta chứng hai bổ đề sau
Bổ đề 1: Mọi hình chữ nhật có kích thước 1 cạnh chia hết cho 3 luôn được lát bởi các viên đa hình đô-
mi-nô 1  3 ô vuông.
Bổ đề này hiển nhiên đúng.
Bổ đề 2: Mọi hình thước thợ tạo bởi 3 hình vuông kích thước 2 n  2n đều được lát bởi
một số viên đá hình đô-mi-nô và 1 viên đá hình thước thợ.
Chứng minh: (quy nạp)
Với n  1 bổ đề luôn đúng.
Giả sử bổ đề đúng với n  k .
Với n  k  1 thì ta chia hình thướt thợ tạo bởi ba hình vuông 2k 1  2k 1 thành hai
hình chữ nhật kích thức 2k  2k.3 , 2k 1  2k.3 và một hình thướt thợ được tạo bởi ba
hình vuông 2k  2k theo giả thiết quy nạp hình thướt thợ này được lát bởi một số viên
đá hình đô-mi-nô và 1 viên đá hình thước thợ.
Bây giờ ta sẽ chứng minh bài toán ban đầu. (cũng bằng quy nạp)

Với n  1 thì hình vuông 2  2 dùng 1 viên đá hình thước thợ suy ra bài toán 2k
đúng với n  1.
Giả sử bài toán đúng với n  k . 2k 1 2
Với n  k  1 ta chia hình vuông 2k 1  2k 1 thành 4 hình vuông 2k  2k có thể 2k 1
đặt tên là hình vuông thứ 1,2,3,4. Hình vuông đặc biệt sẽ nằm ở một trong 4 hình 4 3
vuông 2k  2k giả sử đó là hình vuông thứ 1.
Theo giả thiết quy nạp thì để lát được hình vuông 1 (không lát ô đặc biệt) ta chỉ
cần dùng không quá k viên đá hình thướt thợ. Mặt khác với hình thướt thợ tạo bởi 2k 1
3 hình vuông thứ 2,3,4 theo bổ đề thì ta chỉ cần dùng 1 hình thước thợ.
Tóm lại để lát kín hình vuông 2k 1  2k 1 ta cần dùng không quá k  1 viên đá hình thướt thợ kích
thước 3 ô vuông. Điều phải chứng minh. 

3. Phương pháp truy hồi.


Bài 6: (Dự tuyển IMO – 1996)

Cho bảng ô vuông n  n (n  1). Hỏi có bao nhiêu cách đánh dấu các ô vuông trong bảng sao cho
trong mỗi hình vuông 2  2 luôn có đúng 2 ô được đánh dấu. (Hai cách đánh dấu được gọi là khác nhau
nếu có một ô vuông nào đó mà trong cách này nó được đánh dấu còn trong cách kia nó không được đánh
dấu).
HD:
Gọi Sn là số cách đánh dấu các ô trong bảng n  n  n  1 .
Xét tập T gồm các ô vuông nằm trong cột cuối cùng (tính từ bên phải sang) và hàng cuối cùng (tính
từ trên xuống).

Gọi An là các cách đánh dấu mà có 2 ô vuông kề nhau trong T cùng được đánh dấu hoặc cùng
không được đánh dấu và Bn là các cách đánh dấu mà các ô trong T được đánh dấu xen kẻ nhau.
Ta thấy mỗi cách đánh dấu thuộc Bn sẽ ứng với một cách đánh dấu trong Bn1 , mỗi cách đánh dấu
thuộc An sẽ ứng với một cách đánh dấu thuộc An1 và một cách đánh dấu thuộc Bn1.
Từ đó ta có Bn  Bn1 , An  An1  Bn 1
Sn  An  Bn  2Sn1  Sn  2 , n  3
S2  6, S3  14
 Sn  8n  10, n  1. 
Bài 7: Có 2n người xếp thành 2 hàng dọc. Hỏi có bao nhiêu cách chọn ra một số người (ít nhất 1) từ 2n
người này, sao cho không có hai người nào đứng kề nhau được chọn. Hai người đứng kề nhau là hai
người có số thứ tự liên tiếp trong một hàng dọc hoặc có cùng số thứ tự ở hai hàng.

HD:
Gọi Sn là số cách chọn ra một số người từ 2n người xếp thành 2 hàng dọc và Tn là số cách chọn ra
một số người từ 2n  1 người xếp thành 2 hàng dọc, trong đó khuyết một chỗ ở đầu của một hàng. Ta có
S1  2, T1  1.
1 2
3 4
Hình 1

1 2

Hình 2
 Xét 2n người xếp thành 2 hàng dọc (như hình 1). Ta xét các cách chọn thoả mãn điều kiện đầu bài.
Xảy ra các khả năng sau :
1) Người ở vị trí số 1 được chọn : Khi đó người ở vị trí số 2 và số 3 không được chọn Có Tn1  1 cách
chọn (+1 là do bổ sung cách chọn “không chọn gì cả”)
2) Người ở vị trí số 2 được chọn : Tương tự, có Tn1  1 cách chọn.
3) Cả hai người ở vị trí số 1 và số 2 đều không được chọn: Có Sn1 cách chọn.
Vậy ta có Sn  Sn 1  2Tn1  2 (1).
 Xét 2n  1 người xếp thành 2 hàng dọc (như hình 2). Ta xét các cách chọn thoả mãn điều
kiện đầu bài. Xảy ra các khả năng sau :
1) Người ở vị trí số 1 được chọn: Khi đó người ở vị trí số 2 không được chọn có Tn1  1 cách chọn.
2) Người ở vị trí số 1 không được chọn: có Sn1 cách chọn.
Vậy ta có Tn  S n1  Tn 1  1 (2).
Từ (1) ta suy ra 2Tn1  Sn – Sn1 – 2  2Tn  Sn1 – Sn – 2. Thay vào (2), ta được
Sn1 – Sn – 2  2Sn 1  Sn – Sn 1 – 2  2
Sn 1  2S n  Sn 1  2
Từ đây dễ dàng tìm được
n 1 n 1

Sn 
1  2  
 1 2 
.
2
2
Bài 8: Có bao nhiêu cách lát đường đi kích thước 3  2n bằng các viên gạch có kích thước 1  2.
HD:
Gọi c n là số cách lát đường đi kích thước 3  2 n. Ta thực hiện phân hoạch các trường hợp theo sơ đồ
sau:
Theo sơ đồ ta có cn  3cn 1  2bn 1 (1)
Xét riêng các bn1 ta có sơ đồ

Theo sơ đồ ta có bn 1  cn 2  bn2
 bn  cn1  bn1 (2)
 cn 1  3cn
bn  2
(1)  cn 1  3cn  2bn  
b  cn  3cn 1
 n 1 2
c  3cn c  3cn1
Thế vào (2) n1  cn 1  n
2 2
 cn1  3cn  2cn1  cn  3cn 1
 cn1  4cn  cn 1
Dễ thấy
c1  3
c2  11

Vậy cn thỏa c1  3, c2  11, cn1  4cn  cn1. 

4. Phương pháp song ánh.


Bài 9: Có bao nhiêu các đi ngắn nhất trong lưới nguyên từ điểm (0,0) đến (m,n). (ứng với m cột, n
hàng)

HD: Mỗi đường đi ngắn nhất lập từ m đoạn đi sang trái và n đoạn đi lên, là một bộ sắp thứ tự T,L,L,T,...
gồm m chữ T và n chữ L.
Dễ thấy số cách đi này là Cmm n (số cách chọn m vị trí trong xâu có độ dài m  n để đặt các chữ T vào).
Bài 10: Cho n là số nguyên dương thỏa mãn các tính chất: nếu n cái domino được đặt trên một bàn cờ
6  6 với mỗi domino chiếm 2 đơn vị diện tích vuông, thì luôn luôn có thể đặt thêm 1 domino lên trên bàn
mà không phải di chuyển bất kỳ domino nào khác. Xác định giá trị lớn nhất của n.

HD:
Ta sẽ chứng minh giá trị lớn nhất của n là 11. Hình 1 là cách sắp xếp 12 domino trên bàn cờ thì không thể
để thêm 1 domino nào nữa. Suy ra n  11.
1 2
3 4
5 6
7 8
9 10
11 12
Hình 1
Ta sẽ chứng minh rằng với 11 domino trên bàn cờ thì luôn đặt thêm được 1 domino nữa. Ta sẽ chứng
minh bằng phản chứng. Giả sử rằng tồn tại cách đặt 11 domino trên bàn cờ mà không thể đặt thêm 1
domino nào nữa.
Khi đó số ô vuông trên bàn chưa bị phủ bởi 11 domino là: 36  22  14.
Đặt S1 là phần trên của bàn cờ ban đầu có kích thước 5x6 (Hình 2). Đặt A là tập các ô vuông trong S1
mà không bị phủ bởi các domino và S2 là hàng cuối của bàn cờ (bàn cờ chia làm 2 phần S1 và S2). Vì ta
không thể điền thêm 1 domino nào vào bàn cờ, nên một trong 2 ô vuông bất kỳ cạnh nhau sẽ được bao
phủ bởi 1 domino, suy ra S2 có nhiều nhất là 3 ô không bị phủ bởi domino (trống), suy ra S1 có ít nhất là
14  3  11 ô trống.

S1
S3

S2
Hình 2
Đặt S3 là phần dưới của bàn cờ và có kích thước 5  6 (Hình 2), B là tập tất cả các domino nằm trong
S3. Ta sẽ định nghĩa một ánh xạ f từ A vào B. Ta có, với mỗi ô vuông trống s trong S1, sẽ tồn tại một ô
vuông t (bị phủ) nằm kế phía dưới s. Suy ra ô vuông t nằm trong S3 và được phủ bởi một domino d. Rõ
ràng domino d phải nằm trong S3 (vì nếu ko thuộc S3 thì nó sẽ gồm 2 ô t và s vô lý). Khi đó ta xác định
ánh xạ f là f (s) = d. Ta sẽ chứng minh rằng f là đơn ánh. Thật vậy, giả sử s1 , s2  A, s1  s2 sao cho
f ( s1 )  f ( s2 )  d . Suy ra d phủ 2 ô vuông ở dưới s1 và s2 . Suy ra s1 và s2 nằm cạnh nhau (Hình 3), như
vậy khi đó ta có thể đặt thêm 1 domino phủ lên s1 và s2 (vô lý). Vậy f là đơn ánh, suy ra A  B hay
B  11 . Nhưng cả bàn cờ chỉ có 11 domino, suy ra B  11. Khi đó thì hàng trên cùng sẽ không bị phủ
bởi 1 domino nào, và sẽ đặt được thêm 1 quân domino nữa (vô lý).

s s1 s2
t d

Hình 3
Vậy giá trị lớn nhất của n là 11. 
5. Phương pháp tô màu kết hợp bất biến
Bài 11: Cho sàn nhà kích thướt 2017  2017. Hỏi sàn nhà này có lát được bằng các viên gạch có kích
thướt 4  4 và 5  5 không?

HD:
Đầu tiên ta tô màu đen cho các hàng các hàng lẻ và tô màu trắng cho các hàng chẵn.
Thế thì số ô đen nhiều hơn trắng là 2017 ô.
Nếu ta dùng viên gạch 4  4 lát thì nó sẽ phủ hết 8 ô trắng và 8 ô đen, nếu ta dùng viên gạch 5  5
lát thì nó sẽ phủ lên 10 ô trắng (đen) và 15 ô đen (trắng).
Do đó nếu ta lát được kín sàn nhà bằng hai loại gạch trên thì số ô trắng đen chênh lệch nhau phải là
bội số của 5. Nhưng 2017 không chia hết cho 5 do vậy ta có kết luận là không thể lát kín sàn nhà như đề
bài được. 
Bài 12: Cho bảng 11×12. Tô màu các ô của bảng theo quy tắc sau: mỗi lần tô 6 hoặc 7 ô chưa có màu
nằm liên tiếp nhau trên cùng một hàng hoặc một cột. Hỏi bằng cách đó có thể tô màu hết tất cả các ô
vuông sau 19 lần tô hay không ? Câu trả lời là thế nào với 20 lần tô.

HD:
Nếu dùng 19 lần: gọi x là số lần tô 6 ô và y là số lần tô 7 ô liên tiếp trên một hàng hoặc một cột.
Ta có hệ phương trình
 x  y  19 x  1
 
6 x  7 y  132  y  18

Hình 1
Tô các ô như hình vẽ, ta thấy mỗi cách tô 6 ô thì ứng với 3 ô đỏ (lẻ ô đỏ), mỗi cách tô 7 ô thì ứng với 4
ô đỏ (chẵn ô đỏ) do vậy nếu tô 1 lần 6 ô và 18 lần tô 7 ô thì tổng số ô đỏ là một số lẻ. Nhưng thực tế số ô
đỏ trên Hình 1 là một số chẵn. Chúng ta đã tìm ra được mâu thuẩn.
Nếu dùng 20 lần tô thì tương tự ta cũng có 8 lần tô 6 ô và 12 lần tô 7 ô dễ dàng chỉ ra cách tô thoả bài
toán như sau.

6 6
Hình 2
Bài 13: Một nền nhà hình chữ nhật được lát bởi các hình 2×2 và 1×4. Nhưng khi lát lại nền nhà thì vỡ
mất một viên 2×2, người ta thay bằng các viên 1×4. Hỏi họ có thể lát được nền nhà lại hay không?

HD:
Ta tô màu các ô theo quy tắc ô (lẻ;lẻ) là màu đỏ, các ô còn lại màu trắng.
1 2 3 …
1
2
3

Với cách tô trên thì viên gạch 2×2 sẽ chứa 1 ô đỏ còn viên gạch 1×4 sẽ chứa 0 hoặc 2 ô đỏ. Thế thì
khi vỡ 1 viên 2×2 tương ứng với 1 ô đỏ thì ta không thể thay thế được cách lát bởi các viên gạch 1×4.
Nhận xét: Mấu chốt ở đây là cần chỉ ra một cách tô mà trong cách tô đó thì tính chất (ở đây là tính
chẵn lẻ) của các viên gạch 2×2 và 1×4 luôn bất biến và các bất biến ở hai loại gạch này phải khác nhau.
Bài 14: Bàn cờ 8  8 được phủ bằng các thanh hình chữ nhật 1  3 và luôn trống 1 ô. Hãy chỉ ra các vị trí
có thể của ô trống?

HD:
1 2 3 1 2 3 1 2
2 3 1 2 3 1 2 3
3 1 2 3 1 2 3 1
1 2 3 1 2 3 1 2
2 3 1 2 3 1 2 3
3 1 2 3 1 2 3 1
1 2 3 1 2 3 1 2
2 3 1 2 3 1 2 3
Hình 1
Ta thấy khi dùng thanh 1  3 để lát (cả dọc và ngang) thì mỗi cách lát để ứng với 1 bộ ba số (1,2,3).
Do đó nếu lát hết hình (trống 1 ô) ta sẽ cần 21 viên ứng với 21 ô 1, 21 ô 2 và 21 ô 3. Vì vậy sẽ có 1 ô số 2
bị thừa (do trên hình có 22 ô số 2).
Suy ra ô trống có thể là 1 trong các ô số 2.
Mặt khác cách đánh số các ô còn có thể thay đổi thông qua việc lấy đối xứng và qua những phép
quay 900, 1800, 2700 nên tổng hợp lại ta có các ô trống chỉ có thể là một trong các ô trắng trên hình 2

Hình 2
Bài 15: Xét hình vuông 7×7 ô. Chứng minh rằng ta có thể xoá đi một ô để phần còn lại không thể phủ
kín bằng 15 quân trimino kích thước 1×3 và 1 quân trimino hình chữ L. Ta chứng minh rằng nếu bỏ đi
một ô ở góc trên bên trái thì phần còn lại không thể phủ được bằng các quân triminô đã cho.

HD: Để làm điều này, ta đánh số các ô vuông như sau

Khi đó, nhận xét rằng 1 quân triminô kích thước 1  3 sẽ che 3 số 1, 2, 3 (nếu nó nằm ngang) hoặc 3
số giống nhau (nếu nó nằm dọc). Như vậy tổng các số mà một quân triminô 1  3 che luôn chia hết cho 3.
Trong khi đó dễ thấy quân triminô hình chữ L che 3 số có tổng không chia hết cho 3.
Bây giờ giả sử ngược lại rằng hình vuông 7  7 bỏ đi ô ở góc trên bên trái có thể phủ được bằng 15
quân triminô 1  3 và 1 quân triminô hình chữ L thì theo lý luận trên, tổng số các số mà các quân triminô
này che sẽ không chia hết cho 3. Điều này mâu thuẫn vì tổng các số trên các ô còn lại bằng
20  1  14  2  14  3  90 chia hết cho 3.
Mâu thuẫn trên chứng tỏ điều giả sử là sai và ta có điều phải chứng minh.
Bài 16: Cho một bảng hình vuông kích thước 2014  2014. Kí hiệu (m,n) là ô ở hàng m cột n. Người ta
tô màu theo quy tắc sau: tô ba ô chưa có màu nằm cạnh nhau trên cùng một hàng hay cùng một cột. Hỏi
bằng cách tô đó có thể tô được tất cả các ô vuông trong phần còn lại của bảng hay không nếu:

a) Bỏ đi bốn ô vuông ở giữa bảng ô vuông.


b) Bỏ đi các ô vuông nằm ở các vị trí 1; a  ,  2; 2 a  ,  3; b  ,  4; 2b  trong đó 1  a , b  1007
HD:
a) Có thể tô màu hết.
Khi ta bỏ đi bốn ô ở giữa bảng hình vuông thì ta có thể chia hình còn lại thành 4 hình chữ nhật gồm
hai hình kích thướt 1008 1006 và hai hình vuông 1006 1008.

Do 1008 chia hết cho 3 nên 4 hình chữ nhật này luôn có thể tô được theo cách trên.
b) Ta sẽ chứng minh câu 2 này không thể tô hết.(phản đảo)
Đánh số các ô như hình vẽ
1 2 3 ... ... ... 2013 2014

1 2 3 ... ... ... 2013 2014

1 2 3 ... ... ... 2013 2014

1 2 3 ... 2013 2014

...
1 2 3 2013 2014

1 2 3 ... ... ... 2013 2014

1 2 3 ... ... ... 2013 2014

1 2 3 ... ... ... 2013 2014

Tổng tất cả các số là 2014. 1  2  ...  2014   2014.1007.2015 không chia hết cho 3.
Khi ta bỏ đi bốn ô 1; a  ,  2; 2 a  ,  3; b  ,  4; 2b  thì ta đang bỏ đi bốn số trong bảng có tổng chia hết
cho 3. Với cách tô màu như giả thiết thì luôn tô được 3 số có tổng chia hết cho 3. (hoặc dọc hoặc ngang)
Như vậy nếu ta tô màu được hết tức là tổng tất cả các số được tô màu và số bị bỏ đi là một số chia
hết cho 3. Điều này mâu thuẩn.
Bài 17: Cho 2n số thực đôi một khác nhau a1 , a2 ,..., an ; b1 , b2 ,..., bn . Viết các số lên bảng n  n như sau ô
(i; j ) hàng i cột j là (ai  b j ). Chứng minh rằng nếu tích tất cả các số trên cột bằng nhau thì tích tất cả các
số trên hàng cũng bằng nhau.

HD:
Tích các số trên cột thứ j bằng  j   b j  a1  b j  a2  ...  b j  an 
Tích các số trên các cột bằng nhau nên ta có  i   j i, j  1, 2,..., n.
Xét đa thức P( x)   x  a1  x  a2  ...  x  an  ta có
P(b1 )  P(b2 )  ...  P(bn )  C
Suy ra đa thức G ( x)  P( x)  C là đa thức bậc n có n nghiệm b1 , b2 ,...bn suy ra
G ( x)   x  b1  x  b2  ...  x  bn 
  x  a1  x  a2  ...  x  an   C   x  b1  x  b2  ...  x  bn  (1)
Thay lần lượt x  ai vào (1) ta được
C   ai  b1  ai  b2  ...  ai  bn 
Tích các số trên hàng thứ i là  ai  b1  ai  b2  ...  ai  bn  luôn bằng nhau. 
Bài 18: Trên mạng lưới ô vuông vô hạn người ta điền vào mỗi ô vuông cơ sở một số thực sao cho mỗi số
này bằng trung bình cộng của bốn số ở bốn hình vuông cơ sở có cạnh kề với nó.
a) Chứng minh rằng: Nếu các số được điền vào các ô vuông cơ sở là những số nguyên dương thì
các số đó phải bằng nhau.
b) Nếu các số được điền là các số hữu tỉ thì các số được điền vào các ô vuông cơ sở có cạnh kề
với nó, có nhất thiết phải bằng nhau không? Giải thích?

HD:
a) Vì các số thực được điền vào các ô vuông là những số nguyên dương nên tồn tại số a nhỏ nhất
trong các số được điền.
Giả sử tồn tại một ô vuông cơ sở có chứa số a mà bốn ô vuông cơ sở có cạnh liền kề có ít nhất một
ô vuông có chứa số b  a . Gọi c, d, e là ba số ở ba ô vuông cơ sở có cạnh liền kề còn lại.
b  a 1
Khi đó:    b  c  d  e   a , trái với giả thiết.
c, d , e  a 4
Như vậy nếu có một ô vuông có chứa số a thì bốn ô vuông có cạnh liền kề với nó cũng chứa số a.
Do đó tất cả các ô vuông đều chứa số a.
b) Ta xây dựng một hệ trục tọa độ vuông góc có các trục tọa độ song song hoặc trùng với các cạnh
của lưới ô vuông và có đơn vị trên mỗi trục bằng độ dài cạnh của ô vuông cơ sở. Ở mỗi hình vuông cơ sở
ta điền một số bằng trung bình cộng hoành độ và tung độ của tâm của hình 6
vuông đó.
Khi đó do tọa độ của tâm các hình vuông cở sở đều là số hữu tỉ nên B

số đặt vào đó cũng là số hữu tỉ. Và số đặt vào bốn ô vuông cơ sở có cạnh 4
kề với nó không bằng nhau. A I
C

Ta chứng minh số điền vào các ô vuông cơ sở bằng trung bình cộng
của bốn số ở bốn ô vuông có cạnh liền kề với nó như sau: Không mất tổng 2
D

quát giả sử có hình vẽ bên. Khi đó tâm của bốn hình vuông cơ sở A, B, C,
D là bốn đỉnh của một hình vuông nhận tâm I của hình vuông cơ sở ở giữa
làm tâm, nên dẫn đến tọa độ điểm I là trung bình cộng tọa độ các điểm A,
5
B, C, D do đó số được đặt trong hình vuông tâm I là số hữu tỉ và là trung
bình cộng của bốn số hữu tỉ được đặt trong các hình vuông tâm A,B,C,D.

Bài 19: Tìm số tự nhiên n để có thể viết các số 1, 2,..., n2 vào một bảng n  n sao cho tổng 4 số trên các

hình con có dạng là một số chẵn.

HD:
Xét một hình chữ thập bất kì. Từ giả thiết của bài toán ta suy ra
a  a  b  c  e    b  c  d  e   a  d đây là một số chẵn suy ra a, d cùng tính
chẵn lẻ.
b e d Một cách tương tự ta suy ra a, b, c, d có cùng tính chẵn lẻ.
c Nhưng nếu a, b, c cùng tính chẵn lẻ và  a  b  c  e  là một số chẵn vậy e cũng
có cùng tính chẵn lẻ với a, b, c, d hay 5 số trên một hình chữ thập luôn có cùng tính
chẵn lẻ.
Mặt khác trên một bảng bất kì ngoại trừ 4 ô góc thì các ô khác đều thuộc
giao của 2 hình chữ thập. Suy ra tất cả các ô trừ 4 ô góc đều có cùng tính chẵn lẻ với
nhau.
Ta sẽ chứng minh n  3 chính là số thoả mãn bài toán.
Thật vậy nếu xét trường hợp n  4 thì từ 1 đến 4 2 có 8 số chẵn và 8 số lẻ
2 1 4 không thể điền lượng số chẵn lẻ này chỉ vào 4 ô góc để đảm bảo các ô còn lại cùng
tính chẵn lẻ.
3 9 5
Trong trường hợp n  3 thì từ1 đến 32 ta có 5 số lẻ, 4 số chẵn. Ta sẽ điền 4
8 7 6 số chẵn vào 4 ô góc và 5 số lẻ vào các ô còn lại. Những cách điền như thế thoả mãn
điều kiện bài toán.

Bài 20: Trên bảng ô vuông 3  3 , người ta đặt một số viên sỏi sao cho mỗi ô vuông có không quá một
viên sỏi. Với mỗi cách đặt ta cho tương ứng với số điểm bằng tổng số: các hàng, các cột, các đường chéo
chứa số lẻ các viên sỏi trên đó. Bảng không có sỏi ứng với 0 điểm.
a) Tồn tại hay không cách đặt sỏi sao cho ô chính giữa bảng không có sỏi và số điểm tương ứng
với cách đặt đó là 8.
b) Chứng minh rằng số cách đặt sỏi với điểm số là một số chẵn bằng số cách đặt sỏi với điểm số
là một số lẻ.
HD:
a) Giả sử ô chính giữa không có sỏi và điểm số của cách đặt là 8. Như vậy 3 hàng, 3 cột và hai
đường chéo đều có một số lẻ viên sỏi. Gọi a,b,c,d là số sỏi trong các ô như hình vẽ, a,b,c,d  0,1 . Khi
đó các ô đối xứng với a,b,c,d qua tâm sẽ có số sỏi tương ứng là a ', b ', c ', d ' sao cho
a  a '  b  b '  c  c '  d  d '  1.
a b c a b c
0 d d' 0 d
c' b' a'

Từ đó  a  b  c    a ' b ' c '   3 suy ra một trong hai tổng a  b  c hoặc a ' b ' c ' là một số
chẵn. Khi đó dòng thứ nhất hoặc dòng thứ ba có tổng số sỏi là một số chẵn, mâu thuẫn với giả thiết ban
đầu.Vậy không tồn tại cách đặt sỏi thỏa mãn điều kiện bài toán.
b) Ta gọi hai cách đặt sỏi là liên hợp với nhau nếu ô trên cùng bên trái của chúng có số sỏi khác
nhau và các ô còn lại tương ứng có số sỏi như nhau.
a b c a' b c
f e d f e d
g h i g h i
(B) (B')
Như vậy, các cách đặt sỏi chia thành từng cặp đôi một liên hợp với nhau.
Xét hai cách đặt liên hợp với nhau (B) và (B'). Tổng số sỏi ở dòng 1, cột 1 và 1 đường chéo cả hai
bảng đôi một khác nhau về tính chẵn lẻ. Các dòng, cột và đường chéo còn lại của hai bảng có số sỏi như
nhau. Do đó điểm số của (B) và (B') khác nhau 3 đơn vị, suy ra số điểm của (B) và (B') có tính chẵn lẻ
khác nhau.
Vậy hai cách đặt liên hợp với nhau, một cách xếp có điểm số chẵn, cách đặt còn lại có điểm số là
một số lẻ suy ra điều phải chứng minh.
Bài 21: Công viên có một khu đất du lịch 100x100m, chia làm 100 lô mỗi lô 10x10m. Người dân muốn
trồng một hoa vào các ô đất trên để khách tham quan. Loài hoa này có tập tính sinh trưởng rất kì lạ. Nếu ô
nào chưa có hoa mà có ít nhất hai ô cạnh nó (có chung cạnh) đã có hoa thì hôm sau ô đó cũng sẽ mọc đầy
hoa. Hỏi nếu chỉ được trồng một lần thì người dân cần trồng vào ít nhất mấy ngày để sau một thời gian thì
cả khu đất đều đầy hoa ?

HD:
Ta đưa bài toán về mô hình bảng hình vuông 10x10 và mỗi ô có hoa sẽ được tô đen. Mỗi bước biến
đổi tương ứng với việc tô đen một ô chữ được tô và có chung cạnh với ít nhất hai ô được tô đen. Do đó ta
cần tìm số ô đen ban đầu để sau một số hữu hạn bước biến đổi thì ta có thể tô đen cả bảng.
Gọi p là chu vi của tất cả các phần được tô đen ta sẽ chứng minh sau mỗi bước biến đổi thì p
không tăng.
Thật vậy ta có các trường hợp sau

Khi cả bảng được tô thì p /  4x10 = 40. Do đó, để tô đen cả bảng ta cần có p  p / . Mà mỗi ô
vuông có chu vi là 4 nên ta cần tô ít nhất là 10.
Ngược lại ta có thể chỉ ra một cách tô cụ thể là tô mười ô trên đường chéo chính số 1 của hình
vuông 10x10 ô
Ngày hôm sau các ô trên đường chéo số 2 sẽ bị tô đen.
Tiếp tục như vậy thì đến ngày thứ 10 cả khu đất sẽ mọc đầy hoa.
Bài 22: (30/4 2016) Cho một bảng có 100x100 ô vuông, mỗi ô vuông điền một dấu +. Ta thực hiện một
phép biến đối như sau. Đổi dấu toàn bộ một hàng hoặc một cột của bảng (+ thành -, - thành +). Hỏi sau
một số lần thực hiện biến đổi như trên thì bảng có thể có đúng 98 dấu – được không?

HD:
Giả sử sau một số lần đổi dấu ta thu được bảng có 98 dấu –
Gọi xi là số lần đổi dấu ở hàng thứ i, y i là số lần đổi dấu ở cột thứ i
Gọi m là số số lẻ trong các số xi và n là số số lẻ trong các số y i
Ta có m, n  1, 2,...,100
Ô  i; j  là dấu – nếu xi  y j lẻ
Ta có số lượng dấu – trên bảng chính bằng
m. 100  n   n 100  m   98   m  50  n  50   43.57 (*)
 m  50 57
Suy ra  m  50  n  50  57  
 n  50 57
Ta lại có 50  m  50  50, 50  n  50  50  m  50  0  n  50  0 mâu thuẩn (*).
Vậy không thể có đúng 98 dấu – trên bảng.

6. Phương pháp phản chứng.


Bài 23: Hình chữ nhật 9  12 được chia thành các hình vuông đơn vị. Ta tô màu các ô bằng các màu đen
và trắng sao cho hai ô liền kề thì khác màu. Ta tô đỏ tâm của các ô vuông trừ 4 ô vuông góc và 8 ô liền kề
với chúng. Hỏi có thể đánh số các tâm đỏ theo thứ tự c1 , c2 , c3 ,..., c96 sao cho
1. Tất cả các đoạn c1c2 , c2 c3 ,..., c96c1 đều có độ dài là 13.
2. Đường gấp khúc khép kín c1c2 c3 ...c96 c1 là đối xứng tâm.
HD:
Nhận xét:
 Mỗi đoạn ci ci 1 được hình thành từ hình chữ nhật 3  4 hoặc 4  3. Và các đoạn này nối hai ô khác
màu lại với nhau. Ta tạm gọi các đoạn ci ci 1 là những đoạn “cơ sở”.
 Mỗi nhóm các đoạn cơ sở nối 2 ô khác màu luôn gồm một số lẻ đoạn “cơ sở”.
Để dễ xác định và phân biệt các điểm ci với nhau theo một cách tự nhiên ta xếp hình chữ nhật lên mặt
phẳng toạ độ như hình vẽ. Khi đó A  2; 2  , B 11;8  , C  6,5;5  .
8 B

6
5 C
4

2 A

2 4 5 6, 8 9 11
Đường gấp khúc đi qua hai điểm A,B. Hai điểm này đối xứng qua C và chi đường này thành 2 đoạn  1
và  2 . Hai điểm A,B khác màu do đó hai đoạn  1 và  2 có số lẻ đoạn “cơ sở”. Mặt khác vì có tổng cộng
là 96 đoạn “cơ sở”, một nửa là 48 đây là một số chẵn thế nên  1 và  2 phải có số lượng đoạn “cơ sở”
khác nhau. Vậy hai phần này không thể đối xứng qua C. Suy ra mỗi phần  1 và  2 phải đối xứng qua C.
Mỗi phần có số lẻ đoạn “cơ sở” suy ra mỗi phần phải chứa một đoạn “cơ sở” đối xứng qua C (nhận C
làm ta đối xứng).
Chỉ có hai đoạn như thế là
Đoạn nối 2 điểm  5;4  và  8;6  .
Đoạn nối 2 điểm  5;6  và  8; 4  .
Suy ra hai đoạn này phải thuộc hai đường gấp khúc  1 và  2 (mỗi đoạn thuộc một đường).
Điểm  2;2 chỉ có thể nối với hai điểm  4;5 và  5;4  nên 3 điểm này phải nối trực tiếp với nhau.
Tương tự  2;8  chỉ có thể nối với hai điểm  5;6  và  4;5  .
11; 2  chỉ có thể nối với hai điểm  8; 4  và  9;5 .
11;8 chỉ có thể nối với hai điểm 8;6  và  9;5 .
Các điểm này nối với nhau và lập thành đường gấp khúc đóng
 5; 4  
  2; 2  
  4;5  
  5;6  
  8; 4  
 11; 2  
  9;5  
 11;8  
  8;6  
  5; 4 
Điều này mâu thuẩn vì hai đoạn “cơ sở”  5;6  
  8; 4  và  8;6  
  5; 4  không thể chung trên
một đoạn gấp khúc  1 hoặc  2 .
Vậy không thể tồn tại cách đánh số thoả bài toán.

7. Nguyên lí cực hạn.


Bài 24:
a) Trên một mạng lưới ô vuông vô hạn trên một mặt phẳng, đặt vào mỗi ô vuông một số tự nhiên
sao cho số trong mỗi ô bằng trung bình cộng của bốn số trong các ô vuông có cạnh kề với ô đó. Chứng
minh tất cả các số bằng nhau.

b) Nếu số trong mỗi ô bằng trung bình cộng của bốn số trong các ô vuông bốn góc. Ta có thể đặt
được tối đa bao nhiêu giá trị của các số?
HD:
a) Tồn tại ô vuông với số được điền bé nhất, 4 ô kề cạnh cũng được đặt cùng số bé nhất đó. Với ô
vuông khác bất kì ta luôn tìm được một dãy các ô vuông kề cạnh bắt đầu từ ô vuông đó tới ô vuông với
số bé nhất, do vậy cũng được đặt số bé nhất. Vậy tất cả các số đều bằng nhau.

b) Tô màu đen trắng theo kiểu bàn cờ, ta chứng minh được tất cả các số trong các ô cùng màu phải
bằng nhau. Tối đa đặt được 2 giá trị của các số vào mạng lưới ô vuông.

a b
e
d c

Bài 25: Đặt các số nguyên 1, 2,3,...n 2 vào một bàn cờ n  n  n  2  một cách ngẫu nhiên, mỗi số đúng
một lần, mỗi ô một số. Chứng minh rằng tồn tại hai ô vuông kề nhau (chung cạnh hoặc chung đỉnh) mà có
giá trị khác nhau ít nhất n  1.

HD:
Với 2 ô bất kì ta luôn tìm được một dãy không quá n các ô vuông kề nhau nối chúng (tính cả 2 ô đó).
(1)
Giả sử ngược lại, hai ô vuông kề nhau bất kì có giá trị khác nhau nhiều nhất n. (2)
2
Xét hai ô vuông chứa số 1 và n . Theo nhận xét (1) ta có một dãy không quá n ô vuông kề nhau nối
n  n  1 .
chúng. Nhưng kết hợp với điều kiện (2), giá trị hai ô này khác nhau không quá Như vậy ta thu
2
n  1  n  n  1  n  1.
được Không thể xảy ra vì n  2. 
Bình luận: Việc chọn ra hai phần tử cực biên là điều khá tự nhiên nếu mục tiêu của chúng ta là chỉ ra
điều vô lí.
Việc nhìn ra đại lượng để áp dụng Nguyên lý cực hạn đôi khi là không đơn giản, ta phải chọn một
“hàm đo” thích hợp nhằm áp dụng được nguyên lý cho tập hợp số để thu được lời giải.
Bài 26: Một bảng hình vuông cỡ 2014  2014 được viết các số. Tổng của các số trong mỗi hình chữ thập
(hình tạo bởi một hàng và một cột) không nhỏ hơn 4027. Hãy tìm cách điền số lên bảng để được tổng S
tất cả các số là nhỏ nhất.

HD:
Xét hàng i với tổng các số trong hàng là nhỏ nhất so với các hàng khác.
S
Kí hiệu tổng ấy là m, suy ra m 
2014
Khi đó tổng của tất cả các hình chữ thập ở hàng i lớn hơn hoặc bằng 2014.4027  a
Suy ra S   2014  1 m  a  S  2013m  a
Ta có
2013
2014.4027  S  2013m  S  S
2014
4027
 2014.4027  S
2014
 S  20142.
Dấu bằng chỉ xảy ra khi tất cả các ô đều bằng 1.

8. Nguyên lí Dirichlet.
Bài 27: Có m  n người lính đứng thành m hàng, n cột. chứng minh rằng trong 37 người đó luôn tồn tại
10 người đứng không liền kề nhau.

HD:

Định hướng: những bài toán thiên về chứng minh tồn tại một số lượng người thoả mãn tính chất
gợi cho ta dấu hiệu của việc sử dụng nguyên lí Dirichlet.
37 người có thể thay cho việc ta đã có số thỏ trong nguyên lí.
Ta cần tìm ra số chuồng để sau cho luôn có chuồng có ít nhất 10 con thỏ trong đó (10 người lính
không liền kề).
Không mấy khó khăn ta thấy số chuồng cần tìm là 4. Ta cần “xây dựng chuồng” hay chỉ ra 4 tập
hợp thoả mãn tính chất “hai người lính luôn không liền kề”.
Từ những định hướng như trên ta có thể tìm ra lời giải như sau:
Đánh số các vị trí như hình vẽ. Ô (lẻ,lẻ) đánh số 1, ô (lẻ, chẵn) đánh số 2, ô(chẵn,chẵn) đánh số 3, ô
(chẵn,lẻ) đánh số 4.
Theo nguyên lí Dirichlet thì trong 37 người lính luôn tồn tại ít nhất 10 người lính mang cùng 1 số.
10 người lính này luôn đứng không liền kề nhau. 
Bài 28: Trong bảng 3x7 (3 hàng,7 cột) người ta tô các ô vuông bởi hai màu đen và trắng, mỗi ô một màu.
Chứng minh rằng với bất kì cách tô nào ta luôn tìm được một hình chữ nhật có các cạnh nằm trên các
đường lưới mà 4 đỉnh nằm ở bốn ô cùng màu.

Phân tích đề bài:


Xét một trường hợp thỏa như hình vẽ và nghĩ đến giả thiết nếu có 2 cột hoặc 2 hàng cùng trạng thái
thì luôn có hình chữ nhật thỏa mãn, nhưng khả năng hai hàng cùng trạng thái là khó xảy ra vì quá ít hàng
(3 hàng). Hai cột cùng trạng thái xem như 2 con thỏ cùng một chuồng.
Thế thì xét về trạng thái các cột ta thấy có tới 23  8 trạng thái khác nhau của 1 cột (8 chuồng)
nhưng giả thiết thì chỉ có 7 cột (7 con thỏ).
Số thỏ ít hơn số chuồng ta nghĩ đến việc chia trường hợp để giảm bớt chuồng. Tất nhiên trong các
chuồng thì ta lưu ý đến 2 trường hợp đặc biệt là toàn trắng hoặc toàn đen.
HD:
TH1: Nếu trong bảng tồn tại một cột được tô bởi duy nhất 1 màu chẳng hạn là màu đen.
 Nếu tồn tại một cột có ít nhất 2 ô màu đen thì bài toán được giải quyết
 Nếu tất cả 6 cột còn lại chỉ có nhiều nhất một ô đen thì các ô đó chỉ có thể ở một trong các trạng
thái tính từ trên xuống là TTĐ,TĐT,ĐTT,TTT thế thì theo Dirichlet luôn có hai cột mang cùng một trạng
thái lúc đó ta luôn có hình chữ nhật cần tìm từ hai cột này.

TH2: Nếu không có cột nào được tô một màu thì ta có 7 cột với 6 trạng thái ô như sau
TTĐ,TĐT,ĐTT,ĐĐT,ĐTĐ,TĐĐ theo Dirichlet sẽ có hai cột có cùng trạng thái và xác định được hình
chữ nhật cần tìm.
Mở rộng bài toán:
Hướng 1: Thay đổi bảng thành 4x7 (4 hàng,7 cột) cũng là hướng khai thác thông minh. Hoặc thay
đổi thành bảng vô hạn.
Hướng 2: Lưới điểm tọa độ nguyên.
VD: Mỗi điểm trong mặt phẳng được tô một trong hai màu xanh hoặc đỏ chứng minh rằng luôn tồn
tại một hình chữ nhật có tọa độ các đỉnh là các số nguyên.

9. Một số dạng khác


Bài 29: Trên bàn cờ 10x10 người ta viết các số từ 1 đến 100. Mỗi hàng chọn ra số lớn thứ ba. Chứng
minh rằng tồn tại một hàng có tổng các số trong hàng đó nhỏ hơn tổng các số lớn thứ ba đã được chọn.

HD: Gọi các số lớn thứ ba là a1  a2  ...  a10 . Khi đó số phần tử lớn hơn a1 nhiều nhất là 20 (nhiều nhất
là 2 phần tử ở mỗi hàng).
Suy ra a1  80 . Tương tự a2  78 mặt khác
a9  a10  1, a8  a10  2,..., a3  a10  7.
Kết hợp điều kiện trên ta có
a1  a2  ...  a10  80  78   a10  7   ...   a10  1  a10  8a10  186
Xét hàng chứa a10 tổng các số hàng này là
S 10   100  99  a10   a10  1  ...   a10  7   8a10  171  8a10  186  a1  a2  ...  a10
Ta có điều phải chứng minh.
Bài 30: Cho dãy số 1,2,3,…,100. Chứng minh rằng với mỗi cách xoá đi 10 số bất kì thì trong 90 số còn
lại luôn tồn tại 10 số lập thành một cấp số cộng.

HD:
Ta sẽ viết các số lên bảng 10 10 theo quy luật như sau
1 2 3 4 5 6 7 8 9 10
11 12 13 20
21 22 23 30

91 92 93 100
Khi đó 10 số trên mỗi một hàng, 10 số trên mỗi một cột hay 10 số liên tiếp từ hàng i đến hàng i  1
đều là một là những cấp số cộng.
Cách xoá để phá vỡ những cấp số cộng trên là xoá mỗi hàng một số xoá mỗi cột một số và nếu ở
hàng i ta xoá phần tử ở cột j thì hàng i  1 ta phải xoá đi phần tử ở cột k  j. Các ô xoá phải không cùng
hàng và cũng phải không cùng cột.
Từ ý tưởng trên ta có cách xoá như hình vẽ.
Nhưng với cách xoá trên thì vẫn còn 10 ô nằm trên đường chéo còn là gồm
1,12,23,34,45,56,67,78,89,100 là một cấp số cộng.
Điều phải chứng minh.
PHƯƠNG PHÁP ÁNH XẠ TRONG CÁC BÀI TOÁN PHƯƠNG TRÌNH HÀM
Giáo viên Phạm Thị Thùy
Trường THPT chuyên Lý Tự Trọng Cần Thơ
I. CƠ SỞ LÝ THUYẾT.
I.1. Ánh xạ
1.1. Định nghĩa. Một ánh xạ f từ tập X đến tập Y là một quy tắc đặt tương ứng mỗi phần tử x
của X với một (và chỉ một) phần tử của Y . Phần tử này được gọi là ảnh của x qua ánh xạ f và
được kí hiệu là f  x  .

i. Tập X được gọi là tập xác định của f . Tập hợp Y được gọi là tập giá trị của f .

ii. Ánh xạ f từ X đến Y được kí hiệu là

f : X Y

x  y  f  x

iii. Khi X và Y là các tập số thực, ánh xạ f được gọi là một hàm số xác định trên X

(iv) Cho a  X , y  Y . Nếu f  a   y thì ta nói y là ảnh của a và a là nghịch ảnh của y qua ánh
xạ f .

 
(v) Tập hợp Y  y  Y x  X , y  f  x  gọi là tập ảnh của f . Nói cách khác, tập ảnh f  X  là
tập hợp tất cả các phẩn tử của Y mà có nghịch ảnh.
I. 2. Đơn ánh, toàn ánh, song ánh
I. 2.1. Định nghĩa. Ánh xạ f : X  Y được gọi là đơn ánh nếu với a  X , b  X mà a  b thì
f  a   f  b  , tức là hai phần tử phân biệt sẽ có hai ảnh phân biệt.

Từ định nghĩa ta suy ra ánh xạ f là đơn ánh khi và chỉ khi với a  X , b  X mà f  a   f  b  , ta
phải có a  b .
I.2.2. Định nghĩa. Ánh xạ f : X  Y được gọi là toàn ánh nếu với mỗi phần tử y  Y đều tồn tại
một phần tử x  X sao cho y  f  x  . Như vậy f là toàn ánh nếu và chỉ nếu Y  f  X  .

I.2.3. Định nghĩa. Ánh xạ f : X  Y được gọi là song ánh nếu nó vừa là đơn ánh vừa là toàn ánh.
Như vậy ánh xạ f : X  Y là song ánh nếu và chỉ nếu với mỗi y  Y , tồn tại và duy nhất một phần
tử x  X để y  f  x  .

I. 3. Ánh xạ ngược của một song ánh

I.3.1. Định nghĩa. Ánh xạ ngược của f , được kí hiệu bởi f 1 , là ánh xạ từ Y đến X gán cho mỗi
phần tử y  Y phần tử duy nhất x  X sao cho y  f  x  . Như vậy
f 1  x   y  f  x   y

I.3.2. Chú ý. Nếu f không phải là song ánh thì ta không thể định nghĩa được ánh xạ ngược

của f . Do đó chỉ nói đến ánh xạ ngược khi f là song ánh.

I. 4. Ánh xạ hợp

I. 4.1. Định nghĩa. Nếu g : A  B và f : B  C và g  A   B thì ánh xạ hợp f  g : A  C được


xác định bởi

 f  g  a   f  g  a   .
Kí hiệu p n  p  p  ...  p .

n
II. VẬN DỤNG ĐƠN ÁNH, TOÀN ÁNH VÀ SONG ÁNH TRONG CÁC BÀI TOÁN VỀ
PHƯƠNG TRÌNH HÀM.
1 . SỬ DỤNG TÍNH CHẤT ĐƠN ÁNH.
Nhận xét:

 Nếu một vế có chứa f  x  và vế còn lại có chứa biến x bên ngoài thì thông thường f là đơn
ánh.
 Nếu một hàm số mà đơn ánh chúng ta rất hay dùng thủ thuật tác động f vào cả hai vế hoặc tạo
ra f   x    f   x      x     x  .

 Chứng tỏ f là đơn ánh ta giả sử x1 , x2  X thỏa f  x1   f  x2  . Khi đó ta cần chứng minh:
x1  x2 ( xét f  x1   f  x2  với x là biến nằm bên ngoài sự tác động của f ).

Lời giải của một bài toán phương trình hàm thường được bắt đầu bằng mệnh đề “ Giả sử tồn tại
hàm số f  x  thỏa mãn các yếu cầu của bài ra” . Khi tìm được các biểu thức của hàm số nghiệm,
ta phải kiểm tra vào phương trình đã cho rồi mới được kết luận nghiệm.
Bài toán 1. Tìm tất cả các hàm f :    thỏa

f  x  f  x   2 f  y    x  f  x   y  f  y  , (*)

với mọi x, y   .

Lời giải.

* Giả sử tồn tại hàm số f  x  thỏa mãn yêu cầu bài ra.

* Ta nhận thấy vế phải của phương trình có sự đối xứng giữa hai biến x, y nên hoán đổi vai trò của
x, y cho nhau rồi so sánh vế trái ta được

f  x  f  x   2 f  y    f  y  f  y   2 f  x   (1)

* Ta cần chứng minh f là đơn ánh

Thật vậy, Giả sử f  y1   f  y 2 

Cố định x hay nói cách khác thay x bởi 1 , từ * ta có

f 1  f 1  2 f  y1    f 1  f 1  2 f  y2  

là đơn ánh.

Vì vậy, từ 1 ta được x  f  x   2 f  y   y  f  y   2 f  x  hay f  x   x  f  y   y  c  c   


Khi đó, ta có f  x   x  c c   .
* Tìm c

Thay f  x   x  c  c    vào * ta được


f  x  f  x   2 f  y    x  f  x   y  f  y   f  2 x  2 y  3c   2 x  2 y  2c

 2 x  2 y  4c  2 x  2 y  2c  4c  2c  c  0 .

Vậy f  x   x. thử lại ta được f  x   x là hàm cần tìm.

Nhận xét:

Mấu chốt trong bài toán này chính là f đơn ánh và cơ sở để ta nghỉ đến việc chứng minh f
là đơn ánh chính là sự xuất hiện của đẳng thức f  x  f  x   2 f  y    f  y  f  y   2 f  x   và cơ
sở để nghĩ đến tính chất đơn ánh chính là của f đó là sự xuất hiện của biến x, y bên ngoài.

Ta tiếp tục xét bài toán tương tự sau.

Bài toán 2. (Đề thi chính thức Olympic 30/4/2006) Hãy tìm tất cả các hàm số f :    thỏa
mãn

f  x  f  y   xf  y    x  xy  y , x, y   . 1

Lời giải

* Giả sử tồn tại hàm số f  x  thỏa mãn yêu cầu bài ra.

* Ta nhận thấy vế phải của phương trình có sự đối xứng giữa hai biến x, y nên hoán đổi vai trò của
x, y cho nhau rồi so sánh vế trái ta được

f  x  f  y   xf  y    f  y  f  x   yf  x    *
* Ta cần chứng minh f là đơn ánh.

Thật vậy, giả sử f  y1   f  y2  . Trong 1 ta cố định x hay nói cách khác là thay x bởi 1 ta được:

f 1  f  y1   f  y1    f 1  f  y2   f  y2    1  y1  y1  1  y2  y2  y1  y 2

 f là đơn ánh nên từ * ta được: x  f  y   xf  y   y  f  x   yf  x   3

Thay x bởi 0 vào  3 ta được f  y   y  f  0   yf  0 

Đặt f  0   a . Theo trên ta có f  y    a  1 y  a, y    4 


* Tìm a

Thay  4  vào 1 ta được  a  1  x  f  y   xf  y    a  x  xy  y, x, y  .

 
  a  1 x   a  1 y  a  x  a  1 y  a   a  x  xy  y, x, y    5 

a  0
Thay x bởi 0 ; y bởi 0 vào  5  ta được  a  1 a  a  0  
 a  2

Khi a  0 ta được f  x   x

Khi a  2 ta được f  x    x  2

Thử lại ta thấy thỏa mãn. Vậy có hai hàm số thỏa mãn đề bài là: f  x   x, x   và
f  x    x  2, x   .

Nhận xét:

Trong bài toán trên tính chất đối xứng của hai biến giữa hai biến x, y tỏ ra khá quan trọng
trong việc tìm ra lời giải của bài toán, và sự kết hợp giữa tính chất đối xứng của hai biến x, y với
tính chất đơn ánh của f làm cho bài toán trở nên dễ dàng hơn. Vần đề đặt ra liệu những bài toán
không có tính chất đối xứng giữa hai biến x, y ta có thể sử dụng một ph ép thế nào đó để đưa về đối
xứng của hai biến x, y hay không bằng cách xét tiếp bài toán sau:

Bài toán 3. Tìm tất cả các hàm số f :  0;     0;   thỏa mãn:

y
f  x  f  y   , x, y   0;   1 .
xy  1

Lời giải

* Giả sử tồn tại hàm số f  x  thỏa yêu cầu bài ra.

y ya
* Với a  0, xét  a  y  axy  a  x   y  0
xy  1 ay

Khi đó, từ 1 ta được

 ya   xa 
f  f  y  a  f   f  x
 ay   ax 

* Ta cần chứng minh f là đơn ánh, thật vậy

Giả sử f  x   f  y  , khi đó ta có:


x y
Vì f  x   f  y   x  f  x   x  f  y   f  x  f  x    f  x  f  y    2

x  1 xy  1

 x 2 y  x  x 2 y  y  x  y  f là đơn ánh.

ya xa
Khi đó từ  2  ta suy ra:  f  y   f  x
ay ax

1 1 1 1 1 1 1
   f  y     f  x   f  y    f  x    c  f  y    c, y   a;  
a y a x y x y

1  1  y 1 y
Thay f  y    c vào 1 ta được: f  x   c    c 
y  y  xy  1 1 xy  1
x c
y

y y
 c   3
xy  1  cy xy  1

Thay x bởi 1 và y bởi 1 vào  3 ta được:


c  0
1 1 1  2c  c 2 1
c     2  4c  2c 2  2  c  2c 2  3c  0  
2c 2 2c 2 c   3
 2

1 1 3
 f  x  hoặc f  x   
x x 2

1 3
Vì f  x    0;   , x   0;   nên ta loại f  x    .
x 2

1
Thử lại, ta thấy f  x   thỏa yêu cầu bài toán.
x

Nhận xét:

Sẽ là sai lầm nếu ta “ nếu ta lập luận ngắn gọn” như sau: Xét

y y 1
 1  y  xy  1  x  (*)
xy  1 y

 y 1   x 1 
Do đó từ 1 suy ra : f   f  y  1  f   f  x   , x  0, y  0  2 
 y   x 

Mà f là đơn ánh nên từ  2  dẫn đến:


y 1 x 1 1 1
 f  y   f  x  , x, y   0;    f  x    f  y   , x, y   0;  
y x x y
1
 f  x   c, x   0;   (c là hằng số).
x

Bài toán “ sai lầm” ở đây chỉ được xét khi x, y  1;   (do điều kiện (*))

Ta xét thêm một bài toán đưa về đối xứng biến x, y của một vế như sau:

Bài toán 4. Tìm tất cả các hàm số f :    thỏa mãn:

f  2 f  x   y   2 x  y  6, x, y  . 1 .
Lời giải

* Giả sử tồn tại hàm số f  x  thỏa yêu cầu bài ra.

* Trong 1 , ta thay y bởi 2x ta được: f  2 f  x   2 x   6, x    2  .

Khi đó, từ  2 ta có: f  2 f  x   2 x   f  2 f  y   2 y  , x, y    3

* Ta chứng minh f là đơn ánh.

Thật vậy, Khi f  x1   f  x2  , từ 1 , ta có:


2 f  x1   y  2 f  x2   y  f  2 f  x1   y   f  2 f  x2   y   2 x1  y  6  2 x1  y  6  x1  x2

 f là đơn ánh. Do vậy, từ  3 ta được: 2 f  x   2 x  2 f  y   2 y  f  x   x  f  x   x  c

Khi đó: f  x    x  c, x    4

Thay  4  vào 1 ta được: f  2 x  2c  y   2 x  y  6, x, y  

 2 x  2c  y  c  2 x  y  6  c  6  f  x    x  6 .

Thử lại, ta thấy f  x    x  6 thỏa yêu cầu bài toán.

Nhận xét:

Ta có bài toán tổng quát, với cách giải tương tự như sau: Tìm hàm f thỏa mãn
f  af  x   by   cx  dy  e, x, y  , trong đó a, b, c, d , e là các số thực khác 0.

Như vậy qua các bài toán trên ta thấy quá trính nhận biết f là đơn ánh khá quan trọng trong
việc tìm ra đáp án của bài toán khi xuất hiện đẳng sự tác động f vào hai vế nhằm xuất hiện
f   x    f   x      x     x  , vấn đề quan trọng ở đây là cần chứng minh f là đơn ánh;
trong một số bài toán tuy có sự xuất hiện của f   x    f   x   nhưng không chứng minh được
f là đơn ánh, khi đó ta có thể biện luận để tìm ra đáp án của bài toán. Bài toán sau là một trường
hợp như trên.

Bài toán 5. Tìm tất cả các hàm số f :    thỏa mãn:

f  1  x  f  y    yf 1  f  x   , x, y   1 .
Lời giải

* Giả sử tồn tại hàm số f  x  thỏa yêu cầu bài ra.

* Trong 1 , ta thay y bởi 1 ta được f  1  x  f 1   f 1  f  x   , x    2

* Từ  2  , ta thấy rằng nếu f là đơn ánh thì bài toán sẽ trở nên dễ dàng, thật vậy

Trường hợp 1: Giả sử f là đơn ánh.

Từ  2  , ta được: 1  x  f 1  1  f  x   f  x   f 1 x  f 1  1, x   .

Khi đó, ta có f  x  có dạng như sau: f  x   ax  b, x   .

Thay vào 1 ta được: f 1  x  ay  b    yf 1  ax  b 

 a 1  x  ay  b   b  y  a 1  ax  b   b  , x, y   ( a, b là hằng số)  3

Thay x bởi 0 , y bởi 0 và  3 ta được: ab  b  0

Thay x bởi 1, y bởi 0 và  3 ta được: 2ab  b  0

ab  b  0
Khi đó ta có hệ phương trình:  b0
2ab  b  0

Thay b  0 vào  3 ta được: a 1  x  ay  y  a 1  ax    a 2 y  a 2 xy  ay  axy

a  0

 ay  axy  y  xy  4

Từ  4  , thay x  1, y  1 ta được: 2a  2  a  1

Thử lại, ta thấy f  x   0 và f  x   x thảo yêu cầu bài toán.

* Trường hợp 2: f không phải là đơn ánh, khi đó y1  y2 sao cho f  y1   f  y2 
Khi đó vì f  y1   f  y2   1  x  f  y1   1  x  f  y2   f  1  x  f  y1    f  1  x  f  y2  

 y 1 f 1  f  x    y 2 f 1  f  x   vì  y 1  y 2  f 1  f  x    0, x  .  5

Thay  5  vào 1 ta được: f  1  x  f  y    0, x, y    6

Giả sử y0  , f  y0   0 , Từ  6  ta được f  x   0, x   (mâu thuẫn với điều giả sử).

Vậy từ  6  ta có f  x   0, x   .

Vậy các hàm số thỏa yêu cầu bài toán là : f  x   0 và f  x   x

Nhận xét:

Quan hệ đơn ánh trong bài này chính là mẫu chốt tìm ra lời giải của bài toán.

Bài toán 6 (IMO 1988, Shortlist). Tìm tất cả các hàm f : *  * thỏa mãn đẳng thức:

f  f  m   f  n    m  n , (1)

với mọi m, n  * .

Lời giải.

* Giả sử tồn tại hàm số f  x  thỏa yêu cầu bài ra.

*Thay m  n vào đẳng thức trên ta được f  2 f  n    2n (1),

Xét : f  n1   f  n2   f  f  n1    f  f  n2   , từ (1)  2n1  2n2  n1  n2 hay suy ra f là đơn


ánh.

Ta có 2n  n  1  n  1  n  n  f  f  n  1  f  n  1   f  f  n   f  n   , và do f là đơn ánh
nên f  n  1  f  n  1  2 f  n   f  n  1  f  n   f  n   f  n  1 , n  2 (2).

Từ đẳng thức (2) ta có:

f  n   f  n  1  f  n  1  f  n  2   ...  f  2   f 1  a ,

suy ra

f  n   a  f  n  1  a  a  f  n  2   2a  f  n  2   2a  a  f  n  3  3a  f  n  3  ... 

= f 1   n  1 a  an  b ; trong đó b  f 1  a .

Thay f  n   an  b vào phương trình (*) ta được:

f  am  b  an  b   m  n  f  a  m  n   2b   m  n  a  a  m  n   2b   b  m  n
 a 2  m  n   2ab  b  m  n

Đồng nhất hệ thức ta được:


a  1
  a  1
a   1   a  1 
 
a 2  1
  

a   1   b  0   f  n   n

2ab  b  0
1    a  1  f  n   n l 
a   b  0  
 2 
 b  0
 b  0

vì f  n   N * .

Vậy f  n   n, n  * .

Nhận xét:
Ta còn có kết quả mạnh hơn

f  m   f  n   f  p   f  q  , m  n  p  q

Từ đó, ta rút ra được phương trình:

f  n  1  f  n  1  f  n   f  n   f  n  1  f  n   f  n   f  n  1 , n  *

Đến đây, ngoài cách làm như trên, ta có thể tìm ra công thức tổng quát của f  n  bằng cách sử
dụng phương trình sai phân.
Ngoài cách giải trên. Ta có thể tham khảo cách giải dưới đây.
Lời giải khác.

Rõ ràng phương trình đồng nhất f  x   x là nghiệm của bài toán. Ta đi chỉ ra điều này bằng cách
chỉ ra f 1  1 sau đó dùng quy nạp.

Giả sử f 1  t  1 . Đặt: s  f  t  1  0 . 1

Ta thấy rằng nếu f  m   n thì f  2n   f  n  n   f  f  m   f  m    m  m  2m . Vì vậy ta có


f  2t   2 và (do 1 ).

Do đó: 2s  2t  f  f  2s   f  2t    f  2t   2 suy ra t  1 mâu thuẫn.

Điều này chứng tỏ f 1  0 hoặc f 1  1

Nếu f 1  0 khi đó

f  0   f  0  0   f  f 1  f 1   2 , f  4   f  2  2   f  f  0   f  0    0  f 1  4  1 (vì


f là đơn ánh) mâu thuẫn. Nên f 1  1 .
Bây giờ bằng quy nạp, nếu f  n   n thì f  n  1  f  f  n   f 1   n  1 . Dẫn đến kết luận
f  n   n, n  .

Một cách tổng quát: Với f :    và k  N * , ta có:

 k  k
f   f  ni     ni , ni    f  n   n, n   .
 i 1  i 1

Không khó có thể nhận thấy cách giải bài toán tổng quát này dựa vào bài toán ban đầu, tức
với k  2 . Thật vậy, ta giữ nguyên mọi biến nk với k  3 , chỉ để hai biến n1 , n2 thay đổi sao

cho tổng không đổi, sử dụng tính đơn ánh ta sẽ suy ra kết quả quen thuộc
f  m   f  n   f  p   f  q  , m  n  p  q

Từ đó giải quyết được bài toán tổng quát trên. Dưới đây là một số bài toán tương tự.
Bài toán 7. Cho k  * . Tìm tất cả các hàm f : *  * thỏa mãn đẳng thức:

f  f  n   m   n  f  m  k  , (1)

với mọi m, n  * .

Lời giải.

* Giả sử tồn tại hàm số f  x  thỏa yêu cầu bài ra.

* Thay m bởi f 1 vào 1 ta được: f  f  n   f 1   n  f  f 1  k 

 f  f  n   f 1   n  1  f  k  k   f  f  n   f 1   f  f  n  1  k   2
* Ta chứng minh f là đơn ánh, thật vậy:

Xét
f  n1   f  n2   m  f  n1   m  f  n2   f  m  f  n1    f  m  f  n2    n1  f  m  k   n2  f  m  k 

 n1  n2  f là đơn ánh nên từ  2  ta được:


f  n   f 1  f  n  1  k  f  n  1  f  n   f 1  k .

Đặt: f 1  k  a khi đó ta có:


f  n  1  f  n   f  n  1  f  n   a  f  n  1  a  f  n   a  a  f  n  1  a  a  a  f  n  2 
 ...  na  f 1
 na  a  k   n  1 a  k
Hay f  n   an  k  3

Thay  3 vào 1 ta được: f  na  k  m   n   m  k  a  k , m, n  N *  4

Thay m  1, n  1 vào  4  ta được:


f  a  k  1  1  k  a  1  k   a  k  1 a  k  1  k  a  1  k  a 2  1  a  1 hoặc a  1

Khi đó : f  n   n  k hoặc f  n   n  k . Vì f  n   N * , n  N * nên f  n   n  k (loại).

Thử lại, ta thấy f  n   n  k thỏa yêu cầu bài toán.

Nhận xét:

Tính chất đơn ánh của f trong bài toán này khá rõ ràng vì xuất hiện biến n nằm ngoài tác
động của hàm f .

Khi chứng minh f là đơn ánh ta nên dựa vào biến nằm ngoài n để xét f  n1   f  n2 
 n1  n2 kết hợp với việc cố định biến m còn lại.

Bài toán 8. Tìm tất cả các hàm f :    thỏa mãn đẳng thức:

f  x  f  y    f  x  xy   yf 1  x  , x, y   , (1)

Lời giải.

* Giả sử tồn tại hàm số f  x  thỏa yêu cầu bài ra.

* Kí hiệu P  u, v  chỉ việc thay x bởi u, y bởi v .

Thực hiện P  0, y  vào 1 ta được f  f  y    f  0   yf 1  2

Thực hiện P  0, 0  vào 1 ta được: f  f  0    f  0 

Trường hợp 1: f 1  0 . Xét f  y1   f  y2   f  f  y1    f  f  y2  

f  0   y1 f 1  f  0   y2 f 1  y1  y2  f là đơn ánh nên từ  3 ta được : f  0   0  4

Khi đó: từ  2  suy ra f  f  y    yf 1


 f  x 
Thực hiện P  ; x  vào 1 ta được:
 2 
 f  x   f  x   f  x   f  x 
f  f  x   f   f  x    xf 1    xf 1  0
 x   x   x   x 
 f  x 
 f 1    0, x    5 
 x 

 f  x  f  x
Từ  4  ,  5 suy ra: f 1    f  0  , x    1   0 (vì f là đơn ánh)
 x  x

 f  x   x, x  0 , kết hợp với f  0   0 suy ra f  x   x, x   thử lại thỏa yêu cầu bài
toán.

Trường hợp 2: f 1  0 , khi đó từ  2  ta có: f  f  y    f  0  , y    6

Thực hiện P 1; f  0   vào 1 ta được: f 1  f  f  0     f 1  f  0    f 2  0  7

Thay  6  vào  7  ta được: f 1  f  0    f 1  f  0    f 2  0   f  0   0

Thực hiện P 1; x  1 vào 1 ta được:

f 1  f  x  1   f  x  1   x  1 f  0   f  x  1  0, x  

 f  x   0, x   . Thử lại ta thấy thỏa mãn.

Nhận xét:

Kỹ thuật “ thế triệt tiêu” như trên sử dụng như sau, chẳng hạn ta cần triệt tiêu
f  y
f  x  f  y   và f  x  xy  ở hai vế ta xét x  f  y   x  xy  x  . Vì vậy thực hiện
y
 f  x 
P ; x  sẽ thu được kết quả như mong muốn.
 x 

Trong lời giải bài này xuất hiện phép thế P 1; x  1 để nhằm xuất hiện
f 1  f  x  1   0 để đưa ra kết quả của bài toán.

Bên cạnh những bài toán chứng minh f là đơn ánh một cách trực tiếp ta cũng có thể
chứng minh f là đơn ánh bằng cách sử dụng phương pháp phản chứng, ta có bài toán sau:

Bài toán 9: Tìm tất cả các hàm f :  0;     0;   thỏa mãn đẳng thức:
x 2  f  x   f  y     x  y  f  yf  x   , x, y  0. (1)

Lời giải.

* Giả sử tồn tại hàm số f  x  thỏa yêu cầu bài ra.

* Thay y bởi x vào 1 ta được: x 2  2 f  x    2 xf  xf  x    xf  x   f  xf  x    2

Thay x bởi 1 vào  2  ta được: f 1  f  f 1   3


* Ta chứng minh f là đơn ánh.

Giả sử tồn tại hai số dương x1 , x2 sao cho x1  x2 và f  x1   f  x2   a  0

 x12  a  f  y     x1  y  f  ya 
Thay x bởi x1 ; x bởi x2 trong 1 ta được:  2
 x2  a  f  y     x2  y  f  ya 

Suy ra x1 và x2 là nghiệm của phương trình: tx 2  ux  v  0 ( t , u, v dương) dẫn đến


v
x1 x2    0 (mâu thuẫn vì x1 , x2  0 ). Vậy f là đơn ánh trên  0;   .
t

Khi đó: Từ  3  f 1  1 .

1
Thay x  1 vào 1 ta được: 1  f  y   1  y  f  y   yf  y   1  f  y   , y   0;  
y

1
Thử lại, ta thấy có duy nhất một hàm số f  x   , x   0;   thỏa yêu cầu bài toán.
x

Nhận xét:

Ta có thể chứng minh f đơn ánh như sau:

x2 f  yf  x  
Từ 1 ta được:  , x, y   0;    4
x  y f  x  f  y 

Giả sử x1  0, x2  0 thỏa f  x1   f  x2  . Khi đó từ  4  ta có:


f  yf  x1   f  yf  x2   x12 x22
    x1 x2  x1  x2   y  x1  x2  x1  x2   0
f  x1   f  y  f  x2   f  y  x1  y x2  y

  x1  x2  x1 x2  y  x1  x2   0  x1  x2 (vì x1 , x2 , y   0;    x1 x2  y  x1  x2  0 )
Từ đó cho ta thấy, khi chứng minh f là đơn ánh ta có thể chứng minh bằng nhiều cách khác
nhau như:

 f u   f v   u  v

 Chứng minh phản chứng , x1  x2 thỏa f  x1   f  x2  , tìm cách dẫn đến điều mâu thuẫn

Khi chứng minh f là đơn ánh, xét f  x1   f  x2  với x1 , x2 là biến đại diện cho biến x nếu
x là biến nằm ngoài sự tác động của f .

Qua các ví dụ trên nếu ta ấp dụng được tính đơn ánh của f thì bài toán trở nên khá nhẹ
nhàng và đơn giản.

2 . SỬ DỤNG TÍNH CHẤT TOÀN ÁNH.


Nhận xét:

 Nếu f là toàn ánh thì ta hay dùng tồn tại một số b sao cho f  b   0
 f  b   0; f  b   1;...
 Nếu quan hệ hàm là hàm bậc nhất của biến ở vế phải ta có thể nghĩ đến tính chất đơn ánh,
toàn ánh và vận dụng tính chất này khi cần thiết.
 Khi giải phương trình hàm dựa vào giá trị của đối số và giá trị của hàm số ta cũng có thể
vận dụng tính toàn ánh của f .

Bài toán 1. Tìm tất cả các hàm số f :    thỏa mãn

f  x  f  y    x  f  y   xf  y  , x, y  

Lời giải.

* Giả sử tồn tại hàm số f  x  thỏa yêu cầu bài ra.

* Ta có thể viết lại quan hệ hàm dưới dạng f  x  f  y    x  f  y   1  f  y   2

TH1: f  y   1, y   , thử lại thỏa yêu cầu bài toán.

TH2: f  y  không đồng nhất với 1, khi đó vế phải của  2  là hàm bậc nhất theo x nên tập giá trị
là  . Khi đó f là toán ánh.

Thay x bởi 0 vào 1 ta được: f  f  y    f  y   3


Vì f là toán ánh nên x  , y   thỏa f  y   x nên từ  3 ta được f  x   x , thử lại ta thấy
f  x   x không thỏa mãn.

Vậy: f  x   1 là hàm số thỏa mãn yêu cầu bài toán.

Nhận xét:

Trong các bài toán này từ điều kiện f  f  y    f  y  giúp ta tìm ra nghiệm của bài toán
nhưng với điều kiện f phải là toán ánh.

Như vậy, tính toán ánh của f trong bài toán này tỏ ra khá hiệu quả giúp bài toán trở nên dễ
dàng hơn trong quá trình tìm bời giải.

Bài toán 2. ( Việt Nam TST 2004). Tìm tất các giá trị của a sao cho tồn tại duy nhất một hàm số
f :    thỏa mãn điều kiện:

2
f  x 2  y  f  y     f  x    ay, x, y   . 1
Lời giải.

* Giả sử tồn tại hàm số f  x  thỏa yêu cầu bài ra.

2
 
* Khi a  0 , từ 1 ta được f x 2  y  f  y    f  x   , x, y   , khi đó ta có hai hàm số thỏa
mãn là f  x   0 và f  x   1  a  0 (loại)

* Khi a  0 . Vì vế phải là hàm bậc nhất nên y có tập giá trị là  . Do đó f là toàn ánh
 b   : f  b   0

Tìm b

2
Thay y bởi b vào 1 ta được: f  x 2  b    f  x    ab , x    2 .
2
Thay x bởi  x vào  2  ta được: f  x 2  b    f   x    ab , x    3

2 2  f  x  f  x
Từ  2  ,  3 ta được:  f  x     f   x      f  b   0 .
 f  x    f   x 

2
Thay y bởi b vào 1 ta được: f  x 2  b    f  x    ab , x    4

Từ  3 ,  4   f  x 2  b   f  x 2  b   2ab  5

Thay x bởi 0 vào  5  ta được: f  b   f  b   2ab  2ab  0  b  0  a  0 


Vậy f  b   0  b  0 .

 Thay x bởi 0 vào 1 ta được: f  y  f  y    ay  6

Thay y bởi 1 vào  6  ta được: f 1  f 1   a  8 

2
Thay y bởi 0 vào 1 ta được: f  x 2    f  x   7

2  f 1  1
Thay x bởi 1 vào  7  ta được: f 1   f 1   
 f 1  0

Vì f  x   0  x  0 nên ta nhận f 1  1

Thay f 1  1 vào  8 ta được: f  2   a

2
Ta có: a 2   f  2    f  22   f  4   f  2 
2

  2    a  f  2  a  2a  aa  02
2  f
2

Vì a  0 nên ta nhận a  2

Vậy a  2 .
2
Khi đó 1 trở thành: f  x 2  y  f  y     f  x    2 y, x, y  . 9
2

Thay y bởi 
 f  x vào  9  ta được :
2

2
  f  x    f  x  2   2 2
  f  x 
2
  f  x  2  
 2
f x  
f      f  x    f  x   f x 
 2
 f    0
 2  2   2  2 
     

2
 f  x    f  x  2 
x 2
  f    0, x   ( vì f  x   0  x  0 )
2  2 
 

  f  x  2   f  x 
2
2

 f    x  10 
 2  2
 

Hay f  y    x 2  y  f  y   y   x 2 , x   11
2

Thay y bởi 
 f  x và f  y   y bởi  y 2 do (11), kết hợp với  7  , khi đó từ  9  ta được:
2
2 2
f  x 2  y 2    f  x     f  y    f  x 2   f  y 2  , x, y   12 

Từ 12  thay x bởi 0 ta được: f   y 2    f  y 2   f là hàm số lẻ.

Khi đó từ 1 ta có: f  x  y   f  x    y    f  x   f   y   f  x   f  y  , x, y  

 f cộng tính.

2 2 2
Ta lại có:  f  x    f  x 2  nên  f  x  y    f  x  y 

2
  f  x   f  y    f  x 2  2 xy  y 2 
2 2
  f  x    2 f  x  . f  y    f  y    f  x 2   f  y 2   f  2 xy 

 2 f  x  f  y   f  2 xy   2 f  x  f  y   f  xy  xy   2 f  x  f  y   f  xy   f  xy 

 f  x  . f  y   f  xy   f nhân tính.

Hàm f vừa nhân tính, vừa cộng tính nên f  x   x , thử lại ta thấy hàm f  x   x thỏa yêu
cầu bài toán.

Nhận xét.

Đây là một bài toán khó, tính chất toàn ánh của f sử dụng khá hiệu quả; dấu hiệu
nhận biết tính chất toàn ánh ở đây chính là vế phải đẳng thức là hàm bậc nhất, khi khẳng
định f là toán ánh ta luôn b   : f  b   0, từ đó ta đi tìm b .

Từ điều kiện f  0   0 ta nhận xét được một loạt tính chất đặc biệt của f như f 1  1,
2
f  x 2    f  x   , tính chất f là hàm số lẻ; tính nhân tính; cộng tính của hàm f làm cơ sở
để tìm ra kết quả của bài toán.

Nếu ban đầu ta không tìm cách vận dụng tính chất toàn ánh của f thì bài này khó có
thể giải quyết được.

Vì vậy khi giải một bài toán phương trình hàm thì việc nhận biết và vận dụng các tính
chất của hàm f là rất quan trọng nó sẽ giúp việc giải quyết bài toán một cách dễ dàng hơn.

Bài toán 3. Tìm tất cả các hàm số f :    thỏa mãn:

f  x  y 2  z   f  f  x    yf  x   f  z  , x, y, z   1

Lời giải
* Giả sử tồn tại hàm số f thỏa đề bài ra.

* Nhận thấy f  x   0, x   thỏa 1

* Giả sử f  x   0 tức tồn tại u   sao cho f  u   0 .

Kí hiệu P  u, v  thay x bởi u , y bởi v vào 1 . Với x   ta có:

 x  f  f  u    f  0    x  f  f  u    f  0  2 
P  u; ;0   f u      x  f là toàn ánh.
 f  u     f u   
    

P  x;0;0   f  x   f  f  x    f  0   f  x   x  f  0 

(vì f là toàn ánh nên f  f  x    x )  2 

Thay  2  vào 1 ta được:


x  y 2  z  f  0   f  x  f  0    y  x  f  0    z  f  0   x  y 2  f  x  f  0   y  x  f  0  

 x  y 2  x  f  0   f  0   y  x  xf  0    y 2  2 f  0   y  x  xf  0    3

Thay y bởi 0 vào  3 ta được: f  0   0 . Khi đó từ  2  ta được: f  x   x, x   .

Thử lại, thấy hàm số này không thỏa 1 . Do đó có duy nhất một hàm số thỏa mãn yêu cầu
bài toán là: f  x   0, x  .

Nhận xét:

Ngoài cách nhận biết tính chất toàn ánh của f dựa vào một vế của đẳng thức là hàm
bậc nhất ta còn có thể chứng minh f là toàn ánh bằng cách y  , x   thỏa f  x   y

x  f  f  u    f  0
Sở dĩ ta có thể nhận biết được cách thay y bởi vào 1 là do
y

P  u, y,0   f  u  y 2   f  f  u    yf  u   f  0  , y  

x  f  f  u    f  0
Với x  , xét x  f  f  u    yf  u   f  0   y  . Vì vậy ta thực hiện
f u 

 x  f  f  u    f  0 
P  u, ,0  để suy ra f là toàn ánh.
 f u  
 
Đối với một bài toán phương trình hàm nếu ta có thể vận dụng tính chất toàn ánh của
f điều dó giúp ta tính được một số giá trị đặc biệt của hàm như f  0  ; f 1 làm cơ sở để
tìm ra đáp án của bài toán.

3 . SỬ DỤNG TÍNH CHẤT SONG ÁNH.


* Nhận xét:
Khi f là một song ánh ta có thể chú ý đến tính chất đơn ánh và toán ánh mà ta đã
vận dụng trong phần 1 và phần 2. Ngoài ra ta có thể chú ý thêm.
 f song ánh và cộng tính trên tập rời rạc khi đó f  x   ax

 f song ánh liên tục (đơn điệu) và cộng tính trên tập  khi đó f  x   ax

Bài toán 1. Tìm tất cả các hàm số f :    thỏa mãn

f  f  x   y   x  f  y  , x, y   1
Lời giải
Giả sử tồn tại hàm số f thỏa yêu cầu bài toán.

* f là đơn ánh, thật vậy:

Xét f  x1   f  x2   f  x1   y  f  x 2   y  f  f  x1   y   f  f  x 2   y 

 x1  f  y   x2  f  y   x1  x2  f là đơn ánh.

* f là toán ánh, vì vế phải là một hàm bậc nhất của x nên f là một toàn ánh

Thay x bởi 0 ; y bởi 0 vào 1 ta được: f  f  0    f  0 

Vì f là đơn ánh  f  0   0

Thay y bởi 0 vào 1 ta được: f  f  x    x  2


Thay  2  vào 1 ta được: f  f  x   y   f  f  x    f  y   3 .
Vì f là toán ánh nên thay f  x  bởi x vào  3 ta được: f  x  y   f  x   f  y 

 f cộng tính  f  x   ax  4 .
Thay  4  vào 1 ta được

f  ax  y   x  ay  a  ax+y   x  ay  a 2 x  ay  x  ay  a  1 hoặc a  1.


Thử lại, ta thấy f  x   x hoặc f  x    x thỏa yêu cầu bài toán.

Nhận xét: Nếu yêu cầu bài toán trên tập  thì ta cần thêm tính chất đơn điệu hoặc liên tục,
ta có thể xét bài toán tương tự sau:
Bài toán 2. (toán học tuổi trẻ 2010). Tìm tất cả các hàm số liên tục f :    thỏa mãn
điều kiện
f  x  f  y   2 y  f  x  1
Lời giải
Giả sử tồn tại hàm số f thỏa yêu cầu bài toán

* f là đơn ánh

Xét f  y1   f  y2  , khi đó:

f  y1   f  y2   x  f  y1   x  f  y2   f  x  f  y1    f  x  f  y2  

 2 y1  f  x   2 y2  f  x   y1  y2  f là đơn ánh.

* f là toán ánh, vì vế phải là hàm bậc nhất theo y nên f là toán ánh.

* Thay x bởi 0, y bởi 0 vào 1 ta được f  f  0    f  0 

Vì f là đơn ánh  f  0   0.

Thay x bởi 0 vào 1 ta được: f  f  y    2 y  2


Thay  2  vào 1 ta được: f  x  f  y    f  f  y    f  x   3
Thay f  y  bởi y vào  3  ta được: f  x  y   f  y   f  x   f cộng tính
 f  x   ax  4
Thay  4  vào 1 ta được:
f  x  ay   2 y  ax  a  x  ay   2 y  ax  a 2 y  2 y  a  2; a   2.

Thử lại ta thấy f  x   2 x hoặc f  x    2 x thỏa yêu cầu bài toán.

Nhận xét:
* Do giả thiết bài toán f là hàm liên tục, vì vậy khi chứng minh được f là song ánh ta cần
thêm tính chất cộng tính của f là có thể suy ra f có dạng f  x   ax . Tương tự như vậy
khi f đơn điệu và cộng tính, nếu ta chứng tỏ được f song ánh ta cũng có thể khẳng định
f  x   ax.

Bài toán 3. (IMO 1992). Tìm tất cả cá hàm số f :    thỏa mãn điều kiện
2
f  x 2  f  y     f  x    y ,  x , y  . 1
Lời giải
* Giả sử tồn tại hàm f thỏa mãn yêu cầu bài toán

* f là đơn ánh, thật vậy nếu f  y1   f  y2  thì


2 2
f  x 2  f  y1    f  x 2  f  y2     f  x    y1   f  x    y2  y1  y2 .

* f là toán ánh, vì vế trái là hàm bậc nhất theo y nên f có tập giá trị toàn bộ là 

Vậy f là một song ánh.

* Tính f  0  . Do f là một song ánh nên tồn tại duy nhất a   sao cho f  a   0 . Thay
2
x  0 vào 1 ta được f  f  y     f  0    y

Thay x  y  a và sử dụng kết quả trên ta được


2
 
f  a2   a  f  a   f f  a2   0   f 0  a 2  f  0  a  0

Từ đây ta thu được quan hệ quen thuộc


2
f  f  x    x, x   và f  x 2    f  x   (thay y  0 )

* Xét tính cộng tính


Khi x  0
Ta có
2 2
 
f  x  y  f 

  x  
 f f  y   f

    f  y
x (theo (1))
2

 f  x   f  y  vì f  x      2 , x  0, y  .
 f x2

Thay y bởi x vào  2  ta được: f  0   f  x   f   x   f  x    f   x   f là hàm lẻ.

Khi x  0
Ta có f  x  y   f     x  y     f   x  y    f   x   f   y   f  x   f  y  , x  0, y  .

 f  x  y   f  x   f  y   f cộng tính.

* f là hàm đơn điệu tăng


2
Vì f  x 2    f  x   nên x  0  f  x   0.

Xét x  y thì x  y  0 nên f  x  y   0


Khi đó f  x   f  x  y   y   f  x  y   f  y   f  y   f cộng tính và đơn điệu nên có
dạng f  x   ax, thay vào ta được a  1. Vậy f  x   x, x   thỏa mãn yêu cầu bài toán.

Nhận xét.
Khi chứng minh f song ánh ta thường liên tưởng đến việc chứng minh f cộng tính và đơn
điệu để sử dụng tính chất f  x   ax nhằm tìm ra kết quả của bài toán.
ĐỊNH LÍ HALL

Lâm Quốc Toàn

Trường THPT Chuyên Nguyễn Thị Minh Khai - Sóc Trăng

1. TÓM TẮT LÝ THUYẾT

1.1. Bài toán đám cưới của Hall

“Với n chàng trai, trong đó cứ k chàng trai bất kỳ thì quen với ít nhất k cô gái
1  k  n  . Chứng minh rằng đây là điều kiện cần và đủ để tổ chức đám cưới tập
thể sao cho mỗi chàng trai cưới một cô gái mình quen”.

Định lí 1 (định lí Hall dạng tập hợp). Gọi Ai , i  1, 2,3,..., n là tập hợp các cô gái

mà chàng trai thứ i quen. Khi đó, với mọi i1 , i2 , i3 ,..., ik  1; 2;3;...; n thỏa mãn:

Ai1  Ai2  Ai3  ...  Aik  k (điều kiện Hall)

khi và chỉ khi tồn tại

 a1; a2 ; a3 ;...; an   A1  A2  A3  ... An sao cho ai  a j với mọi i  j .

Chứng minh

 Giả sử  a1; a2 ; a3 ;...; an   A1  A2  A3  ...  An sao cho ai  a j với mọi i  j .

Với mọi ik  1; 2; 3;...; n thì do ai  Ai , ai  Ai , ai  Ai ...., ai  Ai , với


1 1 2 2 3 3 k k

ait  ais khi s  t . Do đó

Ai1  Ai2  Ai3  ...  Aik  k .

 Giả sử hệ  A1 ; A2 ; A3 ;...; An  thỏa mãn điều kiện Hall.

Ta bắt đầu bỏ đi phần tử thuộc mỗi tập Ai sao cho điều kiện Hall vẫn được

thỏa mãn. Cuối cùng thu được hệ tối giản  B1 ; B2 ; B3 ;...; Bn  vẫn thỏa mãn điều kiện

Trang 1
Hall, với Bi  Ai , hệ tối giản ở đây được hiểu là nếu như bỏ đi một phần tử từ một
tập Bi bất kì nào đó thì điều kiện Hall không còn được thỏa mãn.

+ Ta chứng minh rằng Bi  1, với i  1, n .

Giả sử ngược lại tồn tại i sao cho Bi  2.

Không mất tính tổng quát giả sử B1  2 , khi đó B1 có 2 phần tử khác nhau là

x và y . Do đó nếu bỏ đi x hoặc y thì điều kiện Hall không còn thỏa mãn. Vì thế

tồn tại hai tập P và Q

P  2;3; 4;....; n và Q  2;3; 4;....; n .

 
Gọi X   B1 \  x     Bi  ;
 iP 

 
Y   B1 \  y     Bi  .
 iQ 

   
Khi đó X   B1 \  x     Bi  và Y   B1 \  y     Bi  không thỏa điều kiện
 iP   iQ 
Hall nên

X  P  1  X  P và Y  Q  1  Y  Q

 
Mặt khác: X  Y  B1    Bi  và  Bi   X  Y  .
 iP Q  iP Q

Từ 2 điều này suy ra X  Y  1  P  Q và X  Y  P  Q .

Theo nguyên lí bù trừ và nhận xét trên, ta được:

P  Q  X  Y  X Y  X Y  1 P  Q  P  Q  1 P  Q .

Điều vô lý này dẫn đến khẳng định Bi  1, với i  1, n .

+ Ta chứng minh các tập Bi đôi một không giao nhau.

Giả sử ngược lại tồn tại i, j, i  j sao cho Bi  B j   x thì theo điều kiện Hall,

ta có

Trang 2
1  Bi  B j  2 (vô lí)

Như vậy chọn được ai  Bi  Ai , i  1, n sao cho ai  a j để

 a1; a2 ; a3 ;...; an   A1  A2  A3  ... An .

1.2. Đồ thị lưỡng phân

Định nghĩa 1. Một đồ thị (hữu hạn) được hiểu là một bộ hai tập hợp hữu hạn: Tập
hợp đỉnh và tập hợp cạnh (có hướng hoặc vô hướng) nối các đỉnh này với nhau.

Ta kí hiệu: đồ thị bởi chữ G ,

tập đỉnh bởi chữ V ,

tập cạnh bởi chữ E .

Đồ thị được kí hiệu là G  V , E  .

Định nghĩa 2. Cho đồ thị G  V , E  . Khi đó:

a) Cạnh nối hai đỉnh a và b của đồ thị được kí hiệu là a, b  ;

b) Hai đỉnh khác nhau của đồ thị được gọi là kề nhau hoặc láng giềng của
nhau nếu như chúng được nối với nhau bởi một cạnh.

Như vậy, nếu hai đỉnh a và b của một đồ thị G  V , E  là kề nhau, ta có


a, b  E .
Định nghĩa 3 (đồ thị lưỡng phân (đồ thị hai phe)). Đồ thị lưỡng phân là đồ thị
G  V , E  mà tập đỉnh V có thể phân hoạch thành hai tập hợp X , Y sao cho tập

cạnh E chỉ gồm các cạnh nối hai đỉnh không cùng một tập hợp.

Ta kí hiệu đồ thị lưỡng phân này là G   X , Y , E  .

Ví dụ 1. Đồ thị biểu diễn một buổi nhảy với các đôi nhảy khác giới là đồ thị lưỡng
phân.

Định lí 2 (định lí Hall dạng graph). Cho đồ thị lưỡng phân G   X , Y , E  . Với

mỗi tập con A của X , gọi G  A là tập các đỉnh thuộc Y kề với một đỉnh thuộc A
nào đó. Khi đó điều kiện cần và đủ để tồn tại một đơn ánh f : X  Y sao cho x kề
với y  f x  là G  A  A với mọi A là tập khác rỗng con X .

Trang 3
Chứng minh

Ta xem:

 Mỗi chàng trai tương ứng với một đỉnh của tập hợp X ;

 Mỗi cô gái tương ứng với một đỉnh của tập hợp Y ;

 Chàng trai quen cô gái nếu một đỉnh thuộc X có cạnh nối với một đỉnh
thuộc Y . E là tập hợp các cạnh nối các đỉnh thuộc X với các đỉnh thuộc Y .

Theo định lí 1, ta được điều phải chứng minh.

Định nghĩa 4 (ghép cặp hoàn hảo). Một ghép cặp từ X đến Y là một tập hợp các
cạnh sao cho không có hai cạnh nào chung nhau đầu mút. Một ghép cặp được gọi là
hoàn hảo khi nó có thể nối tất cả các đỉnh từ X đến Y (lưu ý không cần phải tất cả
các đỉnh của Y ), tức là tồn tại một đơn ánh từ X đến Y .

Định lí 3. Cho đồ thị lưỡng phân G   X , Y , E  . Với mỗi tập con A khác rỗng của

X , kí hiệu G  A là tập các đỉnh thuộc Y kề với một đỉnh nào đó thuộc A. Khi đó

điều kiện cần và đủ để tồn tại một ghép cặp hoàn hảo từ X đến Y là G  A  A ,

  A  X .

Tính chất 1. Cho đồ thị lưỡng phân G   X , Y , E  và X  Y  n . Nếu bậc của tất

cả các đỉnh đều nguyên dương và bằng nhau thì tồn tại cách nối 1  1 từ các đỉnh của
X đến các đỉnh của Y .

Lời giải

Gọi số nguyên dương trên là m. Ta xem:

 Mỗi chàng trai tương ứng với một đỉnh của tập hợp X ;

 Mỗi cô gái tương ứng với một đỉnh của tập hợp Y ;

 Chàng trai quen cô gái nếu một đỉnh thuộc X có một cạnh nối với một
đỉnh thuộc Y . E là tập hợp các cạnh nối các đỉnh thuộc X với các đỉnh thuộc Y .

Theo đề bài, ta có mỗi chàng trai quen với đúng m cô gái, do đó cứ k chàng
trai bất kì quen đúng m.k cô gái, tức là cứ k chàng trai bất kỳ thì quen với ít nhất k
cô gái.

Trang 4
Áp dụng bài toán đám cưới của Hall và X  Y  n , ta có thể tổ chức đám

cưới tập thể sao cho mỗi chàng trai cưới một cô gái mình quen tức là tồn tại cách
nối 1  1 từ các đỉnh của X đến các đỉnh của Y .

2. BÀI TẬP

Bài 1 (Hậu Giang-2015). Có 1000 hộp bi, mỗi hộp chứa đúng 10 viên bi. Biết rằng
không có 11 viên bi nào cùng màu. Chứng minh rằng có thể lấy từ mỗi hộp một
viên bi sao cho tất cả 1000 viên bi được lấy ra đều có màu khác nhau.

Lời giải

Có 1000 hộp mà mỗi hộp chứa đúng 10 viên bi nên tổng số các viên bi là:
1000.10  10000 (viên bi).

Mà không có 11 viên bi nào cùng màu nên theo nguyên lí Dirichlet thì có ít
nhất 1000 màu bi khác nhau.

Ta xây dựng đồ thị lưỡng phân G   X , Y , E  với:

 X là tập hợp các đỉnh mà mỗi đỉnh của nó tương ứng với một hộp bi;

 Y là tập hợp các đỉnh mà mỗi đỉnh của nó tương ứng với một màu bi;

 Một đỉnh thuộc X có cạnh nối với một đỉnh thuộc Y nếu hộp bi tương ứng
với đỉnh thuộc X có màu tương ứng với đỉnh thuộc Y . E là tập hợp các cạnh nối
các đỉnh thuộc X với các đỉnh thuộc Y .

Gọi A là tập khác rỗng bất kỳ con X , G  A là tập hợp tất cả các đỉnh thuộc Y
kề với các đỉnh thuộc A .

Ta chứng minh A  G  A .

Thật vậy, giả sử A  k .

Tức là A gồm k đỉnh biểu diễn cho k hộp, mà mỗi hộp có 10 bi và có tối đa
10 viên bi cùng màu nên số lượng màu tối thiểu của 1 hộp này là 1 màu, do đó số
lượng màu tối thiểu của k hộp này là k màu.

Vì vậy các đỉnh thuộc Y có cạnh được nối với các đỉnh thuộc A ít nhất là k .

Trang 5
Hay A  G  A , với mọi A  X .

Theo định lí Hall thì tồn tại một ghép cặp hoàn hảo từ X đến Y .

Do đó ta có thể lấy từ mỗi hộp một viên bi sao cho tất cả 1000 viên bi được lấy
ra đều có màu khác nhau.

Bài 2 (Kazakhstan 2003). Cho hai tờ giấy hình vuông cùng có diện tích bằng 2003.
Mỗi hình vuông này được chia thành 2003 đa giác, mỗi đa giác có diện tích bằng 1
(cách chia cho hai tờ giấy này khác nhau). Người ta đặt hai tờ giấy này chồng lên
nhau. Chứng minh rằng có thể đặt 2003 cây kim để tất cả 4006 đa giác đó đều bị
đâm thủng.

Lời giải

Ta xây dựng đồ thị lưỡng phân G   X , Y , E  với:

 X là tập hợp 2003 đỉnh, mỗi đỉnh tương ứng với một đa giác có diện tích
bằng 1 của tờ giấy thứ nhất;

 Y là tập hợp 2003 đỉnh, mỗi đỉnh tương ứng với một đa giác có diện tích
bằng 1 của tờ giấy thứ hai;

 Một đỉnh thuộc X có cạnh nối với một đỉnh thuộc Y nếu ta đặt hai tờ giấy
lên nhau thì chúng có ít nhất một điểm chung. E là tập hợp các cạnh nối các đỉnh
thuộc X với các đỉnh thuộc Y .

Gọi A là tập gồm k đỉnh con X , khi đó A là tập gồm k đa giác của tờ giấy
thứ nhất, k đa giác này phủ một diện tích bằng k . Do đó để phủ được hết k đa giác
này thì tờ giấy thứ hai phải chứa ít nhất là k đa giác. Vì thế

G  A  k  A .

Theo định lí Hall tồn tại một ghép cặp hoàn hảo từ X đến Y . Khi đó ta chỉ
cần đặt 2003 cây kim tương ứng với các cạnh trong ghép cặp hoàn hảo đó thì thỏa
yêu cầu bài toán.

Trang 6
Bài 3. Các ô vuông của một bảng vuông 2017  2017 (2017 hàng ngang, 2017 hàng
dọc) được tô các màu trắng hoặc đen sao cho trên mỗi hàng cũng như trên mỗi cột
đều có đúng 3 ô được tô màu đen. Chứng minh rằng trong mọi cách tô như vậy luôn
có thể tìm ra 2017 ô được tô màu đen sao cho không có 2 ô nào nằm cùng một hàng
hay cùng một cột.

Ý tưởng

Các bài toán áp dụng định lí Hall có một giả thiết quan trọng là đồ thị lưỡng
phân G   X , Y ; E  . Trong tổ hợp ta thấy bảng (có hàng ngang và hàng dọc) là một

cấu hình tương ứng.

Lời giải

Ta xây dựng đồ thị lưỡng phân G   X , Y , E  với:

 X là tập hợp các đỉnh mà mỗi phần tử của nó tương ứng với một hàng,
X  h1 ; h2 ;...; h2017  ;

 Y là tập hợp các đỉnh mà mỗi phần tử của nó tương ứng với một cột,
Y  c1 ; c2 ;...; c2017  ;

 Một đỉnh thuộc X có cạnh nối với một đỉnh thuộc Y nếu hàng cắt cạnh tại
một ô màu đen. E là tập hợp các cạnh nối các đỉnh thuộc X với các đỉnh thuộc Y .

Theo đề bài, ta được X  Y  2017 và bậc của tất cả các đỉnh đều bằng 3.

Áp dụng tính chất 1, tồn tại cách nối 1  1 từ các đỉnh thuộc X đến các đỉnh
thuộc Y hay ta luôn có thể tìm ra 2017 ô được tô màu đen sao cho không có 2 ô nào
nằm cùng một hàng hay cùng một cột.

Bài 4 (VN TST 2010). Có n nước, mỗi nước có k đại diện  n  k  1 . Người ta

chia n.k người này thành n nhóm, mỗi nhóm k người sao cho không có 2 người
nào cùng nhóm đến cùng một nước. Chứng minh rằng có thể chọn ra n người đến
từ các nhóm khác nhau và đến từ các nước khác nhau.

Lời giải

Trang 7
Ta xây dựng đồ thị lưỡng phân G   X , Y , E  với:

 X là tập hợp đỉnh mà mỗi phần tử của nó tương ứng với một nhóm,
X  a1 ; a2 ; a3 ;...; an  ;

 Y là tập hợp đỉnh mà mỗi phần tử của nó tương ứng với một nước,
Y  b1 ; b2 ; b3 ;...; bn  ;

 Một đỉnh thuộc X có cạnh nối với một đỉnh Y nếu trong nhóm X có đại
diện nước mà đỉnh thuộc Y biểu diễn. E là tập hợp các cạnh nối các đỉnh thuộc X
với các đỉnh thuộc Y .

Ta có một nước X bất kì có k người đại diện trong k nhóm Y khác nhau và
một nhóm Y bất kì có k người đại diện của k nước X khác nhau.

Từ đó, ta được đồ thị lưỡng phân G   X , Y , E  , X  Y  n và bậc của tất cả

các đỉnh đều bằng k , nên theo tính chất 1, tồn tại cách nối 1  1 từ các đỉnh của X
đến các đỉnh của Y hay ta có thể chọn ra n người đến từ các nhóm khác nhau và
đến từ các nước khác nhau.

Bài 5. Một tập P chứa 2017 số nguyên tố phân biệt. Gọi A là tập hợp gồm các tích
của 1008 phần tử của P và B là tập hợp gồm các tích của 1009 phần tử của P .
Chứng minh rằng tồn tại một phép tương ứng 1  1 f từ A đến B thỏa mãn a chia
hết f  a  , với mọi a  A .

Hướng dẫn giải

Ta xây dựng đồ thị lưỡng phân G   X , Y , E  với:

 X là tập hợp đỉnh mà mỗi phần tử của nó tương ứng với một phần tử của
tập hợp A ;

 Y là tập hợp đỉnh mà mỗi phần tử của nó tương ứng với một phần tử của
tập hợp B ;

 Một đỉnh thuộc X có cạnh nối với một đỉnh Y nếu a chia hết b với a  A,
b  B . E là tập hợp các cạnh nối các đỉnh thuộc X với các đỉnh thuộc Y .

Trang 8
Khi đó, ta có mỗi đỉnh thuộc X có cạnh nối với đúng 1009 đỉnh thuộc Y .
1008
Từ đó, ta được đồ thị lưỡng phân G   X , Y , E  , X  Y  C2017 và bậc của tất

cả các đỉnh đều bằng 1009.

Theo tính chất 1, tồn tại một phép tương ứng 1  1 f từ A đến B thỏa mãn a
chia hết f  a  , với mọi a  A .

Bài 6. Bảng n  n được gọi là bảng hoán vị nếu các số trên bảng là 0 và 1 sao cho
trên mỗi hàng và mỗi cột có đúng một số 1. Cho G là 1 bảng n  n gồm các số
nguyên không âm sao cho tổng các số trên mỗi hàng và trên mỗi cột bằng nhau và
bằng c . Chứng minh rằng G có thể viết dưới dạng tổng các bảng hoán vị.

Ý tưởng

Ta thấy sự giống nhau của bảng hoán vị và bảng G là bảng hoán vị có tổng
các hàng và các cột bằng nhau. Nên thay vì biểu diễn G thành tổng các bảng hoán
vị, ta sẽ trừ đi 1 ở các ô được chọn tương ứng trong bảng G .

Lời giải

Ta sẽ chứng minh luôn tìm được n số nguyên dương nằm ở các hàng và cột
khác nhau.

Ta xây dựng đồ thị lưỡng phân G   X , Y , E  với:

 X là tập hợp đỉnh mà mỗi phần tử của nó tương ứng với một hàng,
X  h1 ; h2 ;...; hn  ;

 Y là tập hợp đỉnh mà mỗi phần tử của nó tương ứng với một cột,
Y  c1 ; c2 ;...; cn  ;

 Một đỉnh thuộc X có cạnh nối với một đỉnh thuộc Y nếu giao của hàng
và cột này là một ô chứa số nguyên dương. E là tập hợp các cạnh nối các đỉnh
thuộc X với các đỉnh thuộc Y .

Ta sẽ chứng minh k đỉnh trong X kề với ít nhất k đỉnh trong Y .

Trang 9
Giả sử như k đỉnh trong X kề với ít hơn k đỉnh trong Y , khi đó k đỉnh trong
X kề với l  l  k  đỉnh trong Y .

Xét bảng con k  l . Tổng các ô trong bảng bằng tổng các ô tính theo các hàng
và bằng tổng các ô tính theo các cột nên có c.k  c.l (vô lí do k  l ).

Do đó k đỉnh trong X phải kề ít nhất k đỉnh trong Y .

Từ đây theo định lí Hall thì ta luôn chọn ra được n ô chứa n số nguyên dương
và các ô này nằm ở các hàng và cột khác nhau. Giảm đi trong các ô chứa số nguyên
dương 1 đơn vị thì tính chất của bảng vẫn không đổi nên tiếp tục áp dụng nhiều lần
thì ta sẽ được 1 bảng hoán vị (điều phải chứng minh).

Bài 7 (Canada 2006). Trong một bảng ô vuông m  n ( m dòng, n cột) chứa các số
không âm, mỗi hàng hoặc cột chứa ít nhất một số dương. Ngoài ra, nếu một hàng
giao một cột là ô chứa một số dương thì tổng các số của hàng và cột bằng nhau.
Chứng minh rằng m  n .

Ý tưởng

Một cách tự nhiên, ta nghĩ đến việc chứng minh m  n và m  n bằng cách
xây dựng đồ thị lưỡng phân với tập hợp các hàng và các cột rồi áp dụng định lí Hall
là có thể xem xét.

Lời giải

Vì tính chất đối xứng, ta có thể giả sử m  n và sẽ chứng minh m  n .

Ta xây dựng đồ thị lưỡng phân G   X , Y , E  với:

 X là tập hợp đỉnh mà mỗi phần tử của nó tương ứng với một hàng,
X  h1 ; h2 ;...; hn  ;

 Y là tập hợp đỉnh mà mỗi phần tử của nó tương ứng với một cột,
Y  c1 ; c2 ;...; cn  ;

 Một đỉnh thuộc X có cạnh nối với một đỉnh thuộc Y nếu giao của hàng
và cột này là một ô chứa số dương. E là tập hợp các cạnh nối các đỉnh thuộc X với
các đỉnh thuộc Y .

Trang 10
Gọi A là một tập hợp các đỉnh trong X ;

G  A là một tập hợp các đỉnh trong Y , mà các đỉnh này kề với ít nhất

một đỉnh trong A .

Ta chứng minh G  A   A .

Thật vậy giả sử ngược lại G  A   A .

Vì hai đỉnh chỉ được nối với nhau, nếu ô giao nhau của hàng và cột là ô chứa
số dương nên ngoại trừ những ô giao nhau của hàng và cột thì tất cả các số còn lại
trên hàng và trên cột đó đều là 0.

Gọi tổng các hàng trong A là s1 , s2 , s3 ,..., sk . Khi đó mỗi cột trong G  A cũng

có tổng là một số si nào đó. Nhìn theo khía cạnh cột thì tổng toàn bộ các số dương
trên hàng cũng chính là tổng của tất cả các số dương trên cột. Điều đó có nghĩa là
tổng các hàng thuộc A chỉ bằng tổng của một tập con các hàng thuộc A (điều này
vô lí vì các số si đều dương).

Do đó G  A   A theo định lí Hall tồn tại một ghép cặp hoàn hảo từ X đến

Y dẫn đến X  Y hay m  n .

Từ đó ta được điều phải chứng minh.

3. BÀI TẬP TƯƠNG TỰ

Bài 1. Cho đồ thị hai phe G , chứng minh rằng nếu các bậc của các đỉnh đều dương
và bằng nhau thì tồn tại một ghép cặp hoàn hảo.

Bài 2. Một bộ bài gồm 52 lá, với thứ tự là: A, 2, 3, 4, 5, 6, 7, 8, 9, 10, J, Q, K và


mỗi lá bài có bốn chất: cơ, rô, bích, chuồng. Người ta chia ngẫu nhiên 52 lá bài này
thành 13 đống, mỗi đống có 4 lá bài. Chứng minh rằng tồn tại một cách lấy từ mỗi
đống một lá bài sao cho 13 lá bài lấy ra đủ thứ tự: A, 2, 3, 4, 5, 6, 7, 8, 9, 10, J, Q,
K.

Trang 11
Bài 3. Bảng n  n gồm các số thuộc 0; 1 sao cho với mọi tập con gồm n ô, trong

đó không có hai ô cùng hàng hoặc cùng cột, chứa ít nhất một số 1. Chứng minh
rằng tồn tại i hàng và j cột với i  j  n  1 , có giao chứa toàn 1.

Bài 4. Cho một bảng n  k với n  k sao cho trong mỗi ô vuông có một số từ 1 đến
n . Biết rằng trong mỗi hàng và mỗi cột không có số nào trùng nhau. Chứng minh
rằng ta có thể mở rộng bảng trên thành bảng n  n với một số từ 1 đến n trong mỗi
ô, sao cho trong mỗi hàng và mỗi cột không có số nào trùng nhau.

Bài 5. Có 100 người đến từ 25 quốc gia, mỗi quốc gia có 4 người và họ ngồi trên
một bàn tròn. Chứng minh rằng ta có thể chia họ thành 4 nhóm sao cho mỗi nhóm
có 25 người của 25 quốc gia khác nhau và không có ai cùng nhóm ngồi cạnh nhau
trên bàn tròn.

Bài 6. Một lớp khiêu vũ có 42 thành viên, trong 31 người bất kì có ít nhất 1 cặp
nam, nữ quen nhau. Chứng minh rằng có thể chọn trong 42 người này 12 cặp nam,
nữ quen nhau.

Bài 7 (USA TST 2014). Cho n là số nguyên dương chẵn và G là đồ thị đơn n đỉnh
n2
với cạnh không có khuyên. Một cặp không tính thứ tự các đỉnh  x; y được gọi
4
là đẹp nếu chúng cùng kề với một đỉnh (tồn tại đỉnh z sao cho xz và yz đều là
cạnh). Chứng minh rằng G có ít nhất n  n  1 cặp đỉnh đẹp.

Trang 12
TÀI LIỆU THAM KHẢO

1. Trần Nam Dũng, Tài liệu bồi dưỡng đội tuyển Việt Nam tham dự IMO
2010.

2. Trần Nam Dũng (Chủ biên), Các phương pháp giải toán qua các kỳ thi
Olympic, 2016.

3. Nguyễn Văn Mậu-Phạm Ngọc Thạch, Kỷ yếu hội thảo khoa học các chuyên
đề bồi dưỡng học sinh giỏi sinh giỏi khu vực duyên hải nam trung bộ và tây nguyên,
Gia Lai-2014.

4. Đề thi học sinh giỏi các tỉnh.

5. Tạp chí Toán học và tuổi trẻ.

6. https://diendantoanhoc.net/.

Trang 13
KHAI THÁC HÀM BẬC NHẤT

Trần Văn Trí (THPT Chuyên Hùng Vương, Bình Dương)

Trong bài viết nhỏ này tôi khai thác một số hướng thường gặp của hàm số y  ax  b trong
phương trình hàm. Nói chung khi dạy phương trình hàm mỗi người có các tiếp cận riêng. Đây là
các tiếp cận của tôi trong quá trình dạy đội tuyển.

HƯỚNG 1: KHAI THÁC TÍNH CỘNG TÍNH

Bài 1: Tìm tất cả f :    thỏa mãn điều kiện

f  x  y   f  x   f  y  , x, y   . (1)

Giải: Giả sử tồn tại f thỏa mãn yêu cầu đề bài

Cho x  y  0 vào (1) ta được f  0   0

Thay y bởi –x vào (1) ta được f   x    x (2)

Thay y bởi x vào (1) ta được f  2 x   2 f  x 

Giả sử k là số nguyên dương sao cho f  kx   kf  x  , x  

Khi đó f   k  1 x   f  kx   f  x    k  1 f  x  , x  
Do đó theo nguyên lí quy nạp, ta có f  nx   nf  x  , x  , n   (3)

Kết hợp (2) và (3) ta suy ra f  nx   nf  x  , x  , n   .

 1 1 1 1
Ta có f  n   nf    f    f 1 , n  
 n n n n

m
Với mọi q  , q  , m  , n   ,  m, n   1
n

m 1 m
f  q   f    mf    f 1
n n n

Vậy f  x   ax, x   với a  f 1 .

Thử lại trực tiếp ta thấy f  x   ax, x   thỏa mãn để bài.


Bài 2 (Singapor 2002) Tìm tất cả f :    thỏa mãn điều kiện

 f  x  y   f  x   f  y   2 xy, x, y  

 f 1  2003

Giải:

Giả sử tồn tại hàm số f thỏa mãn yêu cầu đề bài

 g 1  2002
Đặt f  x   g  x   x 2 . Khi đó ta có 
 g  x  y   g  x   g  y  , x, y  

Theo trên g  x   2002 x

ĐS: f  x   x 2  2002 x .

Chú ý: bài này có thể mở rộng theo các phép đổi biến tương tự như lớp hàm côsi chuyển về cộng
tính

Bài 3: Tìm tất cả f :    thỏa mãn điều kiện

 f  x  y   f  x   f  y   30 xy  x  y  4   2012, x, y  

 f 1  2011

f  x   g  x   10 x3  60 x 2  2012

HƯỚNG 2: KHAI THÁC TÍNH ĐƠN ÁNH

Bài 1: Tìm tất cả các f :    sao cho

f  f  x  y   f  x  y    2 x, x, y  

Giải:

+ Cho y = 0 vào (1) ta được f  2 f  x    x

+ Ta chứng minh f là đơn ánh. Thật vậy giả sử f  x   f  y  khi đó

2 f  x  2 f  y   f  2 f  x  f  2 f  y   x  y

+Ta có
f  f  x  y   f  x  y    2 x  f  2 f  x   và f đơn ánh

Nên f  x  y  f  x  y  2 f  x . (2)

Đặt a  x  y, b  x  y . Khi đó (2) trở thành

 ab
f  a   f b  2 f   (3)
 2 

Cho b  0 vào (3) ta được

a
f  a   f  0  2 f   (4)
2

Sử dụng (4) vào (3) ta được

f  a   f b   f  a  b  f  0 (5)

Đặt g  a   f  a   f  0  . Khi đó (5) trở thành

g  a   g b  g  a  b

g  x   ax  f  x   ax  b . Thử lại f  x   x, f  x    x .

HƯỚNG 3: SỬ DỤNG TÍNH TOÀN ÁNH

Bài 1: (Dự tuyển IMO -2002)Tìm tất cả các hàm f :    thỏa mãn điều kiện

f  f  x   y   2 x  f  f  y   x  , x, y   . (1)

Giải: Dự đoán f  x   x  c

+ Chứng minh f là toàn ánh

Thay y bởi  f  x  vào (1) ta được

f  0   2 x  f   f  x   x  , x  

Hay f   f  x   x   2 x  f  0  , x  

Vì 2 x  f  0  có tập giá trị  nên f là toàn ánh.


+ Vì f là toàn ánh nên tồn tại a   : f  a   0 .

+ Thay x =a vào (1) ta được f  y   2a  f  f  y   a  , y  

Hay f  f  y   a    f  y   a   a, y  

Mà f là toàn ánh nên f  x   x  a, x  

Thử lại ta thấy f x   x , x   thỏa mãn yêu cầu.

Bài 2:(Kore 2003) Tìm tất cả hàm số f :    thỏa điều kiện

f  x  f  y    f  x   xf  y   f  f  y   , x, y  

Giải:

+ Kiểm tra hàm hằng ta được f  x   0 thỏa mãn yêu cầu đề bài.

+ Giả sử f  x   0 .

 Thay x bởi f(y) vào (1) ta được f  0   f 2  y   2 f  f  y  


f 2  y  f  0
Hay f  f  y      , y   (2)
2 2

+ Do f  x   0 nên tồn tại y0   sao cho f  y0   a  0

Cho y  y0 vào (1) ta được

f  x  a   f  x   ax  f  a 
 f  x  a   f  x   ax  f  a   *

Vì vế phải của (*) là hàm bậc nhất theo biến x nên    f  x  a   f  x  | x    T

Vậy T   .

+ Nhận xét 2: Như vậy theo dự đoán ban đầu và (2) nên ta sẽ tìm cách chứng minh
2
 f  x   f  y  
f  f  x   f  y     C
2
Thật vậy , thay x bỡi f  z  vào (1) ta được

f  f  z   f  y   f  f  z   f  z  f  y   f  f  y 
f 2  z  f  0 f 2  y  f  0
 f  z f  y    
2 2 2 2
2
 f  z   f  y  
=-   f  0
2

x2
Vì T   nên f  x     f  0  , x   . Do đó f  0   0 .
2

x2
Thử lại ta thấy f  x    , x   thỏa mãn yêu cầu.
2

Các bài tương tự

Bài 1: (Czech – 2012)Tìm tất cả hàm số f :    thỏa mãn

4
f  x  f  y    f  x    x  f  y    x 4 , x, y  

Bài 2:Tìm tất cả các hàm số f :    thoả mãn điều kiện

f  x  f  y    f  x   f  f  y    2 xf  y   f  y   3, x, y  

Bài 3 Tìm tất cả hàm số f :    thỏa điều kiện sau

f ( x  f ( y ))  3 f ( x)  2 x  f ( y ) x, y   . (1)

Bài 4: Tìm tất cả các hàm số f :    thoả mãn điều kiện

f  x  f  y    2 f  x   x  f  y  , x, y  

HƯỚNG 4: KHAI THÁC HẰNG ĐẲNG THỨC

Bài 1: Tìm tất cả các hàm số f :    sao cho

f  x3   f  y 3    x  y   f  x   f  x  f  y    f  y   , x, y  
2 2
(1)

Giải:
+ Kiểm tra nghiệm riêng: giả sử f  x   c là nghiệm của (1). Khi đó theo (1) ta có

2c   x  y  c 2 , x, y    c  0

Vậy hàm số f  x   0, x   thỏa mãn yêu cầu đề bài

2
Cho y = 0 vào (1) ta được f  x3   x  f  x   , x   (2)

Cho x = y = 0 vào (1) ta được f  0   0 .

Cho x = y = 1 vào (1) ta được 2 f 1  2 f 2 1  f 1  0 hoặc f 1  1

Thay (2) vào (1) ta được

2 2
x  f  x   y  f  y    x  y   f  x   f  x  f  y    f  y   , x, y  
2 2

2 2
Suy ra y  f  x    x  f  y     x  y  f  x  f  y   0, x, y  

hay  f  x   f  y    yf  x   xf  y    0, x, y   (3)

Cho y = 0 vào (3) ta được

 f  x   f 1   f  x   xf 1   0, x  

Suy ra với mỗi x   thì f  x   c hoặc f  x   kx .

Kết hợp với giả thiết f  0   0 ta suy ra f  x   0 hoặc f  x   kx .

+ Ta chứng minh f  x   0, x   hoặc f  x   kx, x   .

Thật vậy, giả sử tồn tại a, b  0 sao cho f  a   0; f  b   kb

Bài 2: (Uzbekistan - 2014) Tìm tất cả các hàm số f :    sao cho

f  x3   f  y 3    x  y   f  x   f  xy    f  y   , x, y  
2 2

Giải:

+Cho x = y =0 vào (1) ta được f  0   0 .


+ Cho y = 0 vào (1) ta được f  x3   xf 2  x  .

Khi đó (1) trở thành

xf 2  x   yf 2  y    x  y   f  x   f  xy    f  y   , x, y  
2 2

Hay xf 2  y   yf 2  x    x  y  f  xy  (2)

+ thay y bỡi – x vào (1) ta được

f  x3   f   x 3   0, x   . Do đó f là hàm số lẻ

+Thay y bởi –y vào (2) ta được

xf 2  y   yf 2  x    y  x  f  xy  (3)

Từ (2) và (3) ta suy ra

xf 2  y   yf  xy  , x, y  

Chọn y  1 , ta được f  x   f 1 x

Thử lại f  x   ax, x   thỏa yêu cầu.

Các bài tương tự

Bài 1:(Mondova 2004) Tìm tất cả các hàm số f :    sao cho

f  x3   f  y 3    x 2  xy  y 2   f  x   f  y   , x, y   (1)

Bài 2: Tìm tất cả các hàm số f :    sao cho

f  x3   f  y 3    x  y   f  x   f  x  f  y    f  y   , x, y  
2 2

Bài 3: Tìm tất cả các hàm số f :    sao cho

f  x3   f  y 3    x  y   f  x   yf  x    f  y   , x, y  
2 2

Bài 4: (IMO shortlist 2009) Tìm tất cả hàm số f :    sao cho

f  xf  x  y    f  yf  x    x 2 , x, y  
Bài 5: (Hàn Quốc 2000) Tìm tất cả các hàm số f :    thỏa mãn điều kiện

f  x 2  y 2    x  y   f  x   f  y   , x , y   (1)

Bài 6: Tìm tất cả các hàm số f , g :    sao cho

f  x   f  y    x  y  g  x  y  , x, y   (1)

Bài 7: Tìm tất cả f :    sao cho

 x  y  f  x  y    x  y  f  x  y   4 xy  x 2  y 2 
Bài 8: Tìm tất cả các hàm số f :    thỏa mãn các điều

1. Có hữu hạn s   sao cho f  s   0


2. f  x 4  y   x3 f  x   f  f  y   , x, y  

HƯỚNG 5: ĐƯA VỀ TÍCH

Bài 1: Tìm tất cả hàm số f :    thỏa điều kiện

 1
 f  0 
 2
a   : f  a  y  f  x   f  a  x  f  y   f  x  y  , x, y   1

Giải:

1
Cho x  y  0 vào (1) ta được f  a   .
2

Cho y = 0 vào (1) ta được

f  a  f  x   f  a  x  f  0   f  x   f  a  x   f  x  , x   (2)

Thay y bởi a  x vào (1) ta được

2 2 1
 f  x    f  a  x   f a 
2
(3)

Từ (2) và (3) ta suy ra

2 1
 f  x  
4
.
1 1
Khi đó với mỗi x ta suy ra f  x   hoặc f  x    .
2 2

Nhận xét: Bây giờ ta chứng minh không xảy ra trường hợp đan hàm. Muốn vậy ta thử hai hàm
trên vào điều kiện (1) xem thử hàm nào sẽ thỏa mãn yêu cầu.

1
Nếu x0   sao cho f  x0    thì
2
2
1 x x  x   x    x 
  f  x0   f  0  0   2 f  0  f  a  0   2  f  0    0 vô lí
2  2 2  2  2   2 

1 1
Vậy f  x   , x   . Thử lại ta thấy f  x   , x   thỏa mãn yêu cầu đề bài.
2 2

Bài 2: (VMO - 1995) Tìm tất cả hàm số f :    thỏa điều kiện

f  x  y    x
2 2 2
 2 yf  x    f  y   , x, y   . (1)

Giải:

Cho x = y = 0 vào (1) ta được


2
f  0   f 0  f  0  0  f  0  1

TH1: f  0   0

Cho y = 0 vào (1) ta được f  x 2   x 2

Cho x = y vào (1) ta được


2
f  0   x 2  2 xf  x    f  x   , x  
2
  f  x   x   0, x    f  x   x, x  

Thử lại ta thấy f  x   x, x   thỏa mãn yêu cầu đề bài.

TH2: f  0   1

Hướng 1:

Cho y = 0 vào (1) ta được f  x 2   x 2  1, x   . Do đó f  x   x  1, x  0 .


Cho x = 0 vào (1) ta được
2
f  y 2   2 y   f  y  
2 2
  f  y    f  y 2   2 y  y 2  2 y  1   y  1

Khi đó với mỗi giá trị y ta suy ra

 f  y  y 1

 f  y    y  1

Ta chứng minh không xảy ra đan hàm

Cách 1: loại thẳng một hàm

Giả sử y0   sao cho f  y0    y0  1

Chọn x  y  y0 vào (1) ta được

2
1  y0 2  2 y0 f  y0    f  y0  
2
  y0  f  y0    1  f  y0   y0  1  f  y0   y0  1

+ Nếu f  y0   y0  1  y0  1   y0  1  y0  0 và f  0   1 (mâu thuẫn)

+ Nếu f  y0   y0  1 thì y0  1   y0  1  y0  1

Cách 2: Chứng minh không xảy ra đan hàm

Vì f  x   x  1, x  0 nên ta có thể giả sử tồn tại hai số a  0, a  1; b  0, b  1 sao cho

f  a   a  1; f  b   b  1

Khi đó, cho x  a; y  b ta được

f  a  b    a
2 2
 2bf  a    f  b  
2

2 2
  a  b   1  a 2  2b  a  1   b  1
 4ab  4b  0

 b  0  a  1 mâu thuẫn giả thiết.


Vậy f  x   x  1, x; f  x    x  1, x . Kết hợp với f  x   x  1, x  0 ta suy ra
f  x   x  1, x .

Thử lại ta thấy f  x   x  1, x

Các bài tương tự

Bài 1: (VMO - 2005) Tìm tất cả hàm số f :    thỏa điều kiện

f  f  x  y    f  y  f  x   f  x   f  y   xy, x, y   (1)

Bài 2: (VMO – 2002) Tìm tất cả các hàm f :    thỏa mãn điều kiện

f  y  f  x    f  x 2002  y   2001 yf  x  , x, y  

Bài 3: (Iran - 1999) Tìm tất cả hàm số f :    thỏa điều kiện

f  f  x   y   f  x 2  y   4 yf  x  , x, y   . (1)

Bài 4: (VMO 2013) Tìm tất cả hàm số f :    thỏa điều kiện

 x  y   f  f 2  x    f  f 2  y      f  x   f  y    f 2  x   f 2  y   , x, y  
PHƯƠNG TRÌNH HÀM ĐA THỨC MỘT BIẾN
Nguyễn Chí Trung

Email: nguyenchitrung12@gmail.com

Giáo viên trường THPT Chuyên Trần Đại Nghĩa

Bài viết giới thiệu một số bài toán phương trình hàm đa thức một
biến và một số hướng tiếp cận để giải các bài toán đó. Để giải các
bài toán dạng này thì ngoài việc cần nắm được các phương pháp
giải phương trình hàm thông thường, ta phải hiểu rõ các đặc điểm
và tính chất riêng của đa thức.

Chắc hẳn bài viết còn nhiều hạn chế nên tác giả rất mong nhận
được nhiều ý kiến đóng góp của quí thầy cô để bài viết được hoàn
thiện hơn. Tác giả xin chân thành cám ơn.

I.MỘT SỐ HƯỚNG TIẾP CẬN ĐỂ GIẢI BÀI TOÁN PHƯƠNG TRÌNH HÀM ĐA
THỨC MỘT BIẾN.
 Chú ý bậc và hệ số của đa thức.

Khi giải bài toán phương trình hàm đa thức thì bậc và hệ số của đa thức là một trong những
điều quan trọng nhất mà ta cần lưu ý.

Bài 1. Tìm đa thức P   x  thỏa mãn

 
P x 2  2 x 2  10 x  2 xP  x  1  3 1
Hướng dẫn giải:

Rõ ràng P không thể là đa thức hằng vì nếu P là đa thức hằng thì vế trái 1 là đa thức bậc 2
còn vế phải 1 lại là đa thức có bậc không quá 1.

Xét deg P  n  1 .

Nếu n  2 thì 1 không thể xảy ra vì vế trái 1 là đa thức có bậc là 2n còn vế phải 1 là
đa thức có bậc n  1 ,bé hơn 2n .

Do đó n  1 . Suy ra P  x   ax  b  a, b  , a  0  .

Khi đó, ta có

Trang 1
1   ax  b   2 x  10 x  2 x  a  x  1  b   3
2 2

  a  2  x  10 x  b  2ax  2  a  b  x  3
2 2

 a  2  2a
 a  2
 10  2  a  b   
 b  3
b  3

Vậy có duy nhất một đa thức thỏa mãn yêu cầu bài toán là P  x   2 x  3 .

Bài 2. [Azerbaijan 2015] Tìm đa thức P   x  thỏa mãn

  
P P  x   x 2  x  1 P  x  , x   1
Hướng dẫn giải:

Rõ ràng P  x   0 thỏa mãn 1 .

Xét deg P  n  0 . So sánh bậc hai vế của 1 ta được n2  n  2 . Suy ra n  2 .

Do đó đa thức cần tìm có dạng P  x   ax 2  bx  c  a, b, c  , a  0  . Khi đó

1  aP  x   bP  x   c   x
2

2
 x  1 ax 2  bx  c , x  
 a  ax  bx  c   b  ax  bx  c   c   x  
2
2 2 2
 x  1 ax 2  bx  c , x 

Tới đây, tương tự như bài trước, ta có thể biến đổi tiếp và tìm được a, b, c . Tuy nhiên ta có
thể tìm a, b, c theo cách khác, ngắn hơn, như sau:

   
Từ 1 ta suy ra được P P  x  và x 2  x  1 P  x  là hai đa thức bằng nhau. Tức là ta có

  
P P  x   x 2  x  1 P  x  , x    2
Từ  2  nếu thay x  x0 , với x0  là một nghiệm của đa thức P , ta sẽ thu được P  0   0 .
Từ đó suy ra c  0 .

Do đó P  x   ax 2  bx  a, b  , a  0  . Khi đó

Trang 2
1  aP  x   bP  x    x  x  1 P  x 
2 2

 aP  x   b  x  x  1  Do P  x   0 
2

 
 a ax 2  bx  b  x 2  x  1
a2  1
 a  1
 ab  1  
b  1 b  1

Do đó P  x   x 2  x .

Vậy có hai đa thức thỏa mãn yêu cầu bài toán là P  x   0; P  x   x 2  x .

Bằng cách sử dụng một tính chất quen thuộc về nghiệm của đa thức (các tính chất về nghiệm
của đa thức sẽ được khai thác nhiều hơn trong các bài sau) ta đã chứng minh được c  0 và
điều này giúp ta dễ dàng xác định được đa thức P .

Bài 3. Tìm đa thức P   


 x  thỏa mãn P P  x   x  P  x  P  x  1 .

Hướng dẫn giải:

Tương tự hai bài trước, ta tìm được các đa thức thỏa mãn yêu cầu bài toán là:

P  x   0; P  x   1; P  x   x 2  bx  c  b, c  .

Bài 4. Tìm các đa thức P, Q   x  thỏa mãn P Q  x   P  x  Q  x   1 .
Hướng dẫn giải:

Xét TH1: P là đa thức hằng.

Đặt P  x   c ( c là hằng số thực ).

c  0
Khi đó 1  c  cQ  x   
Q  x   1

Do đó, trong trường hợp này, ta tìm được các đa thức thỏa mãn yêu cầu bài toán là:

P là đa thức không và Q là đa thức có hệ số thực tùy ý hoặc P  x   c và Q  x   1 ( c là


hằng số thực khác 0).

Xét TH2: P không là đa thức hằng và Q là đa thức hằng.

Đặt Q  x   d ( d là hằng số thực). Khi đó 1  P  d   P  x  .d

Trang 3
Nếu d  0 thì 1  P  0   0 .

P d 
Nếu d  0 thì 1  P  x   (loại vì P không là đa thức hằng)
d

Do đó, trong trường hợp này, ta tìm được các đa thức thỏa mãn yêu cầu bài toán là:

P  x   xH  x  và Q  x   0 , trong đó H là đa thức có hệ số thực tùy ý khác đa thức không.

Xét TH3: P và Q không là đa thức hằng, tức là P và Q có bậc lớn hơn hoặc bằng 1.

Ta chứng minh được deg P  deg Q  2 . Suy ra

P  x   ax2  bx  c, Q  x   dx 2  ex  f  a, b, c, d , e, f  ; a, d  0  .

So sánh hệ số cao nhất của hai vế của (1) ta suy ra d  1 .

Gọi x0  là một nghiệm của đa thức Q .Từ 1 thay x  x0 ta được P  0   0 . Do đó c  0


.

Lúc này ta được P  x   ax 2  bx , Q  x   x 2  ex  f . Do đó

1  Q  x   aQ  x   b    ax 2  bx  Q  x   aQ  x   b  ax 2  bx
 b
 e
ea  b  a
 eax  af  b  bx   
af  b  0 f b

 a

Như vậy, trong trường hợp này, ta tìm được các đa thức thỏa mãn yêu cầu bài toán là:

b b
P  x   ax 2  bx và Q  x   x 2  x  a, b  , a  0
a a

Tóm lại, tất cả các đa thức P và Q thỏa mãn yêu cầu bài toán là:

P là đa thức không và Q là đa thức có hệ số thực tùy ý;

P  x   c và Q  x   1 ( c là hằng số thực khác 0);

P  x   xH  x  và Q  x   0 ( H là đa thức có hệ số thực tùy ý khác đa thức không);

b b
P  x   ax 2  bx và Q  x   x 2  x ( a, b là các hằng số thực, a  0 ).
a a

Trang 4
Ngoài việc phải xét các trường hợp nhỏ một cách cẩn thận thì trong bài toán này ta cần phải
lưu ý rằng ở trường hợp 3 để xác định bậc của hai đa thức P và Q ta phải giải được phương
trình nghiệm nguyên pq  p  q , trong đó p  deg P  1, q  deg Q  1 .

Bài 5. [Iran 2006] Tìm tất cả đa thức P   x  thỏa mãn

  
P x  P  x  P x  P P x  1
Hướng dẫn giải:

Xét TH1: P  x   ax  b  a, b  .
1  a  x   ax  b    b  ax  b  a  ax  b   b
 a  a  1 x  ab  b  a  a  1 x  ab  2b
b0

Như vậy, trong trường hợp này, ta tìm được các đa thức thỏa mãn yêu cầu bài toán là
P  x   ax , trong đó a là hằng số thực tùy ý (nếu a  0 thì P là đa thức không).

  
Xét TH2: deg P  n  2 . Ta có 1  P x  P  x   P P  x   P  x   2
Dễ thấy  2  không thể xảy ra vì vế trái của  2  là đa thức có bậc là n  n  1  1 và vế phải
của  2  là đa thức có bậc là n mà n  n  1  1  n  1  n , n  2 .

Vậy tất cả đa thức cần tìm là P  x   ax , trong đó a là hằng số thực.

Bài 6. [Greece 2016] Tìm tất cả các đa thức đơn khởi P, Q không là đa thức hằng, có hệ số
  x  12    x  12 
thực thỏa mãn P 1  1 và 2P  x   Q   Q  , x  1 .
 2   2 
   

Hướng dẫn giải:

Đặt deg P  n và degQ  m . Do Q là đa thức đơn khởi nên Q có dạng


Q  x   x m  qm1 x m1  ...  q1 x  q0 qi  , i  0,  m  1 .
Khi đó

Trang 5
  x  12    x  12 
Q  Q 
 2   2 
   
m
  x  12    x  12 
m
 m 1 m 1

         x  12  x  12 
2 2
 x  1 x  1 
    q
m 1 
     ...  q1   
 2   2   2   2    2 2 
          


m m
  x  12    x  12 
   
 2   2 
   
  x  12  x  12     x  12    x  12  
m 1 m 2 m 1
  x  12    x  12 
  
   

   .   ...  
 2 2  2   2   2   2  
         
 m 1 m 2
  x  12  
m 1

     x  12    x  12 
2
 x  1
 2x      .   ...    
   2   2   2  
 2        

  x  12    x  12 
nên ta suy ra hạng tử có bậc cao nhất của Q   Q  là m .x 2 m 1 .
 2   2  2m 2
   

Mặt khác do P là đa thức đơn khởi nên hạng tử có bậc cao nhất của 2P  x  là 2 x n .

Từ đó ta suy ra n  2m  1 và m  2m1 . Từ m  2m1 ta suy ra m  1 hoặc m  2 .

Với m  1 ta suy ra n  1 . Suy ra P  x   x  p0 , Q  x   x  q0  p0 , q0  .


Do P 1  1 , ta suy ra P  x   x .

Dễ dàng kiểm tra được P  x   x và Q  x   x  q0  q0   thỏa mãn yêu cầu bài toán.
Với m  2 ta suy ra n  3 .

Từ 1 lần lượt thay x  0, 1,  1 ta được

P  0   0,2P 1  Q 1  Q  0  ,2P  1  Q  0   Q 1 .

Suy ra P  1  1 .

Trang 6
Đặt H  x   P  x   x 3 .

Giả sử H  x   0 . Suy ra H là đa thức có bậc không quá 2  * .


Mặt khác từ P  0   0, P 1  1, P  1  1 ta suy ra H  0   H 1  H  1  0 , tức là H có
ba nghiệm phân biệt (mâu thuẫn với  * ).

Do đó H  x   0 , tức là P  x   x 3 .

Kết hợp với Q  x   x 2  q1 x  q0 ta có

2 2
  x  12    x  12    x  12  x  12 
1  2 x   2    2   q1  2  2 
3

     
 
 2 x 3  2 x x 2  1  2q1 x  q1  1

Suy ra các đa thức thỏa mãn yêu cầu bài toán trong trường hợp này là P  x   x 3 và
Q  x   x 2  x  q0 , với q0 là hằng số thực.

Vậy tất cả đa thức cần tìm là:

P  x   x và Q  x   x  q0 ; P  x   x 3 và Q  x   x 2  x  q0 , trong đó q0 là hằng số thực.

Qua các bài toán trên ta đã thấy được vai trò quan trọng của việc so sánh bậc và hệ số cao
nhất của hai đa thức. Tiếp theo, ta sẽ tìm hiểu một số bài toán mà ngoài hạng tử có bậc cao
nhất, ta cần xét đến các hạng tử khác của đa thức.

 
 x  thỏa mãn P x 2  P  x  .
2
Bài 7. Tìm tất cả đa thức P 

Hướng dẫn giải:

Xét TH1: P là đa thức hằng.

Đặt P  x   c , c là hằng số thực. Khi đó

c  0
P  x2   P  x   c  c2  
2

c  1

Vậy, trong trường hợp này, các đa thức thỏa mãn yêu cầu bài toán là:

P  x   0; P  x   1 .

Trang 7
Xét TH2: deg P  n  1 .


Khi đó P  x   an x n  an1 x n1  ...  a1 x  a0 ai  , i  0, n , an  0 . 

Giả sử tồn tại i  0, n  1 sao cho ai  0 . Gọi k  max i  0, n  1 ai  0 . 
Khi đó P  x   an x n  ak x k  ...  a0 . Suy ra

 
P x 2  an x 2 n  ak x 2 k  ...  a0 và


P  x   an x n  ak x k  ...  a0 
2 2
 an2 x 2 n  2an ak x nk  ...  a02

Vì 2n  n  k  2k nên ta suy ra P x 2  P  x  . 
2

Như vậy, ta phải có ai  0, i  0, n  1 . Suy ra P  x   an x n  an  0  . Khi đó

 
P x 2  P  x   an x 2 n  an2 x 2 n  an  an2  an  1 .
2

Do đó, trong trường hợp này, các đa thức thỏa mãn yêu cầu bài toán là P  x   x n .

Vậy tất cả đa thức cần tìm là P  x   0, P  x   1, P  x   x n n   


.
Trong trường hơp 2, ta có thể trình bày lại lời giải theo cách khác như sau:

Gọi hệ số cao nhất của P  x  là an  an  \ 0 .

   
2
So sánh hệ số cao nhất của hai đa thức P x 2 và P x ta suy ra an  an2 hay an  1 .

Xét đa thức G thỏa mãn G  x   P  x   x n .

Giả sử đa thức G khác đa thức không.

Đặt deg G  x   m , ta suy ra m  , m  n  1 . Mặt khác ta có

P  x2   P  x   x2 n  G  x 2    x n  G  x    G  x 2   G  x    2G  x  x n *
2 2 2

So sánh bậc của hai vế của  * ta suy ra

2m  m  n  m  n (mâu thuẫn với điều kiện m  n  1 ).

Trang 8
Do đó đa thức G phải là đa thức không hay P  x   x n .

Thử lại ta thấy với mọi n  


, đa thức P  x   x n thỏa mãn yêu cầu bài toán.

Vậy tất cả đa thức thỏa mãn yêu cầu bài toán là:

P  x   0; P  x   1; P  x   x n  n  
.
Bài 8. [Chọn đội tuyển HCM 2014 - 2015] Tìm tất cả đa thức P   x  (không đồng nhất


không) sao cho P  x    
n
 P x n , x  và n là số nguyên lớn hơn 1.

Hướng dẫn giải:

Tương tự bài trước, ta tìm được các đa thức thỏa mãn yêu cầu bài toán là:

P  x   x m , với m  , nếu n chẵn và P  x    x m , với m  , nếu n lẻ.

   
 x  thỏa mãn P  x   P x 2  2 x 4 1
2
Bài 9. Tìm tất cả đa thức P 

Hướng dẫn giải:

Dễ thấy đa thức P phải có bậc không bé hơn 2. Do đó, P có dạng P  x   ax n  Q  x  , trong


đó a  0, n  2 và Q   x  hoặc là đa thức có bậc bé hơn n hoặc là đa thức không.

1   ax  
 Q  x   ax 2 n  Q x 2    2x
2
n 4

 
 a2  a x 2 n  2ax nQ  x   Q 2  x   Q x 2  2 x 4   2
So sánh bậc của hai vế của  2  ta thu được các kết quả:

Nếu a  1 thì 2n  4 , tức là n  2 .

Nếu a  1 thì n  m  4 , với m  deg Q  0 .

Lần lượt xét n  2, n  3, n  4 ta tìm được tất cả đa thức thỏa mãn yêu cầu bài toán là:

P  x   2 x 2 , P  x    x 2 , P  x   x 3  x, P  x   x 4  1 .

Bài 10. [Colombia 2015] Tìm tất cả đa thức P   x  thỏa mãn

   
P P  x   P x n  P  x   1 , trong đó n là bậc của đa thức P 1
Hướng dẫn giải:

Trang 9
Xét các trường hợp n  0, n  1 ta tìm được hai đa thức thỏa mãn yêu cầu bài toán là:

P  x   1; P  x   2 x  1 .

Xét n  2 .

Giả sử đa thức P chỉ có một hạng tử có hệ số khác 0, tức là P  x   an x n  an  , an  0  .

1  a  a x     
n n 2

n n
n
 an x n  an x n  1  ann1  an x n  an x n  1  0 (vô lý)

Do đó đa thức P phải có ít nhất hai hạng tử có hệ số khác 0, tức là đa thức P có dạng

P  x   an x n  am x m  G  x  , trong đó n, m  , m  n và an , am  \ 0 , G   x  hoặc là


đa thức có bậc nhỏ hơn m hoặc là đa thức không. Ta có

       G  P  x 
P P  x   an P  x   am P  x 
n m

 a  a x  a x  G  x   a  P  x  
 G Px 
n m
n m
n n m m

 
P x n  P  x   1  an x n      
 G x n  an x n  am x m  G  x   1
n m
 am x n

So sánh hạng tử có bậc cao thứ hai của hai vế của 1 ta thu được các kết quả sau:

Nếu m  1 thì nann am x    am x n.m (vô lí vì n  n  1  m  n.m )


n n 1  m

Nếu m  1 và am  an  0 thì nann am x     am  an  x n (vô lí vì n  n  1  1  n )


n n 1 1

Nếu m  1 và am  an  0 thì nann am x    am x (vô lí vì n  n  1  1  1 )


n n 1 1

Nếu m  0 thì nann am x    an x n . Suy ra n  2 . Khi đó P  x   a2 x 2  a0 a2 , a0 


n n 1
 \ 0 
Thử lại, ta thấy mọi đa thức có dạng P  x   a2 x 2  a0 a2 , a0   
\ 0 không thỏa mãn yêu
cầu bài toán.

Vậy tất cả đa thức thỏa mãn yêu cầu bài toán là P  x   1; P  x   2 x  1 .

1
Trong đề bài toán này, nếu ta thay đổi 1 thành P P  x   P x n  P  x       4
thì các đa

1 1 1
thức thỏa mãn yêu cầu bài toán là P  x   ; P  x   2 x  ; P  x   x 2  .
4 4 2

Trang 10
 Chú ý nghiệm của đa thức.

Bên cạnh bậc và hệ số của đa thức thì nghiệm của đa thức cùng các tính chất liên quan được
khai thác rất nhiều trong các bài toán về phương trình hàm đa thức.

Bài 11. Tìm tất cả đa thức P   x  thỏa mãn P  x   P  x  1 .

Hướng dẫn giải:

Cách 1: So sánh bậc và hệ số của đa thức.

Dễ thấy mọi đa thức P  x   c , với c là hằng số thực thỏa mãn yêu cầu bài toán.

Xét P  x   an x n  an1 x n1  ...  a1 x  a0 n   



, ai  , i  0, n , an  0 .

       ...  a1  x  1  a0
n n 1
Khi đó P x  1  an x  1  an1 x  1

So sánh hệ số của lũy thừa bậc n  1 của hai đa thức P  x  và P  x  1 , ta suy ra

an1  nan  an1 mà điều này không thể xảy ra vì nan  0 .

Vậy tất cả đa thức thỏa mãn yêu cầu bài toán là P  x   c , với c  .

Cách 2: Sử dụng tính chất về nghiệm của đa thức.

Đặt P  0   c . Từ giả thiết ta suy ra P  k   P  0   c, k  .

Đặt G  x   P  x   c . Ta suy ra G  k   0, k  . Như vậy đa thức G có vô số nghiệm.


Suy ra G là đa thức không. Do đó P  x   c .

Thử lại, ta thấy tất cả đa thức thỏa mãn yêu cầu bài toán là P  x   c , với c  .

Bài 12. [Greece 2014] Tìm tất cả đa thức P   x  thỏa mãn

x 2
  
 6 x  8 P  x   x 2  2 x P  x  2  , x  1
Hướng dẫn giải:

x 2
  
 6 x  8 P  x   x 2  2 x P  x  2    x  2  x  4  P  x   x  x  2  P  x  2 

Từ 1 lần lượt thay x  2, x  4, x  2 ta suy ra được P  0   P  2   P  2   0 .

Do đó đa thức P viết được dưới dạng P  x   x  x  2  x  2  Q  x  với Q   x  . Khi đó

Trang 11
1   x  2 x  4 x  x  2 x  2 Q  x   x  x  2 x  2 x  4 xQ  x  2 , x  2
Từ  2  ta suy ra  x  2  Q  x   xQ  x  2  , x   3 .
Từ  3 thay x  0 ta được Q  0   0 . Do đó đa thức Q có thể viết được dưới dạng
Q  x   xR  x  với R   x  . Khi đó

3   x  2 xR  x   x  x  2 R  x  2 , x   4
Từ  4  ta suy ra R  x   R  x  2  và làm tương tự Bài 11, ta suy ra được R  x   c , với c là
hằng số thực.

Từ đó, ta suy ra tất cả đa thức thỏa mãn yêu cầu bài toán là P  x   cx 2  x  2  x  2  , c  .

Bài 13. Cho a, b  


. Tìm tất cả đa thức P   x  không là hằng số thỏa mãn

xP  x  a    x  b  P  x  1 .
Hướng dẫn giải:


Đặt deg P  n  1 và P  x   an x n  an1 x n1  ...  a1 x  a0 ai  , i  0, n , an  0 . 
So sánh hệ số lũy thừa bậc n của hai vế của 1 , ta suy ra b  na .

Từ 1 lần lượt thay x  0, x  a,..., x   n  1 a ta suy ra


P  0   P  a   ...  P  n  1 a  0 . 
Đa thức P có bậc n và có n nghiệm là 0, a,...,  n  1 a nên P có dạng


P  x   cx  x  a  ... x   n  1 a  c   .
 
Dễ thấy với b  na , đa thức P  x   cx  x  a  ... x   n  1 a thỏa mãn yêu cầu bài toán.

Kết luận:

b 
Nếu  thì không tồn tại đa thức thỏa mãn yêu cầu bài toán.
a

b
Nếu
a
 n 
thì tất cả đa thức cần tìm là P  x   cx  x  a  ... x   n  1 a  c   .

Trang 12
Tiếp theo, ta sẽ xét các bài toán phương trình hàm đa thức có dạng sau:

    
P F  x P G x  P H  x Qx , 
trong đó F, G, H , Q   x  và deg F  deg G  deg H .

Một số kết quả liên quan đến nghiệm của bài toán tổng quát này đã được TS. Trần Nam Dũng
trình bày trong chuyên đề “Về một dạng phương trình hàm đa thức”. Tuy đã được nhắc đến
từ rất lâu nhưng đến nay dạng bài này vẫn thường xuyên xuất hiện trong các đề thi học sinh
giỏi.

Trong phần trước ta đã gặp hai bài toán có dạng này là Bài 7 và Bài 9. Bây giở, ta sẽ tiếp tục
tìm hiểu thêm một số bài toán với nhiều hướng giải khác.

Bài 14. Tìm tất cả đa thức P   x  thỏa mãn P  x  P  x  1  P  x 2  1 .

Hướng dẫn giải:

Xét TH1: P là đa thức hằng.

Trong trường hợp này ta dễ dàng tìm được các đa thức thỏa mãn yêu cầu bài toán là:

P  x   0; P  x   1 .

Xét TH2: P không là đa thức hằng.

Gọi z0  là một nghiệm của đa thức P  x  .

Từ 1 ta suy ra z02 cũng là nghiệm của đa thức P  x  và tương tự ta có z0 , z02 , z04 ,..., z02 ,...
k

cũng là nghiệm của đa thức P  x  .

Nếu z0  1 thì đa thức P  x  có vô số nghiệm (vì z0  z02  ...  z02  ... )


k

Nếu 0  z0  1 thì đa thức P  x  có vô số nghiệm (vì z0  z02  ...  z02  ... )


k

Do đó z0  0;1 .

Mặt khác từ 1 ta cũng suy ra  z0  1 cũng là nghiệm của đa thức P  x  . Suy ra
2

z0  1 0;1 .

Trang 13
Giả sử z0  0;1 . Khi đó z0  1 và z0  1  1 . Đặt z0  cos   i sin     0; 2   .

1   5   1 3 1 3 
z0  1  1   cos  1  sin 2   1  cos      ;   z0    i; 
2
i
2 3 3   2 2 2 2 


1 3 1 3  
 1 3 1 3 

Mà nếu z0    i;  i  thì z02    i;   i .
2 2 2 2 
   2 2
 2 2 

Suy ra z02  1  3  0;1 (!)

Do đó z0  0;1 .

Như vậy, ta đã chứng minh được nếu z0  là nghiệm của đa thức P  x  thì z0  0;1 .

Suy ra P  x   cx k  x  1
m
 c  \ 0 , k , m  , k 2  m2  0  .

c  1
Khi đó 1  cx k  x  1 .c.  x  1 x m  c.x 2 k  x 2  1
2m

m k
.
k  m

Vậy tất cả đa thức cần tìm là P  x   0; P  x   1; P  x   x m  x  1


m
m   
.

Bài 15. Tìm tất cả đa thức P   x  thỏa mãn P  x  P  x  1  P  x 2  1 .

Hướng dẫn giải:

Xét TH1: P là đa thức hằng.

Trong trường hợp này, ta tìm được các đa thức thỏa mãn yêu cầu bài toán là:

P  x   0; P  x   1 .

Xét TH2: P không là đa thức hằng.

Giả sử đa thức P  x  có nghiệm thực x0 .

Từ (1) suy ra đa thức P  x  có nghiệm thực x1   x0  1 .


2

Đặt xn   xn1  1 n  
2
.

Với mọi n  , ta chứng minh được xn là nghiệm của đa thức P .


Mà xn  xn1  xn21  xn1  1  0 n  nên ta suy ra đa thức P có vô số nghiệm (vô lý)

Vậy đa thức P không có nghiệm thực. Suy ra đa thức P có bậc chẵn.

Trang 14
Đặt deg P  2k  k   và Q  x   P  x    x  x  1 .
 2 k

Mặt khác, từ (1) ta suy ra hệ số cao nhất của P là 1.

Do đó hoặc là Q  x   0 hoặc là deg Q  m  2k  1 .

Giả sử Q không là đa thức không, tức là đa thức Q có bậc m  2k  1 .

1  Q  x    x  x  1  Q  x 1   x  x  1   Q  x    x  x  1
2 k 2 k 2 4 2 k

 Q  x  Q  x  1  Q  x   x 2  x  1   x 2  x  1 Q  x  1  Q  x 2   2
k k

So sánh bậc của 2 vế của (2) ta được m  2k  2m  m  2k (mâu thuẫn với m  2k  1 )

Do đó ta suy ra Q phải là đa thức không, tức là P  x    x 2  x  1 k   .


k 

, đa thức P  x    x 2  x  1 thỏa mãn yêu cầu bài toán.


 k
Thử lại, ta thấy với mọi k 

Vậy tất cả đa thức cần tìm là P  x   0; P  x   1; P  x    x 2  x  1  k  .


k 

Bài 16. Tìm tất cả đa thức P  


 x  thỏa mãn P  x  P  x  1  P x 2  x  1  1 .
Hướng dẫn giải:

Xét TH1: P là đa thức hằng.

Trong trường hợp này, ta tìm được các đa thức thỏa mãn yêu cầu bài toán là:

P  x   0; P  x   1 .

Xét TH2: P không là đa thức hằng.

Giả sử z0  là một nghiệm của đa thức P . Khi đó từ 1 lần lượt thay x  z0 , x  z0  1 ta

   
suy ra P z02  z0  1  P z02  z0  1  0 , tức là z1  z02  z0  1, z2  z02  z0  1 cũng là nghiệm
của đa thức P .

Chọn z0 là nghiệm phức có môđun lớn nhất của đa thức P , ta suy ra z0  z1 , z0  z2 .

Mà z1  z2  2z0 nên ta có 2 z0  z1  z2  z1  z2  2 z0 . Suy ra

z1  z2  z1  z2 và z1  z2  z0

Từ đây ta suy ra được z1  z2 hay z02  1  0 . Vậy z0 i; i  2 .

Trang 15
Vì P   x  nên từ  2  ta suy ra P có hai nghiệm là i và i . Do đó P viết được dưới

 
dạng P  x   x 2  1 Q  x  , trong đó m 
m

và Q   x  thỏa mãn Q  i  , Q  i   0 .

Khi đó

1  Q  x  Q  x  1  Q  x 2
 x 1   3
Vì Q  i  , Q  i   0 nên từ  3  ta suy ra được Q phải là đa thức hằng và cũng do Q  i   0
nên Q  x   c , với c  \ 0 . Khi đó  3  c2  c  c  1 .

Suy ra các đa thức thỏa mãn yêu cầu bài toán trong TH2 là P  x   x 2  1   m   .
m

Vậy tất cả đa thức thỏa mãn yêu cầu bài toán là P  x   0, P  x   1, P  x   x 2  1 , m   


m

.

Bài 17. Tìm tất cả đa thức P   x  thỏa mãn P  x  P  2 x 2   P  2 x3  x 2  1 .


Hướng dẫn giải:

Xét TH1: P là đa thức hằng.

Trong trường hợp này, ta tìm được các đa thức thỏa mãn yêu cầu bài toán là:

P  x   0; P  x   1 .

Xét TH2: P không là đa thức hằng.

Từ 1 ta suy ra P  0  0;1 .

Nếu P  0   0 thì tồn tại đa thức G   x và k  


thỏa mãn

P  x   x k G  x  và G  0   0 .

1  x k G  x   2 x 2  G  2 x 2    2 x3  x 2  G  2 x3  x 2 
k k

 2k x k G  x  G  2 x 2    2 x  1 G  2 x3  x 2   2
k

Từ  2  ta suy ra G  0   0 (mâu thuẫn với G  0   0 ).

Nếu P  0   1 thì tồn tại đa thức H   x và k  


thỏa mãn

P  x   x k H  x   1 và H  0   0 .

Trang 16
1   x k H  x   1  2x  H  2x   1   2x  x  H  2x  x   1
2 k 2 3 2 k 3 2

 2k x3k H  x  H  2 x 2   x k H  x   2k x 2 k H  2 x 2   x 2 k  2 x  1 H  2 x 3  x 2 
k

 2k x 2 k H  x  H  2 x 2   H  x  2k x k H  2 x 2   x k  2 x  1 H  2 x 3  x 2   3
k

Từ  3 ta suy ra H  0   0 (mẫu thuẫn với H  0   0 )

Như vậy, trong trường hợp này, ta không tìm được đa thức nào thỏa mãn yêu cầu bài toán.

Tóm lại, tất cả đa thức đa thức thỏa mãn yêu cầu bài toán là P  x   0; P  x   1.

Trong TH2, ta cũng có thể giải bằng cách so sánh các hạng tử tương ứng như sau:

Gọi hạng tử có bậc thấp nhất của P là ax p  a  0, p  .


So sánh hạng tử có bậc thấp nhất của hai vế của 1 ta suy ra a2 2 p x 3 p  ax 2 p , do đó
p  0, a  1 .

Đặt Q  x   P  x   1 . Gọi hạng tử có bậc thấp nhất của Q là bx q b  0, q   


.
So sánh tổng hai hạng tử có bậc thấp nhất của hai vế của 1 ta suy ra bx q  1  bx 2 q  1 *
Mà b  0, q  0 nên  * không thể xảy ra.

Vậy không tồn tại đa thức khác hằng nào thỏa mãn yêu cầu bài toán.

Bài 18. Tìm tất cả đa thức P   x  thỏa mãn

P  x  P  2 x 2   P  x3  x  1
Hướng dẫn giải:

Xét TH1: P là đa thức hằng.

Trong trường hợp này, ta tìm được các đa thức thỏa mãn yêu cầu bài toán là:

P  x   0; P  x   1 .

Xét TH2: P không là đa thức hằng.

Đặt deg P  n  n  
 . Từ 1 ta suy ra P  0 0;1 .
Tương tự bài trước ta chứng minh được P  0   0 , suy ra P  0   1 .

Trang 17
Từ 1 ta cũng suy ra được hệ số cao nhất của đa thức P là
1
.
2n

Gọi x1 , x2 ,..., xn là n nghiệm phức của đa thức P .

Theo định lý Viét ta có x1 x2 ...xn  2n . Suy ra tồn tại k 1, 2,..., n thỏa mãn xk  2 .

Đặt z0  xk , z1  z03  z0 , z2  z13  z1 ,..., zm  zm3 1  zm1 ,...


Do z0  2 nên ta chứng minh được zm  zm1  2 m  .

Mà từ (1) ta suy ra zm là nghiệm của P  x  với mọi m nên đa thức P  x  có vô số


nghiệm (vô lý). Do đó, trong trường hợp này, ta không tìm được đa thức nào thỏa mãn yêu
cầu bài toán.

Vậy tất cả đa thức đa thức thỏa mãn yêu cầu bài toán là P  x   0; P  x   1.

 
 x  thỏa mãn 2P 2 x 2  1  P  x   1 1 .
2
Bài 19. Tìm tất cả đa thức P 

Hướng dẫn giải:

Đặt P 1  c .

Từ 1 thay x  1 ta suy ra 2c  c2  1 hay c  1  2,1  2 .  


  
  
k
Ta có thể viết lại P dưới dạng P x  x  1 Q x  c , trong đó k  
và Q   x  hoặc

là đa thức không hoặc là đa thức thỏa mãn Q 1  0 .

 x  1 Q  x   c 
2

1  2   2 x   
 2 Q 2 x 2  1  c  
k k
2
1

 
 2k 1  x  1  x  1 Q 2 x 2  1   x  1 Q  x   2c  x  1 Q  x 
k k 2k 2 k

 2k 1  x  1 Q 2 x 2   1   x  1 Q  x    2cQ  x   2 
k k 2

Từ  2  thay x  1 ta suy ra 22 k 1Q 1  2cQ 1 hay Q 1  0 .

Do đó Q  x   0 , tức là P  x   c .

Dễ thấy P  x   1  2 , P  x   1  2 là các đa thức thỏa mãn yêu cầu bài toán.

Trang 18
Bài 20. Tìm tất cả đa thức P   x  thỏa mãn P  x  P   x   P x 2   1 .
Hướng dẫn giải:

Dễ thấy có tất cả hai đa thức hằng thỏa mãn yêu cầu bài toán P  x   0, P  x   1 .

Xét P không là đa thức hằng.


k
Giả sử P có nghiệm phức w khác 0. Suy ra w2 , w4 ,..., w2 ,... cũng là nghiệm của P . Mà P
m m
chỉ có hữu hạn nghiệm nên phải tồn tại nghiệm w1  0 sao cho w12  w1 hay w12 1
 1 với m
m1
là số nguyên dương nào đó. Chọn m1 là số nguyên dương nhỏ nhất sao cho w12  w1 . Khi đó
m11
w1 , w12 ,..., w12 là các nghiệm phân biệt của P .

Suy ra ta có thể viết P dưới dạng P  x   Q1  x  P1  x  , trong đó P1   x  khác đa thức

không và Q1  x    1  x  w   x  w  ... x  w  với w


m1 2 2
m 11 m1

1 1 1
2
1
1
 1.

Khi đó

1   1  x  w  ...  x  w


m1 2
1
2 2
1
m11

 P  x  P   x    1  x  w  ... x  w  P  x 
1 1
m1 2
1
2 2
1
m11

1
2

 P  x  P x   P  x 
1 1 1
2

Như vậy đa thức P1 có tính chất P1  x  P1   x   P1 x 2 (giống như đa thức P ) nhưng có bậc  
bé hơn P . Lặp lại các bước suy luận trên ta suy ra P có dạng P  x   Q1  x  ...Qk  x  .Pk  x 

trong đó Qj  x    1
mj
 x  w   x  w  ... x  w
j
2
j
2
j
m j 1

 , P  x
k
 x  là đa thức khác đa

thức không và không có nghiệm phức khác 0.

Suy ra Pk  x  có dạng cx n  n  , c  0  và Pk  x  Pk   x   Pk x 2 .  
           
n n
Từ đó suy ra Pk x  1 x n , tức là P x  1 x nQ1 x ...Qk x .

       
n
Thử lại, ta thấy đa thức P x  1 x nQ1 x ...Qk x , trong đó n, k  và

Qj  x    1  x  w   x  w  ... x  w  với w


mj m j 1 mj

j
2
j
2
j
2
j
1
 1 , thỏa mãn yêu cầu bài toán.

Trang 19
Vậy tất cả đa thức thỏa mãn yêu cầu bài toán là P  x   0 , P x  1 x nQ1 x ...Qk x ,       
n

và Qj  x    1  x  w   x  w  ... x  w 
mj m j 1 mj

trong đó n, k  j
2
j
2
j
với w j  , w2j 1
 1.

Lưu ý, các đa thức với hệ số thực có dạng

P  x    1 x n  x  1 P1  x  ...Pk  x  ,
nm m

và Pj  x   1  x  x 2  ...  x
m j 1
trong đó n, m, k  , với m j là số nguyên dương lẻ thỏa
mãn yêu cầu bài toán.

 Chú ý các tính chất đặc biệt của đa thức.

Đa thức có rất nhiều đặc điểm và tính chất riêng. Việc phải vận dụng linh hoạt các tính chất
của đa thức để giải bài toán phương trình hàm đa thức là đặc điểm thú vị nhất của dạng toán
này.

Bài 21. [Iran 2015] Tìm tất cả đa thức P   x  thỏa mãn

 P  5x     1 .
2
 3  P 5x 2  1

Hướng dẫn giải:

Dễ dàng tìm được các đa thức hằng thỏa mãn yêu cầu bài toán là:

1  13 1  13
P x  ,Px  .
2 2

Xét trường hợp P không là đa thức hằng.

Trong trường hợp này, ta chứng minh được P   x  .

5 5  5 5  
1  13 1  13 
Sau đó, từ 1 thay x 
10
ta suy ra P  
 10   2
;
2
 2 .
   

1 1 
Mà P   x  nên P   5  phải có dạng a  b 5 với a, b  (mâu thuẫn với  2  )
 2 10 

Suy ra không tồn tại đa thức khác hằng nào thỏa mãn yêu cầu bài toán.

Vậy tất cả đa thức cần tìm là:

1  13 1  13
P x  ,Px  .
2 2

Trang 20
Bài 22. [Serbia EGMO TST 2015] Tìm tất cả đa thức P   x  thỏa mãn

 x  100 P  x   xP  x  1  1 , x  1 .
Hướng dẫn giải:

1
Đặt Q  x   P  x   . Khi đó Q   x  và ta có
100

1   x  100 Q  x   xQ  x  1 , x  2


Từ  2  ta tìm được Q  x   cx  x  1 ...  x  99  c   . Suy ra tất cả đa thức cần tìm là:
1
P  x   cx  x  1 ...  x  99   c  
100

Bài 23. [Putnam 2010] Tìm tất cả các cặp đa thức p  x  và q  x  có hệ số thực và thỏa mãn

p  x  q  x  1  p  x  1 q  x   1 1
Hướng dẫn giải:

Dễ thấy p, q khác đa thức không.

Từ 1 , thay x bởi x  1 ta được p  x  1 q  x   p  x  q  x  1  1 2  .


  
Từ 1 ,  2  ta suy ra p  x  q  x  1  q  x  1  q  x  p  x  1  p  x  1   3
 
Lưu ý rằng từ 1 ta cũng suy ra p  x  , q  x   1 . Do đó từ  3  ta suy ra

 p  x  1  p  x  1 p  x   4
Mà p  x  , p  x  1 , p  x  1 có cùng bậc và hệ số cao nhất nên

p  x  1  p  x  1  2 p  x   5
Đặt r  x   p  x   p  x  1 , ta suy ra r   x  và r  x  1  r  x  . Do đó r là đa thức hằng.

Tương tự, đặt s  x   q  x   q  x  1 , ta cũng suy ra được s là đa thức hằng.

Từ đó ta tìm được các đa thức thỏa mãn yêu cầu bài toán là:

p  x   ax  b, q  x   cx  d với a, b, c, d  thỏa mãn bc  ad  1 .

Trang 21
Bài 24. [Georgia TST 2005] Tìm tất cả đa thức P   x  thỏa mãn

  
P 2P  x   2P P  x   2 P  x  , x    1
2

Hướng dẫn giải:

1
Dễ thấy tất cả đa thức hằng thỏa mãn yêu cầu bài toán là P  x   0, P  x    .
2

 
Xét deg P  1 . Chú ý rằng từ 1 ta suy ra P 2P  x   2P P  x   2 P  x  , x      2
2

Mà với mọi x  đều tồn tại z  sao cho P  z   x nên từ  2  ta suy ra

P  2 x   2P  x   2 x 2 , x   3
Từ  3  ta tìm được P  x   x 2  bx  b  .
1
Thử lại, ta thấy các đa thức cần tìm là P  x   0, P  x    , P  x   x 2  bx  b  .
2

Bài 25. [Olympic chuyên KHTN 2017] Tìm tất cả đa thức P   x  sao cho

P  a  P  b  P c   P  a  b  c   2 ,
2 2 2 2

với mọi bộ số  a, b, c  thỏa mãn ab  bc  ca  1  0 .

Hướng dẫn giải:

Chú ý rằng  2 x  1 2 x  1   2 x  1  x     x  2 x  1  1  0 nên từ giả thiết ta suy ra

P  2 x  1  P  2 x  1  P   x   P  3x   2, x  1
2 2 2 2

Từ 1 ta suy ra P  x   mx  n với m, n  . Khi đó

P  a   P  b   P  c   P  a  b  c   2,   a, b, c  : ab  bc  ca  1
2 2 2 2

 
 m 2 a2  b2  c 2  3n2  m 2  a  b  c   n2  2,   a, b, c  : ab  bc  ca  1
2

 2n2  2m 2  2  m 2  n2  1

Vậy tất cả đa thức cần tìm là P  x   mx  n ,với m, n thỏa mãn m2  n2  1 .

Ta cũng có thể viết nghiệm của bài toán dưới dạng P  x   cos  x sin  ,với    0;2  .

Trang 22
II. BÀI TẬP ĐỀ NGHỊ.
Bài 1. Tìm tất cả đa thức P   x  có bậc n  
và thỏa mãn

    
P 1  P  x   P x 2  ...  P x n  1  x  x 2  ...  x n P  x  , x   .

   
 x  thỏa mãn 16P x 2  P  2 x  .
2
Bài 2. Tìm tất cả đa thức P 

Tìm tất cả đa thức P   x  thỏa mãn P  x  2 x    P  x  2   .


2
2
Bài 3.

Xác định tất cả đa thức W   x  thỏa mãn W  x  W  x   W  x  


5
Bài 4. 2 3
, x  .

Bài 5. Tìm tất cả đa thức P   x  thỏa mãn P  x  P  2 x   P  2 x  x  . 2 3

Bài 6. Tìm tất cả đa thức P   x  thỏa mãn P  x  P  2 x   P  2 x  x  . 2 3 2

Bài 7. [Olympic Toán sinh viên 2010] Tìm tất cả đa thức P   x thỏa mãn
P  x  P  x 2   P  x3  2 x  , x  .

Bài 8. [Olympic 30/4 lần thứ XX - 2014] Xác định đa thức P   x  thỏa mãn

  
P  x  P x 2  P x 3  3x , x   .

 
 x  thỏa mãn P  x   1  4P x 2  4 x  1 .  
2
Bài 9. Tìm tất cả đa thức P 

   
 x  thỏa mãn P x 2  2  P  x   2 .
2
Bài 10. Tìm tất cả đa thức P 

 x  thỏa mãn P  x  1   P  x    1 .
2
2
Bài 11. Tìm tất cả đa thức P 

 x  thỏa mãn P  x  x  1   P  x    P  x   1 .
2
2
Bài 12. Tìm tất cả đa thức P 

Bài 13. [VMO 2006] Tìm tất cả đa thức P   x  thỏa mãn

    
P x 2  x 3P  x   P   x   P  x   2 x 2 , x  
2
.

Bài 14. Tìm tất cả đa thức P   x  có hệ số thực thỏa mãn

 x  1 P  x  1   x  1 P  x  1  4  x2  1 P  x  .
3 3

Bài 15. Tìm tất cả đa thức đơn P, Q   x  thỏa mãn


P 1  P  2   ...  P  n   Q 1  2  ...  n  , n  1 .

Bài 16. Tìm tất cả đa thức P   x  thỏa mãn


P  2015  2025 và P  x   10   
2
 P x 2  3  13, x  .

Trang 23
Bài 17. [Romani 2002] Tìm tất cả đa thức f  x  , g  x  có hệ số thực thỏa mãn

x 2
 x  1 f  x 2  x  1   x 2  x  1 g  x 2  x  1 , x  .

Bài 18. [Greece TST 2014] Tìm tất cả đa thức P   x  khác đa thức không và thỏa mãn

 
P3  x   3P  x   P x 3  3P   x  , x 
2
.

Bài 19. [Baltic Way 2015] Cho số nguyên n  1 . Tìm tất cả đa thức P   x  không là đa

 n n 1 
     
thức hằng và thỏa mãn P  x  P x 2 P x 3 ...P x n  P  x 2  , x 
 
.
 
Bài 20. Tìm tất cả đa thức P     
 x  thỏa mãn P  x  P x 2  1  P x 2 P  x  1 .

Bài 21. Cho số nguyên k  2 . Tìm tất cả đa thức P   x  thỏa mãn

   
P  x  P 2 x k  1  P x k P  2 x  1 .

Bài 22. [IMO 2004] Tìm tất cả đa thức f   x  sao cho

f  x  y   f  y  z   f  z  x   2 f  x  y  z  , với mọi x, y, z thỏa mãn xy  yz  zx  0 .

Bài 23. [Costa Rica 2008] Tìm tất cả đa thức P  x    x  thỏa mãn

P   
3 a  b  P  
3 b  c  P 3 c  a 
 P  2a  b  c   P  a  2b  c   P  a  b  2c  , a, b, c 
Bài 24. [Romani TST 2013] Cho số nguyên n  2 . Tìm tất cả đa thức f   x  thỏa mãn

  
1 f xn 1  f  x  . 
n

Bài 25. Tìm tất cả đa thức P   x  chỉ có nghiệm thực và thỏa mãn:


P  x  P  x   P x2 1 . 

Trang 24
TÀI LIỆU THAM KHẢO

[1] Nguyễn Hữu Điển, Đa thức và ứng dụng, Nhà xuất bản Giáo dục, 2006.

[2] Lê Hoành Phò, Chuyên khảo Đa thức, Nhà xuất bản Đại học Quốc gia Tp.HCM, 2003.

[3] Nguyễn Tài Chung, Bồi dưỡng học sinh giỏi Phương Trình Hàm, Nhà xuất bản Đại học
Quốc gia Hà Nội, 2014.

[4] Trần Nam Dũng, Về một dạng phương trình hàm đa thức,

http://diendantoanhoc.net/forum/index.php?app=core&module=attach&section=attach&attac
h_id=20496.

[5] https://artofproblemsolving.com.

Trang 25
BẤT ĐẲNG THỨC JENSEN CHO HÀM HAI BIẾN
---------o0o---------
Nguyễn Hoàng Vinh
Giáo viên chuyên Lương Thế Vinh, Đồng Nai
I – Đặt vấn đề
Chúng ta đã biết nếu hàm số f  x  thỏa điều kiện f ''  x   0, x  K (Với K là một khoảng, đoạn nào

đó) thì với mọi bộ số  ;  không âm và     1 thì có bất đẳng thức Jensen như sau

 f  a    f  b   f  x   y  a, b  K

Vấn đề đặt ra ở đây, nếu thay hàm số đã cho thành một hàm hai biến là f  x; y  thì f  x; y  cần thỏa

điều kiện gì để
 f  x1; y1    f  x2 ; y2   f  x1   y1; x2   y2 

II – Ý tưởng
Để giải quyết vấn đề đặt ra, chúng ta đi tìm hiểu một phương pháp chứng minh bất đẳng thức Jensen
như sau
Đầu tiên, ta có khai triển Taylor cho hàm số f  x  tại điểm x0 là

1
f  x   f  x0   f '  x0  x  x0   f ''  x0  x  x  x0 
2

2
Với   0;1 , x  x  x0

Theo giả thiết, f ''  x   0, x  K nên ta có bất đẳng thức

f  x   f  x0   f '  x0  x  x0  , x, x0  K (1)

(Ta còn gọi đây là bất đẳng thức tiếp tuyến)


Bây giờ, lấy a, b  K thì  a   b  K và thay x0   a   b , thay x lần lượt là a, b vào (1) để có hai bất

đẳng thức sau


f  a   f  a   b   f '  a   b  a   a   b 

f  b   f  a   b   f '  a   b b   a   b 

Từ đó suy ra
 f  a    f  a   b    f '  a   b  a   a   b  (2)
 f  b   f  a   b    f '  a   b b   a   b  (3)
Cộng theo vế (2) và (3) với lưu ý     1 ta sẽ có

 f  a    f  b   f  x   y  a, b  K

Với cách chứng minh này, đã sử dụng giả thiết đạo hàm cấp hai của hàm số f  x  không âm trên một

khoảng nào đó. Vậy, ta sẽ cố gắng chuyển hàm số f  x; y  về hàm số một biến, từ đó xét đạo hàm cấp

hai, dùng khai triển Taylor và thực hiện các thao tác chứng minh tương tự.
III – Khai triển Taylor cho hàm hai biến
Chú ý: Các hàm số được nói ở đây đều khả vi cấp hai trên miền lồi K.
Tất nhiên, bằng kiến thức của hàm nhiều biến, ta đã biết khai triển Taylor cho hàm số f  x; y  tại điểm

M  x0 ; y0  đến cấp hai như sau

f  x; y   f  x0 ; y0   f x  x0 ; y0  x  f y  x0 ; y0  y  y0 
1
  f x2  x0  x; y0  y   2 x  2 f xy  x0  x; y0  y  xy  f y2  x0  x; y0  y   2 y 
2 

Kí hiệu: f x , f y là đạo hàm cấp một của hàm số theo biến x, y. f x2 , f y2 là đạo hàm cấp hai của hàm số

theo biến x, y và f xy là đạo hàm của hàm số f x theo y.

Ở đây, tôi sẽ đưa ra một cách tiếp cận gần gũi với phổ thông và phù hợp với ý tưởng đã nêu ở mục trên.
Cho K là một miền lồi và hai điểm A  x; y  , M  x0; y0  nằm trong K. Khi đó, mọi điểm B trên

đoạn AM có tọa độ là B  x0  t x; y0  t y  với t   0;1 . Khi đó, ta xét hàm số mới như sau

g  t   f  x0  t x; y0  t y 

Rõ ràng đây là hàm một biến và xét khai triển Taylor cho hàm số này tại t  0
1
g  t   g  0   g '  0  .t  g ''   t  t 2 ,   0;1
2
Ta biết rằng: g '  t   f x  x0  t x; y0  t y  x  f y  x0  t x; y0  t y  y và

g ''  t   f x2  x0  t x; y0  t y   2 x  2 f xy  x0  t x; y0  t y  xy  f y2  x0  t x; y0  t y   2 y

Khi đó, cho t  1 ta có đẳng thức cần chứng minh.


IV – Các bất đẳng thức nền tảng xây dựng trên khai triển Taylor
Để cho thuận lợi, ta kí hiệu f x2  f x2  x; y  , fxy  fxy  x; y , fy2  fy2  x; y .
Khi đó, xét biệt thức
  f x2  2 x  2 f xy xy  f y2  2 y

Ta nhận thấy, nếu với một điều kiện P của các điểm A  x; y  , M  x0; y0  và các giá trị

x  x  x0 , y  y  y0 sao cho AM thuộc P và   0 với mọi điểm B trên đoạn AM thì từ khai triển
trên, sẽ dẫn đến bất đẳng thức
f  x; y   f  x0 ; y0   f x  x0 ; y0  x  x0   f y  x0 ; y0  y  y0 

Ta gọi đây là bất đẳng thức tiếp diện


Bất đẳng thức 1: Cho P là một miền lồi thỏa   0, A  x; y   P khi đó, với mọi điểm

A1  x1; y1  , A2  x2; y 2  thuộc P ta luôn có

f  x1; y1   f  x2 ; y2   f x  x2 ; y2  x1  x2   f y  x2 ; y2  y1  y2 

xy 2
3 
Ví dụ 1: Ta xét hàm số f  x; y   thì    xy  yx   0 x; y  0 . Từ đó dẫn đến
2

x y  x  y
bất đẳng thức
xy

ab

1
x  y a  b  a  b 2
b2  x  a   a2  y  b  x; y; a; b  0
Ví dụ 2: Cho hàm hai biến f  x; y  thỏa   0 trên miền lồi P. Khi đó, với mọi bộ n điểm

Ai  xi ; yi   P thì
n n

 f x
i 1
x i 1 ; yi 1  xi  xi 1    f y  xi 1 ; yi 1  yi 1  yi 
i 1

Quy ước: xn1  x1 , yn1  y1

e x y
Hệ quả: Xét hàm số f  x   ln  e x  e y  thì biệt thức   y 
x  y   0 nên áp dụng bất
2

e e
x

đẳng thức trên cho bộ ba điểm A  x1; y1  , B  x2 ; y 2 ,C x3 ; y 3  có tọa độ dương, ta có

y1 
e x1  x3  x1   e y1  y3  y1    0
1
e
cyc
x1
e

Để đơn giản hơn, đặt ai  e xi , bi  e yi i  1, 2,3 . Ta đưa bất đẳng thức về dạng
a1 a b1 b a2 a b2 b a3 a b3 b
 2  2  3  3  1  1
a1 b1 a2 b2 a1 b1 a2 b2 a2 b2 a3 b3 a2 b2 a3 b3 a3 b3 a1 b1 a3 b3 a1 b1
a
1 .b
1 .a2 .b
2 .a
3 .b
3 1
Tiếp tục vấn đề, ta nghiên cứu cách mở rộng bất đẳng thức Jesen cho hàm hai biến trên nền tảng
bất đẳng thức tiếp diện. Xét hai điểm A  x1; y1  , B  x2 ; y 2  trên miền lồi P sao cho hàm số f  x; y  có biệt

thức  không âm trên miền này. Khi đó, đặt S  x1   x2 ; y1   y2    s1 ; s2   P với

    1,  ,   0 và áp dụng bất đẳng thức tiếp diện ta có


f  x1; y1   f  s1; s2   f x  s1; s2  x1  s1   f y  s1; s2  y1  s2  (1)

f  x2 ; y2   f  s1; s2   f x  s1; s2  x2  s1   f y  s1; s2  y2  s2  (2)

Nhân (1) với  , (2) với  rồi cộng lại ta có

 f  x1; y1    f  x2 ; y2   f  x1   x2 ; y1   y2 

Bất đẳng thức 2: Cho hàm số f  x; y  có biệt thức  không âm trên miền lồi P, khi đó với mọi điểm

A  x1; y1  , B  x2 ; y 2  thuộc P và mọi bộ số  ;  thỏa     1,  ,   0 ta luôn có

 f  x1; y1    f  x2 ; y2   f  x1   x2 ; y1   y2 

Ví dụ 3: Xét hàm số f  x; y   xy , khi đó biệt thức   2x.y sẽ dương khi x; y cùng dấu hay với

hai điểm A  x1; y1  , B  x2 ; y 2  thỏa x1  x2 , y1  y2 ta sẽ có

 x1 y1   x2 y2   x1   x2  y1   y2 

Với mọi bộ  ;  thỏa     1,  ,   0 .


Rõ ràng, đây là một mở rộng của bất đẳng thức Chebyshev.

Tất nhiên, ta biết rằng nếu P là một miền lồi và n điểm Ai  xi ; yi  i  1, 2,3..., n thuộc P thì điểm

 n n
 n
A    i xi ;   i yi  với  i  1,  i  0 cũng thuộc P. Từ đó, lặp lại quá trình chứng minh trên, ta có
 i 1 i 1  i 1

bất đẳng thức mở rộng như sau


Bất đẳng thức 3: Cho hàm số f  x; y  có biệt thức  không âm trên miền lồi P, khi đó với mọi bộ n
n
điểm Ai  xi ; yi  i  1,2,3..., n thuộc P và mọi bộ i , i  1, 2,3.., n thỏa 
i 1
i  1,  i  0 thì ta có

n
 n n

i f  xi ; yi   f  i xi ; i yi 
i 1  i 1 i 1 
Ví dụ 4: Ta quay trở lại hàm số f  x; y   xy và xét bộ n điểm Ai  xi ; yi  i  1,2,3..., n thỏa

x1  x2  x3  ...  xn , y1  y2  ...  yn và bộ n điểm này thỏa điều kiện P là làm cho biệt thức  không
n
âm. Khi đó, với mọi bộ n số i , i  1, 2,3.., n thỏa 
i 1
i  1,  i  0 thì ta có

1 x1 y1   2 x2 y2  ...   n xn yn  1 x1   2 x2  ...   n xn 1 y1   2 y2  ...   n yn 

Hoàn toàn tương tự, ta vẫn có vẫn có thể định nghĩa biệt thức  cho hàm nhiều biến f  x1; x2 ; x3 ;...; xn 

như sau

 
1i , j  n
f xi x j  x1 ; x2 ;...; xn xi x j

Và cách chứng minh không bị ảnh hưởng nên ta mở rộng tiếp sang hàm nhiều biến với kí hiệu cho tiện
việc trình bày là f  A  f  x1; x2 ;...; xn  A  x1; x2 ;...xn 

Bất đẳng thức 4: Cho hàm số f  x1; x2 ; x3 ;...; xn  và miền lồi P sao cho biệt thức  của hàm số này

không âm trên P. Khi đó, với mọi bộ n điểm Ai thuộc P và n số thực không âm  i có tổng là 1, ta luôn


n
 n

i f  Ai   f  i Ai 
i 1  i 1 
Ví dụ 5: Xét hàm số f  x; y; z  có biệt thức


1
 xy  yz  zx  2

 z 2  2 x  x 2  2 y  y 2  2 z  0 x; y; z
x 2
y z 2

2 3

Nên từ đó, ta có bất đẳng thức


2 2 2
n
 n
  n
  n

i xi2  yi2  zi2   i xi    i yi    i zi 
i 1  i 1   i 1   i 1 
Với mọi xi ; yi ; zi i  1, 2,.., n và mọi bộ số thực không âm  i có tổng là 1.

V – Một số bất đẳng thức ứng dụng


Bài 1: Cho bốn số x; y; z; t  0;1 . Chứng minh rằng tồn tại một hoán vị a, b, c, d của bốn số này và thỏa

 sin  ab    sin  cd   sin   a   b  .  c   d    ,   0,    1



Xét hàm số f  x; y   sin xy có biệt thức   2cos xyxy  sin xy  yx  xy . Do đó, để biệt thức
2

này không dương, ta cần chọn a  c, b  d để xy  0 và lưu ý các số đề bài cho thuộc  0;1 nên

cos xy  0, sin xy  0 . Từ đó có bất đẳng thức cần chứng minh.


Bài 2: Cho các số thực dương a, b, c, d . Chứng minh

2ab 2cd  a  c  b  d    a  d  c  b 
 
ab cd a bcd

xy 2
3 
Ta xét f  x, y   có biệt thức    yx  x y   0 x ; y  0 và áp dụng bất đẳng thức
2

x y  x  y
Jensen cho hai điểm A  a; b  , B  c; d  ta có điều phải chứng minh

Chú ý: theo biệt thức delta, ta chỉ cần x  y  0 là đủ hay giả thiết bài toán có thể ghi lại là “Cho các số
thực a; b; c; d có a  b  0, c  d  0 ”
Bài 3: Cho các số thực dương x1 , x2 , y1 , y2 , z1 , z2 và các số thực không âm  ;  có tổng là 1. Chứng

minh rằng

 x1  y1  z1  . x2  y2  z2 
 
 x1 y1  x2 y2  x2 y2


Vì x1 , x2 , y1 , y2 , z1 , z2 là các số thực dương nên tồn tại a1 , a2 , b1 , b2 , c1 , c2 thỏa x1  ea1 , y1  eb1 , z1  ec1 …

e x y z
Ta xét hàm số f  x; y; z   ln  e x  e y  e z  có biệt thức    x  y  z   0 và áp
2

e x
e ey

z 2

dụng bất đẳng thức Jensen cho hàm ba biến.


Bài 5: Chứng minh rằng

 x  x  
1 1 1

1.  x  y  x y     y1  y2   n  1, x1 , x2 , y1 , y2  0 
n n n n n n n n n
1 1 2 2 1 2

 
1 1 1

2.  x1n  y1n  n   x2n  y2n  n   x1  x2    y1  y2   0  n  1, x1 , x2 , y1 , y2  0 


n n n


1 1

     yx  xy 
2
Xét hàm số f  x, y   x n  y n có biệt thức    n  1 x n2 y n2 x n  y n
2
n n
và căn cứ

vào giá trị của n, áp dụng bất đẳng thức cho các bộ số ta được điều phải chứng minh.
Bài 6: Cho x1; x2 ; x3 ; y1; y2 ; y3 dương và các số không âm  ,  ,  có tổng là 1. Chứng minh

  x1  y1     x2  y2     x3  y3    x1 x2 x3  y1 y2 y3 


n n n n
nếu n > 1


Vì đây là các số không âm nên tồn tại a1; a2 ; a3 ; b1; b2 ; b3 sao cho xi  eai , yi  ebi i  1, 2,3 . Khi đó, xét


hàm số f  x; y   e x  e y 
n
có biệt thức

  n  n  1  e x  e y   e x  e y   n ex  e y  e  x  e  y   0
n2 x y 2 n 1 x 2 y 2

Áp dụng bất đẳng thức Jensen với bộ các số a1; a2 ; a3 ; b1; b2 ; b3 ta có điều cần chứng minh.

VI – Ý nghĩa hình học


Hàm số z  f  x; y  là một mặt cong trong không gian, ví dụ z  R 2  x 2  y 2 là nửa mặt cầu,

z  x 2  y 2 là mặt paraboloid hyperbolic…


Với mỗi bộ điểm A, B trong mpOxy, ta đã xây dựng một mặt phẳng (P) qua A, B và cắt mặt cong
z  f  x; y  theo 1 đường cong có phương trình tham số là

 x  x0  at

 C  :  y  y0  bt
 z  f x  at , y  bt
  0 0 
Từ đó, ta nghiên cứu các tính chất của đường cong (C) này như trong mặt phẳng (O’uv) bằng cách đặt
hệ trục mới sao cho trục Ou là AB và trục Ov song song Oz.
Ta thấy có khả năng như sau
1. Cắt (P) bởi mọi mặt phẳng thì luôn thu được một đường cong lõm.
2. Cắt (P) bởi lớp các mặt phẳng thỏa tính chất nào đó thì được đường cong lõm.
Ta đưa ra định nghĩa như sau
Định nghĩa 1: Trong mặt phẳng tọa độ Oxyz, đường cong (C) nằm trong mặt phẳng (P) song song Oz
gọi là lõm nếu ta đặt hệ trục O’uv sao cho O’u là giao tuyến của (P) với (Oxy) và O’v song song Oz thì
(C) trở thành đồ thị 1 hàm số lồi.
Định nghĩa 2: Hàm số f  x; y  gọi là lồi toàn phần trên miền D nếu cắt mặt cong z  f  x; y  bởi mọi

mặt phẳng song song Oz, ta luôn thu được 1 đường cong lõm trên D.

Ví dụ: Nửa mặt cầu z  R2  x2  y 2 là 1 hàm số lồi toàn phần.


Định nghĩa 3: Hàm số f  x; y  gọi là lồi một phần trên miền D nếu cắt mặt cong z  f  x; y  bởi 1 lớp

các mặt phẳng song song Oz ta mới có đường cong lồi trên D.
x  a
Ví dụ: Khi cắt mặt z  x 2  y 2 bởi mặt phẳng x  a thì ta thu được đường cong  và khi
z  a  y
2 2

chuyển đường cong này vào mặt phẳng tọa độ mới thì nó là đường parabol y  a 2  x 2 nên đây là đường

cong lồi (không lõm). Ngược lại, cắt bởi mặt phẳng y = b ta có đường cong là parabol y  x 2  b2 trong
mặt phẳng tọa độ mới, đây là đường cong lồi.
Lưu ý:
1. Nếu các điểm A, B lấy sao cho  x1  x2  y1  y2   0 thì đường giao tuyến của mặt cắt với Oxy có

vector chỉ phương v  a, b  đều có tung và hoành cùng dấu. Ta gọi hàm số f  x; y  lồi theo chiều dương

nếu giao tuyến của các mặt cắt với (Oxy) là đường thẳng có vector chỉ phương với tung và hoành cùng
dấu.
2. Tương tự, ta cũng có khái niệm lồi theo chiều âm nếu giao tuyến của các mặt cắt với (Oxy) là đường
thẳng có vector chỉ phương với tung và hoành trái dấu.
Tài liệu tham khảo
1. Toán cao cấp giải tích hàm nhiều biến, Nguyễn Đình Trí, nhà xuất bản giáo dục.
2. Sáng tạo bất đẳng thức, Phạm Kim Hùng, nhà xuất bản ĐHSP.

You might also like